Сумма модулей: Доказательства свойств модуля

Доказательства свойств модуля

Существуют следующие свойства модуля действительных чисел:

1) |a + b| ≤ |a| + |b|;

2) |ab| = |a| × |b|;

3) , a ≠ 0;

4) |a – b| ≥ |a| – |b|.

Проведем доказательства, рассматривая различные случаи значений a и b.

Доказательство 1) |a + b| ≤ |a| + |b|:

Если a и b – положительные числа, то их модули совпадают с их значениями: |a| = a, |b| = b. Из этого следует, что |a + b| = |a| + |b|.

Если a – отрицательное число, а b – положительное число, то выражение |a + b| можно записать как |b – a|. Выражение же |a| + |b| равно сумме абсолютных значений a и b, что больше, чем b – a. Поэтому |a + b| < |a| + |b|.

Если b – отрицательное число, а a – положительное, то |a + b| принимает вид |a – b|, что также меньше суммы модулей |a| + |b|.

Если a и b – отрицательные числа, то получим |–a – b|. Результат этого выражения равен |a + b| (т. к. |–a – b| = |–(a + b)| = |a + b|). Но уже было доказано, что |a + b| = |a| + |b|, следовательно и |–a – b| = |a| + |b|.

Доказательство 2) |ab| = |a| × |b|:
Здесь, в отличие от сложения, рассматривать все случаи особо не требуется, т. к. абсолютное значение произведения любых чисел (положительных ли, отрицательных ли) не зависит от знаков множителей. В выражении |ab| мы сначала перемножаем числа, а потом «отбрасываем» знак (отрицательный, если он есть), в выражении |a| × |b| сначала избавляемся от знаков, а потом перемножаем. Но от того, в какой момент был взят модуль (до или после умножения), не зависит абсолютное значение произведения.

Доказательство 3) , a ≠ 0:

Если a – положительное число, то |a| = a и, следовательно, доказываемое равенство верно, т. к. и правая и левая части равны 1/a.

Если a – отрицательное число, то имеем . Взятие модуля в обоих выражениях приведет к делению единицы на абсолютное значение a. 2-1}{4}d$. Равенство в нем достигается, когда $a_1 = \ldots = a_m \lt a_{m+1} = \ldots = a_{2m+1}$.

Поделиться ссылкой:

Похожее

Внеклассный урок — Модуль числа

Модуль числа

Модулем числа называется само это число, если оно неотрицательное, или это же число с противоположным знаком, если оно отрицательное.

Например, модулем числа 5 является 5, модулем числа –5 тоже является 5.

То есть под модулем числа понимается абсолютная величина, абсолютное значение этого числа без учета его знака.

Обозначается так: |5|, |х|, |а| и т.д.

Правило:

                                                                     |а| = а, если а ≥ 0.

                                                                     |а| = –а, если а < 0.

 

Пояснение:

|5| = 5
Читается так: модулем числа 5 является 5.

|–5| = –(–5) = 5
Читается так: модулем числа –5 является 5.

|0| = 0
Читается так: модулем нуля является ноль.

 

Свойства модуля:

1) Модуль числа есть неотрицательное число:

|а| ≥ 0

2) Модули противоположных чисел равны:

|а| = |–а|

3) Квадрат модуля числа равен квадрату этого числа:

|а|2 = a2

4) Модуль произведения чисел равен произведению модулей этих чисел:

|а · b| = |а| · |b|

6) Модуль частного чисел равен отношению модулей этих чисел:

|а : b| = |а| : |b|

7) Модуль суммы чисел меньше или равен сумме их модулей:

|а + b| ≤ |а| + |b|

8) Модуль разности чисел меньше или равен сумме их модулей:

|аb| ≤ |а| + |b|

9) Модуль суммы/разности чисел больше или равен модулю разности их модулей:

|а ± b| ≥ ||а| – |b||

10) Постоянный положительный множитель можно вынести за знак модуля:

|m · a| = m · |а|, m >0

11) Степень числа можно вынести за знак модуля:

|аk| = |а|k, если аk существует

12) Если |а| = |b|, то a = ± b

 

Геометрический смысл модуля.

Модуль числа – это величина расстояния от нуля до этого числа.

Для примера возьмем снова число 5. Расстояние от 0 до 5 такое же, что и от 0 до –5 (рис.1). И когда нам важно знать только длину отрезка, то знак не имеет не только значения, но и смысла. Впрочем, не совсем верно: расстояние мы измеряем только положительными числами – или неотрицательными числами. Пусть цена деления нашей шкалы составляет 1 см. Тогда длина отрезка от нуля до 5 равна 5 см, от нуля до –5 тоже 5 см.

На практике часто расстояние отмеряется не только от нуля – точкой отсчета может быть любое число (рис.2). Но суть от этого не меняется. Запись вида |a – b| выражает расстояние между точками а и b на числовой прямой.

 

Пример 1. Решить уравнение |х – 1| = 3.

Решение.

Смысл уравнения в том, что расстояние между точками х и 1 равно 3 (рис.2). Поэтому от точки 1 отсчитываем три деления влево и три деления вправо – и наглядно видим оба значения х:
х1 = –2, х2 = 4.

Можем и вычислить.

х – 1 = 3
х – 1 = –3

х = 3 + 1
х = –3 + 1

х = 4
х = –2.

Ответ: х1 = –2; х2 = 4.

 

Пример 2. Найти модуль выражения:

3√5 – 10.

Решение.

Сначала выясним, является ли выражение положительным или отрицательным. Для этого преобразуем выражение так, чтобы оно состояло из однородных чисел. Не будем искать корень из 5 – это довольно сложно. Поступим проще: возведем в корень 3 и 10. Затем сравним величину чисел, составляющих разность:

3 = √9. Следовательно, 3√5 = √9 · √5 = √45

10 = √100.

Мы видим, что первое число меньше второго. Значит, выражение отрицательное, то есть его ответ меньше нуля:

3√5 – 10 < 0.

Но согласно правилу, модулем отрицательного числа является это же число с противоположным знаком. У нас отрицательное выражение. Следовательно, надо поменять его знак на противоположный. Выражением, противоположным 3√5 – 10, является –(3√5 – 10). Раскроем в нем скобки – и получим ответ:

–(3√5 – 10) = –3√5 + 10 = 10 – 3√5.

Ответ:

|3√5 – 10| = 10 – 3√5.

 

Модуль — сумма — Большая Энциклопедия Нефти и Газа, статья, страница 1

Модуль — сумма

Cтраница 1

Модуль суммы не может превзойти сумму модулей слагаемых.  [1]

Модуль суммы двух или нескольких комплексных чисел не превосходит суммы модулей этих чисел.  [2]

Модуль суммы двух или нескольких чисел меньше или равен сумме модулей этих чисел.  [3]

Модуль суммы индексов всех особых точек невырожденного векторного поля v степени т ( обозначается Ind v) не превосходит числа Петровского — — Олейник II ( т) и сравним по модулю 2 с числом и. Никаких других ограничений на Irul v не существует.  [4]

Заменим модуль суммы в правой части ( 20) суммой модулей и потребуем выполнения полученного неравенства. В этом случае ( 20) будет выполняться автоматически.  [5]

Докажите, что модуль суммы двух перемещений не превосходит суммы модулей составляющих перемещений. В каком случае модуль суммы равен сумме модулей слагаемых перемещений.  [6]

Известно, что модуль суммы меньше или равен сумме модулей слагаемых.  [7]

Доказать, что модуль суммы двух комплексных чисел не превосходит суммы модулей этих чисел.  [8]

Установим теперь свойства модуля суммы и разности двух комплексных чисел.  [9]

Теорема о том, что модуль суммы не больше суммы модулей слагаемых, легко распространяется на случай абсолютно сходящихся рядов.  [10]

Теорема о том, что модуль суммы не больше суммы модулей слагаемых, легко распространяется на случай абсолютно сходящихся рядов.  [11]

Поскольку разложить в ряд Фурье модуль суммы гармоник в общем виде нельзя, укажем, что при незначительных искажениях несущей выходной сигнал будет подобен детектированному.  [12]

Принципиальный интерес представляет способ выделения модуля суммы и разности входных — величин, предложенный в.  [13]

Установим теперь важные для дальнейшего свойства модуля суммы и разности двух комплексных чисел.  [14]

Такое отображение, фактически представляющее собой натягивание модуля суммы гауссовскнх полей на параболоиды в направлении внешней нормали, переведет гладкие параболоиды в некоторые случайные геометрические тела. {2}}\Leftrightarrow \\xy>\left| x \right|\cdot \left| y \right|\Leftrightarrow \\xy>\left| xy \right|,\end{array}\)

а это противоречит определению модуля.

Следовательно, таких \( x;y\in \mathbb{R}\) не существует, а значит, при всех \( x,\text{ }y\in \mathbb{R}\) выполняется неравенство \( \left| x+y \right|\le \left| x \right|+\left| y \right|.\)

Примеры для самостоятельного решения:

1) Докажите свойство №6.

2) Упростите выражение \( \left| \frac{31}{8}-\sqrt{15} \right|+\left| \frac{15}{4}-\sqrt{15} \right|\).

Ответы:

1) Воспользуемся свойством №3: \( \left| c\cdot x \right|=\left| c \right|\cdot \left| x \right|\), а поскольку \( c>0\text{ }\Rightarrow \text{ }\left| c \right|=c\), тогда

\( \left| cx \right|=c\cdot \left| x \right|\), ч.т.д.

2) \( \left| \frac{31}{8}-\sqrt{15} \right|+\left| \frac{15}{4}-\sqrt{15} \right|\). {2}}\overset{<}{\mathop{\vee }}\,15\cdot 16\text{ }\Rightarrow \text{ }\)

\( \frac{15}{4}-\sqrt{15}\text{ }<0\text{ }\Rightarrow \text{ }\left| \frac{15}{4}-\sqrt{15} \right|=\sqrt{15}-\frac{15}{4}\).

Складываем значения модулей:

\( \displaystyle \left| \frac{31}{8}-\sqrt{15} \right|+\left| \frac{15}{4}-\sqrt{15} \right|=\frac{31}{8}-\sqrt{15}+\sqrt{15}-\frac{15}{4}=\frac{1}{8}=0.125\)

В уравнении сумма равна 0

Мы уже рассматривали уравнения, равные нулю (типа «произведение равно нулю»). К виду «произведение равно нулю» сводятся многие уравнения из разных разделов алгебры.

Если в уравнении сумма равна нулю, в некоторых случаях его можно решить, применяя следующее свойство функций:

Сумма нескольких неотрицательных функций равна нулю тогда и только тогда, когда каждая из функций равна нулю.

Таким образом, уравнение

   

где

   

   

   

   

равносильно системе уравнений

   

В частности,

   

   

где 2n — чётное натуральное число

   

   

Примеры уравнений, решение которых основано на этом свойстве функций.

   

ОДЗ: x∈R.

Сумма модулей равна нулю, если каждое из слагаемых равно нулю. Поэтому данное уравнение равносильно системе

   

Найдём корни каждого уравнения:

   

Оба модуля обращаются в нуль при x=2.

Ответ: 2.

   

ОДЗ: x∈[-4;2].

Сумма корней чётной степени равна нулю, если каждое из слагаемых рано нулю. Следовательно, это уравнение равносильно системе

   

Решаем каждое уравнение:

   

Оба слагаемых обращаются в нуль при x= -4.

Ответ: -4.

Для нахождения корней достаточно решить только одно из уравнений и проверить, удовлетворяют ли полученные корни остальным уравнениям системы.

   

   

ОДЗ: x∈(-∞; 1]U[9; ∞).

Сумма неотрицательных функций равна нулю, если каждая каждая из функций равна нулю:

   

Корень третьего уравнения — x=9 — удовлетворяет также 1-му и 2-му уравнениям системы.

Ответ: 9.

   

ОДЗ: x∈[-1; 1].

Правая часть уравнений — сумма неотрицательных функций. Соответственно, уравнение равносильно системе

   

Корни второго уравнения

   

x=1 и x= -1. Оба корня удовлетворяют и первому уравнению.

Ответ: ±1.

Как в другие уравнения из курса алгебры, решаемые с применением свойств функций,  уравнения, в которых сумма неотрицательных функций равна нулю, при первом рассмотрении могут производить впечатление сложных. На самом деле, решить их достаточно просто, если помнить соответствующий теоретический материал.

Сумма модулей отклонений — Студопедия

плюсы:

— нечувствительность к выбросам.

минусы:

— сложность вычислительной процедуры;

— возможность больших отклонений между фактическими и проектными функциями;

— неоднозначность значений параметров и т.д.

Исходя из этих преимуществ и недостатков, обычно в качестве меры близости выбирают сумму квадратов и выбирают такую функцию y=f(x), у которой эта сумма квадратов достигает минимума.

Для того, чтобы лучше понять сущность метода наименьших квадратов, необходимо вначале вспомнить математические основы определения экстремума функции нескольких переменных.

В первую очередь, введем некоторые определения:

Определение 1.Функция Z = ƒ (x, y)имеет максимум в точке М00, у0), если значение функции в этой точке больше значений ее в точках, достаточно близких к точке М00, у0), т.е.

ƒ (х0, у0) > ƒ (х0 + Δх, у0 + Δу).

Это означает, что полное приращение функции Z = ƒ (х, у), вызванное переходом от точки (х0, у0) к соседней точке, будет величиной отрицательной:

ΔZ = ƒ (х0 + Δх, у0 + Δу) — ƒ (х0, у0) < 0. (2.1)

Определение 2.Функция Z = ƒ (x, y)имеет минимум в точке М00, у0), если значение функции в этой точке меньше значений ее в точках, достаточно близких к точке М00, у0), т. е.


ƒ (х0, у0) < ƒ (х0 + Δх, у0 + Δу).

Это означает, что полное приращение функции Z = ƒ (х, у), будет величиной положительной:

ΔZ = ƒ (х0 + Δх, у0 + Δу) — ƒ (х0, у0) > 0. (2.2)

Допустим, что функция Z = ƒ (х, у) имеет в точке М00, у0) максимум или минимум (экстремум).Тогда для функции должно выполняться одно из неравенств (3.1) или (3.2) при любых, достаточно малых Δх, Δу.

Предположим, что Δу = 0; тогда функция Z = ƒ (х, у) сделается функцией только одной переменной х. Эта функция по условию имеет экстремум.

Таким образом, условия обращения в нуль частных производных функции или несуществование хотя бы одной из них являются необходимыми условиями, но недостаточными условиями экстремума функции.

Имеем систему:

. (2.4)

Условия (2. 4) являются необходимыми для существования экстремума функции. Но может случиться, что эти условия в некоторых обстоятельствах невыполнимы.

Достаточные условия существования экстремума функции нескольких переменных имеют более сложный вид.

Пусть в точке М00, у0) частные производные обращаются в нуль, т.е.

, .

Подсчитаем значения частных производных второго порядка функции

Z = ƒ (х, у) в этой точке и обозначим их соответственно буквами: А, В, С:

тогда:

1. Если АС — В2 > 0, то функция Z = ƒ (х, у) имеет в точке М00, у0) экстремум, а именно:

при А < 0 максимум,

при А > 0 минимум.

2. Если АС — В2 < 0, то функция Z = ƒ (х, у) не имеет в точке М00, у0) экстремума.

3. Если АС — В2 = 0, то вопрос о существовании экстремума функции в точке М00, у0) остается открытым и требуются дополнительные исследования.


Метод наименьших квадратов является одним из важных применений теории экстремума функции нескольких переменных.

Предположим, что в результате некоторого опыта или наблюдения установлена зависимость между переменными величинами х и у, выражаемая в виде таблицы:

х х1 х2 хn
у у1 у2 уn

Пусть требуется перейти от табличного метода задания функции к аналитическому (т.е. выраженному в виде формулы), причем, если это нельзя сделать точно, постараемся получить аналитическую связь приближенно.

Теперь обратимся к графическому изображению данной системы. Рассматривая значения х и у как координаты точек в прямоугольной системе координат, наносим эти точки на график. Пусть, например, построенные точки расположены достаточно близко к некоторой прямой. Поэтому можно приблизительно считать, что между х и у существует линейная зависимость, выражаемая формулой,

у = ах + b.

Поставим задачу аналитического определения неизвестных коэффициентов а и b.

В основе аналитического метода определения а и b лежит метод наименьших квадратов. Точки, полученные на основании опытных данных, вообще говоря, не лежат на искомой прямой. Если бы некоторая точка (хi, уi) лежала на прямой, то ее координаты удовлетворяли бы уравнению прямой, т.е. имело бы место равенство:


уi = axi + b или axi + b — yi = 0

Однако в общем случае подстановка координат точки в уравнение прямой дала бы:

axi + b — yi = εi,

где εi ─ какая то малая величина.

Прямая сумма модулей — Примеры задач

В абстрактной алгебре прямая сумма — это конструкция, которая объединяет несколько модулей в новый, более крупный. В некотором смысле прямая сумма модулей — это «самый общий» модуль, который содержит данные модули как подпространства.

Наиболее известные примеры этой конструкции встречаются при рассмотрении векторных пространств (модулей над полем) и абелевых групп (модулей над кольцом Z целых чисел). Конструкция также может быть расширена для покрытия банаховых и гильбертовых пространств.

Конструкция векторных пространств и абелевых групп

Мы даем конструкцию первой в этих двух случаях в предположении, что у нас есть только два объекта. Затем мы обобщаем на произвольное семейство произвольных модулей. Ключевые элементы общей конструкции более четко определяются при более глубоком рассмотрении этих двух случаев.

Конструкция для двух векторных пространств

Предположим, что V и W — векторные пространства над полем K .Мы можем превратить декартово произведение V × W в векторное пространство над K , определив операции покомпонентно:

  • ( v 1 , w 1 ) + ( v 2 , w 2 ) = ( v 1 + v 2 , ш 1 + ш 2 )
  • α ( v , w ) = (α v , α w )

для v , v 1 , v 2 дюймов V , w , w 1 , w 2 в W и α в K .

Результирующее векторное пространство называется прямой суммой V и W и обычно обозначается знаком плюса внутри круга:

V⊕W {\ displaystyle V \ oplus W}

Подпространство V × {0} V W изоморфно V и часто идентифицируется с V ; аналогично для {0} × W и W . (См. Внутреннюю прямую сумму ниже.) При такой идентификации верно, что каждый элемент V W может быть записан одним и только одним способом как сумма элемента V и элемента W . Размер V W равен сумме размеров V и W .

Эта конструкция легко обобщается на любое конечное число векторных пространств.

Конструкция для двух абелевых групп

Для абелевых групп G и H , которые записываются аддитивно, прямое произведение также называется прямой суммой.Таким образом, мы превращаем декартово произведение G × H в абелеву группу, определяя операции покомпонентно:

  • ( г 1 , ч 1 ) + ( г 2 , ч 2 ) = ( г 1 + г 2 , h 1 + h 2 )

для г 1 , г 2 дюймов G и h 1 , h 2 дюймов Н .

Обратите внимание, что мы также можем расширить операцию взятия целых кратных до прямой суммы:

для г в G , h в H и n целое число. Это аналогично расширению скалярного произведения векторных пространств до указанной выше прямой суммы.

Результирующая абелева группа называется прямой суммой из G и H и обычно обозначается знаком плюса внутри круга:

G⊕H {\ displaystyle G \ oplus H}

Подпространство G × {0} из G H изоморфно G и часто идентифицируется с G ; аналогично для {0} × H и H .(См. Внутреннюю прямую сумму ниже.) С этой идентификацией верно, что каждый элемент G H может быть записан одним и только одним способом как сумма элемента G и элемента из H . Ранг G H равен сумме рангов G и H .

Эта конструкция легко обобщается на любое конечное число абелевых групп.

Построение произвольного семейства модулей

Следует заметить явное сходство между определениями прямой суммы двух векторных пространств и двух абелевых групп.Фактически каждый из них является частным случаем построения прямой суммы двух модулей. Кроме того, изменяя определение, можно учесть прямую сумму бесконечного семейства модулей. Точное определение выглядит следующим образом.

Предположим, что R — некоторое кольцо, а { M i : i in I } — семейство левых модулей R , индексированных набором I . Прямая сумма для { M i } затем определяется как набор всех функций α с областью I , таких что α ( i ) ∈ M i для все i I и α ( i ) = 0 для всех, кроме конечного числа индексов i .

Две такие функции α и β можно добавить, записав (α + β) ( i ) = α ( i ) + β ( i ) для всех i (обратите внимание, что это снова ноль для все, кроме конечного числа индексов), и такая функция может быть умножена на элемент r из R , записав ( r α) ( i ) = r (α ( i )) для все и . Таким образом, прямая сумма становится левым модулем R . Обозначим его через

⨁i∈IMi {\ displaystyle \ bigoplus _ {i \ in I} M_ {i}}

Свойства

При правильной идентификации мы снова можем сказать, что каждый элемент x прямой суммы может быть записан одним и только одним способом как сумма конечного числа элементов M i .

Если M i на самом деле являются векторными пространствами, то размерность прямой суммы равна сумме размеров M i . То же верно и для ранга абелевых групп и длины модулей.

Каждое векторное пространство над полем K изоморфно прямой сумме достаточно большого количества копий K , поэтому в определенном смысле следует учитывать только эти прямые суммы. Это неверно для модулей над произвольными кольцами.

Тензорное произведение распределяется по прямым суммам в следующем смысле: если N — некоторый правильный R -модуль, то прямая сумма тензорных произведений N на M i (что являются абелевыми группами) естественно изоморфно тензорному произведению N на прямую сумму M i .

Прямые суммы также коммутативны и ассоциативны, что означает, что не имеет значения, в каком порядке формируется прямая сумма.

Группа линейных гомоморфизмов R из прямой суммы в некоторую левую R -модуль L естественно изоморфна прямому произведению групп R -линейных гомоморфизмов из M i до L .

Внутренняя прямая сумма

Предположим, что M — это некий модуль R , а M i — это подмодуль M для каждых i в I .Если каждое x в M может быть записано одним и только одним способом как сумма конечного числа элементов M i , то мы говорим, что M — это внутренняя прямая сумма субмодулей M i . В этом случае M естественно изоморфна (внешней) прямой сумме M i , как определено выше.

Прямое слагаемое из M — это подмодуль N , такой, что есть другой подмодуль N ‘ из M , такой, что M является внутренней внутренней прямой суммой N и N ′ .

Категориальная интерпретация

На языке теории категорий прямая сумма является копроизведением и, следовательно, копределом в категории левых R -модулей, что означает, что она характеризуется следующим универсальным свойством. Для каждых i в I учитывайте естественное вложение

ji: Mi → ⨁i∈IMi {\ displaystyle j_ {i}: M_ {i} \ rightarrow \ bigoplus _ {i \ in I} M_ {i}}

, который отправляет элементы M i к тем функциям, которые равны нулю для всех аргументов, кроме i .Если f i : M i M являются произвольными линейными картами R для каждого i , то существует ровно одна линейная карта R

f: ⨁i∈IMi → M {\ displaystyle f: \ bigoplus _ {i \ in I} M_ {i} \ rightarrow M}

, так что f o j i = f i для всех i .

Прямая сумма модулей с дополнительной структурой

Если рассматриваемые нами модули несут некоторую дополнительную структуру (например,грамм. норма или внутренний продукт), то прямая сумма модулей часто также может содержать эту дополнительную структуру. В этом случае мы получаем копродукт в соответствующей категории всех объектов, несущих дополнительную структуру. Два наиболее ярких примера относятся к банаховым и гильбертовым пространствам.

Прямая сумма банаховых пространств

Прямая сумма двух банаховых пространств X и Y является прямой суммой X и Y , рассматриваемых как векторные пространства, с нормой || ( x , y ) || = || x || X + || y || Y для всех x дюймов X и y дюймов Y .

Как правило, если X i , где i пересекает набор индексов I , представляет собой набор банаховых пространств, то прямая сумма ⊕ i I X i состоит из всех функций x с доменом I , так что x ( i ) ∈ X i для всех i I и

∑i∈I‖x (i) ‖Xi конечно. {\ displaystyle \ sum _ {i \ in I} \ | x (i) \ | _ {X_ {i}} {\ mbox {конечно.}}}

Норма определяется суммой выше. Прямая сумма с этой нормой снова является банаховым пространством.

Например, если мы возьмем набор индексов I = N и X i = R , то прямая сумма ⊕ i N будет пробелом l 1 , который состоит из всех последовательностей ( a i ) вещественных чисел с конечной нормой || a || = ∑ i | a i |.

Прямая сумма гильбертовых пространств

Если дано конечное число гильбертовых пространств H 1 , …, H n , можно построить их прямую сумму, как указано выше (поскольку они являются векторными пространствами), а затем повернуть прямую сумму в гильбертово пространство, определив внутренний продукт как:

⟨(x1, . .., xn), (y1, …, yn)⟩ = ⟨x1, y1⟩ + … + ⟨xn, yn⟩ {\ displaystyle \ langle (x_ {1}, …, x_ {n}), (y_ {1}, …, y_ {n}) \ rangle = \ langle x_ {1}, y_ {1} \ rangle +… + \ langle x_ {n}, y_ {n} \ rangle}

Это превращает прямую сумму в гильбертово пространство, которое содержит данные гильбертовы пространства как взаимно ортогональные подпространства.

Если дано бесконечно много гильбертовых пространств H i для i в I , мы можем провести такое же построение; обратите внимание, что при определении внутреннего продукта только конечное число слагаемых будет отличным от нуля. Однако результатом будет только внутреннее пространство продукта, и оно не будет полным.Затем мы определяем прямую сумму гильбертовых пространств H i как завершение этого внутреннего пространства продукта.

В качестве альтернативы и эквивалентно, можно определить прямую сумму гильбертовых пространств H i как пространство всех функций α с областью I , так что α ( i ) является элементом H i для каждых i в I и:

∑i‖α (я) ‖2 <1 {\ displaystyle \ sum _ {i} \ left \ | \ alpha _ {(i)} \ right \ | ^ {2} <{\ mathcal {1}} }

Тогда скалярное произведение двух таких функций α и β определяется как:

⟨α, β⟩ знак равно ∑i⟨αi, βi⟩ {\ displaystyle \ langle \ alpha, \ beta \ rangle = \ sum _ {i} \ langle \ alpha _ {i}, \ beta _ {i} \ rangle}

Это пространство заполнено, и мы получаем гильбертово пространство. {2}}. Сравнивая это с примером для банаховых пространств, мы видим, что прямая сумма банахова пространства и прямая сумма гильбертова пространства не обязательно совпадают. Но если имеется только конечное число слагаемых, то прямая сумма банахова пространства изоморфна прямой сумме гильбертова пространства.

Каждое гильбертово пространство изоморфно прямой сумме достаточно большого числа копий основного поля (либо R , либо C ).

de: Direkte Summe es: Suma directa fr: somme directe он: סכום ישר ja: 直 和

(PDF) Подмодули типов и разложение модулей по прямой сумме

98 J.DAUNS AND Y. ZHOU

(4) ⇒ (2). Пусть K — естественный класс. Чтобы показать (2), достаточно показать

, что для любых подмодулей X и Y из M, если X и Yare в K, тогда

будет X + Y. По лемме Цорна существует подмодуль Pmaximal

относительно X⊆P∈K и подмодуль Qmaximal относительно

до Y⊆Q∈K. Тогда P и Q дополняют подмодули M,

P∩Q≤eP и P∩Q ≤eQ. Таким образом, P и Q оба являются замыканиями

P∩Qin M. Если P = Q, по (4) существует 0– = X⊆P + Q, например

P∩X = 0 и X → P∩Q.Тогда X∈K и P⊂P⊕X∈K, противоречие

. Итак, P = Q и, значит, X + Y⊆P∈K.

(5) ⇒ (1). Предположим, что (1) не выполняется. Тогда существуют подмодули типа

T1- = T2 Mof типа K для естественного класса K. Отсюда следует

, что T1∩T2- = 0, T1∩T2≤eTifor i = 1,2, и T1∩T2 не является существенным

. в T1 + T2. Таким образом, существует 0  = A⊆T1 + T2, такое что

T1∩T2∩A = 0. Отсюда следует, что Ti∩A = 0fori = 1,2. Поскольку

каждый Ti является подмодулем типа M, мы имеем TiTA. Мы знаем, что

A = A / (T1∩A) ∼

= (A + T1) / T1⊆ (T2 + T1) / T1∼

= Т2 / (Т1∩Т2).Тогда

A∼

= B / (T1∩T2) для некоторого B с T1∩T2≤eB⊆T2. Обратите внимание, что B⊥A,

и поэтому B∩A = 0 иB⊕A⊆M.

(3) ⇒ (5). Предположим, что существует вложение X⊕ (X / Y) α

→ M

, где Y — собственный существенный подмодуль X и X⊥ (X / Y). Возьмем

x∈X, но x / ∈Y, и пусть m1 = α (x) и m2 = α (x + Y). Тогда m1R⊥

m2R. Чтобы в этом убедиться, пусть m1aR ∼

= m2bR для некоторых a, b ∈R. Отсюда следует, что

α (xaR) ∼

= α ((x + Y) bR). Это дает xaR ∼

= (x + Y) bR.Должно быть

xaR = 0, поскольку X⊥ (X / Y). Итак, m1aR = 0. Таким образом, m1R⊥m2R.

Кроме того, m⊥

1⊆m⊥

2 и m⊥

2 / m⊥

1≤eR / m⊥

1.Wenextprovem2 = 0,

, что дает противоречие. Определим β: m1R → m2R по β (m1r) = m2r,

r∈R. Тогда β является гомоморфизмом и ker (β) = m1m⊥

2. Пусть L будет замыканием типа

для ker (β) inm1R. Определим f: m1R → m1R⊕m2R (⊆M)

как f (x) = x + β (x), x∈m1R. Тогда это мономорфизм. Поскольку L

является замыканием типа ker (β) inm1R, f (ker (β)) параллельно f (L).Это

дает, что ker (β) параллельно f (L). Пусть Ltc и f (L) tc — замыкания типа

Земли f (L) inM соответственно. Тогда и Ltc, и f (L) tc

являются замыканиями типа ker (β) в M. По (3) Ltc = f (L) tc. Отсюда следует, что

L + f (L) является параллельным расширением L.ПримечаниеL — это подмодуль типа

m1R. Так как m1R⊥m2R, Lis — подмодуль типа для m1R⊕m2R. Это

означает, что L = L + f (L), т. Е. F (L) .L. Отсюда следует, что β (L) ⊆L.

прямая сумма в nLab

Прямые суммы и слабые прямые произведения

Контекст

Пределы и пределы

пределы и коллимиты

1-категориальный

  • предел и копредел

    • лимитов и копределов на примере

    • Коммутативность пределов и копределов

    • малый лимит

    • отфильтрованный colimit

    • колимит просеянный

    • подключенный лимит, широкий откат

    • сохраненный лимит, отраженный лимит, созданный лимит

    • продукт, продукт волокна, изменение базы, сопродукт, откат, выталкивание, изменение базы, эквалайзер, коэквалайзер, соединение, встреча, конечный объект, исходный объект, прямой продукт, прямая сумма

    • конечный предел

  • Канский добавочный номер

  • взвешенный лимит

  • конец и коэнда

2-категоричный

(∞, 1) -категория

Модельно-категориальная

Идея

Понятие прямой суммы или слабого прямого произведения — это понятие из алгебры, которое действительно имеет смысл в любой категории CC с нулевыми морфизмами (то есть любой категории, обогащенной над замкнутой моноидальной категорией заостренных множеств), поскольку пока существуют необходимые (со) лимиты.

Базовый и знакомый пример — прямая сумма V1⊕V2V_1 \ oplus V_2 двух векторных пространств V1V_1 и V2V_2 над некоторым полем или, в более общем смысле, двух модулей над некоторым кольцом. Как правило, для II — множество и {Vi} i∈I \ {V_i \} _ {i \ in I} — индексируемое II семейство векторных пространств или модулей, их прямая сумма ∈i∈IVi \ bigoplus_ {i \ in I } V_i — это набор формальных линейных комбинаций элементов в каждом из ViV_i. Это может частично мотивировать терминологию: элемент в прямой сумме — это сумма элементов , по крайней мере, в этих случаях.

Это обобщает двумя разными способами, которые мы называем прямой суммой и слабым прямым произведением . Во многих случаях (как в примере выше) они совпадают, но не всегда. Также во многих случаях прямые суммы будут такими же, как и сопутствующие продукты. В любом случае конечные слабые прямые продукты такие же, как и продукты, но бесконечные версии (почти всегда) разные.

Терминология

Название «слабый прямой продукт» происходит от понятия прямого продукта в алгебре для продукта в конкретной категории, созданного с помощью функтора забывчивости; слабый прямой продукт будет подобъектом прямого продукта (и всего прямого продукта в конечных случаях). Но здесь мы не будем ограничиваться контекстом такой конкретной категории.

Термин «прямая сумма» происходит от конечного побочного продукта (одновременно продукта и сопутствующего продукта) в аддитивных категориях. Аддитивный характер этих побочных продуктов распространяется в бесконечном случае (где побочные продукты обычно больше не появляются) на побочные продукты, а не на продукт. Даже когда прямая сумма не совпадает с побочным продуктом, он все равно сохраняет часть этого аромата.

В классических примерах CC прямая сумма и слабое прямое произведение совпадают.Однако приведенные ниже общие определения различают их в некоторых случаях, и мы используем термины «прямая сумма» и «слабый прямой продукт», чтобы лучше всего вызвать ощущения «как сопутствующий продукт» и «часть продукта».

Определения

Пусть 𝒞 \ mathcal {C} — категория с произведениями и копроизведениями, а также с нулевыми морфизмами. Пусть II — множество, и пусть (Ai) i∈I (A_i) _ {i \ in I} — II-индексированное семейство объектов в 𝒞 \ mathcal {C}, следовательно, функция A: I → Obj ( 𝒞) A: I \ to Obj (\ mathcal {C}).

Теперь мы определим как прямую сумму, так и слабое прямое произведение этого семейства.AiA_i будем называть прямыми слагаемыми или (слабыми) прямыми множителями .

Прямая сумма

Здесь мы должны предположить, кроме того, что 𝒞 \ mathcal {C} — обычная категория (или иначе имеет хорошее представление об изображении).

Определение

Пусть rr — морфизм копроизведения ∐iAi \ coprod_i A_i в произведение ∏iAi \ prod_i A_i, характеризующийся наличием следующих компонентов

(Ai → ∐A → r∏A → Aj) = {IdAiifi = j0ijifi ≠ j, \оставил( A_i \ to \ coprod A \ stackrel {r} {\ to} \ prod A \ to A_j \верно) знак равно \оставил\{ \множество{ Id_ {A_i} & if \; я = j \\ 0_ {ij} & если \; я \ neq j ,} \верно.\,

, где 0ij0_ {ij} — нулевой морфизм от AiA_i к AjA_j.

Прямая сумма по семейству {Ai} \ {A_i \} — это изображение

∐iAi → coimr⨁iAI → imriAi \ coprod_i A_i \ overset {\ coim r} \ to \ bigoplus_i A_I \ overset {\ im r} \ to \ prod_i A_i

морфизма рр.

Слабое прямое произведение

Здесь мы рассматриваем финишные изделия

∏i∈FAi \ prod_ {i \ in F} A_i

, поскольку FF изменяется на конечных подмножествах индексного множества II. (В конструктивной математике используйте здесь «конечно индексированные» или «конечные по Куратовски» … хотя если II имеет разрешимое равенство, как это имеет место в обычных примерах, то каждое конечно индексированное подмножество II на самом деле конечно в самом строгом смысле.)

Эти конечные произведения образуют прямую систему, индексируемую направленным множеством 𝒫finI \ mathcal {P} _ {fin} I конечных подмножеств II (упорядоченных по включению) с отображением

∏i∈FAi → ∏i∈GAi, \ prod_ {i \ in F} A_i \ to \ prod_ {i \ in G} A_i,

, где F⊆GF \ substeq G, заданная формулой

∏i∈FAi≅∏i∈FAi × ∏i∈G ∖ F1 → (id, 0) ∏i∈FAi × ∏i∈G ∖ FAi≅∏i∈GAi. \ prod_ {i \ in F} A_i \ cong \ prod_ {i \ in F} A_i \ times \ prod_ {i \ in G \ setminus F} 1 \ stackrel {(id, 0)} {\ to} \ prod_ { i \ in F} A_i \ times \ prod_ {i \ in G \ setminus F} A_i \ cong \ prod_ {i \ in G} A_i. wk_i A_i определяется как направленный копредел этой прямой системы.

Примеры

Пример

В категориях Grp или Ab (абелевых) групп прямая сумма и слабое прямое произведение согласуются. Для конечного числа объектов это то же самое, что и прямой продукт, который является продуктом в обеих категориях.

Предложение

В этих примерах прямая сумма также может быть описана в более элементарных терминах как подгруппа прямого произведения:

⨁i: IAi = {(ai) i: I | ess∀ (i: I), ai = 0}, \ bigoplus_ {i: I} A_i = \оставил\{ (a_i) _ {i: I} \; | \; ess \ forall (i: I), \; a_i = 0 \верно\} \ ,,

, где «ess∀ess \ forall» означает «для всех, кроме конечного множества».Это проясняет, что прямая сумма равна прямому продукту, когда задействовано только конечное число объектов.

Для 𝒞 = \ mathcal {C} = Ab, RRMod это группа формальных линейных комбинаций элементов в слагаемых.

Пример

Для RR кольца прямые суммы в категории RRMod или модулей над RR даются суммами на нижележащих абелевых группах.

Пример

В категории заостренных множеств прямая сумма и слабое прямое произведение различаются.p прямых сумм для 1≤p≤∞1 \ leq p \ leq \ infty, хотя я не знаю, каким универсальным свойствам они все удовлетворяют.) В этом случае прямая сумма совпадает с копроизведением, а слабое прямое произведение — то же самое, что и произведение даже для бесконечно большого числа объектов. См. Прямую сумму банаховых пространств.

Внутренние прямые суммы

Дан объект BB и семейство подобъектов? AiA_i группы BB (или, в более общем смысле, семейство морфизмов Ai → BA_i \ to B, или эквивалентно отображение ∐iAi → B \ coprod_i A_i \ to B), предположим, что существует прямая сумма ⨁iAi \ bigoplus_i A_i.Предположим далее, что отображение ∐iAi → B \ coprod_i A_i \ в B факторизуется через отображение ∐iAi → ⨁iAi \ coprod_i A_i \ в \ bigoplus_i A_i (что означает, что оно уникально множится, если ∐iAi → ⨁iAi \ coprod_i A_i \ to \ bigoplus_i A_i эпично, так как должно быть в обычной категории). Наконец, предположим, что (или) фактор-отображение ⨁iAi → B \ bigoplus_i A_i \ to B является изическим. Затем мы говорим, что BB — это внутренняя прямая сумма AiA_i.

Напротив, абстрактно определенная прямая сумма ⨁iAi \ bigoplus_i A_i может называться внешней прямой суммой .Эти термины обычно используются с конкретными категориями, где AiA_i может быть задан независимо (для внешней прямой суммы) или как подмножество некоторого окружающего пространства (либо BB, либо что-то из того, что BB является подмножеством) для внутренней прямой суммы. В слишком абстрактном контексте разницы нет: с одной стороны, любая внутренняя прямая сумма тем более изоморфна любой внешней прямой сумме; с другой стороны, для внешней прямой суммы существует естественное отображение ∐iAi → ⨁iAi \ coprod_i A_i \ to \ bigoplus_i A_i, относительно которого внешняя прямая сумма является внутренней прямой суммой.3 | y \ in \ mathbb {F} \} \), то уравнение (4.4.2) остается в силе.

Если \ (U = U_1 + U_2 \), то для любого \ (u \ in U \) существуют \ (u_1 \ in U_1 \) и \ (u_2 \ in U_2 \) такие, что \ (u = u_1 + u_2. \)

Если так получилось, что \ (u \) можно однозначно записать как \ (u_1 + u_2 \), то \ (U \) называется прямой суммой \ (U_1 \) и \ (U_2. \)

Определение 4.4.3: Прямая сумма

Предположим, что каждое \ (u \ in U \) может быть однозначно записано как \ (u = u_1 + u_2 \) для \ (u_1 \ in U_1 \) и \ (u_2 \ in U_2 \).{2m + 1} \}. \]
Тогда \ (\ mathbb {F} [z] = U_1 \ oplus U_2. \)

Предложение 4.4.6 . Пусть \ (U_1, U_2 \ subset V \) — подпространства. Тогда \ (V = U_1 \ oplus U_2 \) тогда и только тогда, когда выполняются следующие два условия:

  1. \ (V = U_1 + U_2; \)
  2. Если \ (0 = u_1 + u_2 \) с \ (u_1 \ in U_1 \) и \ (u_2 \ in U_2 \), тогда \ (u_1 = u_2 = 0. \)

Доказательство.
\ ((«\ Rightarrow») \) Предположим, \ (V = U_1 \ oplus U_2 \).Тогда по определению выполняется условие 1. Конечно, \ (0 = 0 + 0 \), и, поскольку по уникальности это единственный способ записать \ (0 \ in V \), мы имеем \ (u_1 = u_2 = 0 \).

\ ((«\ Leftarrow») \) Предположим, что выполнены условия 1 и 2. По условию 1 для всех \ (v \ in V \) существуют \ (u_1 \ in U_1 \) и \ (u_2 \ in U_2 \) такие, что \ (v = u_1 + u_2 \). Предположим, \ (v = w_1 + w_2 \) с \ (w_1 \ in U_1 \) и \ (w_2 \ in U_2 \). Вычитая два уравнения, получаем

\ [0 = (u_1 — w_1) + (u_2 — w_2), \]

, где \ (u_1 — w_1 \ in U_1 \) и \ (u_2 — w_2 \ in U_2 \).По условию 2 это подразумевает \ (u_1 — w_1 = 0 \) и \ (u_2 — w_2 = 0 \), или, что эквивалентно, \ (u_1 = w_1 \) и \ (u_2 = w_2 \), как требуется.

Предложение 4.4.7. Пусть \ (U_1, U_2 \ subset V \) будут подпространствами. Тогда \ (V = U_1 \ oplus U_2 \) тогда и только тогда, когда выполняются следующие два условия:

  1. \ (V = U_1 + U_2; \)
  2. \ (U_1 \ cap U_2 = \ {0 \}. \)

Доказательство.
\ ((«\ Rightarrow») \) Предположим, \ (V = U_1 \ oplus U_2 \).Тогда по определению выполняется условие 1. Если \ (u \ in U_1 \ cap U_2 \), то \ (0 = u + (−u) \) с \ (u \ in U_1 \) и \ (- u \ in U_2 \) (почему?). По предложению 4.4.6 имеем \ (u = 0 \) и \ (- u = 0 \), так что \ (U_1 \ cap U_2 = \ {0 \}. \)

\ ((«\ Leftarrow») \) Предположим, что выполнены условия 1 и 2. Чтобы доказать, что выполняется \ (V = U_1 \ oplus U_2 \), предположим, что

\ [0 = u_1 + u_2, \ rm {~ где ~} u_1 \ в U_1 \ rm {~ и ~} u_2 \ в U_2. \ tag {4.3} \]

По предложению 4.4.6 достаточно показать, что \ (u_1 = u_2 = 0 \).3 \ neq U_1 \ oplus U_2 \ oplus U_3 \), поскольку, например,

\ [(0, 0, 0) = (0, 1, 0) + (0, 0, 1) + (0, -1, -1). \]

Но \ (U_1 \ cap U_2 = U_1 \ cap U_3 = U_2 \ cap U_3 = \ {0 \} \), так что аналог предложения 4.4.7 не выполняется.

Авторы

Версии этого учебника в твердом и мягком переплете доступны на сайте WorldScientific. com.

О. В. Камловский, “Сумма модулей коэффициентов Уолша для некоторых сбалансированных булевых функций”, Матем.Вопр. Криптогр., 8: 4 (2017), 75–98













Эта статья цитируется в научной статье 1 (всего в статье 1 )

Сумма модулей коэффициентов Уолша для некоторых сбалансированных булевых функций

О.В. Камловский

ООО «Центр Сертификационных Исследований», Москва

Аннотация: Мы рассматриваем следующие сбалансированные булевы функции: а) построенные из нормальной бент-функции методом Доббертина, б) мажоритарная функция, в) функции, значения единиц которых последовательно расположены в таблице истинности. Получены точные формулы и оценки сумм модулей коэффициентов Уолша.

Ключевые слова: Логические функции, коэффициенты Уолша, генераторы фильтрации, генераторы комбинирования.

DOI: https://doi.org/10.4213/mvk240

Полный текст: PDF-файл (225 kB)
Ссылки : PDF файл HTML файл

Библиографические базы данных:


УДК: 519.12 + 519.719.2
Поступила 11.V.2017

Образец цитирования: О. В. Камловский, “Сумма модулей коэффициентов Уолша для некоторых сбалансированных булевых функций”, Матем. Вопр. Криптогр., 8: 4 (2017), 75–98

Цитирование в формате AMSBIB

\ RBibitem {Kam17}
\ by О. ~ В. ~ Камловский
\ paper Сумма модулей коэффициентов Уолша для некоторых сбалансированных булевых функций
\ jour Матем. Вопр. Криптогр.
\ год 2017
\ vol 8
\ issue 4
\ pages 75--98
\ mathnet {http://mi. mathnet.ru/mvk240}
\ crossref {https://doi.org/10.4213/mvk240 }
\ mathscinet {http://www.ams.org/mathscinet-getitem?mr=3770676}
\ elib {https://elibrary.ru/item.asp?id=32641310}

Варианты соединения:

  • http://mi.mathnet.ru/eng/mvk240
  • https://doi.org/10.4213/mvk240
  • http://mi.mathnet.ru/eng/mvk/v8/i4/p75

    Цитирующие статьи в Google Scholar: Русские цитаты, Цитаты на английском языке
    Статьи по теме в Google Scholar: Русские статьи, Английские статьи

    Эта публикация цитируется в следующих статьях:

    1. О.А. Логачев, С. Н. Федоров, В. В. Ященко, “О $ \ Delta $ -эквивалентности булевых функций”, Дискретная математика. Appl., 30: 2 (2020), 93–101
  • Количество просмотров:
    Эта страница: 295
    Полный текст: 143
    Ссылки: 38
    Первая страница:

    прямой перевод% 20sum% 20of% 20modules — английский французский перевод прямого% 20sum% 20of% 20modules

    Ваш поиск не дал результатов

    EN Слова, похожие на прямые% 20sum% 20of% 20modules

    • отстранение ,
    • Därstetten ,
    • du reste ,
    • Drust IX des Pictes ,
    • Друк Цендхен ,
    • Drucat ,
    • Дроге де Травейл ,
    • Дроге-де-Травей ,
    • Дроэда ,
    • Driss Jettou ,
    • Дрезденко ,
    • Дрезднер Банк ,
    • Дрезден ,
    • Дрезде

    FR Слова, похожие на прямые% 20sum% 20of% 20modules

    • Därstetten ,
    • Дурресский район ,
    • Сухих дрожжей ,
    • Стены из сухого камня ,
    • Стена из сухого камня ,
    • Сухой камень ,
    • галантерея ,
    • Друст IX пиктов ,
    • Друк Цендхен ,
    • аптека ,
    • Торговля наркотиками ,
    • Наркотуризм ,
    • Дизайн лекарств ,
    • наркозависимый ,
    • наркозависимость

    Прямая сумма — go2kanid

    Прямые суммы определены для ряда различных видов математических объектов,

    включая подпространства, матрицы, модули и группы.

    Прямая сумма матрицы определяется как

    (Ayres 1962, стр. 13-14).

    Прямая сумма двух подпространств и представляет собой сумму подпространств, в которых и имеют общий только нулевой вектор (Розен 2000, стр. 357).

    Важным свойством прямой суммы является то, что она является копродуктом в категории модулей (т. Е. Прямой суммой модулей). Это общее определение как следствие дает определение прямой суммы абелевых групп и (поскольку они являются -модулями, т.е., модули над целыми числами) и прямую сумму векторных пространств (поскольку они являются модулями над полем). Обратите внимание, что прямая сумма абелевых групп такая же, как прямое произведение группы, но термин прямая сумма не используется для неабелевых групп.

    Обратите внимание, что прямые продукты и прямые суммы различаются для бесконечных индексов. Элемент прямой суммы равен нулю для всех элементов, кроме конечного, в то время как элемент прямого произведения может иметь все ненулевые элементы.

    СМОТРИ ТАКЖЕ: Abelian Group, Direct Product, Direct Summand, Group Direct Product, Group Direct Sum, Matrix Direct Sum, Module, Module Direct Sum

    Части этой записи предоставлены Тоддом Роулендом

    ССЫЛКИ:

    Ayres , Ф.Jr. Очерк теории и проблем матриц Шаума. New York: Schaum, 1962.

    Rosen, K.H. (Ed.). Справочник по дискретной и комбинаторной математике. Boca Raton, FL: CRC Press, 2000.

    Прямая сумма двух подпространств и является суммой подпространств, в которых и имеют общий только нулевой вектор (Rosen 2000, стр. 357).

    Важным свойством прямой суммы является то, что она является сопутствующим продуктом в категории модулей (т.е., модульная прямая сумма). Это общее определение как следствие дает определение прямой суммы абелевых групп и (поскольку они являются -модулями, т. Е. Модулями над целыми числами) и прямой суммы векторных пространств (поскольку они являются модулями над полем).

    Логарифмы примеры и решения 10 класс: Свойства логарифмов и примеры их решений (ЕГЭ — 2021)

    {\frac{1}{2}}}=\sqrt{4}=2\)), а вот \( \displaystyle {{\log }_{-4}}2\) не существует.

    Поэтому и отрицательные основания проще выбросить, чем возиться с ними.

    Ну а поскольку основание a у нас бывает только положительное, то в какую бы степень мы его ни возводили, всегда получим число строго положительное.

    Значит, аргумент должен быть положительным.

    Например, \( \displaystyle {{\log }_{2}}\left( -4 \right)\) не существует, так как \( 2\) ни в какой степени не будет отрицательным числом (и даже нулем, поэтому \( \displaystyle {{\log }_{2}}0\) тоже не существует).

    В задачах с логарифмами первым делом нужно записать ОДЗ. 

    Приведу пример:

    Решим уравнение \( \displaystyle {{\log }_{x}}\left( x+2 \right)=2\).

    Вспомним определение: логарифм \( \displaystyle {{\log }_{x}}\left( x+2 \right)\) – это степень, в которую надо возвести основание \( x\), чтобы получить аргумент \( \displaystyle \left( x+2 \right)\).

    И по условию, эта степень равна \( 2\): \( \displaystyle {{x}^{2}}=x+2\).{2}}-x-2=0\).

    Решим его с помощью теоремы Виета: сумма корней равна \( 1\), а произведение \( -2\). Легко подобрать, это числа \( 2\) и \( -1\).

    Но если сразу взять и записать оба этих числа в ответе, можно получить 0 баллов за задачу на ЕГЭ.

    Почему?

    Давайте подумаем, что будет, если подставить эти корни в начальное уравнение?

    \( \displaystyle x=2\text{: }{{\log }_{2}}\left( 2+2 \right)={{\log }_{2}}4=2\) – верно.

    \( \displaystyle x=-1\text{: }{{\log }_{-1}}\left( -1+2 \right)=2\) – это явно неверно, так как основание не может быть отрицательным, то есть корень \( x=-1\) – «сторонний».

    Чтобы избежать таких неприятных подвохов, нужно записать ОДЗ еще до начала решения уравнения:

    \( \displaystyle \left\{ \begin{array}{l}x>0\\x\ne 1\\x+2>0\end{array} \right.\text{ }\Leftrightarrow \text{ }\left\{ \begin{array}{l}x>0\\x\ne 1.\end{array} \right.\)

    Тогда, получив корни \( x=2\) и \( x=-1\), сразу отбросим корень \( -1\), и напишем правильный ответ.

    Пример 1 (попробуй решить самостоятельно)

    Найдите корень уравнения \( \displaystyle {{\log }_{x+1}}\left( 2x+5 \right)=2\). Если корней несколько, в ответе укажите меньший из них.

    Решение:

    \( \displaystyle {{\log }_{x+1}}\left( 2x+5 \right)=2\).

    В первую очередь напишем ОДЗ:

    \( \displaystyle \left\{ \begin{array}{l}x+1>0\\x+1\ne 1\\2x+5>0\end{array} \right.\text{ }\Leftrightarrow \text{ }\left\{ \begin{array}{l}x>-1\\x\ne 0\\x>-\frac{5}{2}\end{array} \right.\text{ }\Leftrightarrow \text{ }\left\{ \begin{array}{l}x>-1\\x\ne 0.\end{array} \right.\)

    Теперь вспоминаем, что такое логарифм: в какую степень нужно возвести основание \( \displaystyle x+1\), чтобы получить аргумент \( \displaystyle 2x+5\)?

    Хотите читать учебник без ограничений? Зарегистрируйтесь:

    Во вторую. То есть:

    \( \displaystyle {{\left( x+1 \right)}^{2}}=2x+5\text{ }\Leftrightarrow \text{ }{{x}^{2}}+2x+1=2x+5\text{ }\Leftrightarrow \text{ }{{x}^{2}}-4=0\text{ }\Leftrightarrow \text{ }\left[ \begin{array}{l}x=2\\x=-2.\end{array} \right.\)

    Казалось бы, меньший корень равен \( \displaystyle -2\). Но это не так: согласно ОДЗ корень \( \displaystyle x=-2\) – сторонний, то есть это вообще не корень данного уравнения. Таким образом, уравнение имеет только один корень: \( \displaystyle x=2\).

    Ответ: \( \displaystyle x=2\).

    Урок 27. логарифмические уравнения — Алгебра и начала математического анализа — 10 класс

    Алгебра и начала математического анализа, 10 класс

    Урок № 27. Логарифмические уравнения.

    Перечень вопросов, рассматриваемых в теме

    1) Понятие простейшего логарифмического уравнения

    2) Основные способы решения логарифмический уравнений

    3) Общие методы в решении логарифмических уравнений

    Глоссарий по теме

    Простейшее логарифмическое уравнение. Уравнение вида , где, a > 0, a ≠ 1.

    Основные способы решения логарифмических уравнений

    1. , где, a > 0, a ≠ 1, то , при условии, что

    2. .

    Общие методы для решения логарифмических уравнений

    1. Разложение на множители.
    2. Введение новой переменной.
    3. Графический метод.

    Основная литература:

    Колягин Ю.М., Ткачева М.В., Фёдорова Н.Е. и др. Математика: алгебра и начала математического анализа, геометрия. Алгебра и начала математического анализа. 10 класс. Базовый и углублённый уровни. – М.: Просвещение, 2014.–384с.

    Открытые электронные ресурсы:

    http://fipi.ru/

    Теоретический материал для самостоятельного изучения

    Уравнение вида , где, a > 0, a ≠ 1 называют простейшим логарифмическим уравнением.

    Данное уравнение имеет единственное решение, которое мы можем получить графически или по определению логарифма: .

    Способы решения логарифмических уравнений:

    1. Если , то (где, a > 0, a ≠ 1,

    Пример 1.

    .

    Воспользуемся определением логарифма

    ;

    .

    Оба корня удовлетворяют неравенству

    Ответ: – 8; 1.

    1. Если

    Если ,

    Пример 2.

    .

    ;

    ;

    ;

    ;

    Ответ: 1.

    Пример 3.

    .

    В данном уравнении систему с ограничивающими условиями можно не составлять, сделав в конце проверку о существовании логарифмов для конкретных значений х.

    Сумму логарифмов в левой части заменим логарифмом произведения:

    .

    Подставим каждый корень в исходное уравнение, получаем верные числовые равенства.

    Ответ: 3; 4.

    Встречаются уравнения, когда нельзя сразу использовать 1 или 2 правило. В этом случае сначала используют общие методы решения уравнений.

    1. Разложение на множители.

    Пример 4.

    Перенесем все в левую часть:

    Можно увидеть общий множитель: .

    Для этого приведем к основанию первый логарифм:

    .

    Вынесем за скобку общий множитель:

    Имеем произведение равное нулю. (Произведение равно нулю тогда и только тогда, когда один из множителей равен нулю)

    , два простейших логарифмических уравнения.

    ;

    Выполняем проверку. Оба числа являются корнями уравнения.

    Ответ: 3; 5.

    1. Введение новой переменной.

    Пример 5.

    Замена: тогда

    Обратная замена:

    Оба числа являются корнями уравнения.

    Ответ: ; 5.

    1. Графический способ решения.

    Строим графики левой и правой частей уравнения, определяем абсциссы точек пересечения графиков.

    Примеры и разбор решения заданий тренировочного модуля

    №1. Решите уравнение:

    Решение.

    Дважды используем определение логарифма:

    Ответ: 6.

    №2 Укажите промежуток, содержащий нули функции

    .

    Возможные варианты ответа:

    Решение: Чтобы найти нули функции, приравниваем ее к нулю.

    Приведем логарифмы к основанию 5: .

    Две равные дроби с равными знаменателями, следовательно, равны и числители. Т. е. Слева и справа логарифмы по одинаковому основанию, значит .

    Ответ: 4

    Логарифмические уравнения. 10-11 класс

    Стоит напомнить всем, что логарифмическими называют уравнения, в которых переменная или функция от «икс» находится под знаком логарифма.
    При равносильных преобразованиях справедливая формула перехода от логарифмического до простого уравнения
    logaf(x)=c⇔f(x)=ac.
    ОДЗ: основание логарифма должно быть больше нуля и не равняться единице,
    функция – положительной
    {x>0, x≠1, f(x)>0}.
    Важно знать частные случаи простейших логарифмических уравнений:
    правая сторjна равна нулю (с=0) или единицы (с=1):
    логарифм основания равен единице
    c=1⇔logaa=1⇔f(x)=a.
    логарифм единицы равен нулю
    c=1⇔loga1=0⇔f(x)=1.
    Эти формулы Вы должны знать на память, поскольку их чаще всего применяют при сведении логарифмов до простейшего типа.
    С целью научить Вас раскрывать логарифмические уравнения, а также подготовить к ВНО тестированию нами решены 40 примеров, которые в полной мере охватывают все известные методы решения логарифмических уравнений, которые Вас учат в 10-11 классе школьной программы, и дальше на первых курсах в ВУЗ-ах.

    Схема вычисления логарифмических уравнений

    1. если возможно, выписать область допустимых значений логарифмов и функций, которые в него входят.
    2. свести уравнение к простейшему типу путем элементарных преобразований, которые заключаются в вынесении степени из основания логарифма (или наоборот), логарифмированию и потенцированию (возведение в степень по основанию (экспонента, основы =10, 2, π)
    3. в случаях сложных уравнений вводят замену переменных и сводят к квадратным или другим известным уравнениям.

    Вычисление уравнений с логарифмом

    Пример 16.1 Решить уравнение logax=c.

    Решение: Имеем простейшее логарифмическое уравнение, которое решается методом сведения к одному основанию логарифмов:
    logax=c
    (здесь a>0, a≠1),
    logax=c•1,
    logax=c•logaa,
    logax= logaac
    Здесь использовали свойства логарифма, единицу расписали как логарифм основания, после чего множитель c внесли под логарифм.
    Далее опустили основы и приравняли выражения в логарифмах:
    x=ac.
    ОДЗ: x>0.
    Ответ: ac – Г.

     

    Пример 16.2 Решить уравнение log1/2(x)=-4.

    А

    Б

    В

    Г

    Д

    ø

    -16

    1/16

    1/16; 16

    16

    Решение: ОДЗ функции под логарифмом: x>0.
    Сводим уравнение к одному основанию логарифмов

    При равных основах приравниваем выражения под логарифмами:
    x=(1/2)-4,
    x=24,
    x=16.
    Ответ: 16 – Д.

     

    Пример 16.3 Решить уравнение log2(-x)=5.

    А

    Б

    В

    Г

    Д

    ø

    32

    -32

    1/32

    -1/32

    Решение: Выполняем раскрытия логарифмов по данной в начале инструкции:
    ОДЗ – -x>0,x<0.
    Упростим уравнения
    log2(-x)=5
    log2(-x)=5•1
    log2(-x)=5• log22
    log2(-x)= log225
    опустим основы и приравняем логарифмические выражения:
    -x=25,
    -x=32,
    x=-32.
    Ответ: -32 – У.

     

    Пример 16.4 Решить уравнение lg(x2-x)=1-lg(5).

    А

    Б

    В

    Г

    Д

    ø

    -3; 2

    -2; 1

    -2; 3

    -1; 2

    Решение: ОДЗ: x2-x>0,
    x(x-1)>0
    Решим неравенство методом интервалов
    x(x-1)=0,
    x1=0,
    x2=1.

    x∈(-∞;1)∪(1;+∞).
    На этом множестве значений и ищем решение уравнения, сперва сведя к одной основе логарифмы

    по теореме Виета:
    x1+x2=1,
    x1•x2=-2.
    x1=-1,
    x2=2.
    Оба корня принадлежат ОДЗ.
    Ответ: -1; 2 – Д.

    ОДЗ неравенства могут быть сложнее, чем сами уравнения, тогда достаточно сами корни уравнения подставить в неравенство (или систему неравенств) и определить, принадлежат ли корни области допустимых значений логарифмческого уравнения.

    Пример 16.5 Сколько корней имеет уравнение lg(x4-10x2)=lg3x3?

    А

    Б

    В

    Г

    Д

    Ни одного

    один

    два

    три

    четыре

    Решение: В логарифме имеем биквадратное выражение, которое при условиях на ОДЗ требует вычислений.2)=lg3x3 имеет один корень.
    Ответ: один – Б.

     

    Пример 16.6 Решить уравнение log6(x-2)+log6(x-1)=1 и указать промежуток, которому принадлежит его корень.

    Решение: Выпишем систему неровностей для ОДЗ:

    По правилу, что сумма логарифмов чисел равна логарифму их произведения ln(a)+ln(b)=ln(a•b) и свойству log66=1, сведем логарифмы к общему основанию:

    При преобразованиях получили квадратное уравнение, корни которого находим по теореме Виета:
    x1+x2=3
    x1•x2=-4.
    x1=-1<2 (не принадлежит ОДЗ)
    x2=4.
    x=4 – единственный корень заданного уравнения, он принадлежит промежутку (3,9;4,1).
    Ответ: (3,9;4,1) – Б.

     

    Пример 16.9 Решить уравнение (log2x)2-2log2x-3=0 и указать сумму его корней.

    Решение: ОДЗ: x>0.
    логарифмическое уравнение
    (log2x)2-2log2x-3=0
    сведем к квадратному заменой log2x=t.
    t2-2•t-3=0
    По формулам Виета имеем:
    t1+t2=2 – сумма корней уравнения;
    t1•t2=3 – их произведение, тогда
    t1=-1 и t2=3 – корни квадратного уравнения.
    Возвращаемся к замене, и вычисляем простые логарифмические уравнения

    Оба корня принадлежат ОДЗ, по условию найдем их сумму:
    x1+x2=0,5+8=8,5.
    Ответ: 8,5 – Д.

    С простых примеров на раскрытие логарифмических уравнений Вы увидели, что достаточно знать несколько формул и базовые свойства логарифма и уже можно самостоятельно решать уравнения. Для простых условий это работает, но напоминаем, что курс ВНО подготовки содержит 40 примеров, причем ряд задач сочетают в себе не только логарифмы, но и корни, модули, показательные выражения. Вы научитесь сводить уравнения к квадратным, логарифмировать и еще много чего нового.

    Алгебра 10 класс — Личный сайт учителя Чендевой Ю.А.

    Логарифмы

     

    Логарифм числа b по основанию а – это показатель степени, в которую нужно возвести число а, чтобы получить число b.

    Формула

    Примеры

     loga b = c (при a > 0, a ≠ 1, b > 0).

    Это означает, что ac = b.

    log5 25 = 2

    Читается так: логарифмом числа 25 по основанию 5 является 2.
    Число 2 является показателем степени.
    Это означает, что 52 = 25.

     

    log464 = 3

    Логарифмом числа 64 по основанию 4 является 3.
    Это означает, что 43 = 64


     

     Говоря иначе, логарифмирование – это действие, обратное возведению в степень.


    Логарифм по основанию 10 называют десятичным логарифмом.

    Примеры десятичного логарифма:

    log10 100

    log10 5

    log10 0,01


    Десятичный логарифм обозначают символом lg. Таким образом:

    вместо log10 100 следует писать lg 100;

    вместо log10 5 пишем lg 5;

    вместо log10 0,01 пишем lg 0,01.

    Логарифмирование и потенцирование.

    Логарифмирование – это нахождение логарифмов заданных чисел или выражений.

                                                                b
    Пример: Найдем логарифм x = a2 · — .
                                                                c

    Решение.

    Последовательно воспользуемся сразу всеми тремя основными свойствами логарифмов, которые изложены выше (логарифм произведения, логарифм частного и логарифм степени):
                          b
    lg x = lg (a2 · —) = lg a2 + lg b – lg c = 2lg a + lg b – lg c.
                          c

    Потенцирование – это нахождение чисел или выражений по данному логарифму числа (выражения).

    Потенцировать – значит освобождаться от значков логарифмов в процессе решения логарифмического выражения.

    Например, надо решить уравнение log2 3x = log2 9.

    Убираем значки логарифмов – то есть потенцируем:

    3х = 9.

    В результате получаем простое уравнение, которое решается за несколько секунд:

    х = 9 : 3 = 3.

    Но потенцирование не сводится к простому и произвольному убиранию значков логарифмов. Для этого в обоих частях уравнения как минимум должно быть одинаковое значение основания (в нашем случае это число 2). Подробнее о потенцировании и его правилах – в следующем разделе.

    Урок по математике на тему «Решение логарифмических уравнений» (10 класс)

    Тема: Решение логарифмических уравнений (10 класс)

    Цель урока: повторить понятие и свойства логарифма; повторить способы решения логарифмических уравнений и закрепить их при выполнении упражнений.

    Задачи:

    — обучающие: повторить определение и основные свойства логарифмов, уметь применять их в вычислении логарифмов, в решении логарифмических уравнений;

    -развивающие: формировать умение решать логарифмические уравнения;

    -воспитательные: воспитывать настойчивость, самостоятельность; прививать интерес к предмету

    Тип урока: урок повторения и закрепления ранее изученного материала.

    Ход урока:

    1. Организационный момент.

    Проверка готовности обучающихся и кабинета к занятию. Объявление темы.

    1. Устная работа.

    Повторение понятия логарифма, повторение его основных свойств и свойств логарифмической функции:

    Разминка по теории:

    1. Дайте определение логарифма.

    2. От любого ли числа можно найти логарифм?

    3. Какое число может стоять в основании логарифма?

    4. Функция y=log0,8 x является возрастающей или убывающей? Почему?

    5. Какие значения может принимать логарифмическая функция?

    6. Какие логарифмы называют десятичными, натуральными?

    7. Назовите основные свойства логарифмов.

    8. Можно ли перейти от одного основания логарифма к другому? Как это сделать?

    1. Работа по карточкам:

      Карточка №1:

      Вычислить: а) log64 + log69 =

      б) log1/336 – log1/312 =

      Решить уравнение:

      log5х = 4 log53 – 1/3 log527

      Карточка №2:

      Вычислить: а) log211 – log244 =

      б) log1/64 + log1/69 =

      Решить уравнение:

      log7х = 2 log75 + 1/2 log736 – 1/3log7125.

    2. Фронтальный опрос класс

    Вычислить:

    log216

    lоg3 √3

    log71

    log(1/625)

    log211 — log 244

    1. log814 + log 832/7

    2. log35 ∙ log53

    3. log5 49

    4. lоg 85 — 1

    5. 25 –log 510

    Сравнить числа:

    1. log½ е и log½π;

    2. log√5/2 и log2√3/2.

    Выяснить знак выражения log0,83 · log62

    1. Повторение решения логарифмических уравнений.

    Класс делится на группы по 4 человека. Каждый из четырех членов группы выбирает один из способов решения, разбирается с ним (при затруднении можно обратиться к учителю), проводит взаимообучение с остальными тремя товарищами. Далее вместе прорешивают четыре примера, ответы проверяются учителем.

    1. Решение уравнений на основании определения логарифма.

     имеет решение .

    На основе определения логарифма решаются уравнения, в которых:

    • по данным основаниям и числу определяется логарифм,

    • по данному логарифму и основанию определяется число,

    • по данному числу и логарифму определяется основание.

    Ответ: 7

    Ответ: 8

    Ответ: 3

    2.Метод потенцирования.

    Под потенцированием понимается переход от равенства, содержащего логарифмы, к равенству, не содержащему их т.е. , то , при условии, что .

    Пример: Решите уравнение 

    3

     — неверно

    Ответ: решений нет.

    ОДЗ:

    3.Уравнения, решаемые с помощью применения основного логарифмического тождества.

    Пример: Решите уравнение 

     – не принадлежит ОДЗ

     – принадлежит ОДЗ

    Ответх=2

    ОДЗ:

      1. Домашнее задание: Решить задание на карточке.

      1. Подведение итогов, рефлексия

    Музыка может возвышать или умиротворять душу

    Живопись – радовать глаз,

    Поэзия – пробуждать чувства,

    Философия – удовлетворять потребности разума,

    Инженерное дело – совершенствовать материальную сторону жизни,

    А математика способна достичь всех этих целей.

    Конспект урока по Математике «Способы решения логарифмических уравнений» 10 класс

    Тема: «Способы решения логарифмических уравнений».

    Цель урока: повторить знания учащихся о логарифме числа, его свойствах; изучить способы решения логарифмических уравнений и закрепить их при выполнении упражнений.

    Задачи:

    — обучающие: повторить определение и основные свойства логарифмов, уметь применять их в вычислении логарифмов, в решении логарифмических уравнений;

    -развивающие: формировать умение решать логарифмические уравнения;

    -воспитательные: воспитывать настойчивость, самостоятельность; прививать интерес к предмету

    Тип урока: урок изучения нового материала.

    Необходимое техническое оборудование: компьютер, проектор, экран.

    Структура и ход урока:

    1. Организационный момент.

    Учитель.

    — Здравствуйте, садитесь! Сегодня тема нашего урока «Решение логарифмических уравнений», на котором мы познакомимся со способами их решения, используя определение и свойства логарифмов. (слайд № 1)

    1. Устная работа.

    Закрепление понятия логарифма, повторение его основных свойств и свойств логарифмической функции:

    1. Разминка по теории:

    1. Дайте определение логарифма. (слайд № 2)

    2. От любого ли числа можно найти логарифм?

    3. Какое число может стоять в основании логарифма?

    4. Функция y=log0,8x является возрастающей или убывающей?Почему?

    5. Какие значения может принимать логарифмическая функция?

    6. Какие логарифмы называют десятичными, натуральными?

    7. Назовите основные свойства логарифмов. (слайд № 3)

    8. Можно ли перейти от одного основания логарифма к другому? Как это сделать? (слайд № 4)

    2. Работа по карточка(3-4 ученика):

    Карточка №1: Вычислить: а) log64 + log69 =

    б) log1/336 – log1/312 =

    Решить уравнение: log5х = 4 log53 – 1/3 log527

    Карточка №2:

    Вычислить: а) log211 – log244 =

    б) log1/64 + log1/69 =

    Решить уравнение: log7х = 2 log75 + 1/2 log736 – 1/3 log7125.

    Фронтальный опрос класса (устные упражнения)

    Вычислить: (слайд № 5)

    1. log216

    2. lоg3√3

    3. log71

    4. log5 (1/625)

    5. log211 — log 244

    Сравнить числа: (слайд № 6)

    1. log½ е и log½π;

    2. log2 √5/2 и log2√3/2.

    Выяснить знак выражения log0,83 · log62/3. (слайд № 7)

    1. Проверка домашнего задания:

    На дом были задания следующие упражнения: №327(неч.), 331(неч.), 333(2) и 390(6). Проверить ответы к данным заданиям и ответить на вопросы учащихся.

    1. Изучение нового материала:

    Определение: Уравнение, содержащее переменную под знаком логарифма, называется логарифмическим.

    Простейшим примером логарифмического уравнения служит уравнение
    loga х =с (а > 0, а≠ 1)
    Способы решения логарифмических уравнений: (слайд № 8)

    1. Решение уравнений на основании определения логарифма. (слайд № 9)

    loga х = с (а > 0, а≠ 1) имеет решение х = ас.

    На основе определения логарифма решаются уравнения, в которых:

    • по данным основаниям и числу определяется логарифм,

    • по данному логарифму и основанию определяется число,

    • по данному числу и логарифму определяется основание.

    Примеры:

    log2 128= х, log16х = ¾, logх 27= 3,

    2х= 128, х =16 ¾ , х3 =27,

    2х = 27, х =2 3 , х3 = 33 ,

    х =7 . х = 8. х =3.

    С классом решить следующие уравнения:

    а) log7(3х-1)=2 (ответ: х=3 1/3)

    б) log2(7-8х)=2 (ответ: х=3/8).

    1. Метод потенцирования. (слайд № 10)

    Под потенцированием понимается переход от равенства, содержащего логарифмы, к равенству, не содержащему их т.е.

    loga f(х) = loga g(х), то f(х) = g(х), при условии, что f(х)>0, g(х)>0 , а > 0, а≠ 1.

    Пример:

    Решите уравнение =

    ОДЗ:

    3х-1>0; х>1/3

    6х+8>0.

    3х-1=6х+8

    -3х=9

    х=-3

    -3 >1/3 — неверно

    Ответ: решений нет.

    С классом решить следующее уравнение:

    lg(х2-2) = lg х (ответ: х=2)

    1. Уравнения, решаемые с помощью применения основного логарифмического тождества. (слайд №11)

    Пример:

    Решите уравнение =log2(6-х)

    ОДЗ:

    6-х>0;

    х>0;

    х≠1;

    log2х2>0;

    х2>0.

    Решение системы: (0;1)Ụ (1;6).

    = log2(6-х)

    х2 = 6-х

    х2+х-6=0

    х=-3 не принадлежит ОДЗ.

    х=2 принадлежит ОДЗ.

    Ответ: х=2

    С классом решить следующее уравнение:

    = (ответ: х=1)

    1. Метод приведения логарифмов к одному и тому же основанию. (слайд № 12)

    Пример:

    Решите уравнение log16х+ log4х+ log2х=7

    ОДЗ: х>0

    ¼ log2х+½ log2х+ log2х=7

    7/4 log2х=7

    log2х=4

    х=16 – принадлежит ОДЗ.

    Ответ: х=16.

    С классом решить следующее уравнение:

    + =3 (ответ: х=5/3)

    1. Уравнения, решаемые с помощью применения свойств логарифма. (слайд № 13)

    Пример:

    Решите уравнение log2 (х +1) — log2 (х -2 ) = 2.

    ОДЗ:

    х+1>0;

    х-2>0. х>1.

    Воспользуемся формулой преобразования разности логарифмов логарифм частного, получаем log2= 2, откуда следует = 4.

    Решив последнее уравнение, находим х = 3, 3>1 — верно

    Ответ: х = 3.

    С классом решить следующие уравнения:

    а)log5 (х +1) + log5 (х +5) = 1 (ответ: х=0).

    б)log9( 37-12х ) log7-2х 3 = 1,

    37-12х >0, х

    7-2х >0, х

    7-2х≠ 1; х≠ 3; х≠ 3;

    log9( 37-12х ) / log3 (7-2х ) = 1,

    ½ log3( 37-12х ) = log3 (7-2х ) ,

    log3( 37-12х ) = log3 (7-2х )2 ,

    37-12х= 49 -28х +4х2 ,

    2-16х +12 =0,

    х2-4х +3 =0, Д=19, х1=1, х2=3, 3 –посторонний корень .

    Ответ: х=1 корень уравнения.

    в) lg(х2-6х+9) — 2lg(х — 7) = lg9.

    2-6х+9) >0, х≠ 3,

    х-7 >0; х >7; х >7.

    lg ((х-3)/(х-7))2 = lg9

    ((х-3)/(х-7))2 = 9,

    (х-3)/(х-7) = 3, (х-3)/(х-7)= — 3 ,

    х- 3 = 3х -21 , х -3 =- 3х +21,

    х =9. х=6 — посторонний корень.

    Проверка показывает 9 корень уравнения.

    Ответ : 9

    1. Уравнения, решаемые введением новой переменной. (слайд № 14)

    Пример:

    Решите уравнение lg2х — 6lgх+5 = 0.

    ОДЗ: х>0.

    Пусть lgх = р, тогда р2-6р+5=0.

    р1=1, р2=5.

    Возвращаемся к замене:

    lgх = 1, lgх =5

    х=10, 10>0 – верно х=100000, 100000>0 – верно

    Ответ: 10, 100000

    С классом решить следующее уравнение:

    log62 х + log6 х +14 = (√16 – х2)22,

    16 – х2 ≥0 ; — 4≤ х ≤ 4;

    х >0 , х >0, О.Д.З. [ 0,4).

    log62 х + log6 х +14 = 16 – х22,

    log62 х + log6 х -2 = 0

    заменим log6 х = t

    t 2 + t -2 =0 ; D = 9 ; t1 =1 , t2 = -2.

    log6 х = 1 , х = 6 посторонний корень .

    log6 х = -2, х = 1/36 , проверка показывает 1/36 является корнем .

    Ответ : 1/36.

    1. Уравнения, решаемые с помощью разложения на множители. (слайд № 15)

    Пример:

    Решите уравнение log4(2х-1)∙ log4х=2 log4(2х-1)

    ОДЗ:

    2х-1>0;

    х >0. х>½.

    log4(2х-1)∙ log4х — 2 log4(2х-1)=0

    log4(2х-1)∙(log4х-2)=0

    log4(2х-1)=0 или log4х-2=0

    2х-1=1 log4х = 2

    х=1 х=16

    1;16 – принадлежат ОДЗ

    Ответ: 1;16

    С классом решить следующее уравнение:

    log3х ∙log3(3х-2)= log3(3х-2) (ответ: х=1)

    1. Метод логарифмирования обеих частей уравнения. (слайд № 16)

    Пример:

    Решите уравнения

    Прологарифмируем обе части уравнения по основанию 3.

    Получим log3 = log3 (3х)

    .

    получаем : log3 х2 log3 х = log3 (3х),

    2log3 х log3 х = log3 3+ log3 х,

    2 log32 х = log3 х +1,

    2 log32 х — log3 х -1=0,

    заменим log3 х = р , х >0

    2 р 2 + р -2 =0 ; D = 9 ; р1 =1 , р2 = -1/2

    log3 х = 1 , х=3,

    log3 х = -1/ 2 , х= 1/√3.

    Ответ: 3 ; 1/√3

    С классом решить следующее уравнение:

    log2 х — 1

    х = 64 (ответ: х=8 ; х=1/4)

    1. Функционально – графический метод. (слайд № 17)

    Пример:

    Решите уравнения: log3 х = 12-х.

    Так как функция у= log3 х возрастающая , а функция у =12-х убывающая на (0; + ∞ ) то заданное уравнение на этом интервале имеет один корень.

    Построим в одной системе координат графики двух функций: у= log3 х и у =12-х.

    При х=10 заданное уравнение обращается в верное числовое равенство 1=1. Ответ х=10.

    С классом решить следующее уравнение:

    1-√х =ln х (ответ : х=1).

    1. Подведение итогов, рефлексия (раздать кружочки, на которых ребята отмечают свое настроение рисунком). (слайд № 18,19)

    Определить метод решения уравнения:



    1. Домашнее задание: 340(1), 393(1), 395(1,3), 1357(1,2), 337(1), 338(1), 339(1)

    Литература

    1. Рязановский, А.Р. Математика. 5 – 11 кл.: Дополнительные материалы к уроку математики/ А.Р.Рязановский, Е.А.Зайцев. – 2-е изд., стереотип. – М.: Дрофа,2002

    2. Математика. Приложение к газете «Первое сентября». 1997. № 1, 10, 46, 48; 1998. № 8, 16, 17, 20, 21, 47.

    3. Скоркина, Н.М. Нестандартные формы внеклассной работы. Для средних и старших классов/ Н.М. Скоркина. – Волгоград: Учитель, 2004

    4. Зив, Б.Г., Гольдич,В.А. Дидактические материалы по алгебре и началам анализа для 10 класса./Б.Г.Зив, В.А.Гольдич. – 3-е изд., исправленное. – СПб.: «ЧеРо-на-Неве», 2004

    5. Алгебра и начала анализа: математика для техникумов/под ред. Г.Н.Яковлева.-М.: Наука, 1987

    6

    Предмет

    Алгебра и начала математического анализа

    Класс

    10

    Тема урока

    «Способы решения логарифмических уравнений», 2 часа

    Базовый учебник

    Ш.А. Алимов, Ю.М. Колягин и др. / М. Просвещение 2014

    1. log814 + log 832/7

    2. log35 ∙ log53

    3. 5 log5 49

    4. 8 lоg85 — 1

    5. 25 log 510

    Изучение логарифмов в старшей школе

    Понятие логарифма

    При решении показательных уравнений удается представить обе части уравнения в виде степеней с одинаковыми основаниями и рациональными показателями. Так, например, при решении уравнения мы заменяем степенью и из равенства степеней с одинаковыми основаниями делаем вывод о равенстве показателей: х = −5/6. Однако, чтобы решить, казалось бы, более простое уравнение 2х = 3, стандартных знаний оказывается недостаточно. Дело в том, что число 3 нельзя представить в виде степени с основанием 2 и рациональным показателем.

    Действительно, если бы равенство , где m и n — натуральные числа, было верным, то, возведя его в степень n, мы должны были бы получить верное равенство 2m = 3n. Но последнее равенство неверно, так как левая его часть является четным числом, а правая — нечетным. Значит, не может быть верным и равенство .

    С другой стороны, график непрерывной функции y = 2x пересекается с прямой y = 3, и, значит, уравнение 2x = 3 имеет корень. Таким образом, перед нами стоят два вопроса: «Как записать этот корень?» и «Как его вычислить?».

    Показатель степени, в которую нужно возвести число a (a > 0, a ≠ 1), чтобы получить число b, называется логарифмом b по основанию a и обозначается logab.

    Теперь мы можем записать корень уравнения 2х = 3:

    х = loga3

    Равенства ax = b и x = logab, в которых число a положительно и не равно единице, число b положительно, а число x может быть любым, выражают одно и то же соотношение между числами a, b и x. Подставив в первое равенство выражение x из второго, получим основное логарифмическое тождество.

    Понятие логарифма в методическом пособии

    Задание

    Решите уравнение: а) 2x = 64; б) ; в) ; г) 4x = 0; д) 7x = −12.

    После проверки ученикам предлагается ответить на вопрос, какое из заданий показалось им наиболее трудным. Вероятный ответ: 2 (в), так как в нем нужно было приводить дробь к степени числа 5. Затем школьникам предлагается высказать мнение о сравнительной с заданием 2 (в) трудности уравнения 2x = 3. На первый взгляд кажется, что это уравнение проще, однако представить 3 в виде степени числа 2 школьникам не удается.

    Дальше изучение нового материала проводится в соответствии с учебником. При этом в зависимости от уровня класса рассматривается или не рассматривается дополнительный материал о невозможности представления 3 в виде 2r , где r = m/n.

    После этого диалог с классом можно строить примерно так:

    — Как вы думаете, имеет ли уравнение 2x = 3 корень? Ответ обоснуйте. [Если построить график функции у = 2x и провести прямую у = 3, то они пересекутся в одной точке, значит, уравнение имеет один корень.]
    — Что можно сказать о корне уравнения ax = b, где а > 0 и а ≠ 1? При всех ли значениях b оно имеет корни?

    Затем вводится определение логарифма числа b по основанию а и записывается основное логарифмическое тождество . При этом выписывание равенства происходит синхронно с повторным чтением определения теперь уже в обратном, по сравнению с учебником, порядке. Теперь можно записать корень уравнения 2х = 3: х = loga3 и предложить школьникам серию самостоятельных работ.

    Логарифмическая функция

    Выразим x из равенства y = logax, получим x = ay. Последнее равенство задает функцию x = ay, график которой симметричен графику показательной функции y = ax относительно прямой y = x.

    Показательная функция x = ay является монотонной, и, значит, разные значения y соответствуют разным значениям x, но это говорит о том, что y = logax, в свою очередь, является функцией x.

    Показательная функция y = ax и логарифмическая функция y = logax являются взаимно обратными. Сравнивая их графики, можно отметить некоторые основные свойства логарифмической функции.

    Свойства функции y = logax, a > 0, a ≠ 11:

    1. Функция y = logax определена и непрерывна на множестве положительных чисел.
    2. Область значений функции y = logax — множество действительных чисел.
    3. При 0 < a < 1 функция y = logax является убывающей; при a > 1 функция y = logax является возрастающей.
    4. График функции y = logax проходит через точку (1; 0).
    5. Ось ординат — вертикальная асимптота графика функции y = loga.

    Математика: алгебра и начала математического анализа, геометрия. Алгебра и начала математического анализа. Углубленный уровень. 10 класс. Учебник

    Учебник входит в учебно-методический комплекс по математике для 10–11 классов, изучающих предмет на углубленном уровне. Теоретический материал в нем разделен на обязательный и дополнительный. Каждая глава завершается домашней контрольной работой, а каждый пункт главы — контрольными вопросами и заданиями. Учебник соответствует Федеральному государственному образовательному стандарту среднего (полного) общего образования, имеет гриф «Рекомендовано» и включен в Федеральный перечень учебников.

    Купить

    Решение логарифмических уравнений и неравенств на основе свойств логарифмической функции

    Освобождаясь от внешнего логарифма, имеющего основание 3, мы ссылаемся на возрастание соответствующей логарифмической функции, то есть на то, что большему значению логарифма соответствует большее значение выражения, стоящего под его знаком. Однако следует иметь в виду, что если функцию y = log3 log0,5(2x + 1) считать логарифмической, то ее аргумент не переменная x, а все выражение log0,5(2x + 1). Если же все-таки рассматривать x как аргумент функции y = log3 log0,5(2x + 1), то эта функция окажется убывающей, так как при увеличении значения x увеличивается значение выражения 2x + 1, уменьшается значение выражения log0,5(2x + 1) и, соответственно, уменьшается значение самой функции.

    Свойства логарифмов

    Связь двух форм записи соотношения между числами a, b и x (речь о ax = b и x = logab) позволяет получить свойства логарифмов, основываясь на известных свойствах степеней.

    Рассмотрим, например, произведение степеней с одинаковым основанием: axay. Пусть x = b и a y = c. Перейдем к логарифмической форме: x = logab и y = logac, тогда bc = a logab × a logac = a logab + logac. От показательной формы равенства bc = a logab + logac перейдем к логарифмической форме:

    loga(bc) = logab + logac

    Заметим, что в левой части формулы числа a и b могут быть отрицательными. Тогда формула будет выглядеть так:

    loga(bc) = loga|b| + loga|c|

    Аналогично можно получить еще два свойства для логарифмов частного и степени.

    • логарифм произведения loga (bc) = loga |b| + loga |c|
    • логарифм частного
    • логарифм степени logabp = p loga|b|

    Последнее свойство дает возможность вывести важную формулу, с помощью которой можно выразить логарифм с одним основанием через логарифм с другим основанием.

    Пусть logab = x. Перейдем к показательной форме ax = b. Прологарифмируем это равенство по основанию c, т.е. найдем логарифмы с основанием c обеих частей этого равенства: logcax = logcb. Применяя к левой части свойство логарифма степени, получим x logca = logcb или , откуда .

    Формула перехода от одного основания логарифма к другому

    Полезно запомнить частный случай формулы перехода, когда одно из оснований является степенью другого:

    Рассмотренные свойства и формула перехода «работают», конечно, только когда все входящие в них выражения имеют смысл.

    Что ещё почитать?

    Логарифмы на ЕГЭ

    Логарифмы встречаются на ЕГЭ: как во второй части (обычно, это задание 15), так и, реже, в первой части. Задания из аттестации — одно из средств мотивации детей на уроках. Зная, что упражнение на доске аналогично заданию ЕГЭ, ученик будет внимательнее следить за его решением.

    Разберем несколько таких заданий.

    Из первой части (определение логарифма на ЕГЭ профильного уровня)

    Решите уравнение log3(x+1)2 + log3|x+1| = 6 . Если корней несколько, укажите наименьший из них.

    Решение. Решаем квадратное относительно log3|x+1| уравнение. Его корни 2 и −3.

    log3|x+1| = 2, |x+1| = 9, x = −10 — это наименьший из корней.

    Ответ: −10.

    Из второй части (логарифмическое неравенство на ЕГЭ профильного уровня)

    Решите неравенство .

    Решение. ОДЗ: x > 0, x ≠ 1. Перейдем к логарифмам по основанию 10:

    ;

    ;

    Умножим числитель и знаменатель на 2, чтобы уйти от радикала:

    ;

    Нули числителя: 2/3, 3, с учетом положительности x, нуль заменяется на 1.

    Ответ:

    Алгебра в таблицах. 7-11 классы. Справочное пособие

    Пособие содержит таблицы по всем наиболее важным разделам школьного курса арифметики, алгебры, начал анализа. В таблицах кратко изложена теория по каждой теме, приведены основные формулы, графики и примеры решения типовых задач. В конце книги помещен предметный указатель. Пособие будет полезно учащимся 7-11 классов, абитуриентам, студентам, учителям и родителям.

    Купить
    Из второй части (логарифмическое уравнение с параметром на ЕГЭ профильного уровня)

    Найдите все значения a, для которых при любом положительном значении b уравнение имеет хотя бы одно решение, меньше 1/3.

    Решение. Найдем ОДЗ:

    Стандартно приводим логарифмы к одному основанию

    ,

    .

    Получили квадратное уравнение относительно .

    Оно должно иметь корень при

    Обозначим, что и рассмотрим квадратичную функцию y = t— bt — 2a.

    Ветви ее графика направлены вверх, а вершина, поскольку b > 0, расположена в левой координатной полуплоскости. Первая ветвь параболы пересекает ось абсцисс правее t = 0, значит при t = 0 y < 0. Получаем −2a < 0 a > 0.

    Ответ: a > 0.

    Учебник «Алгебра и начала математического анализа. Углубленный уровень. 10 класс» схож по структуре с учебником базового уровня, однако предполагает больше часов на изучение сложных задач. Эти и другие издания линейки вы можете апробировать прямо сейчас, воспользовавшись акцией «5 учебников бесплатно». Методическое пособие представлено в свободном доступе. Приглашаем познакомиться с другими вебинарами экспертов и порекомендовать нам интересующую вас тему для последующих трансляций.


    #ADVERTISING_INSERT#


    Решение логарифмических функций — объяснения и примеры

    В этой статье мы узнаем, как вычислять и решать логарифмические функции с неизвестными переменными.

    Логарифмы и экспоненты — две тесно связанные между собой темы в математике. Поэтому полезно взять краткий обзор показателей.

    Показатель степени — это форма записи многократного умножения числа на само себя. Показательная функция имеет вид f (x) = b y , где b> 0

    Например, , 32 = 2 × 2 × 2 × 2 × 2 = 2 2 .

    Экспоненциальная функция 2 2 читается как « два в степени пяти, » или « два в возведении в пятерку » или « два в пятой степени.

    С другой стороны, логарифмическая функция определяется как функция, обратная возведению в степень. Снова рассмотрим экспоненциальную функцию f (x) = b y , где b> 0

    y = журнал b x

    Тогда логарифмическая функция имеет вид;

    f (x) = log b x = y, где b — основание, y — показатель степени, а x — аргумент.

    Функция f (x) = log b x читается как «log base b of x». Логарифмы полезны в математике, потому что они позволяют нам выполнять вычисления с очень большими числами.

    Как решать логарифмические функции?

    Для решения логарифмических функций важно использовать экспоненциальные функции в данном выражении.Натуральное бревно или ln — это обратное значение e . Это означает, что один может отменить другой, то есть

    ln (e x ) = x

    e ln x = x

    Чтобы решить уравнение с логарифмом (ами), важно знать их свойства.

    Свойства логарифмических функций

    Свойства логарифмических функций — это просто правила для упрощения логарифмов, когда входные данные имеют форму деления, умножения или показателя степени логарифмических значений.

    Некоторые из объектов недвижимости перечислены ниже.

    Правило произведения логарифмов гласит, что логарифм произведения двух чисел, имеющих общее основание, равен сумме отдельных логарифмов.

    ⟹ log a (p q) = log a p + log a q.

    Правило частного логарифмов гласит, что логарифм отношения двух чисел с одинаковыми основаниями равен разности каждого логарифма.

    ⟹ log a (p / q) = log a p — log a q

    Правило степени логарифма утверждает, что логарифм числа с рациональной экспонентой равен произведению показателя степени и его логарифма.

    ⟹ журнал a (p q ) = q журнал a p

    ⟹ журнал a p = журнал x p ⋅ журнал a x

    ⟹ log q p = log x p / log x q

    ⟹ журнал p 1 = 0.

    Другие свойства логарифмических функций включают:

    • Основания экспоненциальной функции и ее эквивалентной логарифмической функции равны.
    • Логарифмы положительного числа по основанию того же числа равны 1.

    журнал a a = 1

    • Логарифмы от 1 до любого основания равны 0.

    log a 1 = 0

    • Журнал a 0 не определено
    • Логарифмы отрицательных чисел не определены.
    • Основание логарифмов никогда не может быть отрицательным или 1.
    • Логарифмическая функция с основанием 10 называется десятичным логарифмом. Всегда принимайте основание 10 при решении с помощью логарифмических функций без маленького индекса для основания.

    Сравнение экспоненциальной функции и логарифмической функции

    Каждый раз, когда вы видите логарифмы в уравнении, вы всегда думаете о том, как отменить логарифм, чтобы решить уравнение. Для этого вы используете экспоненциальную функцию . Обе эти функции взаимозаменяемы.

    В следующей таблице описан способ записи и перестановки экспоненциальных функций и логарифмических функций . В третьем столбце рассказывается о том, как читать обе логарифмические функции.

    Экспоненциальная функция Логарифмическая функция Читать как
    8 2 = 64 журнал 8 64 = 2 журнал, основание 8 из 64
    10 3 = 1000 журнал 1000 = 3 лог по основанию 10 из 1000
    10 0 = 1 журнал 1 = 0 лог по основанию 10 из 1
    25 2 = 625 журнал 25 625 = 2 бревно, база 25 из 625
    12 2 = 144 журнал 12 144 = 2 бревно, основание 12 из 144

    Давайте воспользуемся этими свойствами для решения пары задач, связанных с логарифмическими функциями.

    Пример 1

    Записываем экспоненциальную функцию 7 2 = 49 в ее эквивалентную логарифмическую функцию.

    Раствор

    Дано 7 2 = 64.

    Здесь основание = 7, показатель степени = 2 и аргумент = 49. Следовательно, 7 2 = 64 в логарифмической функции;

    ⟹ лог 7 49 = 2

    Пример 2

    Запишите логарифмический эквивалент 5 3 = 125.

    Раствор

    База = 5;

    показатель степени = 3;

    и аргумент = 125

    5 3 = 125 ⟹ лог 5 125 = 3

    Пример 3

    Решить относительно x в журнале 3 x = 2

    Раствор

    журнал 3 x = 2
    3 2 = x
    ⟹ x = 9

    Пример 4

    Если 2 log x = 4 log 3, найдите значение «x».

    Раствор

    2 журнала x = 4 журнала 3

    Разделите каждую сторону на 2.

    журнал x = (4 журнал 3) / 2

    журнал x = 2 журнал 3

    журнал x = журнал 3 2

    журнал x = журнал 9

    х = 9

    Пример 5

    Найдите логарифм 1024 по основанию 2.

    Раствор

    1024 = 2 10

    журнал 2 1024 = 10

    Пример 6

    Найдите значение x в журнале 2 ( x ) = 4

    Раствор

    Перепишите логарифмическую функцию log 2 ( x ) = 4 в экспоненциальную форму.

    2 4 = x

    16 = x

    Пример 7

    Найдите x в следующей логарифмической функции log 2 (x — 1) = 5.

    Решение
    Записываем логарифм в экспоненциальной форме как;

    журнал 2 (x — 1) = 5 ⟹ x — 1 = 2 5

    Теперь решите относительно x в алгебраическом уравнении.
    ⟹ х — 1 = 32
    х = 33

    Пример 8

    Найдите значение x в логарифме x 900 = 2.

    Раствор

    Запишите логарифм в экспоненциальной форме как;

    х 2 = 900

    Найдите квадратный корень из обеих частей уравнения, чтобы получить;

    x = -30 и 30

    Но поскольку основание логарифмов никогда не может быть отрицательным или 1, то правильный ответ — 30.

    Пример 9

    Решить относительно заданного x, log x = log 2 + log 5

    Раствор

    Используя правило продукта Log b (m n) = log b m + log b n получаем;

    ⟹ журнал 2 + журнал 5 = журнал (2 * 5) = журнал (10).

    Следовательно, x = 10.

    Пример 10

    Журнал решения x (4x — 3) = 2

    Раствор

    Записываем логарифм в экспоненциальной форме, чтобы получить;

    x 2 = 4x — 3

    Теперь решите квадратное уравнение.
    x 2 = 4x — 3
    x 2 — 4x + 3 = 0
    (x -1) (x — 3) = 0

    x = 1 или 3

    Поскольку основание логарифма никогда не может быть 1, единственное решение — 3.

    Практические вопросы

    1. Выразите следующие логарифмы в экспоненциальной форме.

    а. 1ог 2 6

    г. журнал 9 3

    г. журнал 4 1

    г. журнал 6 6

    e. журнал 8 25

    ф. журнал 3 (-9)

    2. Найдите x в каждом из следующих логарифмов

    а. журнал 3 (x + 1) = 2

    г. журнал 5 (3x — 8) = 2

    г.журнал (x + 2) + журнал (x — 1) = 1

    г. журнал x 4 — журнал 3 = журнал (3x 2 )

    3. Найдите значение y в каждом из следующих логарифмов.

    а. журнал 2 8 = y

    г. журнал 5 1 = y

    г. журнал 4 1/8 = y

    г. журнал y = 100000

    4. Решите относительно xif log x (9/25) = 2.

    5. Решить журнал 2 3 — журнал 2 24

    6. Найдите значение x в следующем логарифме log 5 (125x) = 4

    7.Учитывая, что Log 10 2 = 0,30103, Log 10 3 = 0,47712 и Log 10 7 = 0,84510, решите следующие логарифмы:

    а. журнал 6

    г. журнал 21

    г. журнал 14

    Предыдущий урок | Главная страница | Следующий урок

    Вопросы по логарифму — Вопросы, основанные на логарифме и решениях

    В доске CBSE главы логарифма включены в программу 9, 10 и 11 класса.Учащимся 9 класса впервые будут представлены вопросы и ответы по логарифму. Следовательно, тщательная практика решения логарифмических задач и ответов требует времени.

    Однако, прежде чем перейти к главе, посвященной логарифму, учащиеся должны быть абсолютно ясны в основных понятиях. Только тогда решение сложных логарифмических вопросов станет значительно проще.

    Вопросы, основанные на логарифме

    Вот несколько вопросов, связанных с логарифмом, которые могут дать учащимся некоторое представление.

    Вопрос 1: Найдите неверное утверждение снизу —

    (a) log (1 + 2 + 3) = log 1 + log 2 + log 3

    (b) log (2 + 3) = log (2 x 3)

    (c) log10 10 = 1

    (d) log10 1 = 0

    Решение: ответ: вариант (b) log (2 + 3) = log (2 x 3).

    Вопрос 2: Каково значение log5512, когда log 2 = 0,3010 и log 3 = 0,4771?

    (а) 3,912

    (б) 3,876

    (в) 2,967

    (г) 2,870

    Решение: ответ — вариант (б) 3.876.

    Вопрос 3: Найдите значение log 9, когда log 27 составляет 1,431.

    (а) 0,954

    (б) 0,945

    (в) 0,958

    (г) 0,934

    Решение: ответ — вариант (а) 0,954.

    Вопрос 4. Каково значение log2 10, когда log10 2 = 0,3010?

    (a) 1000/301

    (b) 699/301

    (c) 0,6990

    (d) 0,3010

    Решение: ответ — вариант (a) 1000/301.

    Вопрос 5: Каково значение log10 80, когда log10 2 = 0.3010?

    (a) 3,9030

    (b) 1,9030

    (c) 1,6020

    (d) Ни один из вышеперечисленных вариантов

    Решение: Ответ — вариант (b) 1.9030.

    Вопрос 6: Сколько цифр в 264, если log 2 = 0,30103?

    (a) 21

    (b) 20

    (c) 18

    (d) 19

    Решение: ответ — вариант (b) 20.

    Вопрос 7: Что из следующего верно, если топор = по?

    (a) log a / log b = x / y

    (b) log a / b = x / y

    (c) log a / log b = y / x

    (d) Ничего из вышеперечисленного option

    Решение: Ответ: вариант (c) log a / log b = y / x.

    Вопрос 8: Каково значение log2 16?

    (a) 8

    (b) 4

    (c) 1/8

    (d) 16

    Решение: Ответ: (b) 4.

    Вопрос 9: Найдите значение y, если logx y = 100 и log2 x = 10.

    (a) 21000

    (b) 210

    (c) 2100

    (d) 210000

    Решение: ответ — вариант (a) 21000.

    Вопрос 10: Найдите значение log10 (0,0001).

    (a) — 1/4

    (b) 1/4

    (c) 4

    (d) — 4

    Решение: ответ — вариант (d) — 4.

    Вопрос 11: Каково значение x, когда log2 [log3 (log2x)] = 1?

    (a) 512

    (b) 12

    (c) 0

    (d) 128

    Решение: ответ — вариант (a) 512.

    Вопрос студентов по логарифмическим вопросам можно пояснить в Веданту. онлайн-классы. У вас также есть возможность скачать материалы в формате PDF с официального сайта. Загрузите приложение сегодня!

    Log-Base-10 — Алгебра II

    Если вы считаете, что контент, доступный через Веб-сайт (как определено в наших Условиях обслуживания), нарушает или другие ваши авторские права, сообщите нам, отправив письменное уведомление («Уведомление о нарушении»), содержащее в информацию, описанную ниже, назначенному ниже агенту.Если репетиторы университета предпримут действия в ответ на ан Уведомление о нарушении, оно предпримет добросовестную попытку связаться со стороной, которая предоставила такой контент средствами самого последнего адреса электронной почты, если таковой имеется, предоставленного такой стороной Varsity Tutors.

    Ваше Уведомление о нарушении прав может быть отправлено стороне, предоставившей доступ к контенту, или третьим лицам, таким как в виде ChillingEffects.org.

    Обратите внимание, что вы будете нести ответственность за ущерб (включая расходы и гонорары адвокатам), если вы существенно искажать информацию о том, что продукт или действие нарушает ваши авторские права.Таким образом, если вы не уверены, что контент находится на Веб-сайте или по ссылке с него нарушает ваши авторские права, вам следует сначала обратиться к юристу.

    Чтобы отправить уведомление, выполните следующие действия:

    Вы должны включить следующее:

    Физическая или электронная подпись правообладателя или лица, уполномоченного действовать от их имени; Идентификация авторских прав, которые, как утверждается, были нарушены; Описание характера и точного местонахождения контента, который, по вашему мнению, нарушает ваши авторские права, в \ достаточно подробностей, чтобы позволить репетиторам университетских школ найти и точно идентифицировать этот контент; например нам требуется а ссылка на конкретный вопрос (а не только на название вопроса), который содержит содержание и описание к какой конкретной части вопроса — изображению, ссылке, тексту и т. д. — относится ваша жалоба; Ваше имя, адрес, номер телефона и адрес электронной почты; а также Ваше заявление: (а) вы добросовестно считаете, что использование контента, который, по вашему мнению, нарушает ваши авторские права не разрешены законом, владельцем авторских прав или его агентом; (б) что все информация, содержащаяся в вашем Уведомлении о нарушении, является точной, и (c) под страхом наказания за лжесвидетельство, что вы либо владелец авторских прав, либо лицо, уполномоченное действовать от их имени.

    Отправьте жалобу нашему уполномоченному агенту по адресу:

    Чарльз Кон Varsity Tutors LLC
    101 S. Hanley Rd, Suite 300
    St. Louis, MO 63105

    Или заполните форму ниже:

    Обогащение: подробнее о логарифмах | Функции

    \ (\ log_ {3} {a} — \ log {\ text {1,2}} = 0 \)

    \ begin {align *} \ log_ {3} {a} — \ log {\ text {1,2}} & = 0 \\ \ log_ {3} {a} & = \ log {\ text {1,2}} \\ \ text {Перейти к экспоненциальной форме:} & \\ 3 ^ {\ log {\ text {1,2}}} & = a \\ \ поэтому a & = \ text {1,09} \ end {выровнять *}

    Альтернативный (более длинный) метод:

    \ begin {align *} \ log_ {3} {a} — \ log {\ text {1,2}} & = 0 \\ \ log_ {3} {a} & = \ log {\ text {1,2}} \\ \ frac {\ log {a}} {\ log {3}} & = \ log {\ text {1,2}} \\ \ log {a} & = \ log {3} \ times \ log {\ text {1,2}} \\ \ log {a} & = \ text {0,037} \ ldots \\ \ поэтому a & = \ text {1,09} \ end {выровнять *}

    \ (\ log_ {2} {(a — 1)} = \ text {1,5} \)

    \ begin {align *} \ log_ {2} {(a — 1)} & = \ text {1,5} \\ \ text {Перейти к экспоненциальной форме:} & \\ 2 ^ {\ text {1,5}} & = a — 1 \\ 2 ^ {\ text {1,5}} + 1 & = a \\ \ поэтому a & = \ text {3,83} \ end {выровнять *}

    Альтернативный (более длинный) метод:

    \ begin {align *} \ log_ {2} {(a — 1)} & = \ text {1,5} \\ \ frac {\ log {(a — 1)}} {\ log {2}} & = \ text {1,5} \\ \ log {(a — 1)} & = \ log {2} \ times \ text {1,5} \\ \ поэтому a — 1 & = \ text {2,83} \ ldots \\ \ поэтому a & = \ text {3,83} \ end {выровнять *}

    \ (\ log_ {2} {a} — 1 = \ text {1,5} \)

    \ begin {align *} \ log_ {2} {a — 1} & = \ text {1,5} \\ \ log_ {2} {a} & = \ text {2,5} \\ \ text {Перейти к экспоненциальной форме:} & \\ 2 ^ {\ text {2,5}} & = а \\ \ поэтому a & = \ text {5,66} \ end {выровнять *}

    Альтернативный (более длинный) метод:

    \ begin {align *} \ log_ {2} {a} — 1 & = \ text {1,5} \\ \ frac {\ log {a}} {\ log {2}} & = \ text {2,5} \\ \ log {a} & = \ log {2} \ times \ text {2,5} \\ \ поэтому a & = \ text {5,66} \ end {выровнять *}

    \ (3 ^ {a} = \ text {2,2} \)

    \ begin {align *} 3 ^ {a} & = \ text {2,2} \\ \ поэтому a & = \ log_ {3} {\ text {2,2}} \\ & = \ frac {\ log {\ text {2,2}}} {\ log {3}} \\ \ поэтому a & = \ text {0,72} \ end {выровнять *}

    \ (2 ^ {(a + 1)} = \ text {0,7} \)

    \ begin {align *} 2 ^ {(a + 1)} & = \ text {0,7} \\ \ поэтому a + 1 & = \ log_ {2} {\ text {0,7}} \\ \ поэтому a & = \ frac {\ log {\ text {0,7}}} {\ log {2}} — 1 \\ & = — \ text {1,51} \ end {выровнять *}

    \ ((\ text {1,03}) ^ {\ frac {a} {2}} = \ text {2,65} \)

    \ begin {align *} (\ text {1,03}) ^ {\ frac {a} {2}} & = \ text {2,65} \\ \ поэтому \ frac {a} {2} & = \ log _ {\ text {1,03}} {\ text {2,65}} \\ \ поэтому a & = 2 \ times \ frac {\ log {\ text {2,65}}} {\ log {\ text {1,03}}} \\ & = \ text {65,94} \ end {выровнять *}

    \ ((\ text {9}) ^ {(1 — 2a)} = \ text {101} \)

    \ begin {align *} (\ text {9}) ^ {(1 — 2a)} & = \ text {101} \\ \ поэтому 1 — 2a & = \ log _ {\ text {9}} {\ text {101}} \\ \ поэтому 1 — \ frac {\ log {\ text {101}}} {\ log {\ text {9}}} & = 2a \\ — \ text {1,10} \ ldots & = 2a \\ \ поэтому — \ text {0,55} & = a \ end {выровнять *}

    Свойства логарифмов | Колледж алгебры

    Результаты обучения

    • Перепишите логарифмическое выражение, используя правило степени, правило произведения или правило частного. {1} = 5 [/ latex].{{\ mathrm {log}} _ {e} 7} = 7 [/ латекс].

      Наконец, у нас есть свойство однозначно .

      [латекс] {\ mathrm {log}} _ {b} M = {\ mathrm {log}} _ {b} N \ text {тогда и только тогда, когда} \ text {} M = N [/ latex]

      Мы можем использовать однозначное свойство для решения уравнения [латекс] {\ mathrm {log}} _ {3} \ left (3x \ right) = {\ mathrm {log}} _ {3} \ left (2x + 5 \ right) [/ latex] для x . Поскольку основы одинаковы, мы можем применить свойство «один к одному», установив равные аргументы и решив для x :

      [латекс] \ begin {array} {l} 3x = 2x + 5 \ hfill & \ text {Установите равные аргументы} \ text {.} \ hfill \\ x = 5 \ hfill & \ text {Вычесть 2} x \ text {.} \ hfill \ end {array} [/ latex]

      А как насчет уравнения [латекс] {\ mathrm {log}} _ {3} \ left (3x \ right) + {\ mathrm {log}} _ {3} \ left (2x + 5 \ right) = 2 [/латекс]? Свойство «один к одному» в данном случае нам не помогает. {a + b} [/ латекс].У нас есть аналогичное свойство для логарифмов, которое называется правилом произведения для логарифмов , которое гласит, что логарифм произведения равен сумме логарифмов. Поскольку бревна — это экспоненты, и мы умножаем их как основания, мы можем складывать экспоненты. Мы будем использовать обратное свойство для вывода правила произведения ниже.

      Для любого действительного числа x и положительных вещественных чисел M , N и b , где [latex] b \ ne 1 [/ latex], мы покажем

      [латекс] {\ mathrm {log}} _ {b} \ left (MN \ right) \ text {=} {\ mathrm {log}} _ {b} \ left (M \ right) + {\ mathrm { log}} _ {b} \ left (N \ right) [/ latex].{m + n} \ right) \ hfill & \ text {Применить правило продукта для показателей степени}. \ hfill \\ \ hfill & = m + n \ hfill & \ text {Применить обратное свойство журналов}. \ hfill \ \ \ hfill & = {\ mathrm {log}} _ {b} \ left (M \ right) + {\ mathrm {log}} _ {b} \ left (N \ right) \ hfill & \ text {Заменить на } m \ text {и} n. \ hfill \ end {array} [/ latex]

      Обратите внимание, что повторное применение правила произведения для логарифмов позволяет упростить логарифм произведения любого количества факторов. Например, рассмотрим [латекс] \ mathrm {log} _ {b} (wxyz) [/ latex].Используя правило продукта для логарифмов, мы можем переписать этот логарифм продукта как сумму логарифмов его множителей:

      [латекс] \ mathrm {log} _ {b} (wxyz) = \ mathrm {log} _ {b} w + \ mathrm {log} _ {b} x + \ mathrm {log} _ {b} y + \ mathrm { log} _ {b} z [/ латекс]

      Общее примечание: правило произведения логарифмов

      Правило произведения для логарифмов можно использовать для упрощения логарифма произведения, переписав его как сумму отдельных логарифмов.

      [латекс] {\ mathrm {log}} _ {b} \ left (MN \ right) = {\ mathrm {log}} _ {b} \ left (M \ right) + {\ mathrm {log}} _ {b} \ left (N \ right) \ text {for} b> 0 [/ latex]

      Пример: использование правила произведения для логарифмов

      Разверните [латекс] {\ mathrm {log}} _ {3} \ left (30x \ left (3x + 4 \ right) \ right) [/ latex].

      Показать решение

      Начнем с написания уравнения равных сумм логарифмов каждого множителя.

      [латекс] {\ mathrm {log}} _ {3} \ left (30x \ left (3x + 4 \ right) \ right) = {\ mathrm {log}} _ {3} \ left (30x \ right) + {\ mathrm {log}} _ {3} \ left (3x + 4 \ right) = {\ mathrm {log}} _ {3} \ left (30 \ right) + {\ mathrm {log}} _ { 3} \ left (x \ right) + {\ mathrm {log}} _ {3} \ left (3x + 4 \ right) [/ latex]

      Последнее расширение выглядит так. Обратите внимание, как коэффициент [латекс] 30x [/ latex] можно разложить до суммы двух логарифмов:

      [латекс] {\ mathrm {log}} _ {3} \ left (30 \ right) + {\ mathrm {log}} _ {3} \ left (x \ right) + {\ mathrm {log}} _ {3} \ left (3x + 4 \ right) [/ латекс]

      Попробуйте

      Разверните [латекс] {\ mathrm {log}} _ {b} \ left (8k \ right) [/ latex].{a-b} [/ латекс]. Правило частного для логарифмов говорит, что логарифм частного равен разности логарифмов. Как и в случае с правилом произведения, мы можем использовать обратное свойство для получения правила частного.

      Для любого действительного числа x и положительных вещественных чисел M , N и b , где [latex] b \ ne 1 [/ latex], мы покажем

      [латекс] {\ mathrm {log}} _ {b} \ left (\ frac {M} {N} \ right) \ text {=} {\ mathrm {log}} _ {b} \ left (M \ справа) — {\ mathrm {log}} _ {b} \ left (N \ right) [/ latex].{2} + 6x} {3x + 9} \ right) & = \ mathrm {log} \ left (\ frac {2x \ left (x + 3 \ right)} {3 \ left (x + 3 \ right)} \ right) \ hfill & \ text {Разложите числитель и знаменатель на множители}. \ hfill \\ & \ text {} = \ mathrm {log} \ left (\ frac {2x} {3} \ right) \ hfill & \ text {Отменить общие множители}. \ Hfill \ end {array} [/ latex]

      Затем мы применяем правило частного, вычитая логарифм знаменателя из логарифма числителя. Затем применяем правило продукта.

      [латекс] \ begin {array} {lll} \ mathrm {log} \ left (\ frac {2x} {3} \ right) & = \ mathrm {log} \ left (2x \ right) — \ mathrm {log } \ left (3 \ right) \ hfill \\ \ text {} & = \ mathrm {log} \ left (2 \ right) + \ mathrm {log} \ left (x \ right) — \ mathrm {log} \ слева (3 \ справа) \ hfill \ end {array} [/ latex]

      Общее примечание: Правило частного для логарифмов

      Правило частного для логарифмов можно использовать для упрощения логарифма или частного, переписав его как разность отдельных логарифмов.

      [латекс] {\ mathrm {log}} _ {b} \ left (\ frac {M} {N} \ right) = {\ mathrm {log}} _ {b} M — {\ mathrm {log}} _ {b} N [/ латекс]

      Как сделать: учитывая логарифм частного, используйте правило частного логарифмов, чтобы записать эквивалентную разницу логарифмов

      1. Выразите аргумент в наименьших числах, разложив числитель и знаменатель на множители и исключив общие термины.
      2. Напишите эквивалентное выражение, вычтя логарифм знаменателя из логарифма числителя.
      3. Убедитесь, что каждый член полностью раскрыт. Если нет, примените правило произведения для логарифмов, чтобы полностью раскрыть логарифм.

      Пример: использование правила частного для логарифмов

      Разверните [латекс] {\ mathrm {log}} _ {2} \ left (\ frac {15x \ left (x — 1 \ right)} {\ left (3x + 4 \ right) \ left (2-x \ right)} \ right) [/ латекс].

      Показать решение

      Сначала отметим, что частное факторизуется в наименьших значениях, поэтому мы применяем правило частного.

      [латекс] {\ mathrm {log}} _ {2} \ left (\ frac {15x \ left (x — 1 \ right)} {\ left (3x + 4 \ right) \ left (2-x \ right) )} \ right) = {\ mathrm {log}} _ {2} \ left (15x \ left (x — 1 \ right) \ right) — {\ mathrm {log}} _ {2} \ left (\ left (3x + 4 \ right) \ left (2-x \ right) \ right) [/ латекс]

      Обратите внимание, что полученные члены являются логарифмами произведений.Для полного расширения мы применяем правило продукта.

      [латекс] \ begin {array} {l} {\ mathrm {log}} _ {2} \ left (15x \ left (x — 1 \ right) \ right) — {\ mathrm {log}} _ {2 } \ left (\ left (3x + 4 \ right) \ left (2-x \ right) \ right) \\\ text {} = \ left [{\ mathrm {log}} _ {2} \ left (15 \ right) + {\ mathrm {log}} _ {2} \ left (x \ right) + {\ mathrm {log}} _ {2} \ left (x — 1 \ right) \ right] — \ left [ {\ mathrm {log}} _ {2} \ left (3x + 4 \ right) + {\ mathrm {log}} _ {2} \ left (2-x \ right) \ right] \ hfill \\ \ text {} = {\ mathrm {log}} _ {2} \ left (15 \ right) + {\ mathrm {log}} _ {2} \ left (x \ right) + {\ mathrm {log}} _ { 2} \ left (x — 1 \ right) — {\ mathrm {log}} _ {2} \ left (3x + 4 \ right) — {\ mathrm {log}} _ {2} \ left (2-x \ right) \ hfill \ end {array} [/ latex]

      Анализ решения

      В этом и последующих примерах есть исключения. {2} + 21x} {7x \ left (x — 1 \ right) \ left (x — 2 \ right)} \ right) [/ латекс].{2} \ right) \ hfill & = {\ mathrm {log}} _ {b} \ left (x \ cdot x \ right) \ hfill \\ \ hfill & = {\ mathrm {log}} _ {b} x + {\ mathrm {log}} _ {b} x \ hfill \\ \ hfill & = 2 {\ mathrm {log}} _ {b} x \ hfill \ end {array} [/ latex]

      Обратите внимание, что мы использовали правило произведения для логарифмов , чтобы найти решение для приведенного выше примера. Таким образом, мы вывели правило мощности для логарифмов , которое гласит, что логарифм степени равен экспоненте, умноженной на логарифм основания. Имейте в виду, что хотя вход логарифма не может быть записан как степень, мы можем изменить его на степень.{2}} \ right) [/ латекс].

      Показать решение

      [латекс] -2 \ mathrm {ln} \ left (x \ right) [/ latex]

      Внесите свой вклад!

      У вас была идея улучшить этот контент? Нам очень понравится ваш вклад.

      Улучшить эту страницуПодробнее

      Основы — примеры проблем с решениями

      Логарифм

      Логарифм положительного числа x по основанию a (a — положительное число, не равное 1) — это степень y, до которой необходимо возвести основание a, чтобы получить число x.


      log a x = y, потому что a y = x a> 0 и a ≠ 1

      Свойства логарифмов:


      1. Запишем в виде логарифмов:


      10 2 = 100 лог 10 100 = 2
      4 5 = 1024 журнал 4 1024 = 5
      13 0 = 1 лог 13 1 = 0
      10 -3 = 0,001 лог 10 0,001 = -3
      64 0,5 = 8 лог 64 8 = 0,5
      5 -2 = 0,04 лог 5 0,04 = -2

      2.Решите и объясните причину:


      журнал 10 1000 журнал 10 1000 = 3, потому что 10 3 = 1000
      журнал 3 81 журнал 3 81 = 4, потому что 3 4 = 81
      log 2 0,5 log 2 0,5 = -1, потому что 2 -1 = 0,5
      журнал 17 1 журнал 17 1 = 0, потому что 17 0 = 1
      журнал 11 11 журнал 11 11 = 1, потому что 11 1 = 11
      log 5 0,2 log 5 0,2 = -1, потому что5 -1 = 0,2
      журнал 15 0 журнал 15 0 =
      журнал 5 (-25) журнал 5 (-25) = n0
      log 0,4 0,4 ​​log 0,4 0,4 ​​= 1, потому что 0,4 1 = 0,4
      журнал 1 49 журнал 1 49 = 0 не определено

      3.Определите x:


      журнал 2 x = 3 x = 2 3 = 8
      журнал 10 x = -4 x = 10 -4 = 0,0001
      журнал 16 x = 0,5 x = 16 0,5 = 4
      журнал 20 x = 1 x = 20 1 = 20
      журнал 25 x = -0,5 x = 25 -0,5 = 0,2
      журнал 0,2487 x = 0 x = 0,2487 0 = 1

      4. Определите:


      журнал a 25 = 2 a = 5
      журнал a 81 = 4 a = 3
      журнал a 100000 = 5 a = 10
      журнал a 512 = 3 a = 8
      журнал a 0,01 = -2 a = 10
      журнал a 5 = 0,5 a = 25
      журнал a 36 = 2 a = 6
      журнал a 64 = 1 a = 64


      5.Логарифмируем следующие выражения (с основанием a)


      6. Определите x:


      7. Пронумеруйте выражение:


      Решение:

      Причина:



      8. Логарифмируем выражение (с основанием a):

      Решение:



      9. Пронумеруйте выражение:


      Решение:

      Причина:



      10.Перечислите выражение:


      Решение:


      Причина:



      11. Используйте десятичный логарифм, чтобы решить уравнение:


      Решение:



      12. Используйте десятичный логарифм, чтобы решить уравнение:


      Решение:



      13. Используйте десятичный логарифм, чтобы решить уравнение:


      Решение:



      14.За t = 50 часов активность радиоактивного натрия снижается до 1/10 от исходного значения. x \), где \ (x \) представляет количество недель, в течение которых прошли.Икс . \ nonumber \]

      Хотя мы создали экспоненциальные модели и использовали их для прогнозов, вы, возможно, заметили, что решение экспоненциальных уравнений еще не упоминалось. Причина проста: ни один из рассмотренных до сих пор алгебраических инструментов не достаточен для решения экспоненциальных уравнений. Рассмотрим уравнение 2 x = 10 выше. Мы знаем, что 2 3 = 8 и 2 4 = 16, поэтому ясно, что x должно быть некоторым значением от 3 до 4, поскольку g ( x ) = 2 x — это увеличивается.Мы могли бы использовать технологию для создания таблицы значений или графика, чтобы лучше оценить решение, но мы хотели бы найти алгебраический способ решения уравнения.

      Нам нужна операция, обратная возведению в степень, чтобы найти переменную, если переменная находится в экспоненте. Как мы узнали на уроке алгебры (предварительное условие для этого конечного курса математики), обратная функция для экспоненциальной функции является логарифмической функцией.

      Мы также узнали, что экспоненциальная функция имеет обратную функцию, потому что каждое выходное значение (y) соответствует только одному входному значению (x).Этому свойству присвоено имя «один к одному».

      Источник: Материал в этом разделе учебника взят из книги Дэвида Липпмана и Мелони Расмуссен, Книжный магазин Open Text, Precalculus: Исследование функций, «Глава 4: Экспоненциальные и логарифмические функции» под лицензией Creative Commons CC BY-SA 3.0 лицензия. Приведенный здесь материал основан на материалах, содержащихся в этом учебнике, но был изменен Робертой Блум в соответствии с этой лицензией.

      Логарифм

      Логарифм (основание b ), записанный журнал b ( x ), является обратной экспоненциальной функции (основание b ), b x .{\ log_ {b} (x)} = x \ nonumber \]

      Поскольку журнал является функцией, его наиболее правильно записать как журнал b ( c ), используя круглые скобки для обозначения оценки функции, как и в случае с f (c) . {- 3} = \ frac {1} {1000} \)

    • Решение

      а.{2} = 9 \)

      Установив связь между экспоненциальными и логарифмическими функциями, теперь мы можем решать основные логарифмические и экспоненциальные уравнения путем переписывания.

      Пример \ (\ PageIndex {3} \)

      Журнал решения 4 ( x ) = 2 для x .

      Решение

      Переписав это выражение в экспоненту, 4 2 = x , поэтому x = 16

      Пример \ (\ PageIndex {4} \)

      Решите 2 x = 10 для x .

      Решение

      Переписав это выражение в виде логарифма, мы получим x = log 2 (10)

      Хотя это и определяет решение, вы можете найти его несколько неудовлетворительным, поскольку трудно сравнить это выражение с десятичной оценкой, которую мы сделали ранее. Кроме того, предоставление точного выражения для решения не всегда полезно — часто нам действительно нужно десятичное приближение к решению. К счастью, это задача, с которой калькуляторы и компьютеры неплохо справляются.К несчастью для нас, большинство калькуляторов и компьютеров оценивают только логарифмы двух оснований: 10 и и . К счастью, это не проблема, так как вскоре мы увидим, что можем использовать формулу «изменения основания» для вычисления логарифмов для других оснований.

      Обычный и натуральный логарифмы

      Общий журнал представляет собой логарифм с основанием 10 и обычно записывается как \ (\ log (x) \), а иногда как \ (\ log_ {10} (x) \). Если база не указана в функции журнала, то используется база b \ (b = 10 \).

      натуральный логарифм — это логарифм с основанием \ (e \), обычно записывается как \ (\ ln (x) \).

      Обратите внимание, что для любой другой базы b, кроме 10, база должна быть указана в виде \ (\ log_b (x) \).

      Пример \ (\ PageIndex {5} \)

      Оцените \ (\ log (1000) \), используя определение общего журнала.

      Решение

      В таблице приведены значения общего журнала

      номер

      число в виде экспоненты

      журнал (номер )

      1000

      10 3

      3

      100

      10 2

      2

      10

      10 1

      1

      1

      10 0

      0

      0.1

      10 -1

      –1

      0,01

      10 -2

      -2

      0,001

      10 -3

      -3

      Чтобы оценить log (1000), мы можем сказать

      \ [x = \ log (1000) \ nonumber \]

      Затем перепишите уравнение в экспоненциальной форме, используя общий логарифм с основанием 10

      . 5 \)
    • \ (\ ln \ sqrt {e} \)
    • Решение

      а.{1/2} \ right) = 1/2 \ nonumber \]

      Пример \ (\ PageIndex {8} \)

      Оцените следующее с помощью калькулятора или компьютера:

      1. \ (\ лог 500 \)
      2. \ (\ ln 500 \)

      Решение

      а. Используя кнопку LOG на калькуляторе для вычисления логарифмов по основанию 10, мы вычисляем LOG (500)

      Ответ: \ (\ log 500 \ приблизительно 2.69897 \)

      г. Используя клавишу LN на калькуляторе для вычисления натуральных логарифмов , , мы вычисляем LN (500)

      Ответ: \ (\ ln 500 \ примерно 6.{x} = \ log _ {c} A \).

      Теперь используется свойство экспоненты для журналов с левой стороны,
      \ [x \ log _ {c} b = \ log _ {c} A \ nonumber \]

      Разделив, мы получим \ (x = \ frac {\ log _ {c} (A)} {\ log _ {c} (b)} \), который является заменой базовой формулы.

      Вычисление логарифмов

      С изменением базовой формулы \ (\ log _ {b} (A) = \ frac {\ log _ {c} (A)} {\ log _ {c} (b)} \) для любых оснований \ (b \), \ (c> 0 \), мы наконец можем найти десятичное приближение к нашему вопросу с начала раздела.х = 10 \) для \ (х \).

      Решение

      Перепишите экспоненциальное уравнение 2 x = 10 в виде логарифмического уравнения

      \ [x = \ log _ {2} (10) \ nonumber \]

      Используя формулу изменения базы, мы можем переписать логарифм по основанию 2 как логарифм любого другого основания. Поскольку наши калькуляторы могут вычислять натуральный логарифм, мы можем использовать натуральный логарифм, который является логарифмической базой e :

      .

      Используя наши калькуляторы, чтобы оценить это, \ (\ frac {\ ln (10)} {\ ln (2)} = \ mathrm {LN} (10) / \ mathrm {LN} (2) \ приблизительно 3.3219 \)

      Это, наконец, позволяет нам ответить на наш первоначальный вопрос из начала этого раздела:
      Для популяции из 50 мух, которая удваивается каждую неделю, потребуется примерно 3,32 недели, чтобы вырасти до 500 мух.

      Пример \ (\ PageIndex {10} \)

      Вычислить \ (\ log_ {5} (100) \), используя изменение базовой формулы.

      Решение

      Мы можем переписать это выражение, используя любую другую основу.

      Метод 1: Мы можем использовать натуральный логарифм с основанием e с заменой основной формулы

      \ [\ log _ {5} (100) = \ frac {\ ln (100)} {\ ln (5)} = \ mathrm {LN} (100) / \ mathrm {LN} (5) \ приблизительно 2 .861 \ nonumber \]

      Метод 2: Мы можем использовать десятичный десятичный логарифм с заменой формулы основания,

      \ [\ log _ {5} (100) = \ frac {\ log (100)} {\ log (5)} = \ operatorname {LOG} (100) / \ mathrm {LOG} (5) \ около 2,861 \ nonumber \]

      Резюмируем взаимосвязь между экспоненциальными и логарифмическими функциями

      Логарифмы

      Логарифм (основание b ), записанный журнал b ( x ), является обратной экспоненциальной функции (основание b ), b x .{q} \ right) = q \ log _ {b} (A) \ nonumber \)

      Свойства журналов: изменение базы: \ (\ log _ {b} (A) = \ frac {\ log _ {c} (A)} {\ log _ {c} (b)} \ text { для любого основания} b, c> 0 \ nonumber \)

      Обратное, экспоненциальное и изменение основных свойств, указанных выше, позволит нам решить уравнения, которые возникают в задачах, с которыми мы сталкиваемся в этом учебнике. Сформулируем для полноты картины еще несколько свойств логарифмов

      .

      Сумма логов: \ (\ log _ {b} (A) + \ log _ {b} (C) = \ log _ {b} (A C) \)

      Разница в свойствах журналов: \ (\ log _ {b} (A) — \ log _ {b} (C) = \ log _ {b} \ left (\ frac {A} {C} \ справа) \)

      Журналы взаимных вычислений: \ (\ log _ {b} \ left (\ frac {1} {C} \ right) = — \ log _ {b} (C) \)

      Взаимные основы: \ (\ log _ {1 / b} C = — \ log _ {b} (C) \)

      Источник: материалы в этом разделе учебника взяты из книги Дэвида Липпмана и Мелони Расмуссен, Книжный магазин Open Text, Precalculus: An Investigation of Functions, «Глава 4: Экспоненциальные и логарифмические функции» под лицензией Creative Commons CC BY-SA 3 .0 лицензия. Приведенный здесь материал основан на материалах, содержащихся в этом учебнике, но был изменен Робертой Блум в соответствии с этой лицензией.

      .

    Эконометрика это: Эконометрика как наука — это… Что такое Эконометрика как наука?

    Профессия — специалист по эконометрике :: Федеральный образовательный портал

    Ольга Демидова, доцент кафедры математической экономики и эконометрики НИУ ВШЭ

    — Ольга Анатольевна, многие считают, что эконометрика и математическая экономика – это одно и то же. Есть ли между ними различие?

    — Термин «эконометрика» был введен в широкий научный обиход норвежским экономистом и статистиком, лауреатом Нобелевской премии Рагнаром Фришем в 1930-е годы. В 1933 году Фриш начал издавать одноименный журнал. До 1950–1960-х годов эконометрикой называли все математические измерения в экономике, а потом произошло разделение на математическую экономику – и эконометрику. Если специалист в области математической экономики стремится выразить утверждения экономической теории в форме математических уравнений, то эконометрист стремится верифицировать (проверить) эти модели с помощью эмпирических данных.

    Для специалиста по математической экономике достаточно сказать, что между такими величинами, как выпуск, труд и капитал существует определенная связь, которую можно описать математически и предложить достаточно абстрактное, универсальное ее описание, например, функцию Кобба-Дугласа. Она характеризует положительную зависимость объема производства от затрат труда и капитала. Специалист по эконометрике сначала соберет данные по объему производства на интересующих его предприятиях. После этого он оценит параметры функции, характеризующей зависимость выпуска от различных факторов для конкретных предприятий в определенных условиях. 

    — Не могли бы вы привести пример использования эконометрики на практике?

    — Приведу один из моих любимых примеров. В 1980-е годы на Нью-Йоркской фондовой бирже комиссионные, которые брокеры получают по итогам сделок, регулировал биржевой комитет. Клиент не мог договориться непосредственно с брокером о размере комиссионных за услуги, а был вынужден платить проценты, установленные биржевым комитетом.

    Американский аналог нашей Федеральной антимонопольной службы усмотрел в этом признаки монополии и потребовал либерализовать цены. На суде представители биржевого комитета привели уравнение регрессии, оцененное по конкретным данным для сделок, из которого следовало, что имеет место естественная монополия. То есть в результате либерализации комиссионные только вырастут. Тогда антимонопольная служба заявила, что уравнение неправильно характеризует зависимость вознаграждения брокеров и объема проданных акций, и на самом деле естественной монополии нет. Их оппоненты не учли проблемы гетероскедастичности[1] данных. Если учесть гетероскедастичность, то результат приводит к противоположным выводам. Прошло несколько заседаний, на которых специалисты выясняли, чьи оценки правильнее. В результате борцы с монополией, видимо, лучше знавшие эконометрику, выиграли процесс.

    — В каких областях чаще всего работают специалисты по эконометрике?

    — В самых разных. На мой взгляд, лучше всего о востребованности специалистов по эконометрике на рынке труда говорит тот факт, что из четырех лауреатов премии «Золотая Вышка» в номинации «Успех выпускника» за последние годы двое были выпускниками кафедры математической экономики и эконометрики, а один (первый заместитель председателя Центрального банка России Владислав Конторович) писал магистерскую работу под руководством профессора нашей кафедры Эмиля Ершова.  

    Многие наши выпускники работают в банковской сфере, где их познания в математике очень востребованы. Так, одна студентка, писавшая под моим руководством магистерскую диссертацию, работала в ВТБ24 и занималась вопросами ипотечного кредитования. Тема ее работы была связана с исследованием, которое она проводила для банка. Нужно было попытаться предсказать поведение людей, берущих ипотечные кредиты: выяснить, кто и с какой вероятностью вернет кредит заранее или выплатит его в срок, или не выплатит вовсе. Банкам не всегда выгодно то, что клиент заранее возвращает кредит. Для них невыгодно получить деньги, на которые они сейчас не рассчитывают. Соответственно банку нужно заранее знать, сколько людей может вернуть кредит раньше срока и на какой объем средств в каждый момент времени он может рассчитывать. В рамках исследования был собран большой объем данных. После их анализа стало ясно, что помимо таких факторов, как возраст и образование, на поведение заемщиков влияет их место жительства. Живущие за Уралом оказались склонны погашать кредиты раньше, чем жители европейской территории.

    Пример из совершенно другой области: один из проектов, в котором я сейчас принимаю участие, называется SEARCH. Он посвящен изучению разнообразных вопросов, связанных с взаимодействием стран – членов ЕС и их соседей, в частности – России. Например, как распределяются потоки мигрантов, торговые потоки, происходит ли «диффузия» инноваций и т.д. Я включена в блок, посвященный оценке социального капитала и пытаюсь с помощью эконометрических моделей сравнить отношение жителей двух выделенных групп стран к основным социальным и политическим институтам.

    Специалисты по эконометрике могут заниматься достаточно широким спектром вопросов. Наши выпускники работают и в рекламе. Здесь важно оценить, в каких средствах массовой информации реклама наиболее эффективна, какая реклама лучше. Многие работают в аналитических отделах достаточно известных торговых фирм и с помощью эконометрических моделей пытаются оценить, допустим, эффект от проведения рекламной компании или обосновать выбор ассортимента торговых точек. Некоторые выпускники нашей кафедры заняты на госслужбе и сделали неплохую карьеру, некоторые принимают участие в работе экспертных групп, нередко дают в средствах массовой информации  комментарии по поводу различных макроэкономических проблем. Немало выпускников кафедры после окончания  магистратуры или аспирантуры занимаются преподавательской и научной деятельностью.

    — Как вы считаете, что является самой большой сложностью для молодых специалистов по эконометрике в их работе?

    — Мне бы хотелось предостеречь своих молодых коллег от одной очень распространенной практики. Часто они, прочитав какую-нибудь статью западного автора, берут использованные им модели и методы и просто переносят их на наши реалии. Это не всегда позволяет получить адекватные результаты. Надо попытаться преодолеть искушение использовать какую-либо модель только потому, что «все так делают». Молодые исследователи, насколько я могу судить по моему опыту участия в конференциях, как отечественных, так и зарубежных, очень любят включать в свои работы модель Арелано-Бонда, модель стохастической границы, метод разностей в разностях и т. д., иногда просто следуя эконометрической моде, благо современные статистические пакеты легко позволяют оценить эти модели. Из полученных результатов зачастую очевидно, что для использованных в работе данных выбранная модель не годится, предварительный анализ данных был проведен достаточно поверхностно, сделанные на основе оцененных моделей выводы весьма сомнительны.

    Работающим с российскими данными надо четко понимать, что западные модели и методы к нашим параметрам могут быть неприменимы. Не стоит просто брать западную статью и оценивать приведенные в ней модели по российским данным, хотя иногда это и может дать интересные результаты. Одна моя дипломница, Полина Ковалева  (сейчас она учится в аспирантуре университета Сити в Лондоне) пыталась вслед за Джеймсом Хекманом выяснить, применимы ли модели типа Минцера для оценки отдачи от образования. Напомню, что в моделях такого типа отдача от образования считается постоянной, т.е. каждый дополнительный год обучения увеличивает заработную плату на одну и ту же величину. Если это так, то профили (графики зависимости) «зарплата – опыт работы» должны быть параллельны для различных лет обучения, т.е. получаться один из другого простым сдвигом. Для американских данных 1980 года параллельность профилей имела место. Однако для российских данных этот факт не подтвердился. Наличие послевузовского образования сначала приводило к достаточно быстрому росту заработной платы, а затем к достаточно быстрому ее иснижению. Из этого результата можно сделать вывод, что полученные знания достаточно быстро устаревают, и если ты не хочешь допустить падения заработной платы, то должен все время повышать свою квалификацию.

    — Часто приходится слышать, что в российских условиях самая трудная задача для аналитика — получить достоверные данные. Так ли это?

    — Это не только российская проблема: зарубежные исследователи тоже жалуются на недостаток данных, проведение социологических опросов – достаточно дорогое удовольствие.

    Но даже когда данные есть, казалось бы, в открытом доступе, их не всегда легко  использовать. При работе в проекте «Анализ системы госзакупок в России» в Институте анализа предприятий и рынков НИУ ВШЭ я вместе с коллегами столкнулась со следующей проблемой. Есть сайт, где публикуется вся информация о государственных контрактах, однако в неудобном для пользователей виде. Если требуется выгрузить информацию о контрактах на группу каких-либо однородных товаров, то система «зависает», а собирать информацию по каждому контракту вручную очень долго.

    — Каким, по-вашему, должен быть хороший учебник по эконометрике?

    — Большинство учебников по эконометрике строится так: сначала теория, потом практические примеры. Причем для реальных данных, увы, не всегда выполняются предположения,  позволяющие получить удобные аналитические формулы, приведенные в учебнике.

    Недаром Марно Вербик, автор очень известного и переведенного на русский язык учебника «Путеводитель по современной эконометрике» приводит высказывание своего слушателя: «Эконометрика была бы гораздо проще без данных». Однако есть точка зрения, что учебники по эконометрике должны идти от практики к теории и изобиловать примерами.

    Есть очень известный американский учебник «Практика эконометрики» профессора Массачусетского технологического института Эрнста Р. Берндта. Он как раз построен по противоположному принципу: берется практическая задача, например, изучение издержек американских фирм-производителей электроэнергии, диверсификация портфеля ценных бумаг, а дальше рассказывается о способах ее решения с применением эконометрики. Студенты сразу погружаются в практику. При составлении своих учебных пособий я попыталась частично взять на вооружение тот же принцип и по максимуму использовать российские кейсы.

    — Каково соотношение теории и практики в вашем лекционном курсе эконометрики?

    — Я пытаюсь соблюсти баланс теории и практики. На каждой лекции я сопровождаю теоретический материал практическими примерами. На компьютерных занятиях мы со студентами работаем с реальными данными, конечно, интереснее всего для них российские.

    Темой последнего семинара были модели бинарного выбора. По данным мониторинга экономического положения и здоровья населения мы исследовали, какие факторы влияют на удовлетворенность человека жизнью. Оказалось, что влияют и доход, и образование, и пол. Тогда студенты решили выяснить, делает ли законный брак человека счастливее. Проверили, оказалось, что женщин – да, а мужчин – нет. Конечно, студенты не остановились на достигнутом и проверили еще много интересных предположений. Возможно, они сделали первый шаг к будущим научным исследованиям.

    — Эконометрика традиционно считается сложным для изучения предметом. Что бы вы посоветовали студентам для успешной сдачи этого курса?

    — Как и в математике, в эконометрике нет «королевской дороги». Изучение эконометрики требует вдумчивой, планомерной работы  с теоретическим материалом и освоения современных статистических пакетов. Настойчивость и трудолюбие будут вознаграждены не только хорошей оценкой за курс, но и возможностью в будущем проводить полноценные эконометрические исследования, а это очень увлекательное занятие!

     

    Беседовала Екатерина Рылько

     

     


     

    [1]    То есть в уравнении регрессии дисперсия ошибки отличалась от наблюдения к наблюдению.

    Объект эконометрики

    Сущность эконометрики

    Определение 1

    Эконометрика — это наука, которая изучает качественные и количественные экономические взаимосвязи, применяя математические статистические методы и модели.

    Современная трактовка объекта эконометрики выработана в соответствии с положениями устава «Эконометрического общества», при этом главными целями стали применение математики и статистики для дальнейшего совершенствования экономической теории.

    Эконометрика является наукой, которая позволяет качественному социальному процессу присвоить количественную характеристику и интерпретировать их с экономической позиции.

    С теоретической точки зрения, эконометрика изучает статистические свойства испытаний и оценок, тогда как с прикладной позиции эконометрика отвечает за применение эконометрических методов для проведения оценки различных экономических теорий. Эконометрика предоставляет инструментарий для проведения экономических измерений, определяет методологию анализа параметров микро- и макроэкономических моделей.

    Помимо этого, эконометрика часто используется для составления прогнозов экономических процессов не только в масштабе экономики и целом, но и на уровне отдельного предприятия, Вместе с тем, эконометрика — это такая же часть экономической теории, как макро- и микроэкономика.

    Замечание 1

    Объектом изучения эконометрики являются экономико-математические модели (эконометрические модели), в основе построения которых — случайные факторы.

    Эконометрика — составной элемент обширного семейства дисциплин, которые посвящены измерениям и использованию статистических инструментов и методов в различных научно-практических областях. Эго семейство, в частности, включает биометрию, технометрику, наукометрию, психометрию, хемометрию, квалиметрию. Особая роль отводится социометрии, которая изучает статистические методы анализа взаимоотношений малых групп и является частью статистического анализов в психологии и социологии.

    Цели и задачи эконометрики

    Основные задачи эконометрики:

    • выявить связи отдельных количественных характеристик экономических объектов для построения математических правил прогнозирования;
    • установить значения каждого числового параметра, входящего в модель;
    • и привести их в соответствие поведению объекта в реальности;
    • получить наилучшие опенки параметров экономико-математических моделей, сконструированных в прикладных целях;
    • проверить теоретико-экономические положения и выводы на фактическом материале;
    • создать универсальные и специальные методы для выявления в экономике статистических закономерностей.

    Замечание 2

    Основной целью эконометрики является поиск адекватного инструмента прогнозирования поведения экономических объектов в разнообразных ситуациях, а также решение на базе прогнозирования практических задач в части оптимального управления объектом; выбора стратегии рыночного поведения.

    Специфика экономических измерений

    Измерение экономических данных характеризуются следующими специфическими особенностями:

    1. Измерению поддаются только данные, определенные операционально.
    2. Экономические измерения при этом подвержены сильному воздействию теоретических мнений относительно данных величин.
    3. Короткие ряды наблюдений и неэкспериментальный характер данных заставляют сомневаться в адекватности полученного результата.
    4. Косвенный характер экономических данных, при этом зачастую первичные изменения не несут экономического характера.
    5. Изменчивый характер единиц измерения.
    6. Острая проблема, связанная с влиянием инструментов измерения на изучаемый объект.

    Что такое Эконометрика — классификация и задачи

    Рубрика: Экономический глоссарий Опубликовано 07.06.2014   ·   Комментарии: 0   ·   На чтение: 2 мин   ·   Просмотры:

    Post Views: 484

    Эконометрика -это наука об экономических измерениях.
    Термин происходит от двух слов: «экономика» и «метрика». Греческий корень «метрон» означает правило определения расстояния между двумя точками в пространстве, «метрия» – измерение.

    Эконометрика — это наука, базирующаяся на анализе связи между различными экономическими показателями с учётом реальных статистических данных, а также с применением математической статистики и методов теории вероятностей. Она позволяет выявить, уточнить, опровергнуть связи, и гипотезы, отражающие существование данных связей между экономическими показателями, предлагаемые конкретной экономической теорией.

    Цель эконометрики заключается в модельном описании количественных взаимосвязей. Данные взаимосвязи обусловливаются общими качественными закономерностями, которые выявляются и оцениваются в экономической теории.

    Предметом исследования являются массовые экономические явления и процессы.

    Основные задачи эконометрики

    1.обнаружение статистических закономерностей в экономике

    2.анализ данных закономерностей

    3.выявление эмпирических экономических зависимостей

    4.построение на их базе эконометрических моделей.

    Классификация задач эконометрики

    Основные признаки классификации всех задач, рассматриваемых эконометрикой:

    1. Классификация задач в зависимости от конечных прикладных целей (прогноз социально-экономических показателей, моделирование возможных вариантов социально-экономического развития системы, выявление факторов, оказывающих наиболее мощное влияние на состояние системы в целом).

    2. Классификация задач в зависимости от уровня иерархии (макроуровень, мезоуровень, микроуровень).

    3. Классификация задач в зависимости от профиля изучаемой экономической системы (рынок,инвестиции,спрос, потребление, ценообразование и прочее).

    Много общих черт существует у эконометрики и статистики – в некоторой степени, эконометрика является часть статистической науки: статистика изучает глобальные явления всех направлений, в том числе и экономических, а эконометрика занимается теми же процессами, но ограничивается исключительно экономическими задачами.

    Post Views: 484

    План. Что такое эконометрика. Как это работает на первый взгляд. Литература. Как это работает на самом деле. три истории

    Probit, logit, heckit модели

    Probit, logit, heckit модели 11 13 июля 2017 г. Курс лекций «Макроэконометрика» 1 Оглавление Дискретные зависимые переменные Probit Logit Цензурированные выборки Tobit Модель Хекмана 2 Дискретные зависимые

    Подробнее

    https://sites.google.com/site/ekonometriya2014

    Тема 1. Введение в эконометрию 1.1. История эконометрии 1.2. Основные понятия и определения эконометрии 1.3. Эконометрическое моделирование 1.4. Литература 1.5. Статистические данные 1.6. Статистические

    Подробнее

    Модель парной регрессии

    Модель парной регрессии 30 25 20 15 10 В статистических данных редко встречаются точные линейные соотношения: y x 1 2 Обычно они бывают приближенными: y x 1 2 5 0 2 4 6 8 10 12 14 16 18 20

    Подробнее

    Эконометрика (базовая) Лекция 1

    Эконометрика (базовая) Лекция 1 Рекомендуемая литература 1. Доугерти К. Введение в эконометрику. М.: ИНФРА-М, 2001. 2. Магнус Я.Р. и др. Эконометрика. Начальный курс. М.: Дело, 2001. 3. Кремер Н.Ш., Путко

    Подробнее

    Правительство Российской Федерации

    Правительство Российской Федерации Федеральное государственное автономное образовательное учреждение высшего профессионального образования «Национальный исследовательский университет «Высшая школа экономики»

    Подробнее

    ЭКОНОМЕТРИКА Программа

    МОСКОВСКИЙ ИНСТИТУТ ЭКОНОМИКИ, МЕНЕДЖМЕНТА И ПРАВА Кафедра общематематических и естественнонаучных дисциплин ЭКОНОМЕТРИКА Программа Москва 2003 1 Составитель: Харламов С.А., доктор технических наук Эконометрика:

    Подробнее

    1 Предисловие. Ф-ПР Рабочая программа

    Предисловие Данная дисциплина рассматривает и изучает эконометрические модели и методы анализа и прогнозирования социально-экономических процессов. Методика преподавания данной дисциплины предусматривает:

    Подробнее

    Эконометрическое моделирование

    Эконометрическое моделирование Лабораторная работа 5 Множественная регрессия Оглавление Множественная регрессия… 3 Мультиколлинеарность… 4 Задание 1. Построение модели множественной регрессии… 5

    Подробнее

    УЧЕБНО-МЕТОДИЧЕСКИЙ КОМПЛЕКС ДИСЦИЛИНЫ

    МИНИСТЕРСТВО ОБРАЗОВАНИЯ И НАУКИ РЕСПУБЛИКИ КАЗАХСТАН ГОСУДАРСТВЕННЫЙ УНИВЕРСИТЕТ имени ШАКАРИМА ГОРОДА СЕМЕЙ Документ СМК 3 уровня УМКД Программа дисциплины «Эконометрика» для студентов УМКД Редакция

    Подробнее

    Введение. кономерности до экономических

    Эконометрика является одной из важнейших составляющих современного экономического образования. В ведущих мировых университетах в программах подготовки экономистов ей уделяется большое внимание. Применение

    Подробнее

    Эконометрика и прогнозирование

    Эконометрика и прогнозирование Автор-составитель: доцент кафедры экономической информатики и математической экономики экономического факультета БГУ, к.ф.-м.н. Васенкова Е.И. Цель и учебная задача Эконометрика

    Подробнее

    Аннотация. Цели и задачи дисциплины

    Цели и задачи дисциплины Аннотация Целью курса является приобретение опыта построения эконометрических моделей и определение возможностей их использования для описания, анализа и прогнозирования реальных

    Подробнее

    ПРОГРАММА ВСТУПИТЕЛЬНОГО ИСПЫТАНИЯ

    ПРОГРАММА ВСТУПИТЕЛЬНОГО ИСПЫТАНИЯ для поступающих на основную образовательную программу магистратуры «Математические методы в экономике» по направлению подготовки 38.04.01 «ЭКОНОМИКА» по предмету «МАТЕМАТИЧЕСКИЕ

    Подробнее

    Рабочая программа дисциплины Эконометрика

    Федеральное государственное автономное образовательное учреждение высшего образования «Национальный исследовательский университет «Высшая школа экономики» Факультет экономических наук Департамент прикладной

    Подробнее

    1. ЦЕЛИ И ЗАДАЧИ ИЗУЧЕНИЯ ДИСЦИПЛИНЫ

    1. ЦЕЛИ И ЗАДАЧИ ИЗУЧЕНИЯ ДИСЦИПЛИНЫ Изучение дисциплины «Эконометрика» ориентировано на получение обучающимися знаний о сложной взаимосвязи различных факторов, оказывающих существенное воздействие на

    Подробнее

    Правительство Российской Федерации

    Правительство Российской Федерации Государственное образовательное бюджетное учреждение высшего профессионального образования «Государственный университет — Высшая школа экономики» Нижегородский филиал

    Подробнее

    ЭКОНОМЕТРИКА. Лекция Введение.

    Лекция 1. ЭКОНОМЕТРИКА 1. Введение. Список рекомендуемой литературы. Основная. 1. Бородич С.А., Эконометрика. Минск, ООО «Новое знание», 2004. 2. Магнус Я.Р., Катышев П.К., Пересецкий А.Л. Эконометрика.

    Подробнее

    РАБОЧАЯ ПРОГРАММА ДИСЦИПЛИНЫ

    МИНИСТЕРСТВО ОБРАЗОВАНИЯ И НАУКИ РФ федеральное государственное бюджетное образовательное учреждение высшего профессионального образования «Мурманский государственный гуманитарный университет» (ФГБОУ ВПО

    Подробнее

    2011 Экономика 2(14)

    ВЕСТНИК ТОМСКОГО ГОСУДАРСТВЕННОГО УНИВЕРСИТЕТА 211 Экономика 2(14) УДК 331.11.3:3 МОТИВАЦИЯ ТРУДОВОЙ ДЕЯТЕЛЬНОСТИ: ФАКТОРЫ И ИХ ОЦЕНКА РЫНКОМ ТРУДА Статья посвящена факторам, влияющим на уровень заработной

    Подробнее

    НОУ ИНТУИТ | Лекция | Сущность и история возникновения эконометрики

    1.1. Предмет исследований эконометрики

    Курс эконометрики занимает важное место в современных программах экономических вузов во всем мире наряду с такими предметами, как микроэкономика, макроэкономика, финансовый анализ. Эконометрические методы используются для прогнозирования в банковском деле, финансах, бизнесе, при государственном регулировании экономики. Что же такое эконометрика? Эконометрика — быстроразвивающаяся отрасль экономической науки, целью которой является количественное описание экономических отношений. Приведем несколько цитат о существе данной науки.

    «Эконометрика позволяет проводить количественный анализ реальных экономических явлений, основываясь на современном развитии теории и наблюдениях, связанных с методами получения выводов» (П. Самуэльсон).

    «Основная задача эконометрики — наполнить эмпирическим содержанием априорные экономические рассуждения» (Л. Клейн).

    «Цель эконометрики — эмпирический анализ экономических законов. Эконометрика дополняет теорию, используя реальные данные для проверки и уточнения постулируемых отношений» (Э. Маленво).

    Термин «эконометрия» впервые ввел П. Цьемпа в 1910 г., который пытался применить методы алгебры и геометрии к анализу хозяйственной деятельности. В настоящее время этот термин используется для того раздела эконометрики и теории экономического анализа, который изучает влияние факторов, формирующих результаты работы фирмы (предприятия).

    В мировой науке термин «эконометрика» обычно употребляется применительно к науке об измерении и анализе экономических явлений. Эта наука возникла на стыке трех дисциплин: экономической теории, методов математического анализа и математической статистики. Основатель журнала «Эконометрика» Р. Фриш привел следующее определение эконометрики: «Эконометрика — это не то же самое, что экономическая статистика. Она не идентична тому, что мы называем экономической теорией, хотя значительная часть этой теории носит количественный характер. Эконометрика не является синонимом приложений математики к экономике. Как показывает опыт, каждая из трех отправных точек — статистика, экономическая теория и математика — необходимое, но не достаточное условие для понимания количественных соотношений в современной экономической жизни. Это — единство всех трех составляющих. И это единство образует эконометрику».

    Однозначного определения эконометрики пока не существует. Есть по крайней мере еще четыре дисциплины, использующие математические методы применительно к экономике:

    1. многомерный статистический анализ данных, тесно связанный с эконометрикой;
    2. финансовая математика, также использующая в своих современных разделах эконометрические методы;
    3. математические модели в экономике — наука, применяющая для подтверждения теоретических концепций эконометрическую технику верификации моделей;
    4. математические методы в экономике (старое название — исследование операций) — наука о постановках и решении оптимизационных задач в экономике, состоящая из таких широко известных разделов, как линейное и нелинейное программирование, сетевое планирование, управление запасами, теория игр. Несколько особняком стоит теория массового обслуживания.

    Некоторые исследователи, в том числе Э. Маленво, придавали широкое толкование эконометрике, интерпретируя ее как любое применение математики или статистических методов к изучению экономических явлений. Однако доминирующим стало мнение, что эконометрика применяет статистические подходы к эконометрическим измерениям. Это обстоятельство обусловило содержание настоящего курса лекций.

    Эконометрика содержит два больших раздела: моделирование данных, неупорядоченных во времени, и теория временных рядов.

    Тест по эконометрике с ответами

    Сборник итоговых тестов по теме Эконометрика с ответами

    Правильный вариант ответа отмечен знаком +

    1. Что является предметом изучения эконометрики?

    — Количественная сторона экономических процессов и явлений

    + Массовые экономические процессы и явления

    — Система внутренних связей между явлениями национальной экономики

    2. Гетероскедастичность – это в эконометрике термин, обозначающий:

    + Неоднородность наблюдений, которая выражается в непостоянной (неодинаковой) дисперсии случайной ошибки эконометрической (регрессионной) модели

    — Однородную вариантность значений наблюдений, которая выражена в относительной стабильности, гомогенности дисперсии случайной ошибки эконометрической (регрессионной) модели

    — Меру разброса значений случайной величины относительно ее математического ожидания

    3. Мультиколлинеарность – это в эконометрике термин, обозначающий:

    — Метод, позволяющий оценить параметры модели, опираясь на случайные выборки

    — Статистическую зависимость между последовательными элементами одного ряда, которые взяты со сдвигом

    + Наличие линейной зависимости между факторами (объясняющими переменными) регрессионной модели

    4. Теорема Гаусса-Маркова в эконометрике опирается на:

    + Метод наименьших квадратов

    — Метод наименьших модулей

    — Метод инструментальных переменных

    5. Эконометрика – это наука, которая изучает:

    — Структуру, порядок и отношения, сложившиеся на основе операций подсчета, измерения и описания формы объектов

    — Возможности применения методов математики для решения экономических задач

    + Количественные и качественные экономические взаимосвязи, и взаимозависимости, опираясь на методы и модели математики и статистики

    6. Коэффициент эластичности (формула в общем виде) в эконометрике имеет вид:

    +

    7. Модели временных рядов в эконометрике – это модели:

    — Которые используются для того, чтобы определить, как себя будет вести тот или иной фактор в течение определенного промежутка времени

    — Которые позволяют максимально точно рассчитать период времени, требующийся для того, чтобы значение фактора изменилось на значимую величину

    + Для построения которых используются данные, характеризующие один объект за несколько последовательных периодов

    8. Метод наименьших квадратов в эконометрике – это метод:

    — Который используется для расчета наименьших отклонений случайных величин, влияющих на конечный результат

    + Который позволяет решать задачи, опираясь на минимизацию суммы квадратов отклонений некоторых функций от искомых переменных

    — Который позволяет оценить значение неизвестного параметра, минимизируя значение функции правдоподобия

    9. Линейный коэффициент корреляции в эконометрике выражается формулой:

    +

    тест 10. Истинный коэффициент детерминации в эконометрике выражается формулой:

    +

    11. Модели в эконометрике – это:

    + Средство прогнозирования значений определенных переменных

    — Экономические и статистические зависимости, выраженные математическим языком

    — Данные одного типа, сгруппированные определенным образом

    12. Какие существуют типы данных в эконометрике?

    — Постоянные, переменные

    — Определенные, неопределенные, качественные, количественные

    + Пространственные, временные, панельные

    13. Зависимая переменная в эконометрике – это:

    — Параметр, состоящий из случайной и неслучайной величин

    + Некоторая переменная регрессионной модели, которая является функцией регрессии с точностью до случайного возмущения

    — Переменная, которая получается путем перевода качественных характеристик в количественные, т.е. путем присвоения цифровой метки

    14. Какова цель эконометрики?

    — Поиск, трактовка (с использованием математического инструментария) и систематизация факторов, которые влияют на поведение экономического объекта

    — Выявление качественных и количественных связей между характеристиками экономических объектов с целью построить экономическую модель их развития

    + Разработка инструментов для прогнозирования поведения экономического объекта в различных ситуациях и на их базе решение практических задач по управлению объектом, выбору поведения в сложившихся экономических условиях и т.д.

    15. Что представляет собой выборочная дисперсия?

    + Несмещенную оценку генеральной дисперсии

    — Смещенную оценку генеральной дисперсии

    — Смещенную оценку моды

    16. Какие приемы используют для идентификации модели?

    — Проверка адекватности, статистический анализ

    + Оценка параметров, статистический анализ

    — Расчет математических ожиданий, проверка адекватности

    17. Предельно допустимое значение средней ошибки аппроксимации составляет … %.

    — Не более 10-12

    — Не более 3-5

    + Не более 8-10

    18. Какие существуют типы переменных в эконометрике?

    + Предопределенные, экзогенные, эндогенные

    — Пространственные, временные, панельные

    — Экзогенные, эндогенные

    19. Назовите ученого, который ввел термин «эконометрика».

    — Н. Кондратьев

    + Р. Фриш

    — К. Грэнджер

    тест_20. Какой показатель измеряет тесноту статистической связи между переменной и объясняющими переменными?

    + Коэффициент детерминации

    — Коэффициент рекурсии

    — Коэффициент корреляции

    21. Укажите, какими способами оценивают параметры линейной регрессии:

    — Дисперсия, метод наименьших квадратов, математическое ожидание

    + Дисперсия, математическое ожидание, ковариация, среднеквадратичное отклонение

    — Математическое ожидание, регрессия, медиана

    22. Критические значения статистики Дарбина-Уотсона зависят от следующих факторов:

    + Количество наблюдений в выборке и число объясняющих переменных

    — Число объясняющих переменных и конкретные значения переменных

    — Количество наблюдений в выборке и конкретные значения переменных

    23. Для установления влияния какого-либо события на коэффициент линейной регрессии при не фиктивной переменной в модель включают:

    — Фиктивную переменную взаимодействия

    + Фиктивную переменную для коэффициента наклона

    — Лаговую переменную

    24. Случайная величина, принимающая отдельные, изолированные друг от друга значения – это:

    + Дискретная величина

    — Вероятностный парадокс

    — Неравномерная величина

    25. Перечислите этапы построения эконометрической модели:

    — Априорный, контекстный, информационный, аналитический, прогностический, идентификация модели

    — Постановочный, контекстный, информационный, аналитический, идентификация модели, параметризация модели

    + Постановочный, априорный, параметризация, информационный, идентификация модели, верификация модели

    26. Эндогенные переменные – это переменные:

    — Внешние, задаваемые вне социально-экономической модели и не зависящие от ее состояния

    + Внутренние, сформированные в результате функционирования социально-экономической системы

    — Которые постоянно изменяются

    27. Что представляет собой априорный этап построения эконометрической модели?

    + Предмодельный анализ экономической сущности изучаемого явления, формирование и формализация априорной информации

    — Сбор и регистрация информации об участвующих в модели факторах и показателях

    — Независимое оценивание значений участвующих в модели факторах и показателях

    28. Если увеличить размер выборки, то оценка математического ожидания:

    — Станет менее точной

    + Станет более точной

    — Не изменится

    тест № 29. Ситуация, при которой нулевая гипотеза была опровергнута, хотя и являлась истинной, называется:

    + Ошибка I рода

    — Системная ошибка

    — Стандартная ошибка

    30. Если предположение о природе гетероскедастичности верно, то дисперсия случайного члена для первых наблюдений в упорядоченном ряду будет … для последних.

    — Такой же, как

    — Выше, чем

    + Ниже, чем

    Эконометрика как учебная дисциплина Текст научной статьи по специальности «Экономика и бизнес»

    УДК 519.2:005.521:633.1:004.8

    01.00.00 Физико-математические науки

    ЭКОНОМЕТРИКА КАК УЧЕБНАЯ ДИСЦИПЛИНА

    Орлов Александр Иванович

    д.э.н., д.т.н., к.ф.-м.н., профессор

    РИНЦ SPIN-код: 4342-4994

    Московский государственный технический

    университет им. Н.Э. Баумана, Россия, 105005,

    Москва, 2-я Бауманская ул., 5, prof-orlov@mail.т

    Статистические методы широко используются в отечественных технико-экономических исследованиях. Однако для большинства менеджеров, экономистов и инженеров они являются экзотикой. Это объясняется тем, что в вузах современным статистическим методам не учат. Обсудим сложившуюся ситуацию, уделив основное внимание статистическим методам в экономических и технико-экономических исследованиях, т.е. эконометрике. В мировой науке эконометрика занимает достойное место. Имеются научные журналы по эконометрике, нобелевские премии по экономике присуждены ряду эконометриков. Положение в области научных и практических работ и особенно преподавания эконометрики в России является неблагополучным. Зачастую за эконометрику выдают отдельные частные построения, например, относящиеся к регрессионному анализу. Статья посвящена эконометрике как учебной дисциплине. Начинается курс с обсуждения структуры современной эконометрики, соотношения прикладной статистики и эконометрических методов. Рассмотрены выборочные исследования (анализ результатов опросов), элементы эконометрики чисел, методы статистической проверки гипотез однородности. Даны понятия о регрессионном анализе, эконометрических методах классификации, современной теории измерений. Важное место занимает статистика нечисловых данных (включая нечеткие множества и их связь со случайными), статистика интервальных данных. Обсуждается проблема устойчивости статистических процедур по отношению к допустимым отклонениям исходных данных и предпосылок модели. Дано представление об эконометрических методах экспертных исследований и управления качеством, анализе и прогнозе временных рядов, эконометрике прогнозирования и риска

    Ключевые слова: СТАТИСТИЧЕСКИЕ МЕТОДЫ, ЭКОНОМЕТРИКА, УПРАВЛЕНИЕ, ЭКОНОМИКА, СТАТИСТИКА НЕЧИСЛОВЫХ И ИНТЕРВАЛЬНЫХ ДАННЫХ, ИНФЛЯЦИЯ

    UDC 519:2:005.521:633.1:004.8

    Physics and mathematical sciences

    ECONOMETRICS AS AN ACADEMIC DISCIPLINE

    Orlov Alexander Ivanovich

    Dr.Sci.Econ., Dr.Sci.Tech., Cand.Phys-Math.Sci.,

    professor

    Bauman Moscow State Technical University, Moscow, Russia

    Statistical methods are widely used in domestic feasibility studies. However, for most managers, economists and engineers, they are exotic. This is due to the fact that modern statistical methods are not taught in the universities. We discuss the situation, focusing on the statistical methods for economic and feasibility studies, ie, econometrics. In the world of science, econometrics has a rightful place. There are scientific journals in econometrics, Nobel Prizes in Economics are given to series of researches in econometrics. The situation in the field of scientific and practical work and especially the teaching of econometrics in Russia is disadvantaged. Often, individual particular constructions replace econometrics in general, such as those related to regression analysis. The article is devoted to econometrics as an academic discipline. Our course begins with a discussion of the structure of modern econometrics, the connections between applied statistics and econometric methods. We consider sample researches (analysis of surveys results), the elements of econometrics numbers, and methods of testing of statistical hypothesis about homogeneity. We have given the concepts of regression analysis, econometric classification methods, modern measurement theory. The important places are occupied by the statistics of non-numerical data (including fuzzy sets and their links with random sets) and the statistics of interval data. The problem of the stability of statistical procedures with respect to the tolerances of input data and model prerequisites is discussed. The representations of the econometric methods of expert research and quality control, analysis and forecasting of time series, econometrics of forecasting and risks are given

    Keywords: STATISTICAL METHODS, ECONOMETRICS, MANAGEMENT, ECONOMICS, STATISTICS OF NON-NUMERIC AND INTERVAL DATA, INFLATION

    йо!: 10.21515/1990-4665-128-050

    1. Введение

    Известно, что современные эконометрические методы — полезные интеллектуальные инструменты инженера, управленца и экономиста [1]. Эконометрика — самостоятельная научная, прикладная и учебная дисциплина. В настоящей статье рассмотрим примерное содержание эконометрики как учебного предмета в соответствии с подходом отечественной научной школы в области эконометрики [2]. Комментарии к разделам курса показывают их место в современной эконометрике и обосновывают необходимость включения соответствующего материала в учебную программу.

    2. Вводный раздел: структура современной эконометрики

    Структура современной статистики: математическая статистика, прикладная статистика, применения статистических методов в конкретных областях. Определение эконометрики как науки о применении статистических методов для анализа конкретных экономических данных, а эконометрических методов — как статистических методов анализа экономических данных. Разрыв между математической статистикой и Госкомстатом РФ. Всегда ли можно верить данным Госкомстата РФ? Пример с оцениванием вузовской науки по числу научных ставок в вузах. Особенности экономических данных и их учет при применении методов прикладной статистики. Понятие об эконометрических моделях (на примерах моделей управления качеством, анализа экспертных оценок и др.). Понятие о конкретных применениях эконометрики (на примере динамики цен и индекса инфляции).

    Содержание этого вводного раздела достаточно полно раскрыто в статье [1]. Приведем содержание только одного из наиболее интересных

    для студентов раздела эконометрики, посвященного изучении динамики цен с помощью индексов инфляции.

    Проблемы и методы анализа динамики цен. Инфляция как рост цен. Рост цен на отдельные товары и услуги и проблемы построения сводной характеристики. Различные варианты потребительских корзин. Определение и расчет индексов инфляции. Динамика инфляции в России. Теорема умножения (связь между значениями индексов инфляции для трех значений времени) и средний индекс (темп) инфляции как среднее геометрическое. Распространенные ошибки при агрегировании индексов инфляции. Принципиальная разница между малыми и большими значениями индексов инфляции. Связь годового индекса и помесячными и средним за год. Теорема сложения для индекса инфляции (правило объединения индексов инфляции для отдельных товарных групп).

    Применения индекса инфляции в различных экономических расчетах. Приведение к сопоставимым ценам. Сравнения средней заработной платы, пенсий, стипендий в различные моменты времени. Реальные результаты индексирования заработной платы и пенсий. Расчет прожиточного минимума, его динамика. Учет индекса инфляции при расчете реального значения процента на вклад в банк, реального процента платы за кредит. Как процент платы за кредит может быть отрицательным? Оценка динамики курса доллара США в России с помощью потребительских паритетов, сравнение с Китаем. Применение индекса инфляции при оценке стоимости основных и оборотных фондов. Опыт прогнозирования индекса инфляции и стоимости потребительской корзины. Применение индекса инфляции при организации производства. Конверсия, индекс инфляции и сохранность технологических цепочек (на примере специальной техники). Динамика макроэкономических показателей России. Использование индекса инфляции при оценивание

    экономического положения предприятий, при финансовом анализе результатов их деятельности.

    3. Прикладная статистика и эконометрические методы

    Области современных статистических методов: статистика чисел (случайных величин), статистика векторов (многомерный статистический анализ), статистика функций (случайных процессов и временных рядов), статистика нечисловых данных. Современная прикладная статистика и эконометрика. Пять точек роста эконометрических методов: непараметрическая статистика, устойчивость статистических процедур, размножение выборок, статистика нечисловых данных, статистика интервальных данных.

    Объем знаний, которым могут овладеть студенты в вузе, ограничен. Перед организаторами образования постоянно стоит проблема — что включать в учебную программу, а что не включать. Эта проблема решается проще, если известно будущее место работы студентов, т.е. подготовка идет для нужд конкретного предприятия. Представляется очевидным, что студент должен владеть современными методами своей специальности. Следовательно, постоянно должна вестись методическая работа по внедрению современных идей, концепций, подходов, методов, расчетных алгоритмов в учебный процесс (этот раздел методики преподавания называют онтодидактикой). Однако онтодидактические работы предполагают хорошее знакомство с современным уровнем знаний и практического применения, личную работу преподавателя на переднем крае науки и постоянное участие в прикладных исследованиях. Кроме того, приходится постоянно пересматривать само содержание преподавания, модернизировать методическое обеспечение — учебники, учебные пособия, задачники, методические разработки по отдельным темам, лабораторные работы, тематики курсовых и дипломных работ.

    Это нелегко. Поэтому вполне естественно, что наблюдается тенденция к консервации. Она выгодна для преподавателя — однажды разработанный курс и его методическое обеспечение используется (с небольшими вариациями) десятилетиями. Она удобна для студентов — те пользуются учебниками, которые за несколько изданий доведены до возможного совершенства, обеспечены заблаговременно изданными задачниками, лабораторными работами и пр. Такая тенденция к консервации обычно наблюдается у курсов «Статистика» (она же «Общая теория статистики») и «Теория вероятностей и математическая статистика». Как следствие консервации, студенты получают знания на уровне 40-60-х годов ХХ в., т.е. с запозданием на 50-70 лет. Наивно думать, что им будет легко самостоятельно осознать и тем более овладеть тем, что возникло за последние полвека.

    При ориентации на современные методы есть свои опасности. Можно неправильно оценить значимость тех или иных научных направлений, пропустить перспективные или, что еще хуже, посвятить много времени тем, чье время быстро пройдет. Однако предшествующие эконометрике курсы «Статистика» и «Теория вероятностей и математическая статистика», как правило, настолько далеки от современности, что за основу выбора можно взять развитие эконометрических идей в 70-90-х годах ХХ в.

    И преподавателю, и студенту труднее при ориентации на курсы с современной тематикой, чем на консервативные. Однако труд студента оправдывается при выходе на самостоятельное поприще по окончании вуза. Автор настоящей статьи более 45 лет разрабатывал и применял эконометрические методы в различных отраслях народного хозяйства и областях науки и смеет надеяться, что опыт научно-прикладной деятельности позволил разработать и внедрить на кафедре «Экономика и организация производства» факультета «Инженерный бизнес и

    менеджмент» Московского государственного технического университета им. Н.Э. Баумана современный курс эконометрики (см. [2]).

    Пользуясь тем, что студенты уже изучили курс «Теория вероятностей и математическая статистика», в разделе 3 даем представление о современной прикладной статистике согласно [3]. Первые три области современных статистических методов: — статистика чисел (случайных величин), статистика векторов (многомерный статистический анализ), статистика функций (случайных процессов и временных рядов) являются достаточно традиционными, хотя в предваряющих эконометрику курсах о них говорится явно недостаточно, а иногда не так и не то. Статистика нечисловых данных как самостоятельная область возникла в 1970-е годы и до сих пор продолжает активно развиваться, поэтому она выделена как одна из точек роста эконометрических методов.

    В соответствии со сказанным выше о необходимости показа студентам современного фронта эконометрических исследований предлагается кратко охарактеризовать основные точки роста, т.е. направления, которые в настоящее время активно развиваются, полезны для практики, но недостаточно представлены в стандартных учебниках. Первой из таких точек указаны непараметрические методы. Хотя они имеют длинную историю, начинающуюся с коэффициентов ранговой корреляции Кендалла и Спирмена начала века и статистик Колмогорова, Смирнова, омега-квадрат тридцатых годов, в преподавании они оттеснены на задний план параметрической статистикой. Почему это произошло? Видимо, тем, что параметрическая статистика, изучающая выборки из семейств распределений, описываемых одним-двумя, максимум тремя числовыми параметрами, имеет дело со сравнительно простым с математической точки зрения объектом. Именно из-за его простоты удалось построить достаточно глубокую, но доступную студентам теорию. Вместе с тем хорошо известно, что распределения реальных данных почти

    никогда не удается адекватно описать с помощью какого-либо параметрического семейства. В частности, распределения погрешностей измерений, вопреки распространенному заблуждению, почти всегда отличны от нормальных [4]. Таким образом, анализ данных с помощью параметрических методов статистики напоминает поиск ключей под фонарем, где светлее, а не в кустах, где они потеряны.

    В настоящее время непараметрическими методами можно решать тот же круг задач, что и параметрическими. При этом вместе неоправданных предположений о принадлежности функций распределения тому или иному параметрическому семейству делаются лишь общие предположения о непрерывности функций распределения. Из общих соображений можно ожидать, что такое расширение области применения может привести к снижению эффективности непараметрических эконометрических методов по сравнению с параметрическими. Однако во многих случаях непараметрические методы оказываются не хуже параметрических, особенно при больших объемах выборок. Иногда, тем не менее, параметрические методы представляются конкурентоспособными. Тогда необходимо изучить устойчивость эконометрических выводов по отношению к допустимым отклонениям исходных данных и предпосылок модели. Общая схема изучения устойчивости эконометрических и экономико-математических методов и моделей дана в [5]. Кроме термина «устойчивость», в данном контексте используются термины «робастность» (от robust — крепкий, грубый (англ.)), чувствительность. Можно выделить два этапа исследования — анализ устойчивости известных процедур, затем поиск новых наиболее устойчивых и вместе с тем достаточно точных. Иногда параметрические и непараметрические методы дают практически одни и те же алгоритмы, особенно при больших объемах выборок. Например, правила доверительного оценивания математического ожидания (при неизвестной дисперсии) в параметрическом случае

    нормального распределения и в непараметрическом случае произвольного распределения (правило строится на основе центральной предельной теоремы теории вероятностей и теорем о наследовании сходимости [6]) отличаются только тем, что в первом случае используются процентные точки распределения Стьюдента, а во втором — процентные точки стандартного распределения. Как известно, при росте объема выборки первые стремятся ко вторым. Однако иногда изучение устойчивости приводит к неожиданным выводам. Так, анализ проблемы обнаружения резко выделяющихся наблюдений (выбросов) привел к выводу о крайней неустойчивости классических методов: если зафиксировать процентную точку (т.е. правило принятия решений), то велик интервал изменения уровня значимости, если же, наоборот, исходить из заданного уровня значимости, то весьма велик интервал для границы, задающей правил отбраковки [4]. Следовательно, правила отбраковки не имеют научного обоснования, их следует рассматривать как эвристические.

    Общеинженерным принципом является указание не только точечного значения измеряемой величины, но и погрешности этого измерения. В экономике и менеджменте, к сожалению, не всегда указывают величины возможных погрешностей измерения. Требование изучения устойчивости статистических процедур нацелено на внедрение указанного общеинженерного принципа в область экономики, инженерного бизнеса и менеджмента. Одним из подходов является статистика интервальных данных (СИД). В ней прослеживается, как погрешности исходных данных, влияют на погрешности выводов, а для этого и исходные данные, и выводы приходится описывать с помощью интервалов, а не чисел, как в классической статистике. СИД активно развивается с 1980-90-х гг., основные ее идеи доступны студентам, поэтому элементы СИД включены в программу по эконометрике.

    Другой подход к изучению устойчивости статистических (эконометрических) выводов предполагает интенсивное использование современной вычислительной техники и основан на «размножении выборок». По выборке можно рассчитать одно число — точечную оценку характеристики или параметра, но этого мало — надо иметь доверительный интервал, т.е. нижнюю и верхнюю границы, между которыми с заданной вероятностью находится истинное значение. Параметрическая теория позволяет это сделать, но ее предпосылки неприемлемы. Поэтому вполне естественным является желание «размножить выборку» — вместо одной получить много похожих, по каждой из них рассчитать оценку, а по совокупности оценок построить распределение оценки, указать доверительный интервал. Метод размножения может быть очень простым: из одной выборки объема п можно получить п выборок объема (п-1), если исключать из исходной выборки последовательно по одному элементу (возвращая ранее исключенные обратно). Есть и много иных методов размножения выборок.

    4. Выборочные исследования (анализ результатов опросов)

    Польза и необходимость выборочных эконометрических исследований. Анкетное исследование (на примере маркетингового исследования потребителей растворимого кофе). Различные виды формулировок вопросов (открытый, закрытый, полузакрытый вопросы), их достоинства и недостатки. Техника интервью. Экономика опросов. Биномиальная и гипергеометрическая модели выборки, их близость в случае большого объема генеральной совокупности по сравнению с выборкой. Асимптотическое распределение выборочной доли (в случае ответов типа «да»-«нет»). Интервальное оценивание доли и метод проверки статистической гипотезы о равенстве долей (на основе теоремы Муавра-Лапласа и таблиц ВЦИОМ).

    Изучение конкретных эконометрических методов естественно начать с методов выборочных опросов, практическая польза которых очевидна (разбираем маркетинговое исследование, выполненное по заказу конкретной фирмы), а математическая сложность невелика. Естественным образом возникает сюжет о проверке однородности (другими словами. обнаружении различий), который продолжается в следующей теме.

    5. Элементы эконометрики чисел

    Определения нормального и логарифмически нормального распределений и их плотностей. Центральная предельная теорема в аддитивном и мультипликативном случае. Методы оценивание параметров логарифмически нормального распределения. Логарифмически нормальное распределение доходов (заработной платы) и «данные» Госкомстата РФ. Непараметрическое оценивание характеристик распределений и доверительных границ для них. Непараметрические методы оценивания плотности.

    Тема начинается с сопоставления нормального и логарифмически нормального распределений. Первое из них иногда допустимо использовать при решении технических задач. В эконометрике его использовать нельзя, поскольку плотность нормального распределения всюду положительна, т.е. вероятность отрицательных значений, а в эконометрике обычно рассматриваются неотрицательны величины. Основная причина популярности нормального распределения в эконометрике — центральная предельная теорема теории вероятностей, в которой речь идет о суммах большого числа независимых случайных величин. Если же вместо сумм имеем дело с произведениями, то вместо нормального распределения получаем логарифмически нормальное, сосредоточенное на положительной полуоси. Им можно приближать распределения различных экономических величин, например, доходов

    населения. В Росстате настолько верят в законность такого приближения, что вместо эмпирических величин публикуют лишь расчетные значения.

    При рассмотрении распределения доходов выявляется целесообразность рассмотрения различных видов средних -математического ожидания, медианы, моды. Непараметрическую статистику начинаем с непараметрического точечного и доверительного оценивания таких характеристик распределения, как математическое ожидание, медиана, дисперсия, среднее квадратическое отклонение, коэффициент вариации. Термин «непараметрическое оценивание» означает, что предположения о принадлежности распределения выборки к какому-либо параметрическому семейству отсутствуют, но предполагается существование моментов нужного порядка. С практической точки зрения это всегда выполнено, поскольку реальные распределения всегда сосредоточены на каком-то отрезке (другими словами, финитны).

    Завершается раздел рассмотрением непараметрических ядерных оценок плотности распределения, использование которых на базе современной вычислительной техники позволит существенно сузить область применения гистограмм — классического способа наглядного представления данных, обладающего неустранимым недостатками -отсутствием обоснования для выбора числа интервалов и потерей информации из-за группирования.

    6. Методы статистической проверки гипотез однородности

    Проблема проверки однородности двух выборок (независимых и связанных). Различные формулировки гипотезы однородности двух выборок. Критерий Крамера-Уэлча для проверки равенства математических ожиданий (на основе Центральной предельной теоремы и теоремы о наследовании сходимости).

    Эмпирическая функция распределения и основанные на ней непараметрические одновыборочные статистические критерии А.Н.Колмогорова, Н.В.Смирнова, омега-квадрат (Крамера-Мизеса-Смирнова). Проверка гипотезы согласия с параметрическим семейством распределений и распространенная ошибка при использовании критериев Колмогорова и омега-квадрат.

    Двухвыборочный критерий Вилкоксона (Манна-Уитни) и его асимптотическая нормальность. Достигаемый уровень значимости. Асимптотическое распределение критерия Вилкоксона при справедливости альтернативной гипотезы и его асимптотическая мощность. Какие выводы можно сделать на основе критерия Вилкоксона? Альтернатива сдвига. Двухвыборочные критерии Смирнова и типа омега-квадрат (Лемана-Розенблатта). Задача обнаружения различия в связанных выборках (письмо главного инженера Рошальского химического комбината). Критерий знаков. Проверка равенства 0 математического ожидания. Одновыборочный критерий Вилкоксона. Проверка симметрии функции распределения относительно 0 с помощью критерия типа омега-квадрат.

    Эта тема — одна из примыкающих к классической тематике математической статистики. Однако различия видны с самого начала. Для проверки равенства математических ожиданий двух выборок используем критерий Крамера-Уэлча. Любопытно, что ни в одном «классическом» пособии по математической статистики нельзя найти доказательства асимптотической нормальности этого критерия. Для доказательства (которое в курсе опущено) приходится ссылаться на теорему о наследовании сходимости [6]. Необходимо обратить внимание на отличие критерия Крамера-Уэлча от критерия Стьюдента, который предпочитают рассматривать в курсах математической статистики. Критерий Стьюдента опирается на две предпосылки — нормальность распределения обеих

    выборок и равенства дисперсий у них. Ни одна из предпосылок обычно не выполняется, особенно для экономических данных.

    Проверку согласия с параметрическим семейством с помощью критериев Колмогорова и омега-квадрат любят включать в учебники по «Общей теории статистики», причем обычно делают это с ошибками, о чем подробно написано в статье [7]. Поэтому уклониться от разбора этого вопроса нельзя. Много недоразумений связано и с двухвыборочным критерием Вилкоксона, самым популярным в переводной литературе (разбору неточных утверждений, связанных со свойствами этого критерия, посвящена статья [8]).

    Необходимость решения вопроса об однородности в связанных выборках естественно начать с практической задачи, например, с письма главного инженера Рошальского химического комбината, в котором тот просит установить, есть ли различия в показаниях двух вискозиметров, используемых при производстве мастики. От решения этого вопроса зависит, включать ли в соответствующий нормативный документ указание на средство измерения вязкости или в этом нет необходимости.

    7. Понятие о регрессионном анализе

    Метод наименьших квадратов для линейной прогностической функции. Вид расчетной таблицы. Оценивание параметров. Критерий правильности расчетов. Точечный и интервальный прогноз. Изменение ширины доверительного интервала при увеличении горизонта прогнозирования. Метод наименьших квадратов в случае линейной функции двух переменных. Вид расчетной таблицы. Оценивание параметров. Критерий правильности расчетов. Более общие варианты метода наименьших квадратов. Модель, линейная по параметрам. Преобразования переменных. Оценивание многочлена. Оценка остаточной

    дисперсии как показатель качества модели. Оценивание размерности модели. Непараметрическая регрессия.

    Регрессионный анализ — одна из наиболее разработанных частей эконометрики. Имеются толстые монографии, посвященные отдельным направлениям регрессионного анализа. На основе регрессионного анализа построена теория планирования эксперимента, дающая большой экономический эффект. Методы восстановления зависимостей на основе наименьших квадратов и наименьших модулей, модели линейной и нелинейной (по параметрам) регрессии, оценивание необходимой степени полинома, различные варианты непараметрической регрессии, дисперсионный анализ, многочисленные модели планирования эксперимента, в том числе экстремального — весь этот перечень тем заслуживает того, чтобы студенты их изучали. Для этого необходимо увеличение продолжительности преподавания эконометрики, переход от одного к системе курсов, как это сделано, например, в Высшей школе экономики — «Эконометрика-1», «Эконометрика-2», «Эконометрика-3» …

    Необходимо рассчитывать не только точечные оценки параметров, но и доверительные границы для прогностической функции. Подробно разбираются методы расчета для линейной регрессии с одной независимой переменной. В то же время вероятностной теории уделяется меньше внимания, хотя и дается общее понятие регрессии как условного математического ожидания, описывается ее непараметрическая оценка.

    8. Эконометрические методы классификации

    Классификация и прогнозирование. Триада: построение классификаций — анализ классификаций — использование классификаций. Лемма Неймана-Пирсона и непараметрический дискриминантный анализ на основе непараметрических оценок плотности в пространствах произвольной природы. Линейный дискриминантный анализ Р.Фишера.

    Многообразие параметрических и непараметрические методов классификации (распознавания образов). Группировки и кластер-анализ. Методы оценки качества алгоритмов классификации.

    Хотя теория классификации несколько менее популярна среди научно-технических работников, количество посвященных ей работ измеряется десятками тысяч. Активно работавшая в 1980-е годы Комиссия ВСНТО по классификации охватывала около 1000 специалистов. Однако классификация как область эконометрики еще не достигла достаточной внутренней стройности. Слишком много отдельных подходов и методов, не связанных друг с другом. О том же свидетельствует и разнобой в терминологии. Построение классификаций называют также распознаванием образов без учителя, автоматической классификацией, кластерным анализом. Использование классификаций — дискриминантным анализом, диагностикой, распознаванием образов с учителем. Тем не менее представляется целесообразным познакомить студентов с основными идеями в области классификации, с алгоритмами кластер-анализа (прежде всего иерархическими агломеративными алгоритмами «ближнего соседа», средней связи, «дальнего соседа»), параметрическими (линейный дискриминантный анализ Р. Фишера) и непараметрическими (на основе непараметрических оценок плотностей, построенных по обучающим выборкам) алгоритмами использования классификаций. Используется также теория принятия статистических решений (при известной матрице потерь из-за ошибочной классификации). Анализ классификаций (разбиений) рассматривается позже, как одна из задач статистики объектов нечисловой природы. Важное место в теме занимают методы оценки качества алгоритмов классификации, в том числе метод пересчета на модель линейного дискриминантного анализа, позволяющий проверять обоснованность использования линейных прогностических индексов. Тема связана как с курсом «Теория вероятностей и математическая статистика»

    (лемма Неймана-Пирсона), так и с курсом «Статистика» (построение группировок).

    Другие многомерные эконометрические методы (многомерное шкалирование, целенаправленное проецирование, метод главных компонент, факторный анализ, канонические корреляции, анализ структуры связей и др.) целесообразно рассмотреть при условии увеличения объема преподавания эконометрики.

    9. Современная теория измерений

    Шкалы наименований, порядка (ранговая), интервалов, отношений, абсолютная. Проблема адекватности эконометрического вывода. Средние величины, результат сравнения которых инвариантен относительно допустимых преобразований шкалы. Применения к расчету рейтингов.

    Рассматриваются основы (репрезентативной) теории измерений: определения, примеры, группы допустимых преобразований для основных типов шкал (наименований, порядка, интервалов, отношений, абсолютной). В качестве основного для эконометрики выдвигается требование устойчивости статистических выводов относительно допустимых преобразований шкал. Сравниваются три вида средних (среднее арифметическое, мода, медиана) для зарплаты работников предприятия. Дается определение средних по Коши. Обсуждается «теорема о медиане» — описание средних, результат сравнения которых устойчив в порядковой шкале. Рассматриваются применения «теоремы о медиане» к рейтингам. Вводятся ассоциативные средние по Колмогорову и дается описание средних, результат сравнения которых устойчив в шкалах интервалов и отношений. Рассматриваются иные применения теории измерений к выбору адекватных методов анализа экономических данных.

    10. Статистика нечисловых данных

    Различные виды нечисловых данных, связи между ними. Люсианы. Нечеткие множества и их связь со случайными. Метрики (показатели различия), эмпирические и теоретические средние, медиана Кемени, асимптотическое поведение решений экстремальных статистических задач, законы больших чисел непараметрические оценки плотности в произвольных пространствах.

    В 1970-х годах стала очевидной большая роль в эконометрике экономических и управленческих данных ранее слабо изучавшихся видов -нечисловых данных, или объектов нечисловой природы. В литературе имеется достаточно подробное описание различных пространств нечисловых данных, а также связей между ними [9]. К нечисловым данным относятся, как уже говорилось, результаты измерений по качественным шкалам (в шкалах наименований и порядка), бинарные отношения (ранжировки (упорядочения), разбиения (отношения эквивалентности), толерантности), последовательности из 0 и 1, множества, нечеткие множества и др. Основные связи между перечисленными видами нечисловых данных были установлены еще в 1970-е годы. Позже была развита, например, теория люсианов (конечных последовательностей испытаний Бернулли с, вообще говоря, различными вероятностями успеха, дающая вероятностную основу для анализа последовательностей из 0 и 1.

    Рассматриваются основы теории нечеткости. Даются определения нечетких множеств и операций над ними. Разбираются примеры нечетких множеств, в частности, нечеткие ответы экспертов, и свойства операций над нечеткими множествами. Анализируется связь нечетких множеств со случайными, позволяющая свести теорию нечеткости к теории случайных множеств и тем самым к теории вероятностей.

    Вводятся метрики (показатели различия) в пространствах произвольной природы — основа методов статистики нечисловых данных. Дан оптимизационный подход к определению эмпирических и теоретических средних в пространствах произвольной природы, проведено сравнение со свойствами среднего арифметического, математического ожидания, теоретической и выборочной медианы. Эмпирическое среднее предлагается рассматривать как агрегирование мнений экспертов. Обсуждается формулировка законов больших чисел в пространствах произвольной природы.

    В качестве эконометрических данных рассматриваются бинарные отношения (ранжировки, разбиения, толерантности). Разбирается их связь с матрицами из 0 и 1 и введение расстояния Кемени между бинарными отношениями. Изучается медиана Кемени, ее асимптотика и свойства при малых объемах выборок и различных предположениях о распределении ранжировок. Вводятся изотропные распределения и устанавливается единственность среднего (медианы). Интерпретация законов больших чисел для нечисловых данных может быть дана в терминах теории экспертного опроса. Устанавливается связь метода средних рангов с коэффициентом ранговой корреляции Спирмена и линейная зависимость расстояния Кемени от коэффициента ранговой корреляции Кендалла. Разбирается метод «идеальной точки» с использованием средних рангов на примере сравнения математических моделей испарения жидкости.

    Вводятся расстояния, теоретические и эмпирические средние в пространстве подмножеств конечного множества. Построение эмпирического среднего (итогового мнения комиссии экспертов) проводится по правилу большинства. Вводятся различные расстояния между нечеткими множествами и применяются для усреднение нечетких ответов экспертов.

    Полученные результаты применяются для изучения асимптотического поведения решений экстремальных статистических задач. Предлагается использовать непараметрические оценки плотности в пространствах произвольной природы, в частности, для дискретных пространств. Обсуждается применение статистики объектов нечисловой природы при построения новой хронологии и значение полученных выводов для современных социально-экономических проблем.

    11. Статистика интервальных данных

    Погрешности измерения и интервальные данные. Нотна -максимально возможное отклонение, вызванное интервальностью статистических данных. Рациональный объем выборки. Их расчет для ряда задач оценивания, проверки гипотез, регрессионного, кластерного и дискриминантного анализов.

    Необходимость учета в эконометрических методах погрешностей измерения, как уже обсуждалось выше, приводит к введению в теорию и практику статистического анализа интервальных данных. Определяются операции над интервальными числами и дается обоснование правил приближенных вычислений. Сравниваются по точности две формулы для выборочной дисперсии. Обсуждается основная модель интервальной статистики. Вводятся основополагающие понятия нотны — максимально возможного отклонения, вызванного интервальностью статистических данных, и асимптотической нотны (при малой абсолютной погрешности). Дается алгоритм расчета асимптотической нотны для квадратичных функций второго порядка. Изучение влияния интервальности дисконт-факторов на величину NPV (чистой приведенной стоимости, net present value) приводит к признанию интервальности самой величину NPV и необходимости использования экспертных оценок при оценке и сравнении инвестиционных проектов.

    Разбираются основные результаты статистики интервальных данных, в том числе второе основополагающее понятие — рациональный объем выборки. Проводится расчет асимптотической нотны, рационального объема выборки и доверительных интервалов при оценивании математического ожидания и дисперсии, а также расчет основных показателей статистики интервальных данных для ряда задач оценивания, проверки гипотез, регрессионного, кластерного и дискриминантного анализов.

    12. Проблема устойчивости статистических процедур по отношению к допустимым отклонениям исходных данных и предпосылок модели

    Робастные методы статистики. Общая схема устойчивости. Метод складного ножа Кенуя. Бутстреп Эфрона и его критика.. Размножение выборок как эффективный способ интенсивного применения вычислительной техники в эконометрике.

    Обсуждаются различные робастные методы и модели статистики и эконометрики, в том числе модель Тьюки-Хубера засорения экономических данных резко выделяющимися наблюдениями и модель Ю.Н. Благовещенского отклонений, имеющих быть на всей числовой оси. Согласно [5] дается формулировка общей схемы изучения устойчивости и разбираются примеры ее применения.

    Рассматриваются методы размножения выборок (различные варианты бутстрепа): метод складного ножа Кенуя, бутстреп Эфрона [31] (и его критика). Размножение выборок выдвигается как эффективный способ интенсивного применения информационно-коммуникационных технологий в эконометрике. Даны различные варианты применения метода размножения выборок в эконометрических исследованиях.

    13. Эконометрические методы экспертных исследований

    Примеры и основные этапы применения методов экспертных оценок. Формирование экспертной комиссии. Метод «снежного кома». Различные виды экспертных процедур (с взаимодействием или без, одно- и многотуровые и др.). Метод средних рангов и метод медиан. Согласование кластеризованных ранжировок. Применение коэффициентов ранговой корреляции, теории люсианов, медианы Кемени и иных методов статистики нечисловых данных.

    Обосновывается необходимость проведения экспертных исследований, особенно в ситуациях быстрого изменения, когда нет возможности опираться на длинные временные ряды статистических данных. Основными представлениями о теории и практике экспертного оценивания должен владеть каждый инженер, экономист, менеджер. Примерами процедур экспертных оценок являются методы оценки техники и артистичности фигуристов, успешности команд КВН, целесообразности финансирования научно-технических проектов, анализируемых в Республиканском исследовательском научно-консультативном центре экспертизы (РИНКЦЭ) на основе методических документов РИНКЦЭ. Они используются на соревнованиях, при выборе, распределении финансирования, при экологических экспертизах, в частности, согласно Закону РФ «Об экологических экспертизах». Другие методы экспертных исследований: метод Дельфи, мозговой штурм, метод средних рангов, метод медиан, метод сценариев.

    Планирование и организация экспертного исследования как один из видов деятельности менеджера. Сравнение ролей лиц, принимающих решения (ЛПР), и специалистов (экспертов) в процедурах экспертиз и принятия решений. Зоны ответственности Рабочей Группы и Экспертной Комиссии. Основные этапы проведения экспертного исследования как основа деятельности менеджера, организующего экспертное исследование.

    Экономические вопросы проведения экспертного исследования — часть знаний менеджера.

    Цели экспертного исследования можно поставить по-разному: сбор информации для ЛПР и/или подготовка проекта решения для ЛПР и др. Используется ряд методов формирования состава экспертной комиссии: методы списков (реестров),» снежного кома», самооценки, взаимооценки. Важна проблема априорных предпочтений экспертов. Достоинства и недостатки процедур, используемых при отборе экспертов, заслуживают подробного обсуждения, равно как и. различные варианты организации экспертного исследования, различающиеся по числу туров (один, несколько, не фиксировано), порядку вовлечения экспертов (одновременно, последовательно), способу учета мнений (с весами, без весов), организации общения экспертов (без общения, заочное, очное с ограничениями («мозговой штурм») или без ограничений).

    Методы анализа экспертных оценок основаны на статистике нечисловых данных, а также на применении непараметрической статистики (коэффициентов ранговой корреляции). Бинарные отношения на конечном множестве часто возникают как способы описания ответы экспертов. Бинарные отношения находятся во взаимно-однозначном соответствии с матрицами из 0 и 1. При анализе мнений экспертов важны такие свойства бинарных отношений, как рефлексивность, симметричность, транзитивность, и соответствующие им подпространства бинарных отношений — подпространства ранжировок (упорядочений), разбиений (отношений эквивалентности), толерантностей. Аксиоматическое введение расстояния Кемени между бинарными отношениями дает возможность применить методы статистики объектов нечисловой природы для анализа ответов экспертов. Применение теории люсианов, вычисление медианы Кемени и использование иных методов статистики нечисловых и интервальных данных — основа работы

    специалиста по анализу экспертных данных. Согласование кластеризованных ранжировок рассматриваем на примере сравнения 8 математических моделей испарения жидкости по экспериментальным данным (при создании банка математических моделей с целью оценки последствий аварий при экологическом страховании).

    Обобщенный показатель (полезность) объекта экспертизы строится как функция частных показателей. Разработан ряд методов построения обобщенного показателя. В частности, есть два подхода к определению весовых коэффициентов линейной функции полезности. Один из них -линейная свертка с коэффициентами, которые оценивают эксперты. Критику такого подхода даем на основе анализа реальных предложений по процедуре выбора технологии уничтожения химического оружия. Недостатки экспертных методов непосредственного определения коэффициентов весомости имеют причиной то, что эксперту свойственно работать в порядковой шкале, а не в количественных шкалах. Рассматривается экспертно-статистический метод и его реализация с помощью метода наименьших квадратов.

    14. Эконометрические методы управления качеством

    Статистический приемочный контроль по альтернативному признаку. Планы контроля. Оперативная характеристика. Приемочный и браковочный уровни дефектности. Предел среднего выходного уровня дефектности. Применение Центральной предельной теоремы теории вероятностей. Статистический приемочный контроль по количественному признаку. Применение статистики люсианов. Всегда ли нужен выходной контроль? Контрольные карты Шухарта и кумулятивных сумм.

    В договорах купли-продажи, создания научно-технической продукции и иных договорах между предприятиями практически всегда имеется

    раздел «Правила приемки и методы контроля». Эти разделы, как и правила сертификации, часто основаны на статистическом приемочном контроле. Под ним понимают выборочный контроль, основанный на эконометрической теории. Его необходимость связана с применением разрушающих методов контроля и с экономической эффективностью, основанной на сокращении затрат на контроль (в машиностроении они составляют в среднем 10% от стоимости продукции).

    Планы контроля по альтернативному признаку разнообразны. Наиболее простым является одноступенчатый контроль. Основной инструмент анализа и синтеза планов контроля — это оперативная характеристика, т.е.функция, определяющая вероятность приемки партии в зависимости от входного уровня дефектности. С помощью оперативной характеристики определяются приемочный и браковочный уровни дефектности. соответствующие заданным рискам поставщика и потребителя. Расчеты наиболее просты для плана (п,0).

    Если применяется контроль с разбраковкой, то находят средний выходной уровень дефектности и его предел (ПСВУД), т.е. максимально возможное значение. Нетрудно рассчитать ПСВУД для плана (п,0). Выбор плана контроля на основе ПСВУД целесообразен в ситуации, когда потребитель должен быть защищен от проникновения в поставляемую ему партию продукции доли брака, превышающей заданную.

    Если поставщик и потребитель по-разному оценивают качества продукции в партии, поступившей от поставщика к потребителю, то возникает арбитражная ситуация. Арбитражная характеристика задает вероятность возникновения арбитражной ситуации. Для уменьшения числа споров применяют принцип распределения приоритетов. Расчет планов контроля поставщика и потребителя на основе принципа распределения приоритетов отличается тем, что приемлемый уровень качества выступает

    для поставщика в качестве браковочного, а для потребителя — в качестве приемочного.

    При достаточно большом объеме выборки (несколько десятков единиц продукции) расчет приемочного и браковочного уровней дефектности для одноступенчатого плана с помощью теоремы Муавра-Лапласа теории вероятностей. Выбор одноступенчатого плана контроля по заданным приемочным и браковочным уровням дефектности также может быть проведен на основе асимптотических соотношений, вытекающих из теоремы Муавра-Лапласа.

    Ограниченность возможности использования экономических показателей при статистическом контроле основана на том, что последствия отказов изделий типа гибели людей нельзя по этическим соображениям описывать в экономических терминах. Поэтому математические модели типа модели Хальда, учитывающие все виды затрат — затраты на контроль, убытки от излишнего забракования и потери от пропуска брака, — остаются теоретическими.

    Основной парадокс теории статистического контроля состоит в том, что чем лучше качество и меньше дефектность, тем больший объем контроля требуется. Поэтому естественно поставить вопрос: всегда ли нужен выходной контроль? Сравнение экономической эффективности сплошного контроля и увеличения объема партии; сплошного контроля и замены дефектных единиц продукции в системе гарантийного обслуживания приводит к выводу о целесообразности во многих случаях отказаться от выходного контроля.

    Обычно выделяют следующие виды статистических методов управления качеством (обеспечения, повышения качества): статистический анализ точности и стабильности технологических процессов (методами прикладной статистики), статистический приемочный контроль по альтернативному и количественному признаку., статистическое

    регулирование технологических процессов (контрольные карты Шухарта и кумулятивных сумм), планирование экспериментов (в том числе экстремальное, с целью добиться максимального выхода полезного продукта), надежность и испытания. При контроле по нескольким альтернативным признакам полезно применение статистики люсианов -одного из разделов статистики объектов нечисловой природы. Используют наглядные диаграммы Парето и диаграммы причин и результатов (известные также как диаграммы Исикавы или «рыбий скелет»).

    Проблемы управления качеством рассматриваются не только на уровне предприятия, но и на отраслевом, государственном и межгосударственном уровнях. Можно указать на проблемы сертификации производства (отдельной партии продукции, конкретного технологического процесса, всего производства в целом) согласно международным стандартам серии ИСО 9000. На всех одиннадцати этапах жизненного цикла продукции (по международному стандарту ИСО 900487). необходимо использование эконометрических методов. В настоящее время переходим от систем контроля качества, основанных на допусках (системы Тейлора), к основанным на функциях потерь (системы Тагути).

    Организационные вопросы управления качеством делятся на вопросы, решаемые государством (стандартизация, аттестация, сертификация), и вопросы, решаемые общественностью (кружки качества, защита прав потребителей). Развиваются комплексные системы управления качеством продукции, используется серия международных стандартов ИСО серии 9000 и лозунг тотального (всеобщего) управление качеством, система «Шесть сигм». Статистические методы управления качеством могут использоваться также при решении экологических задач мониторинга, контроля, управления, других экологических проблем на предприятиях и в регионах, при аудите (контроле массива документов), мониторинге социально-экономического положения и в других областях.

    15. Анализ и прогноз временных рядов

    Методы выделения трендов. Спектральный анализ. Оценивание периода. Модели авторегрессии. Системы эконометрических уравнений.

    Применяют методы восстановления временных зависимостей на основе наименьших квадратов и наименьших модулей. Среди них важное место занимают модели линейной (по параметрам) регрессии. Большое значение приобретает задача оценивание необходимой степени полинома. Полезны модели авторегрессии, в том числе простейшая эмпирическая модель экспоненциального сглаживания. Оценка длины периода может быть сделана на основе методов статистики объектов нечисловой природы путем минимизации в функциональном пространстве. Выделение циклов во временных рядах имеет давнюю историю.. Рассматриваются системы эконометрических моделей и примеры их практического применения.

    16. Эконометрика прогнозирования и риска

    Статистические и экспертные прогнозы. Неопределенности и риски, их моделирование в эконометрике. Диверсификация и страхование. Принятие решений в условиях неопределенности. Применение экспертных оценок при оценке и сравнении инвестиционных проектов.

    Среди пяти основных функцией менеджмента первая -прогнозирование и планирование. Необходимость для менеджера владеть различными методиками прогнозирования не вызывает сомнения. Из различных видов прогнозов выделим статистические и экспертные, качественные и количественные. К частным видам прогнозирования относятся прогнозы невозможности, самоосуществляющиеся прогнозы. Прогнозирование является основой планирования.

    Вопрос о целях предприятия непрост. Неопределенность выражения «максимизация прибыли» без указания интервала времени показывает это.

    Полезно прогнозирование экономической (и не только!) ситуации методом сценариев. Выделены этапы процесса планирования работы отрасли и предприятия, практически на всех нужны те или иные прогнозы.. Часто используется иерархия: миссия и ценности -стратегические цели — задачи — конкретные задания, что соответствует этапам планирования, основанного на прогнозировании: разработка стратегии, бизнес планирование и оперативное планирование.

    Неопределенности и риски сопровождают менеджера постоянно. Различные виды рисков в работе предприятия перечислить трудно, но необходимо. Разработаны методы декомпозиции риска, в частности, используются деревья причин и результатов (диаграммы типа «рыбий скелет» — см. выше).

    Разработаны различные методы описания неопределенностей, прежде всего вероятностно-статистические (наиболее распространенные), основанные на теории нечетких множеств и на статистике интервальных данных. Вероятностно-статистические методы описания риска широко используются, тесно связаны с теорией надежности, с вероятностным анализом безопасности( в атомной энергетике). Выделяют риск события (когда риск состоит в осуществлении нежелательного события) и риск ущерба (количественный). Есть ряд вариантов количественной оценки риска: по среднему ущербу (математическому ожиданию), по медиане ущерба, по квантилю, близкому к 1, по линейной комбинации среднего ущерба и среднего квадратического отклонения, по функции потерь и даже по дисперсии. Плата за риск может быть выявлена при наблюдении за поведением людей, которые обычно стремятся уменьшить риск, например, путем страхования. Плату за риск можно оценить, вычитая среднюю цену акций и цены гарантированных государством облигаций (в США). Один из приемов уменьшения риска — диверсификация деятельности, в том числе диверсификация при управлении пакетом ценных бумаг в условиях риска.

    Дается понятие о вероятностных моделях страхования, в том числе экологического.

    Имеются различные подходы к принятию решений в условиях неопределенности: подход пессимиста, основанный на максимизации выигрыша в наихудшей ситуации (это — подход теории антагонистических игр), подход оптимиста (максимизация выигрыша в наилучших условиях), подход на основе среднего выигрыша (возможно, дополненный оценкой доверительного интервала), подход, основанный на минимизации упущенной выгоды, и др. При несовпадении рекомендаций, даваемых различными подходами, необходимо применение оценок экспертов. В частности, имеются различные подходы к оценке эффективности и выбору инвестиционных проектов. С чисто финансовой точки зрения сравнение инвестиционных проектов сводится к сравнению потоков платежей. Очевидна необходимость изучения устойчивости (чувствительности) выводов по отношению к отклонениям коэффициентов дисконтирования и величин платежей. Столь же очевидна необходимость использования метода экспертных оценок в случае получения противоречивых рекомендаций после проведения анализа чувствительности, например, когда интервалы для используемых характеристик потоков платежей перекрываются.

    17. Заключение

    Подведем итоги. В настоящей статье продемонстрирована необходимость обучения будущих менеджеров, экономистов, инженеров эконометрическим методам. Рассмотрено место курса эконометрики в системе высшего технического образования: опираясь на курсы «Теория вероятностей и математическая статистика» и «Статистика», он призван довести знания студентов до уровня современности. Указаны связи курса эконометрики со многими иными учебными предметами — менеджментом,

    маркетингом, экологией, стандартизацией, метрологией и управлением качеством, инвестиционной, инновационной, контрольной и контроллинговой деятельностью, оценкой финансового состояния предприятия, прогнозированием и технико-экономическим планированием, экономико-математическим моделированием

    производственных систем и др.

    Разработано содержание основного курса эконометрики, который реализован в нескольких конкретных вариантах (многочисленные учебники представлены в [2]). Есть перспективы для его развертывания при увеличении количества часов. о чем подробнее сказано выше. Особенно важным представляется развертывание наиболее современных разделов эконометрики — статистики нечисловых и интервальных данных.

    Наряду с очевидными преимуществами ориентация курса эконометрики на последние научные достижения имеет свои отрицательные стороны, в частности, слабым является методическое обеспечение.

    Литература

    1. Орлов А.И. Современные эконометрические методы — интеллектуальные инструменты инженера, управленца и экономиста // Политематический сетевой электронный научный журнал Кубанского государственного аграрного университета. 2016. № 116. С. 484 — 514.

    2. Орлов А.И. Отечественная научная школа в области эконометрики / А.И. Орлов // Политематический сетевой электронный научный журнал Кубанского государственного аграрного университета (Научный журнал КубГАУ) [Электронный ресурс]. — Краснодар: КубГАУ, 2016. — №07(121). С. 235 — 261. — IDA [article ID]: 1211607006. — Режим доступа: http://ej .kubagro.ru/2016/07/pdf/06.pdf

    3. Орлов А.И. Прикладная статистика — состояние и перспективы // Политематический сетевой электронный научный журнал Кубанского государственного аграрного университета. 2016. № 119. С. 44-74.

    4. Орлов А.И. Распределения реальных статистических данных не являются нормальными // Политематический сетевой электронный научный журнал Кубанского государственного аграрного университета. 2016. № 117. С. 71-90.

    5. Орлов А.И. Новый подход к изучению устойчивости выводов в математических моделях // Политематический сетевой электронный научный журнал Кубанского государственного аграрного университета. 2014. № 100. С. 146-176.

    6. Орлов А.И. Теоретические инструменты статистических методов // Политематический сетевой электронный научный журнал Кубанского государственного аграрного университета. 2014. № 101. С. 253-274.

    7. Орлов А.И. Непараметрические критерии согласия Колмогорова, Смирнова, омега-квадрат и ошибки при их применении // Политематический сетевой электронный научный журнал Кубанского государственного аграрного университета. 2014. № 97. С. 32-45.

    8. Орлов А.И. Двухвыборочный критерий Вилкоксона — анализ двух мифов // Политематический сетевой электронный научный журнал Кубанского государственного аграрного университета. 2014. № 104. С. 91 — 111.

    9. Орлов А.И. О развитии статистики объектов нечисловой природы // Политематический сетевой электронный научный журнал Кубанского государственного аграрного университета. 2013. № 93. С. 41-50.

    References

    1. Orlov A.I. Sovremennye jekonometricheskie metody — intellektual’nye instrumenty inzhenera, upravlenca i jekonomista // Politematicheskij setevoj jelektronnyj nauchnyj zhurnal Kubanskogo gosudarstvennogo agrarnogo universiteta. 2016. № 116. S. 484

    — 514.

    2. Orlov A.I. Otechestvennaja nauchnaja shkola v oblasti jekonometriki / A.I. Orlov // Politematicheskij setevoj jelektronnyj nauchnyj zhurnal Kubanskogo gosudarstvennogo agrarnogo universiteta (Nauchnyj zhurnal KubGAU) [Jelektronnyj resurs].

    — Krasnodar: KubGAU, 2016. — №07(121). S. 235 — 261. — IDA [article ID]: 1211607006. -Rezhim dostupa: http://ej.kubagro.ru/2016/07/pdf/06.pdf

    3. Orlov A.I. Prikladnaja statistika — sostojanie i perspektivy // Politematicheskij setevoj jelektronnyj nauchnyj zhurnal Kubanskogo gosudarstvennogo agrarnogo universiteta. 2016. № 119. S. 44-74.

    4. Orlov A.I. Raspredelenija real’nyh statisticheskih dannyh ne javljajutsja normal’nymi // Politematicheskij setevoj jelektronnyj nauchnyj zhurnal Kubanskogo gosudarstvennogo agrarnogo universiteta. 2016. № 117. S. 71-90.

    5. Orlov A.I. Novyj podhod k izucheniju ustojchivosti vyvodov v matematicheskih modeljah // Politematicheskij setevoj jelektronnyj nauchnyj zhurnal Kubanskogo gosudarstvennogo agrarnogo universiteta. 2014. № 100. S. 146-176.

    6. Orlov A.I. Teoreticheskie instrumenty statisticheskih metodov // Politematicheskij setevoj jelektronnyj nauchnyj zhurnal Kubanskogo gosudarstvennogo agrarnogo universiteta. 2014. № 101. S. 253-274.

    7. Orlov A.I. Neparametricheskie kriterii soglasija Kolmogorova, Smirnova, omega-kvadrat i oshibki pri ih primenenii // Politematicheskij setevoj jelektronnyj nauchnyj zhurnal Kubanskogo gosudarstvennogo agrarnogo universiteta. 2014. № 97. S. 32-45.

    8. Orlov A.I. Dvuhvyborochnyj kriterij Vilkoksona — analiz dvuh mifov // Politematicheskij setevoj jelektronnyj nauchnyj zhurnal Kubanskogo gosudarstvennogo agrarnogo universiteta. 2014. № 104. S. 91 — 111.

    9. Orlov A.I. O razvitii statistiki ob#ektov nechislovoj prirody // Politematicheskij setevoj jelektronnyj nauchnyj zhurnal Kubanskogo gosudarstvennogo agrarnogo universiteta. 2013. № 93. S. 41-50.

    Что такое эконометрика? Типы, этапы и функции

    Недавно мы изучали методы маркетинга, и основной целью маркетинга является увеличение прибыли компании. Прежде чем тратить большие деньги, менеджер по маркетингу должен быть уверен в том, как будет работать кампания и как на нее отреагирует аудитория.

    В таких ситуациях эконометрика используется для определения взаимосвязи между маркетинговыми усилиями и продажами, например:

    • Какой дополнительный доход можно получить от дополнительных сотен рупий, потраченных на рекламу?

    • Какой вид рекламы (цифровая, телевизионная, газетная и т. Д.)) оказывает наибольшее влияние на продажи?

    Вопросы такого типа можно решить с помощью эконометрических методов.

    Введение в эконометрику

    Эконометрика — это количественное применение статистических выводов, экономической теории и математических моделей с использованием данных для разработки теорий или проверки существующих гипотез в экономике и для прогнозирования будущих тенденций на основе огромного количества данных, полученных с течением времени.

    Его функция заключается в преобразовании реальных данных в статистические исследования, а затем в сравнении результатов с теорией или теориями, которые проверяются на аналогичные закономерности.

    Другими словами, анализирует теоретические экономические модели и использует их для разработки экономической политики .

    Основная функция эконометрики — для преобразования качественных отчетов в количественные .

    Согласно книге Stock and Watson (2007), «Эконометрические методы используются во многих отраслях экономики, включая финансы, экономику труда, макроэкономику, микроэкономику и экономическую политику».

    Лоуренс Кляйн, Рагнар Фриш и Саймон Кузнец считаются пионерами эконометрики, а также получили Нобелевскую премию по экономике в 1971 г. за свой вклад. Сегодня он широко используется как учеными, так и практиками, такими как трейдеры и аналитики с Уолл-стрит.

    В зависимости от того, заинтересованы ли вы в проверке существующей теории или в использовании существующих данных для разработки новой гипотезы, основанной на этих выводах, эконометрику можно разделить на следующие категории: теоретическая и прикладная эконометрика.

    Типы эконометрики

    1. Теоретическая эконометрика

    Это исследование свойств существующих статистических моделей и процедур для определения неизвестных значений в модели.При этом мы стремимся разработать новые статистические процедуры, которые действительны, несмотря на то, что природа экономических данных может изменяться одновременно.

    Теоретическая эконометрика в значительной степени опирается на математику, теоретическую статистику и числовые величины, чтобы доказать, что новые процедуры способны делать правильные выводы .

    (Также проверьте: Статистический анализ данных)

    Теоретическая эконометрика фокусируется на таких вопросах, как общая линейная модель, модели одновременных уравнений, распределенные задержки и вспомогательные темы.Большинство из этих проблем возникло при работе над эмпирическими исследованиями.


    Типы эконометрики


    2. Прикладная эконометрика

    Это специальное использование эконометрических методов для преобразования качественных экономических отчетов в количественные, в отличие от теоретического подхода. Поскольку прикладные эконометристы приобретают более близкий опыт работы с данными, они часто сталкиваются с проблемами, связанными с атрибутами данных, которые указывают на ошибки в существующем наборе методов оценки, а также предупреждают их теоретиков-эконометристов об аномалиях.

    Прикладная эконометрика посвящена темам производства товаров и их производительности, спросу на рабочую силу, теории арбитражного ценообразования, вопросам спроса на жилье .

    Например, эконометрист может обнаружить, что дисперсия данных (насколько отдельные значения в серии отличаются от общего среднего) всегда меняется и никогда не фиксируется с течением времени.

    Главный инструмент эконометрики

    Основным инструментом эконометрики является модель линейной множественной регрессии, которая помогает оценить, как изменение одной из независимых переменных влияет на работу модели, от объясняемой переменной до изменений, происходящих в зависимой переменной.В современной эконометрике многие статистические инструменты оказались в центре внимания, но простая линейная регрессия по-прежнему является наиболее часто используемой отправной точкой для анализа.

    Этот шаг необходим, потому что регрессия имеет тенденцию оценивать предельное влияние конкретной объясняющей переменной после учета дисперсии, вызванной влиянием других объясняющих переменных на модель .

    Например, модель может попытаться дифференцировать влияние увеличения налогов на 1 процентный пункт на средние потребительские расходы домохозяйства, предполагая, что другие факторы потребления, такие как доход до налогообложения, богатство и процентные ставки, статичны.


    Этапы эконометрики

    Методология эконометрики довольно проста.


    Этапы эконометрики


    1. Предложение теории

    Первый шаг — предложить теорию или гипотезу, чтобы начать изучение определенной части данных.Объясняющие переменные в модели указываются заранее, а знак и / или величина взаимосвязи между каждой отдельной независимой переменной и зависимой переменной четко устанавливаются, чтобы не вызывать путаницы.

    На этом этапе в игру вступают прикладные эконометристы, которые в значительной степени полагаются на экономическую теорию, чтобы успешно сформулировать гипотезу на основе предоставленных данных.

    (Предлагается прочитать: 7 основных разделов дискретной математики)

    Например, философия международной экономики заключается в том, что цены через открытые границы идут рука об руку после того, как разрешен паритет покупательной способности.Эмпирическая взаимосвязь между внутренними ценами и иностранными ценами (с поправкой на сценарии номинального обменного курса) всегда должна быть положительной, и они должны всегда стараться поддерживать паритет.

    1. Определение статистической модели

    Второй шаг — определить статистическую модель, отражающую суть теории. Экономист пытается предложить уникальную связь между зависимой переменной и независимыми переменными через модель.

    Безусловно, самый простой подход — это допустить линейность, то есть любое изменение независимой переменной всегда будет вызывать аналогичное изменение зависимой переменной. Конечно, невозможно учесть каждое небольшое влияние на зависимую переменную, поэтому в статистическую модель добавляется переменная, чтобы свести на нет внешние возмущения.

    Роль новой переменной здесь состоит в том, чтобы представить все детерминанты зависимой переменной, которые не могут быть учтены.В основном из-за сложности данных.

    Просто чтобы быть последовательным и обеспечить выполнение всех условий для статистической модели, экономисты обычно предполагают, что этот термин «ошибка» в среднем равен нулю и непредсказуем.

    (Рекомендуемое чтение: Типы статистического анализа)

    1. Расчетные переменные

    Третий шаг — оценить неизвестные переменные модели с использованием имеющихся экономических данных.Обычно для этого используется соответствующая статистическая процедура и пакет эконометрических программ.

    Это называется самой простой частью анализа благодаря легкому доступу к обширным экономическим данным и отличным эконометрическим методам и программному обеспечению. Эконометрика по-прежнему основывается на принципах известного стиля вычислений GIGO (мусор на входе, мусор на выходе).

    1. Корректура

    Это четвертый шаг, а также самый важный из всех.На этом этапе нужно задать себе правильные вопросы. Например,

    • Соответствуют ли знаки и взаимосвязь оценочных параметров, связывающих зависимую переменную с независимыми переменными, с предсказаниями экономической теории?

    • Если расчетные параметры не имеют смысла, как следует отредактировать статистическую модель специалисту по эконометрике, чтобы получить соответствующие результаты?

    • А более точная оценка гарантирует получение экономически значимой модели?

    На этом этапе, в частности, проверяются навыки и опыт эконометриста в данной области.

    Проверка гипотезы

    Основным инструментом четвертого этапа является проверка гипотез, статистическая процедура, в которой исследователь отмечает истинное значение экономического параметра, и проводится статистический тест, чтобы выяснить, является ли оцениваемый параметр синонимом конкретной гипотезы.

    Если это не так, исследователь должен либо отвергнуть гипотезу, либо внести изменения в статистическую модель и начать все сначала.

    Если все четыре этапа проходят успешно, результатом является модель, которую можно использовать в качестве инструмента для оценки эмпирической достоверности экономической модели.

    Эмпирическая модель также может использоваться для прогнозирования зависимой переменной, потенциально помогая политикам принимать важные решения об изменениях в денежно-кредитной и / или налогово-бюджетной политике, чтобы экономика оставалась на стабильной платформе.

    Студенты, изучающие эконометрику, часто увлекаются возможностью линейной множественной регрессии для прогнозирования экономических отношений.

    Стоит помнить о трех основах эконометрики;

    • Первый , качество вывода параметров зависит от текущего рабочего состояния экономической модели.

    • Вторая , если релевантная объясняющая переменная исключена из модели, наиболее вероятно, что оценки параметров станут ненадежными и неточными.

    • В-третьих, , у оценок параметров очень малая вероятность совпадения с фактическими значениями параметров, которые генерируются статистическими данными, даже если эконометрист идентифицирует процесс как источник исходных данных.

    В конечном итоге оценки используются, потому что они станут точными по мере того, как будет доступно больше данных, а оценки будут соответствовать широте охвата.


    Функции эконометрики

    Эконометрика выполняет в основном три тесно взаимосвязанные функции.

    • Первая функция эконометрики — проверка экономических теорий или гипотез, выдвинутых желанными эконометриками.Например, напрямую связано потребление с доходом? Связано ли количество спроса на определенный товар обратно пропорционально его цене?

    • Вторая функция эконометрики заключается в предоставлении числовых оценок переменных экономических отношений. Это очень важно для принятия решений.

    Например, государственному разработчику политики необходимо иметь точную оценку коэффициента взаимосвязи между потреблением и доходом, чтобы понять стимулирующий эффект предлагаемого снижения налога и принять правильное решение.

    • Третья функция эконометрики — предсказывать экономические события. Это также необходимо для того, чтобы лица, определяющие политику, предприняли экономически обоснованные действия, если уровень безработицы или инфляции, согласно прогнозам, вырастет в будущем.


    Заключение

    Ни для кого не секрет, что экономика правит миром, и с помощью эконометрики ежедневно подтверждаются новые теории, делаются новые выводы и статистические модели отвергаются или утверждаются.

    Эконометрика делит мир на безграничные возможности для формирования новых теорий, которые дополняются данными, предоставляемыми эконометристам.

    Политики считают эти данные очень информативными и полагаются на эти выводы при формировании очень важных политик и решений.

    Эконометрика — Изучение экономики

    Эконометрика интересна тем, что предоставляет инструменты, позволяющие нам извлекать полезную информацию о важных вопросах экономической политики из имеющихся данных.Студенты, которые приобретают знания в области эконометрики, также обнаруживают, что они улучшают свои перспективы трудоустройства.

    Guy Judge , старший преподаватель кафедры количественной экономики Портсмутского университета


    Изображение Dunechaser на Flickr

    Эконометрика — это использование статистических методов для понимания экономических проблем и проверки теорий. Без доказательств экономические теории абстрактны и могут не иметь отношения к реальности (даже если они абсолютно строгие). Эконометрика — это набор инструментов, которые мы можем использовать для сопоставления теории с данными реального мира.

    Если после получения первой степени вы заинтересованы в том, чтобы продолжить свою экономическую деятельность (будь то дальнейшее обучение или профессиональный экономист в правительстве или частном секторе), вам может помочь эконометрика. Большинство магистерских курсов включают в себя обязательные продвинутые компоненты эконометрики, и большинство работодателей экономистов ищут людей, которые смогут вычислять числа и, что особенно важно, интерпретировать результаты.

    Целью прикладного эконометрического исследования может быть проверка гипотезы — например, определение того, какая часть «гендерного разрыва в оплате труда» может быть объяснена различиями в образовании и опыте.В качестве альтернативы, исследование может оценить ключевой параметр, например эластичность спроса на нефть по цене. Или же для составления прогнозов можно использовать эконометрические методы, подобные тем, которые Банк Англии использует для определения уровня, на котором базовая процентная ставка должна устанавливаться каждый месяц.

    Изучение эконометрики увеличивает ваш человеческий капитал двумя способами. Во-первых, он позволяет вам самостоятельно проводить прикладные эконометрические исследования, что может быть очень полезно, если вы также пишете диссертацию. Современное эконометрическое программное обеспечение значительно облегчает процесс формулирования, оценки и проверки модели и предоставляет полезную графическую информацию, а также таблицы результатов.

    Во-вторых, он позволяет критически оценивать эмпирические работы других. Это может быть полезно при обсуждении соответствующей академической литературы в любом другом модуле — и возможность прокомментировать, почему тот или иной метод может быть подходящим, а может и не подходящим, часто может быть очень впечатляющим.

    Эконометрика — непростой вариант. Изначально многие студенты находят это сухим и скучным. Но те, кто настойчиво с этим справляются, обычно получают вознаграждение за свои усилия. Когда вы понимаете, что делаете, проведение эконометрического исследования и поиск чего-то нового в этом мире приносит удовлетворение и, осмелюсь сказать, весело.

    Ссылки

    Нам прислали это отличное видео, которое понравится вам всем экономическим категориям

    Наш список источников данных в сети

    Предыдущая: Экономика развития

    Далее: Экономический анализ закона

    Введение в эконометрику Глава 1: Введение

    Глава 1. Введение

    В этой главе мы обсуждаем содержание этой книги, включая основные идеи
    мы пытаемся передать и используемые инструменты анализа.Начнем с нашего определения темы: Эконометрика — это применение статистических методов и анализы к изучению проблем и вопросов в экономике.


    Термин эконометрика был придуман в 1926 году норвежцем Рагнаром А. К. Фришем. экономист, получивший первую Нобелевскую премию по экономике в 1969 г. еще один пионер эконометрики Ян Тинберген. Хотя многие экономисты использовал данные и провел расчеты задолго до 1926 года, Фриш чувствовал, что ему нужно новое слово для описания того, как он интерпретировал и использовал данные в экономике.Сегодня эконометрика — это широкая область изучения экономики. Поле постоянно меняется по мере добавления новых инструментов и методов. Однако его центр содержит стабильный набор фундаментальных идей и принципов. Эта книга о сути эконометрики. Мы объясним основную логику и метод эконометрики, сосредоточившись на точном воплощении основных идей. Мы делим изучение эконометрики в этой книге на следующие два основные части:

      Часть 1.Описание
      Часть 2. Вывод

    В каждой части регрессионный анализ будет основным инструментом. Показывая регресс снова и снова в различных контекстах мы подкрепляем идею о том, что это является мощный и гибкий метод, определяющий большую часть эконометрики. В то же время, однако, мы описываем условия, которые должны быть выполнены для его надлежащего использование и ситуации, в которых регрессионный анализ может привести к катастрофическим ошибочные выводы при несоблюдении этих условий.

    В дополнение к регрессионному анализу мы будем использовать моделирование Монте-Карло. на протяжении второй части книги, чтобы смоделировать роль случая в процесс генерации данных. Метод Монте-Карло является неотъемлемой частью нашего подход к обучению, который подчеркивает конкретное, визуальное понимание. В разделе 1.2 используется пример исследования спроса на сигареты для проиллюстрировать цели и методы эконометрического анализа и то, как регрессия вписывается в это предприятие.Далее мы обсудим концепцию Монте. Карло анализ в главе 9, первой главе части 2 книги.

    Эконометрика | Encyclopedia.com

    Краткая история

    Обзор эконометрики

    БИБЛИОГРАФИЯ

    Кратко определенная, эконометрика — это исследование экономической теории в ее отношении к статистике и математике. Основная предпосылка состоит в том, что экономическая теория поддается математической формулировке, обычно как система отношений, которая может включать случайные величины.Экономические наблюдения обычно рассматриваются как образец, взятый из вселенной, описываемой теорией. Используя эти наблюдения и методы статистического вывода, эконометрист пытается оценить отношения, составляющие теорию. Затем эти оценки могут быть оценены с точки зрения их статистических свойств и их способности предсказывать дальнейшие наблюдения. Качество оценок и природа ошибок прогноза могут, в свою очередь, привести к пересмотру самой теории, с помощью которой были организованы наблюдения и на основе которой были выведены постулируемые числовые характеристики Вселенной.Таким образом, существует взаимная связь между формулировкой теории и эмпирической оценкой и проверкой. Отличительной чертой является явное использование математики и статистических выводов. Нематематические теории и чисто описательная статистика не являются частью эконометрики.

    Объединение экономической теории, математики и статистики было больше стремлением эконометриста, чем ежедневным достижением. Многое из того, что принято называть эконометрикой, является математической экономической теорией, которая не опирается на эмпирическую работу; и часть того, что известно как эконометрика, представляет собой статистическую оценку специальных отношений, которые имеют лишь хрупкую основу в экономической теории.Однако это достижение не оправдывает ожиданий, однако его не следует обескураживать. Частью процесса развития науки является то, что теории могут выдвигаться непроверенными и что поиск эмпирических закономерностей может предшествовать систематическому развитию теоретической основы. Однако следствием этого является то, что, хотя слово «эконометрика» явно подразумевает измерение, многие абстрактные математические теории, которые могут или не могут в конечном итоге поддаются эмпирической проверке, часто упоминаются как часть эконометрики.Значение этого слова часто применялось как к математической экономике, так и к статистической экономике; а в просторечии «эконометрист» — это экономист, опытный и интересующийся применением математики, будь то математическая статистика или нет. В этой статье я приму это расширенное определение и рассмотрю как эконометрику в ее узком смысле, так и математическую экономическую теорию.

    Использование математики и статистики в экономике возникло не недавно.Во второй половине семнадцатого века сэр Уильям Петти написал свои эссе о «политической арифметике» [ см. Биографию Петти]. Эта молодая работа, замечательная для своего времени, была эконометрической по своей методологической основе даже с современной точки зрения. Несмотря на то, что на нее не ссылался Адам Смит, она оказала заметное влияние на более поздних авторов. В 1711 году итальянский инженер Джованни Чева призвал использовать математический метод в экономической теории. Хотя за прошедшие годы появилось много статистических исследований, революционное влияние математического метода проявилось только во второй половине девятнадцатого века.Леон Вальрас, профессор Лозаннского университета, больше, чем кто-либо другой, признан создателем экономики общего равновесия, которая является базовой структурой современной математической экономики [ см. Биографию Вальраса]. Его работа, оторванная от каких-либо непосредственных статистических приложений, разработала всеобъемлющую систему отношений между экономическими переменными, включая деньги, чтобы объяснить взаимное определение цен и количества товаров и капитальных благ, производимых и обмениваемых.Вальрас рассматривал экономику как функционирующую в соответствии с классической механикой, при этом состояние экономики определяется балансом сил между всеми участниками рынка. Однако его система общего равновесия была по существу статичной, потому что значения экономических переменных сами по себе не определяли их собственные временные скорости изменения. По этой причине термин «равновесие» употребляется неправильно, поскольку, поскольку общая система Вальраса не была явно динамической, ее решение нельзя описать как состояние равновесия.Тем не менее, как это все еще верно в большинстве экономических теорий, были сторонние дискуссии о регулирующих свойствах экономики, и поэтому, в более широком контексте, решение можно рассматривать как результат уравновешивания динамических сил адаптации.

    Значительное сочетание математической теории и статистической оценки впервые произошло в работе Генри Ладделла Мура, профессора Колумбийского университета в начале двадцатого века [ см. Биографию Мура, Генри Л.]. Мур проделал настоящую эконометрическую работу над экономическими циклами, определением ставок заработной платы и спросом на определенные товары. Его главной публикацией, кульминацией которой стало около трех десятилетий работы, была книга Synthetic Economics , вышедшая в 1929 году. Невероятно, но эта работа, имеющая такое основополагающее значение для последующего развития значительной области социальных наук, было продано всего 873 экземплярами (Stigler 1962 ).

    Эконометрика приобрела свою идентичность как отдельный подход к изучению экономики в течение 1920-х годов.Число людей, посвятивших себя этой детской области, неуклонно росло, и 29 декабря 1930 года они основали международную ассоциацию под названием Эконометрическое общество. Это было достигнуто во многом благодаря энергии и настойчивости Рагнара Фриша из Университета Осло при помощи и поддержке выдающегося американского экономиста Ирвинга Фишера, профессора Йельского университета [ см. Биографию Фишера, Ирвинга] . Назвать это небольшое меньшинство экономистов культом означало бы приписать им слишком узкие и евангельские взгляды; тем не менее, у них было чувство миссии «продвигать исследования, которые направлены на объединение теоретико-количественного и эмпирическо-количественного подходов к экономическим проблемам, и которые пронизаны конструктивным и строгим мышлением, аналогичным тому, которое стало доминировать в мире. естественные науки »(Frisch 1933).

    Их идеи и амбиции были хорошо обоснованы. В последующие годы и в ходе многих методологических споров о роли математики в экономике (тема сейчас довольно устарела) их число росло, а их влияние в более широкой экономической профессии неуклонно расширялось. Сегодня все основные экономические факультеты университетов в западном мире, в том числе совсем недавно в странах советского блока, предлагают работу по эконометрике, и многие придают ей значительный упор.Специальные курсы по эконометрике были введены даже на уровне бакалавриата; написаны учебники; молодое поколение экономистов, поступающих в аспирантуру, прибывает с улучшенной подготовкой по математике и статистическим методам, тяготеет к тому, что, по-видимому, становится все больше, к специализации по эконометрике и вскоре превосходит своих учителей в знании эконометрических методов. Членство в Эконометрическом обществе увеличилось со 163 в 1931 году до более 2500 человек в 1966 году.Журнал общества, Econometrica , за эти годы практически удвоился в размерах, и почти все другие научные журналы по экономике регулярно публикуют статьи, математическая и статистическая сложность которых поразила бы основателей движения в 1920-х и 1930-х годах.

    Сферы применения эконометрики в экономике неуклонно расширяются. В настоящее время едва ли есть область прикладной экономики, в которую не вошли бы математическая и статистическая теория, включая экономическую историю.С ростом интереса к эконометрике и ее концентрации со стороны экономистов само понятие специализации стало размытым. Благодаря своему успеху в качестве основного интеллектуального движения в экономике, эконометрика теряет свою идентичность и исчезает как особая отрасль дисциплины, становясь теперь почти совпадающей со всей областью экономики. Однако эти замечания нельзя неправильно понимать. В экономике остается много проблем и много исследований, которые не являются ни математическими, ни статистическими, и хотя общий уровень подготовки современного экономиста и его интерес к математике и статистике намного превосходит уровень его предшественников, вполне надлежащая градация этих навыков и интересов неизбежно продолжается. существовать.Более того, повторяю, многое из того, что известно как эконометрика, все еще не соответствует взаимосвязи математико-теоретического и статистического, что является целью, содержащейся в определении этой области.

    Поскольку эконометрика больше не является маленьким анклавом в экономической науке, обзор ее предмета должен охватывать большую часть самой экономики.

    Общее равновесие

    Следуя концепции Вальраса об общем экономическом равновесии, экономисты-математики в последние годы занимались гораздо более тщательным анализом проблемы, чем предлагал Вальрас [ см. Экономическое равновесие].В более ранней работе общее экономическое равновесие описывалось системой равенств, включающей бесконечно большое количество экономических переменных, но число, равное количеству независимых уравнений. Предполагалось, что система одновременных уравнений с тем же числом неизвестных, что и независимые, будет иметь «равновесное» решение. Это свободная математика, и в последнее время теоретики-экономисты были озабочены тем, чтобы заново разработать более раннюю теорию с большей строгостью.Равенство уравнений и неизвестных не является ни необходимым, ни достаточным условием существования или единственности решения. Следовательно, нельзя быть уверенным в том, что ранняя теория адекватна для объяснения общего состояния равновесия, к которому экономика, как предполагается, должна сходиться. Это может быть связано с тем, что теория не накладывает условий, необходимых для обеспечения существования общего состояния равновесия, или потому, что теория может быть неопределенной, поскольку она подразумевает несколько различных решений.Поэтому современный теоретик равновесия попытался определить необходимые и достаточные условия для существования и уникальности общего экономического равновесия.

    Концепция равновесия — это состояние, в котором никакие силы внутри модели, действующие во времени, не приводят к дисбалансу системы. Даже если можно продемонстрировать существование такого состояния в рамках какой-либо модели общего равновесия, остается вопрос, является ли оно стабильным или нестабильным, то есть стремятся ли при любом отклонении системы от него силы восстановить прежнее состояние. исходное равновесие или сдвинуть систему дальше.Анализ этих вопросов, которые являются более сложными, чем предложенные здесь, требует явного введения отношений динамической настройки.

    Вопросы существования, уникальности и стабильности равновесия в данном контексте — это не вопросы о реальной экономике, а вопросы, касающиеся свойств теоретической модели, утвержденной для описания реальной экономики. В этом смысле их исследование ориентировано на лучшее понимание последствий альтернативных уточнений самой теории, а не на улучшенное эмпирическое понимание того, как работает наша экономика.

    Большая часть этой работы, кроме того, была ограничена исследованием модели общего равновесия конкурентоспособной экономики , что действительно является частным случаем. Тем не менее, это случай особого интереса, потому что, согласно идеализированным предположениям, экономисты благосостояния приписали конкурентному равновесию характеристики, которые удовлетворяют критериям, которые считаются интересными для социальной оценки экономической деятельности [ см. Экономика благосостояния], согласно Согласно концепции Парето, состояние экономики (не считающееся уникальным) считается оптимальным, если нет другого технологически выполнимого состояния, в котором какой-либо человек находился бы в положении, которое он предпочитает, в то время как ни один человек не был бы в положении, которое он считает худшим [ см. биографию Парето].Таким образом, условия, при которых общее экономическое равновесие было бы оптимальным в этом смысле, подвергались жесткой проверке. Таким образом, экономика благосостояния Парето тесно связана с современными исследованиями систем общего равновесия, но она недостаточно развита как эмпирическое исследование.

    Экономист-позитивист, озабоченный прогнозированием, в принципе также интересовался системами общего равновесия, но с другой точки зрения. Его центральный вопрос: как изменение экономического параметра (коэффициента или, возможно, значения некоторой автономной переменной, которая сама не определяется системой) вызывает изменение равновесного значения одной или нескольких других переменных, которые определяются системой? ? Короче говоря, как равновесное решение зависит от параметров? Это проблема в сравнительной статике , которая противопоставляет два различных равновесия, определяемых разницей в значениях одного или нескольких параметров.

    Сравнительная статика — частичное равновесие

    Именно в проблеме сравнительной статики — сравнении альтернативных состояний равновесия — мы можем наиболее точно провести различие между экономикой общего равновесия и экономикой частичного равновесия , что является знакомым контрастом в литературе.

    Предположим, что в окрестности равновесия общая система одновременных экономических отношений полностью дифференцируется по отношению к изменению определенного параметра, так что все прямые и косвенные эффекты этого изменения учтены.Затем можно надеяться установить направление изменения конкретной экономической переменной по отношению к этому параметру. Например, если повышается определенная налоговая ставка или предпочтения потребителей смещаются в пользу определенного товара, будет ли увеличиваться, уменьшаться или оставаться неизменным количество спроса на какой-либо другой товар? На этот вопрос иногда можно ответить на основе совокупности знаков (плюс, минус, ноль) многих или всех частных производных функций, составляющих систему (при условии, что они являются непрерывно дифференцируемыми).Теоретические соображения или здравый смысл могут позволить априори указать знаки этих частных производных, например, чтобы утверждать, что эластичность спроса отрицательна или перекрестная эластичность спроса положительна. Однако в некоторых случаях теоретику неудобно делать такие утверждения о производной, и, следовательно, некоторые признаки могут быть оставлены неопределенными. Вопрос в том, достаточны ли ограничения, которые теоретик готов наложить априори, для определения того, является ли полная производная интересующей экономической переменной по данному параметру положительной, отрицательной или нулевой.Формальное рассмотрение необходимых и достаточных ограничений, необходимых для решения этого вопроса, однозначно составляет исследование качественной экономики и представляет собой самостоятельную математическую проблему (Samuelson 1947; Lancaster 1965). В некоторых ситуациях может быть важно знать не только знаки различных частных производных, но и их относительные алгебраические величины. Это указывает на необходимость статистической оценки этих производных, что относится к эконометрике в ее самом узком смысле.

    Иногда также полезно знать, что некоторые производные достаточно близки к нулю, и если их принять равными нулю, это не повлияет на вывод о знаке исследуемой полной производной. Уловка или искусство решать, когда рассматривать определенные частные производные как ноль, то есть решить, что определенные экономические переменные не входят в какие-либо существенные отношения в определенные отношения, является сущностью анализа частичного равновесия, названного так потому, что он стремится изолировать часть общей системы из других частей, которые мало с ней взаимодействуют.Таким образом, анализ частичного равновесия является частным случаем анализа общего равновесия, в который были введены более смелые априорные ограничения с целью получения более конкретных и значимых результатов в сравнительной статике. Так же, как экономика общего равновесия обычно ассоциируется с именем Вальраса, так и экономика частичного равновесия ассоциируется с работами Альфреда Маршалла [ см. Биографию Маршалла].

    В качественной экономике некоторый свет проливает свет на признаки частных производных системы с учетом динамической устойчивости модели.С допущениями о природе отношений динамической регулировки можно найти соответствия между условиями, необходимыми для того, чтобы равновесие было устойчивым, и знаками частных производных. Таким образом, точно так же, как стабильность зависит от предположений о том, входят ли различные переменные в данное отношение, положительно или отрицательно, точно так же, как эти переменные входят в данное отношение, иногда можно вывести из предположения, что равновесие является стабильным. Это знаменитый принцип соответствия, созданный Самуэльсоном.[ См. Статика и динамика в экономике.]

    Пространственные модели

    В большинстве моделей общего равновесия экономика рассматривается как существующая в одной точке пространства, игнорируя, таким образом, транспортные расходы, региональную специализацию ресурсов и предпочтения местоположения. Некоторые исследования, однако, явно вводят пространственное измерение, в котором происходит общее равновесие. Это обеспечивает основу для изучения межрегионального расположения, специализации и взаимозависимости в обмене.[ См. Пространственная экономика, статья , посвященная подходу общего равновесия ]. Эти модели, из-за их большей сложности, обычно включают более специальные допущения, такие как линейность отношений и отсутствие возможностей для замены среди факторных услуг в производстве. . Однако они также более непосредственно поддались эмпирической работе.

    При применении анализа частичного равновесия к задачам пространственной экономики, кроме того, предполагается, что местоположения определенных видов экономической деятельности определяются независимо от решений о местоположении в отношении других видов экономической деятельности, и, следовательно, первое можно рассматривать как фиксированное в анализ последнего.[ Обсуждение этого направления исследования см. В Пространственная экономика, в статье о подходе частичного равновесия.]

    Агрегационные и агрегативные модели

    Поскольку системы общего равновесия рассматриваются как охватывающие миллионы индивидуальных отношений, они, очевидно, не поддаются количественной оценке. Поэтому большой интерес представляет уменьшение размерности системы, так что существует некоторая возможность эконометрической оценки.Это означает, что отношения общего типа, например отношения, описывающие поведение фирм в данной отрасли или домохозяйства определенного характера, должны быть объединены в единое отношение, описывающее поведение совокупности сопоставимых экономических агентов. Условия, необходимые для того, чтобы такое агрегирование стало возможным, и используемые методы все еще находятся на довольно предварительной стадии изучения. Но литература по этому поводу развивается. [ См. Агрегирование .]

    Более старая проблема — это просто объединение в одну переменную множества похожих переменных. Это известная проблема «индексов» * — например, как лучше всего представить цены на большое количество различных товаров с помощью единого индекса цен. Таким образом, проблема с индексными числами имеет свои теоретические аспекты [ см. индексные номера, статью о теоретических аспектах], а также свои статистические аспекты [ см. индексные номера, статей по практическим приложениям, и выборка].Теория оказалась полезной при интерпретации альтернативных статистических формул.

    Основные усилия по эмпирическому исследованию систем общего равновесия, которые до некоторой ограниченной степени были агрегированы, относятся к заголовку «Анализ затрат-выпуска». Этот подход, предложенный Василием Леонтьевым в конце 1930-х годов, состоит, по сути, в рассмотрении экономики как системы одновременных линейных отношений и рассмотрении как постоянных относительных величин входов в производственный процесс, которые необходимы для производства продукции процесса. .Эти входы, конечно, могут быть выходами других процессов. Таким образом, с фиксированными коэффициентами, относящимися к входам и выходам интегрированной производственной структуры, можно определить, какой «перечень товаров» может быть произведен, с учетом количества различных «первичных» непроизведенных ресурсов, которые доступны. В качестве альтернативы также могут быть определены количества первичных ресурсов, необходимых для производства данной товарной накладной. Коэффициенты такой системы можно оценить, наблюдая за соотношениями входов и выходов для различных процессов в данный год, или усредняя эти отношения по последовательности лет, или используя инженерные оценки.Это может быть сделано для экономики, разделенной на большое количество различных секторов (сотня или более), или это может быть сделано для частей экономики, таких как мегаполис. Более того, секторирование экономики может быть как по регионам, так и по отраслям, и первое делает метод применимым к изучению межрегиональных или международных торговых отношений. Было проведено большое количество эмпирических исследований по моделям затрат-выпуска, таблицы коэффициентов к настоящему времени разработаны для более чем сорока стран.Количественный анализ работы этих моделей, как легко предположить, потребовал наличия крупномасштабных компьютеров. [ См. Анализ затрат – выпуска.]

    Агрегативные модели в экономике могут относиться к типу частичного или общего равновесия. Те из них, которые относятся к типу частичного равновесия, имеют дело с отдельным сектором экономики изолированно, исходя из предположения, что внешние экономические переменные, которые имеют важное влияние на этот сектор, в свою очередь, не зависят от его поведения.Так, например, рыночная модель спроса и предложения на конкретный товар может рассматривать общий доход потребителей и его распределение как определяемые независимо от цены и выпуска конкретного изучаемого товара. Тем не менее, функции рыночного спроса и предложения являются совокупностью функций спроса и предложения многих людей и фирм. Агрегированные модели типа общего равновесия могут объяснить взаимное определение многих основных экономических переменных, которые являются агрегатами огромного числа индивидуальных переменных.Примерами агрегированных переменных являются общая занятость, общий объем импорта, общие инвестиции в товарные запасы и т. Д. Эти модели обычно называются макроэкономическими моделями , , в отличие от микроэкономических моделей , , которые имеют дело в смысле частичного равновесия с отдельным домохозяйством, фирмы, профсоюзы и т. д. Многие макроэкономические модели рассматривают не только так называемые реальных переменных, которые представляют собой физические запасы и потоки товаров и производственных услуг, но также денежные переменные , такие как уровни цен, количество денег, стоимость общего выпуска и процентная ставка.Подобные модели были особенно распространены с 1936 года, поскольку их стимулировала книга Джона Мейнарда Кейнса «Общая теория занятости, процента и денег» Джона Мейнарда Кейнса и созданная на ее основе литература.

    Один тип агрегированной макроэкономической модели — это модель, которая выделяет несколько важных секторов экономики или связывает макроэкономические переменные двух или более экономик, взаимосвязанных в торговле. Большая часть теории международной торговли имеет дело с подобными моделями [ см. Международная торговля, статья по математической теории ].Фактически, поскольку это был естественный способ анализа международных экономических проблем, теория международной торговли исторически была одной из самых оживленных областей для развития экономической теории, как математической, так и иной. Более узкие эконометрические исследования в этой области были сосредоточены на оценках эластичности импортного спроса.

    Более того, макроэкономические модели пригодились для изучения экономических изменений, и именно с этими моделями была проведена наиболее значительная работа в экономической динамике.Динамические системы в экономике — это такие системы, в которых значения экономических переменных в данный момент времени определяют либо их собственные скорости изменения (непрерывные модели дифференциальных уравнений), либо их значения в последующий момент времени (дискретные модели разностных уравнений). . [ Для общего обсуждения динамических моделей см. Статика и динамика в экономике.] Таким образом, динамические модели включают как переменные, так и меру их изменений во времени. Первые часто встречаются как «запасы», а вторые — как «потоки».«Когда и запасы, и потоки входят в данную модель, возникают сложности с согласованием желаемых количеств каждого из них. Эти проблемы становятся особенно важными, когда вводятся денежные переменные, например, когда мы рассматриваем желание людей как удерживать определенную стоимость денежных активов, так и откладывать (добавлять к активам) по определенной ставке. [ Конкретные проблемы моделей потока запасов обсуждаются в Анализ потока запасов.]

    Динамические модели возникают как в теории долгосрочного экономического роста [ см. Экономический рост, статья по математической теории ], где обе макроэкономические использовались полностью дезагрегированные модели общего равновесия, а также в теории деловых колебаний или деловых циклов [ см. Деловые циклы, статья о математических моделях], где макроэкономические модели являются наиболее распространенными.Не все модели, предназначенные для объяснения уровня деловой активности, должны иметь циклический характер. Современный упор делается на макроэкономические модели, циклические или нет, которые объясняют уровень деловой активности и его изменение динамической системой, которая реагирует на внешние переменные. К ним относятся переменные экономической политики (государственный дефицит, политика центрального банка и т. Д.) И другие переменные, которые, хотя и оказывают важное влияние на экономику, имеют свое объяснение за пределами теории, например, рост населения и темпы роста населения. технологические изменения.Таким образом, внешние переменные, известные как экзогенные или автономные переменные, воздействуют на динамическую экономическую систему и порождают колебания во времени, которые не обязательно должны быть периодическими. Эти модели поддаются эмпирическому исследованию, и для их оценки была проделана большая работа [ см. Эконометрические модели, совокупность]. Структура этих моделей была уточнена и доработана в результате эмпирической работы.

    Огромное преимущество агрегированных моделей, конечно, состоит в том, что они существенно сокращают огромное количество переменных и уравнений, которые появляются в системах общего равновесия, и тем самым делают возможными оценки.Даже в этом случае эти модели могут быть довольно сложными либо потому, что они все еще содержат большое количество переменных и уравнений, либо из-за нелинейностей в их функциональных формах. Однако современный компьютер позволяет оценивать системы такой степени сложности. Но если кто-то заинтересован в анализе динамического поведения этих систем, трудности часто выходят за рамки наших возможностей в математическом анализе. На помощь снова приходит компьютер. С помощью компьютера можно моделировать сложные системы рассматриваемого типа, управлять ими с помощью экзогенных переменных и шокировать их случайными возмущениями, извлеченными из определенных распределений вероятностей.Таким образом можно исследовать производительность этих систем при различных предположениях относительно поведения экзогенных переменных и для большой выборки случайных величин.

    [ Симуляционные исследования такого рода обсуждаются в Simulation, статье об экономических процессах ].

    Переменные, которые обычно возникают в макроэкономических моделях, — это совокупные потребительские расходы, инвестиции в товарные запасы и инвестиции в оборудование. Совокупные потребительские расходы или потребление отражают поведение домохозяйств при принятии решения о том, сколько потратить на потребительские товары, которые в некоторых исследованиях могут быть далее разбиты на такие категории, как потребительские товары длительного пользования, товары краткосрочного пользования и услуги.Используя методы регрессии, потребительские расходы устанавливаются в зависимость от других переменных, некоторые из которых имеют экономический характер (доход потребителей, изменение дохода, самый высокий прошлый доход, уровень потребительских цен и скорость их изменения, процентные ставки и условия потребительского кредита). , ликвидные активы и т. д.), некоторые из которых являются демографическими (раса, размер семьи, проживание в городе или деревне и т. д.). Эмпирическое исследование зависимости потребительских расходов от этих переменных проводилось интенсивно в течение последних двадцати лет.[Для обзора этой работы см. Функция потребления.]

    Динамика инвестиций в товарно-материальные запасы также была объектом интенсивного изучения, как с точки зрения того, как запасы менялись с течением времени относительно общего уровня деловой активности, так и с точки зрения того, как инвестиции в товарные запасы отреагировали на такие переменные, как процентная ставка, изменения продаж, невыполненные заказы и т. д. [ Эта работа рассмотрена в статье Inventories, , посвященной поведению запасов .] Есть некоторые тонкие вопросы, связанные с формулировкой функции инвестирования в запасы. Иногда запасы накапливаются, когда фирмы предполагают, что они должны это делать, а в других случаях они накапливаются, несмотря на желание фирм сократить их, например, когда продажи быстро падают относительно способности фирм изменять темпы выпуска продукции. Таким образом, теоретическая работа, посвященная оптимальному поведению фирм в вопросах политики запасов, может дать некоторую основу для выбора и интерпретации роли различных переменных в функции инвестирования в запасы.[ См. Запасы, статья по теории управления запасами.]

    Зависимость инвестиций в машины и оборудование от таких переменных, как коммерческие продажи, изменения продаж, коммерческая прибыль, ликвидность и т.д., также может быть изучена с помощью эконометрических методов. и различные теории были выдвинуты в поддержку представлений об относительной важности этих различных переменных. Как и в случае с функцией потребления и определением инвестиций в запасы, функция инвестиций в машины и оборудование также была предметом интенсивных эмпирических исследований в течение последних двух десятилетий.[ Эта работа рассматривается в Investment, статье о совокупной инвестиционной функции.]

    Принятие решений

    Хотя для экономиста методологически правильно постулировать ad hoc взаимосвязи между макроэкономическими переменными (Peston 1959), это будет более отрадным, более объединяющим экономическую теорию, если поведение макропеременных может быть выведено из элементарных предположений относительно поведения микропеременных, совокупностями которых они являются.Это проблема агрегирования, о которой говорилось ранее. Предполагается, что аксиоматическая теория поведения отдельного лица, принимающего экономические решения, особенно индивидуума (или домохозяйства) и фирмы, может служить основой для теорий взаимодействия агрегированных макропеременных. Однако большая часть поведенческой теории фирм и домашних хозяйств проводится в контексте анализа частичного равновесия, поскольку отдельный экономический агент не заботится о том, чтобы учесть очень незначительное влияние, которое его собственные решения оказывают на рынок или на экономику как целое.Таким образом, каждое домохозяйство и каждая конкурирующая (но не монополистическая) фирма считает рыночные цены фиксированными и не зависит от их собственного выбора. Но, связывая вместе такие модели частичного равновесия поведения огромного множества отдельных домохозяйств и фирм, нельзя игнорировать влияние их совместного поведения на те самые рыночные переменные, которые они считают константами. Таким образом, микромодели частичного равновесия должны быть преобразованы в более общие модели, учитывающие эти индивидуально не воспринимаемые, но коллективно важные взаимодействия.

    Микроэкономическая теория в значительной степени дедуктивна, она систематически исходит от аксиом, касающихся предпочтения и выбора, к теоремам об экономическом поведении. Для тщательного изучения логических сложностей этой дедуктивной теории часто используется формальная математика. Рыночные решения экономических агентов обычно предполагаются как осмотрительные или рациональные решения, что означает, что они в целом соответствуют определенным основным критериям принятия решений, которые, как считается, имеют широкую интуитивную привлекательность как предписания осмотрительного или рационального выбора.Ситуации, в которых лицо, принимающее решения, может сделать выбор, можно сформулировать по-разному. Бывают «статические» ситуации, когда не предполагается, что решение имеет временный или последовательный характер. Существуют «динамические» ситуации, в которых необходимо принимать последовательность решений, причем определенным образом. Проблема принятия решения также может быть классифицирована в зависимости от того, насколько лицо, принимающее решения, знает о последствиях своих решений. Одна крайность — это случай полной уверенности, когда предполагается, что последствия полностью известны заранее.Другие случаи связаны с риском и возникают, когда предполагается, что лицо, принимающее решения, знает только распределение вероятностей различных результатов, которые могут возникнуть в результате принятого им решения. Наконец, с другой стороны, проблема принятия решения может рассматриваться как включающая почти полную неопределенность, и в этом случае лицо, принимающее решение, знает, каковы возможные результаты, но не имеет априорной информации об их вероятностях. [Для обсуждение различных критериев, предлагаемых для этих различных ситуаций, см. Принятие решений, статью , посвященную экономическим аспектам .] Однако фундаментальным является представление о том, что лицо, принимающее решения, имеет предпочтения и использует их в пределах доступного ему диапазона выбора. Индекс, определяющий его предпочтения, обычно называется полезностью и рассматривается как функция от выбранных им объектов. В частности, когда человек с фиксированным доходом выбирает среди различных «рыночных корзин» товаров, полезность обычно постулируется как функция компонентов рыночной корзины. Аксиоматические системы, необходимые и достаточные для существования такой функции, были объектом интенсивного изучения экономистов-математиков.[ Эта центральная проблема и многие ее тонкие аспекты рассматриваются в Полезности.] Большие усилия, возможно, с небольшой пользой для эмпирической экономики, были направлены на уточнение аксиоматики теории полезности или теории потребительского выбора; К сожалению, было сделано гораздо меньше работы по укреплению предположений теории и увеличению ее эмпирического содержания. Из теории поведения потребителей вытекает концепция функции спроса потребителя на конкретный товар, зависящей, как правило, от всех цен и дохода.

    Что касается теории фирмы, то также предполагается осмотрительное, целенаправленное поведение, и в наиболее распространенной формулировке теории предполагается, что фирма желает в какой-то мере максимизировать свои предпочтения среди потоков будущих прибылей. Это должно происходить с учетом цен, которые фирма должна платить за факторные услуги, рыночных возможностей, с которыми она сталкивается при продаже своей продукции, и ее внутренней технологии производства. Из этого анализа вытекает теория производства и предложения.[Для теория производства фирмы см. Производство; для эконометрических исследований производственных отношений и себестоимости продукции, см. Производство и анализ затрат; , а для эконометрических исследований спроса и предложения см. Спрос и предложение, статью «О эконометрических исследованиях».

    При выводе теории поведения потребителей и теории фирмы целеустремленное и осмотрительное поведение обычно ассоциировалось с понятием, что лицо, принимающее решения, пытается максимизировать некоторую функцию с учетом рыночных и технологических ограничений.Таким образом, математика ограниченной максимизации служила экономисту самым важным инструментом в его профессии. В попытке разработать модели максимизирующего поведения, которые лучше поддаются количественной формулировке и решению, интерес сосредоточился на задачах, в которых максимизируемая функция является линейной, а ограничения составляют набор линейных неравенств. Методы решения таких задач получили название линейного программирования. С дальнейшим развитием были введены нелинейности и случайные элементы, и этот метод стал применяться также к задачам последовательного принятия решений.Вся эта область теперь известна как математическое программирование [ см. Программирование]. В силу своей практической полезности эти методы позволяют анализировать различные конкретные проблемы планирования и оптимизации, особенно проблемы, связанные с деятельностью фирмы. Стимулируемый доступностью этих методов, а также достижениями теории вероятностей и некоторым военным опытом в области системного анализа, расцвел современный количественный подход к проблемам производства и управления бизнесом.Это известно как наука об управлении или исследование операций [ см. Исследование операций]. Это развитие представляет собой случай расщепления, поскольку наука управления теперь рассматривается отдельно от эконометрики, хотя обе области имеют много общего, и у них есть много профессоров и практиков.

    Самыми сложными проблемами в области осмотрительного принятия решений являются те, которые связаны со стратегическими соображениями. По сути, это означает, что последствия решения или действия, предпринятого одним участником, зависят от действий, предпринятых другими; но их действия, в свою очередь, зависят от действий каждого из других участников.Таким образом, структура проблемы заключается не в простом максимизации даже перед лицом риска или неопределенности, а в стратегической игре. [ См. Game Theory, статья о теоретических аспектах.] Основываясь на соображениях разумной стратегии отдельного участника и стимулов для подмножеств участников формировать коалиции, теорию игр можно представить как общую задачу равновесия. и стал тесно связан с современной работой в области экономики общего равновесия.В более частном контексте теория игр оказалась применимой к решению проблем фирм в ситуациях олигополистической и двусторонней монополии. Они характеризуются тем, что каждая фирма, выбирая наилучший образ действий, должна учитывать влияние своих действий на действия других фирм, которые также действуют осмотрительно. В целом, ранний энтузиазм по поводу применения теории игр к этим проблемам промышленного поведения пока подтвержден лишь в ограниченной степени.[Обзор приложений теории игр к деловому поведению см. В Теория игр, в статье о экономических приложениях.]

    Процессы распределения

    Давно озабоченным в экономике было распределение экономических переменных по размеру. Что определяет распределение семейных доходов или распределение активов или продаж фирм в данной отрасли? В прошлые годы эти проблемы решались описательно путем подгонки частотных распределений к данным по разным странам, разным годам или различным отраслям.Соответствие данным из разных источников можно было бы объявить эмпирическим «законом»; таким образом, закон Парето распределения доходов. В последние годы проблема распределения по размерам была пересмотрена. Эконометристы теперь рассматривают его как формулировку динамического процесса роста или распада со случайными элементами. Задача состоит в том, чтобы оценить параметры процесса и определить, существует ли равновесное распределение размеров единиц и каково это распределение. Таким образом, хорошее соответствие может иметь теоретический механизм, а параметры могут зависеть от других экономических переменных, которые могут изменяться или могут контролироваться.[ В этой связи см. Распределения размеров в экономике и Цепи Маркова.]

    Статистические методы

    В естественных науках исследователь должен проводить свои собственные измерения. В экономике, однако, сама экономика генерирует данные в огромных количествах. Налогоплательщики, коммерческие фирмы, банки и т. Д. Регистрируют свои операции, и во многих случаях эти записи доступны экономисту. К сожалению, эти данные не всегда именно те, которые нужны экономисту, и их необходимо часто корректировать в научных целях.В последние десятилетия правительство все активнее занимается сбором и обработкой экономических данных. Это оказало огромную помощь в развитии эконометрики. Это касается не только правительств США и стран Западной Европы, но и данные накапливаются в странах с плановой экономикой, где они имеют решающее значение для операций планирования. [ См. Экономические данные.] Отсутствие адекватных данных наиболее остро ощущается при изучении слаборазвитых экономик, хотя через Организацию Объединенных Наций и другие организации собирается все больший объем данных по этим частям мира и сопоставлено.

    Основная форма представления экономических данных — это последовательные записи экономических наблюдений за определенный период времени. Таким образом, могут существовать данные о ценах на определенные товары, данные о занятости и т. Д. За многие годы. Следовательно, эконометристы традиционно серьезно занимались анализом временных рядов [ см. Временные ряды] и особенно использованием методов регрессии, где различные наблюдения упорядочены во временной последовательности. Это привело к разработке уравнений динамической регрессии, пытающихся объяснить наблюдение конкретной даты как функцию не только других переменных, но также одного или нескольких прошлых значений одной и той же переменной.Таким образом, отношение динамической регрессии представляет собой разностное уравнение, включающее случайный член. Когда в разностное уравнение вводится много прошлых значений переменной, так что это уравнение очень высокого порядка, становится трудно оценить коэффициенты этих прошлых переменных без потери многих степеней свободы. В результате эконометрист попытался наложить определенную схему взаимосвязи на эти коэффициенты, чтобы все они могли быть оценены как функции относительно небольшого числа параметров.Это метод регрессии с распределенным запаздыванием. [ См. Распределенные запаздывания.]

    Только что описанные методы в значительной степени пришли на смену более старым методам декомпозиции временных рядов, в соответствии с которыми временной ряд разбивается на такие компоненты, как тренд, циклы различной длины, сезонный образец изменения , и случайный компонент. Эти методы предполагали взаимодействие повторяющихся воздействий регулярной периодичности и амплитуды. С переходом к разностному уравнению и подходу регрессии были введены экзогенные переменные, и случайные возмущения стали кумулятивными в их эффектах.Таким образом, временные характеристики временного ряда описываются меньше в терминах некоторого внутреннего закона периодичности и больше в терминах последовательности реакций на случайные воздействия и временные вариации других причинных переменных. Таким образом, прогнозирование не является неумолимой экстраполяцией ритмов, а является пересмотренным прогнозом, период за периодом, возрастающей взаимосвязи, зависящей от настоящих и прошлых значений, от экзогенных переменных и случайных элементов. [ См. Прогнозирование и прогнозирование, экономическое.]

    Тем не менее, всегда было разумно предположить довольно строгую периодичность для сезонной составляющей из-за повторяющегося характера сезонов, праздников и т. Д. В результате, при изучении временных рядов, где наблюдения проводятся ежедневно, еженедельно или ежемесячно. , принято сначала оценивать и снимать сезонное влияние. [ Методы для этого обсуждаются в Временных рядах, статье о сезонной корректировке .]

    Другой вид данных, которые использует экономист, — это перекрестные данные.Например, он может использовать выборку наблюдений, сделанных примерно в одно и то же время, за активами, доходами и расходами разных домохозяйств, фирм или отраслей. [ См. Поперечный анализ.] Наблюдая за различиями в поведении людей в выборке и, опять же, обычно с помощью регрессионного анализа, приписывая эти различия различиям в других переменных, находящихся вне контроля этих людей, эконометрист пытается сделать вывод как изменилось бы поведение аналогичных экономических единиц со временем, если бы изменились значения независимых переменных.Есть много ошибок в этом процессе вывода изменений во времени для данной фирмы или домохозяйства на основе различий между фирмами и домохозяйствами в данный момент времени. Особенно полезными становятся данные, которые являются как поперечными, так и временными рядами по своему характеру, как, например, когда наблюдаются бюджеты выборки домашних хозяйств, каждое за несколько последовательных лет. Для получения полезной информации поперечного сечения или поперечного сечения и сортировки временных рядов обычно требуется разработка выборочного обследования.[ Применение методов обследования в экономике обсуждается в Анализ обследований, статья О приложениях в экономике.]

    Очень распространенная проблема в эконометрике возникает, когда разные переменные связаны по-разному. Например, совокупные инвестиции зависят от национального дохода, но национальный доход по-разному зависит от совокупных инвестиций. В анализе спроса и предложения равновесное обмениваемое количество и рыночная цена должны одновременно удовлетворять как функцию спроса, так и функцию предложения.Эта одновременность множественных отношений между одними и теми же переменными представляет особые проблемы при применении методов регрессии. Эти проблемы были тщательно изучены в течение последних двадцати лет, и теперь доступны различные устройства для их решения. Эти методы часто довольно сложны, но с развитием статистической теории и доступности данных и с использованием крупномасштабного компьютера они стали широко использоваться при оценке как частичного равновесия, так и макроэкономических моделей, иногда довольно больших измерение.Хотя здесь упоминается лишь кратко, эта важнейшая проблема статистической методологии, возможно, является самой центральной особенностью эконометрического анализа и является предметом ряда текстов и трактатов. Это также, вероятно, самый большой блок материалов, охватываемых большинством специальных курсов по эконометрике. [ См. Одновременное вычисление уравнений.]

    Тем, кто занимается исследованиями на переднем крае любой науки, прогресс всегда кажется чрезвычайно медленным; но обзор достижений эконометристов как в развитии экономической теории, так и в ее количественной оценке и проверке за последние два или три десятилетия дает ощущение большого достижения.Но по мере того, как решаются старые проблемы, изобретаются новые. Таким образом, развитие эконометрики не ослабевает.

    Роберт Х. Штроц

    Работы, посвященные природе и истории эконометрики: Divisia 1953; Frisch 1933; Tintner 1953; 1954. Основные работ в области: Аллен 1956; Malinvaud 1964; Самуэльсон 1947.

    Аллен Р. Г. (1956) 1963 Математическая экономика. 2-е изд. Нью-Йорк: Сент-Мартинс; Лондон: Макмиллан. Divisia, Franéois 1953 La Société d’Économétrie a atteint sa Majorité. Econometrica 21: 1–30.

    [Фриш, Рагнар] 1933 г. От редакции. Econometrica 1: 1–4.

    Ланкастер, К. Дж. 1965 Теория качественных линейных систем. Econometrica 33: 395–408.

    Малинво, Эдмонд (1964) 1966 Статистические методы в эконометрике. Чикаго: Рэнд МакНалли. → Впервые опубликовано на французском языке.

    Пестон, М. Х. 1959 Взгляд на проблему агрегирования. Обзор экономических исследований 27, вып. 1: 58–64.

    Самуэльсон, Пол А. (1947) 1958 Основы экономического анализа. Гарвардские экономические исследования, Vol. 80. Кембридж, Массачусетс: Harvard Univ. Нажмите. → Издание в мягкой обложке было опубликовано в 1965 году издательством Atheneum.

    Стиглер, Джордж Дж. 1962 Генри Л. Мур и статистическая экономика. Econometrica 30: 1–21.

    Тинтнер, Герхард 1953 Определение эконометрики. Econometrica 21: 31–40.

    Тинтнер, Герхард 1954 г. Преподавание эконометрики. Econometrica 22: 77–100.

    Что следует знать об эконометрике

    Есть много способов определить эконометрику, самый простой из которых — это статистические методы, используемые экономистами для проверки гипотез с использованием реальных данных. В частности, он количественно анализирует экономические явления в связи с текущими теориями и наблюдениями, чтобы сделать краткие предположения о больших наборах данных.

    Вопросы типа «Связана ли стоимость канадского доллара с ценами на нефть?» или «Действительно ли бюджетные стимулы стимулируют экономику?» На это можно ответить, применив эконометрику к наборам данных по канадским долларам, ценам на нефть, бюджетным стимулам и показателям экономического благосостояния.

    Университет Монаша определяет эконометрику как «набор количественных методов, которые полезны для принятия экономических решений», в то время как «Экономический словарь» The Economist определяет ее как «создание математических моделей, описывающих математические модели, описывающие экономические отношения (например, требуемое количество товара зависит положительно от дохода и отрицательно от цены), проверяя обоснованность таких гипотез и оценивая параметры, чтобы получить меру силы влияний различных независимых переменных.»

    Базовый инструмент эконометрики: модель множественной линейной регрессии

    Эконометристы используют множество простых моделей, чтобы наблюдать и находить корреляцию в больших наборах данных, но наиболее важной из них является модель множественной линейной регрессии, которая функционально предсказывает значение двух зависимых переменных как функцию независимой переменной.

    Визуально модель множественной линейной регрессии можно рассматривать как прямую линию, проходящую через точки данных, которые представляют парные значения зависимых и независимых переменных.При этом специалисты по эконометрике пытаются найти объективные, эффективные и последовательные оценщики для прогнозирования значений, представленных этой функцией.

    Таким образом, прикладная эконометрика использует эти теоретические практики для наблюдения за данными из реального мира и формулирования новых экономических теорий, прогнозирования будущих экономических тенденций и разработки новых эконометрических моделей, которые создают основу для оценки будущих экономических событий, связанных с наблюдаемым набором данных.

    Использование эконометрического моделирования для оценки данных

    В тандеме с моделью множественной линейной регрессии эконометристы используют различные эконометрические модели для изучения, наблюдения и формирования кратких наблюдений за большими наборами данных.

    «Экономический глоссарий» определяет эконометрическую модель как модель, «сформулированную таким образом, чтобы ее параметры можно было оценить, если сделать предположение, что модель верна». По сути, эконометрические модели — это модели наблюдений, которые позволяют быстро оценить будущие экономические тенденции на основе текущих оценок и исследовательского анализа данных.

    Эконометристы часто используют эти модели для анализа систем уравнений и неравенств, таких как теория равновесия спроса и предложения, или для прогнозирования изменений рынка на основе экономических факторов, таких как фактическая стоимость внутренних денег или налог с продаж на этот конкретный товар или услугу. .

    Однако, поскольку эконометристы обычно не могут использовать контролируемые эксперименты, их естественные эксперименты с наборами данных приводят к множеству проблем с данными наблюдений, включая смещение переменных и плохой причинно-следственный анализ, который приводит к искажению корреляций между зависимыми и независимыми переменными.

    эконометрики (определение, примеры) | Что такое эконометрика для финансов?

    Что такое эконометрика?

    Эконометрика — это понимание взаимосвязей экономических данных путем использования ссылок на статистические модели и получения наблюдения или закономерностей из предоставленных данных для разработки приближенного будущего тренда.Эконометрика просто экономична с добавлением математики и статистики и помогает в прогнозировании и оценке с помощью статистических методов.

    Методы эконометрики

    Наиболее распространенные методы:

    Вы можете свободно использовать это изображение на своем веб-сайте, в шаблонах и т. Д. Пожалуйста, предоставьте нам ссылку с указанием авторства Ссылка на статью с гиперссылкой
    Например:
    Источник: Econometrics (wallstreetmojo.com)

    Примеры эконометрики для финансов

    Ниже приведены примеры эконометрики для финансов

    Пример № 1 по эконометрике

    Майкл имеет доход 50000 долларов.Структура расходов его дохода составляет 10000 — фиксированная арендная плата и другие домашние расходы составляют 50% его валового дохода, полученного в течение периода.

    Множественная линейная регрессия — один из лучших инструментов для развития отношений на основе прошлых тенденций.

    Уравнение будет иметь вид = B 0 (точка пересечения) + B 1 + e (член ошибки)

    Используя уравнение, можно получить сумму, которую Майкл потратит на основе своего заработанного дохода.

    • Расходы = B 0 (фиксированная арендная плата) + B 1 (эксп.) + e (Член ошибки)
    • = 10000 + 50% (50000)
    • = 35000

    Термин ошибки показывает, что может быть небольшое отклонение вверх или вниз от результата, полученного с помощью статистических инструментов.

    Пример эконометрики 2

    Узнаем заработную плату человека исходя из его опыта работы

    Минимальная заработная плата: 10 тысяч долларов

    На основе регрессии по заработной плате человека получается, что B 1 = 2000

    Таким образом, применяя метод, можно понять, что человек получит минимальную заработную плату в размере 10000 + (2000 * No.лет опыта)

    Эти 10K и 2K являются гипотетическими значениями и должны быть проверены с помощью статистических инструментов, таких как t-тест. T-тест — это метод определения того, значительно ли отличаются друг от друга средние значения двух групп. Это метод логической статистики, который упрощает проверку гипотез.Подробнее и F-тест. Если они существенно не отличаются от 0, то предполагаемое значение не имеет значения, и необходимо повторить проверку, чтобы получить другое значение.

    Как эконометрика работает в финансах?

    Входные данные Выходные данные
    Теории, на которые ссылаются Параметры, используемые в данных
    Выбранные модели Нарисованная область достоверности Проведение теста на предположение
    Применяемые методы Используемые графические инструменты

    Преимущества эконометрики

    Вот преимущества эконометрики.

    • Используя инструменты или прикладную эконометрику, можно преобразовать данные в конкретную модель с целью принятия решения, которое поддерживает эмпирические данные.
    • Помогите получить указанный узор или результат из разбросанных данных.
    • Позволяет нам извлекать релевантную информацию из корзины информации.

    Недостатки эконометрики

    Эконометрика имеет некоторые недостатки.

    • Иногда построение отношений с помощью экономических инструментов является ложным i.е. даже не существует никакой связи между двумя переменными, но модель показывает закономерность на основе прошлой информации. Бывший. Корреляция между дождем и выплаченными дивидендами
    • Это показывает, что всякий раз, когда в квартале выпадает дождь, только компания объявляет дивиденды за этот период. Даже дождь не имеет отношения к выплаченным дивидендам, но, согласно установившейся тенденции, он может давать ложные сигналы, которые могут привести к неправильному решению.
    • Всегда есть выбор между простотой и точностью.Спецификация модели — очень важная задача в прикладной экономике. Выбор меньшего количества переменных может помочь в упрощении и обеспечить более быстрый результат, но он может быть неточным из-за недостатка информации или из-за высокого «нет». переменной, то модель может быть критической, неэкономичной или гигантской.
    • Может возникнуть проблема мультиколлинеарности между переменными, используемыми в данных. Очень важно, чтобы выбранная переменная имела низкую корреляцию между двумя независимыми переменными.Модель оставила этот раздел на пользователя модели.

    Важные моменты

    • Инструменты эконометрики очень критичны. Окончательный вывод может варьироваться от пользователя к пользователю.
    • Результат зависит от типа и спецификации модели. Результаты ориентированы на модели.
    • Данные экономичны, осуществимы, пора получить результаты, которые следует учитывать при применении модели.
    • Может применяться как к данным поперечного сечения, так и к данным временного ряда.
    • Должен быть периметр или тест, необходимый для проведения итоговой эффективности, такой как f-тест в Excel, T-тест, таблица статистики, анализ таблицы ANOVA с использованием пакетов инструментов.

    Заключение

    • Всегда не забывайте проверять, являются ли полученные результаты статистически значимыми для принятия решений.
    • Они развиваются из рассматриваемой модели или периметра.
    • Результат должен быть как эмпирически, так и футуристически благоприятным.
    • Это повторяющееся упражнение, и различные модели также могут быть применены к одной проблеме, чтобы получить лучшее понимание.
    • Переоснащение или недостаточное соответствие результатов может быть разбавлено улучшенной спецификацией модели.

    Рекомендуемые статьи

    Это был путеводитель по эконометрике и ее определению. Здесь мы обсуждаем основные методы и примеры эконометрики для финансов, а также их преимущества и недостатки. Подробнее о наших статьях по бухгалтерскому учету вы можете узнать ниже —

    Что такое эконометрика? | GoCardless

    Термин «эконометрика» впервые был введен польским экономистом Павлом Чомпа в 1910 году. Однако именно Рагнар Фриш и Ян Тинберг определили его современное использование и значение, в результате чего в 1969 году им была присуждена Нобелевская премия по экономике.Вот посмотрите, как сегодня используются методы эконометрики.

    Понимание эконометрики для финансов

    Эконометрика использует сочетание статистических и математических методов для проверки теорий и прогнозирования будущих экономических тенденций. Благодаря сочетанию статистических выводов, экономической теории и основных математических принципов эконометрика для финансов помогает описывать современные экономические системы.

    По сути, он превращает качественные идеи в количественные результаты. Например, эконометрический анализ можно использовать для преобразования теоретической модели в реальный инструмент или результат, который политики могут применить на практике.Для этого эконометристы — это те, кто просеивает огромные груды данных, превращая их в количественные утверждения с помощью моделирования и анализа.

    Эконометрика против статистики

    Хотя между эконометрикой и статистикой есть некоторое совпадение, эти два термина различаются по значению. Эконометрика действительно использует статистические теории и данные при анализе экономических теорий, но она включает в себя больше, чем просто цифры. При применении статистических методов он смотрит на более широкую картину экономики.

    Есть еще много общего в изучении эконометрики и статистики. Для выполнения эконометрического анализа экономист может использовать статистические инструменты, в том числе:

    • Вероятность

    • Частотные распределения

    • Статистический вывод

    • Корреляционный анализ

    • 8 9004 Методы временных рядов

      Какова цель эконометрики?

      Эконометрический анализ используется для проверки гипотезы, будь то существующая экономическая теория или совершенно новая идея.Его также можно использовать для прогнозирования будущих финансовых или экономических тенденций с использованием текущих данных. Это делает эконометрику для финансов повседневным инструментом трейдеров с Уолл-стрит и финансовых аналитиков.

      Идея эконометрики может быть применена для проверки многих теорий. Например, экономист может захотеть проверить гипотезу о том, что по мере увеличения компанией прибыли ее расходы соответственно увеличиваются. Эконометрика будет использовать данные, лежащие в основе этого предположения, а затем использовать статистические инструменты, такие как регрессионный анализ, для более глубокого изучения взаимосвязи между прибылью и расходами.

      Основы эконометрики: теоретические и прикладные

      Эконометрика состоит из двух основных компонентов: теоретической и прикладной. Вот еще немного информации о том, как работают эти элементы эконометрики для финансов.

      Теоретическая эконометрика

      Этот вид эконометрического анализа рассматривает свойства существующих статистических процедур или тестов для оценки любых неизвестных. Теоретики-эконометристы могут разработать новые статистические методологии, учитывающие аномалии в экономических данных.Этот раздел эконометрики опирается в первую очередь на теоретическую статистику, числовые данные и математику с целью доказать, что новые процедуры действительно жизнеспособны.

      Прикладная эконометрика

      Второй компонент эконометрики использует методы преобразования качественных заявлений в количественные. Когда теоретики-эконометристы разрабатывают новые статистические процедуры, это часто является ответом на работу эконометристов-прикладников, которые обнаружили необъяснимые различия в наборах данных.Затем специалисты по прикладной эконометрике могут использовать эти новые методы для проверки своих гипотез.

      Методы эконометрики

      Стандартные методы эконометрики включают несколько этапов. Самый первый шаг — выбрать набор данных для анализа. Это может быть что угодно, от уровня инфляции до показателей безработицы или исторических цен на акции финансовых технологий.

      1. Выбрав данные, эконометрист предлагает гипотезу или теорию для их объяснения. Эта модель должна определять различные задействованные переменные, а также величину взаимосвязи между переменными.Экономическая теория играет большую роль на этом начальном этапе эконометрического анализа.

      2. Следующим шагом является определение статистической модели, которая наилучшим образом соответствует проверяемой экономической теории. Обычно предполагается линейная зависимость, что означает, что любое изменение независимых переменных приведет к тому же уровню изменения зависимой переменной для линейной прогрессии.

      3. Затем вы воспользуетесь статистической процедурой для оценки любых неизвестных параметров или коэффициентов модели.Программное обеспечение для эконометрики обычно быстро справляется с этим шагом.

      4. Гипотеза должна быть проанализирована логически, чтобы увидеть, имеет ли она смысл в рамках предполагаемых параметров и экономических теорий.

      5. Наконец, пришло время проверить гипотезу, чтобы убедиться, что оцениваемый параметр верен.

    Как посчитать коэффициент рождаемости на 1000 человек – CGI script error

    Рекомендации для самостоятельной работы студентов -Расчет демографических показателей

    Рекомендации для самостоятельной работы студентов

    Расчет демографических показателей (рождаемость, смертность, естественный прирост).

    1.Возрастная структура населения определяется по следующей формуле (1.1):

    В= число лиц определенной возрастной группы : среднегодовая численность населения х100% (1.1)

    где В — доля лиц соответствующей возрастной группы.

    2. Половая структура населения рассчитывается по формуле (1.2):

    П= число лиц определенного пола : среднегодовая численность населения х100% (1.2)

    где П — доля лиц соответствующего пола.

    3. Общий показатель рождаемости (рождаемость) представляет собой общее число родившихся живыми в течение года детей, приходящееся на 1000 населения, и рассчитывается по формуле (1.3):

    ОПР = общее число родившихся в течение года: среднегодовая численность населения x1000 (1.3)

    где ОПР — общий коэффициент рождаемости (рождаемость).

    4.Коэффициент плодовитости рассчитывают по следующей формуле (1.4):

    П = общее число родившихся в течение года : среднегодовая численность женского населения фертильного возраста (15-49 лет) х1000 (1.4)

    где П — общий коэффициент плодовитости.

    5. Общий показатель смертности рассчитывается по формуле (1.5):

    ОПС = общее число умерших в течение года : среднегодовая численность населения х1000 (1.5)

    где ОПС — общий показатель смертности.

    6.Показатель естественного прироста, определяемый разностью между показателями рождаемости и общей смертности (формула 1.6).

    ЕП = ОПР-ОПС, (1.6)

    где ЕП — показатель естественного прироста,

    ОПР — общий коэффициент рождаемости (рождаемость),

    ОПС — общий показатель смертности.

    Воспользовавшись вышеприведенными формулами, можно определить основные демографические показатели на территории области, города, в районе обслуживания поликлиники или на конкретном терапевтическом участке.

    Пример. Среднегодовая численность населения, находящегося под наблюдением семейного врача, составляет 1500 человек, из них в возрасте до 14 лет — 225 человек, лиц старше 50лет — 300, женщин — 780, в течение года в семьях родилось 15 детей, умерло 18 человек. Таким образом, используя формулы 1.1-1.6, можно определить:

    *​ возрастной тип населения;

    *​ рождаемость;

    *​ смертность;

    *​ естественный прирост населения.

    1)определение возрастного типа населения:

    доля лиц до 14 лет =225:1500х100% = 15 %

    доля лиц старше 50 лет =300 :1500 х100% = 20 %

    2)структура населения по полу:

    доля женщин = 780:1500 х100% =52%;

    доля мужчин =100 % — 52% =48%

    3) общий показатель рождаемости:

    ОПР =15:1500 х 100% =10 %

    4)общий показатель смертности:

    ОПС = 18:1500 х 100% = 12%

    5)естественный прирост:

    10-12=-2 (%)

    Таким образом, характеризуя демографическую ситуацию среди семей, находящихся под наблюдением, можно отметить, что возрастной тип населения является регрессивным с преобладанием женского населения, рождаемость находится на низком уровне, смертность — на среднем уровне, естественный прирост населения отрицателен, то есть можно говорить о противоестественной убыли населения. На основании этих данных можно сделать вывод о необходимости усиления деятельности медицинской сестры по планированию семьи, следует также уделить внимание охране здоровья женщин и лиц пожилого возраста.

    infourok.ru

    3) Специальный коэфициент разводимости

    Спец. коэфф. разводимости = ×1000

    Специальные коэффициенты брачности и разводимости могут быть рассчитаны только за год (или двухгодичный период), когда проводится перепись населения, потому что только по данным переписи можно определить численность лиц, состоящих и не состоящих в браке.

    Специальный коэффициент смертности не рассчитывается потому, что совпадает с общим коэффициентом смертности.

    С помощью расчета специальных коэффициентов можно устранить второй недостаток общих коэффициентов, т.е., несопоставимость числителя и знаменателя дроби при вычислении общих коэффициентов рождаемости, брачности и разводимости. Однако первый недостаток общих коэффициентов (то есть, их зависимость от возрастной структуры) при этом не устраняется.

    Например, специальный коэффициент рождаемости также зависит от возрастного состава продуцирующего контингента (женщины 15-49 лет).

    Если в этом контингенте высока доля женщин от 20 до 29 лет, которые часто рожают детей, то специальный коэффициент рождаемости будет относительно высоким.

    Если же в этом контингенте высока доля женщин от 40 до 49 лет, которые очень редко рожают детей, то специальный коэффициент рождаемости будет относительно низким.

    Однако есть еще и третий (и самый главный) недостаток общих коэффициентов. Он состоит в том, что ни один из общих коэффициентов не дает представления об основных параметрах того демографического процесса, к которому он относится.

    Общий коэффициент рождаемости не дает представления о том, сколько детей рожает одна женщина за всю жизнь. Специальный коэффициент рождаемости (в расчете на 1000 женщин 15-49 лет) тоже не дает представления об этом. Для этого требуется другой показатель – СУММАРНЫЙ

    КОЭФФИЦИЕНТ РОЖДАЕМОСТИ. Он характеризует среднее число детей, которых одна женщина рожает за всю жизнь.

    Общий коэффициент смертности (число умерших на 1000 населения) не дает представления о том, в каком возрасте умирают люди. Для этого существует другой показатель – СРЕДНЯЯ ОЖИДАЕМАЯ ПРОДОЛЖИТЕЛЬНОСТЬ ЖИЗНИ.

    Общий коэффициент брачности (число браков на 1000 населения) не дает представления о действительно важных параметрах брачности, то есть, о том, в каком возрасте мужчины и женщины вступают в брак, какая часть мужчин и женщин не вступает в брак вообще, в течение всей жизни, а какая часть мужчин и женщин вступает в брак по нескольку раз. Для этого используются другие показатели.

    Общий коэффициент разводимости не дает представления о том, какая часть браков заканчивается разводами. Чтобы получить представление об этом, надо сравнивать число разводов не с общей численностью населения, а с числом браков, или разделить общий коэффициент разводимости (в 2010 г. – 4,5‰) на общий коэффициент брачности (в 2010 г. – 8,5‰)

    Коэффициент неустойчивости браков = Число разводов на 100 браков =

    = число разводов/число браков =

    = общий коэффициент разводимости/общий коэффициент брачности =

    = 4,5:8,5≈0,52 или 52%

    Коэффициент неустойчивости браков=×1000‰

    В России в последние годы на каждые 100 браков приходится от 50 до 60 разводов, то есть большая часть браков заканчивается разводами.

    Поэтому общие коэффициенты могут использоваться лишь на начальном этапе демографического анализа, главным образом для изучения динамики рождаемости, смертности, брачности и разводимости в одной стране или регионе. Если эти коэффициенты за один год повышаются или понижаются на 0,3‰ и более, это свидетельствует о реальном повышении или понижении интенсивности данных демографических процессов. Изменение коэффициентов на 0,1-0,2‰ не надо воспринимать всерьез, поскольку оно может быть вызвано влиянием изменений в возрастном составе населения, который за один год в одной стране сильно измениться не может.

    С ПОМОЩЬЮ ОБЩИХ КОЭФФИЦИЕНТОВ, КАК ПРАВИЛО, НЕЛЬЗЯ СРАВНИВАТЬ РАЗНЫЕ СТРАНЫ ИЛИ РЕГИОНЫ, поскольку возрастной состав их населения может сильно различаться. Такое сравнение допустимо лишь в тех случаях, когда один из сравниваемых общих коэффициентов превосходит другой в несколько раз.

    Если в книге или в статье, посвященной проблемам демографии, которые в наше время вызывают всеобщий интерес, приводится только общая численность населения, темпы его роста и прироста, абсолютные числа родившихся и умерших, а также вышеупомянутые общие коэффициенты рождаемости, смертности или естественного прироста, и не приводится никаких других демографических показателей, то сразу понятно, что АВТОР ЭТОГО ТЕКСТА НЕ ЯВЛЯЕТСЯ ПРОФЕССИОНАЛЬНЫМ ДЕМОГРАФОМ.

    УРОВНИ СЛОЖНОСТИ ДЕМОГРАФИЧЕСКИХ КОЭФФИЦИЕНТОВ (уровни глубины демографического анализа)

    studfiles.net

    Коэффициент рождаемости — это… Что такое Коэффициент рождаемости?

    Абсолютные демографические показатели

    Общая численность населения

    P= P0 + (N — M) + (V+ — V) = P0 + E + Vпр (уравнение демографического баланса)

    P — общая численность населения
    P0 — численность населения на начало года
    N — общее число родившихся
    M — общее число умерших
    E — естественный прирост населения
    V+ — число прибывших
    V — число выбывших

    Общий прирост населения

    P1 — P0 = Pпр
    Р0 — численность населения на начало периода (обычно год)
    Р1 — на конец периода

    Естественный прирост населения

    N — M = E
    N — общее число родившихся
    M — общее число умерших
    Значение показателя может быть отрицательным, если имеет место естественная убыль населения (в России с 1992)

    Миграционное сальдо (чистая миграция)

    V+ — V = Vпр
    V+ — число прибывших (иммигранты)
    V — число выбывших (эмигранты)

    V = O — E (косвенный метод)

    O = P1 — P0

    В отношении прибывших и выбывших во многих странах существует недоучет, в США эмигрантов вообще не учитывают. Тогда ищут сальдо косвенно из уравнения демографического баланса, предложенного ООН в 1960-х.

    Доля женщин репродуктивного возраста

    Общие демографические коэффициенты

    Для общих коэффициентов характерно: стоящее в числителе число демографических событий относится ко всему населению, а не только к той его части, которая порождает данное событие; при этом наступление данного события не уменьшает величину знаменателя.

    Коэффициенты рождаемости и смертности

    ;
    ;
    Nx — число рождений за данный год
    Мx — число смертей за данный год
    Px — общая численность населения

    Специальный коэффициент рождаемости

    W15-49 — средняя численность женщин репродуктивного возраста
    см. Коэффициент фертильности

    Коэффициент интенсивности рождений

    Nx — число рождений у женщин возраста x лет
    Wx — их среднегодовая численность

    Коэффициент младенческой смертности

    ; (формула Ратса)

    ;

    M0 — число умерших в возрасте от 0 до 1 года
    M0-1 — число детей, умерших в возрасте до года из числа родившихся в предыдущем году
    N0 — число родившихся в отчетном году
    N-1 — число родившихся в предыдущем году

    Коэффициент брачности

    B — общее число браков
    P — среднее население в трудоспособном возрасте

    Коэффициент разводимости

    R — общее число разводов

    Индекс разводимости

    Показатель средней продолжительности предстоящей жизни

    Tx — число человеколет, которое предстоит прожить после достижения точного возраста x лет lx — число доживающих до возраста x лет

    Подробнее о показателе: Ожидаемая продолжительность жизни

    Wikimedia Foundation. 2010.

    dic.academic.ru

    Решенные задачи к экзамену по общественному здоровью и здравоохранению, страница 3

    Для решения задачи необходимо рассчитать следующие коэффициенты.

    1. Общий коэффициент рождаемости = Общее число родившихся живыми за год   =

                                                                          Среднегодовая численность населения

    =  = 11, 2‰.

    2. Коэффициент плодовитости (фертильности)=Общее число родившихся живыми за год=

                                                                                                           Среднегодовая численность женщин детородного

                                                                                                            (фертильного) возраста (15-49 лет)

    = = 2,07%

    3. Суммарный коэффициент рождаемости равен сумме возрастных коэффициентов рождаемости, рассчитанным по однолетним или пятилетним возрастным группам.:

    (25,5*5 + 159*5 + 126*5 + 97*5 + 50*5 + 19,1*5 + 4,4*5)/1000 = 2,405.

    4. Брутто-коэффициент воспроизводства женского населения ― это число девочек, рожденных одной женщиной за весь репродуктивный период жизни. Он равен произведению суммы возрастных коэффициентов рождаемости на долю девочек среди родившихся в те годы, для которых вычислен коэффициент.

    = 2,405 · 0,477 = 1,15.

    5. Смертность(общий коэффициент смертности) =  Общее число умерших за год_0    =                                                                                                                     .                                                                                      Среднегодовая численность населения

    =  = 14,9%0.

    6. Естественный прирост (убыль) населения:

    Коэффициент естественного прироста = Общий коэффициент рождаемости ― Общий коэффициент смертности =

    = 11,2 %о-14,9 %о = -3,74‰.

    7.                                Число детей, умершие на 1-м году жизни

    Младенческая =                         в течение года_____________     х 1000 =

    смертность              Число родившихся живыми в данном году

    =  = 23,2‰.

    8. Мертворождаемость =   Родились мертвыми в течение года    х 1000 =

    Родились живыми и мертвыми

    =  = 16,05‰.

    9.                                        Число родившихся     Число умерших в пер

    Перинатальная =           мертвыми         + вые 168 часов жизни   х  1000 =

    смертность          Общее число родившихся живыми и мертвыми

    =  = 22,3‰.

    10. Неонатальная        Число умерших в первые четыре недели

    смертность    =    жизни ребенка (до 28 дней) в данном году_   х 1000 =

                                  Число родившихся живыми в данном году

    =  = 16,3‰.

    11. Ранняя неонаталь-       Число умерших в возрасте 0-7 дней

    ная смертность       =  (до 168 часов) в данном году_________ х 1000 =

    (постнатальная)          Число родившихся живыми в течение года

    =  = 6,32‰.

    12. Поздняя неонаталь- 

    ная смертность  (на        =    Число детей, умерших на 2-4 неделе жизни  х 1000 =

    2-4 неделе жизни)                  Число детей, ро-      —   Число детей, умерших

                                                      дившихся живыми       в 1-ю неделю жизни

    =  = 10,1‰.

    13. Постнеона-         Число детей, умерших в период с 29 дня

    тальная       =                                   до 1 года жизни______________   х 1000 =

    смертность        Число детей, родив-   —    Число детей, умерших в

                                    шихся живыми               первые 4 недели жизни

    =  = 6,96‰.

    № 2

    Статистика населения и медицинская демография, Шаршакова, Дорофеев, 2009

    Общий коэффициент рождаемости

    Общее число родившихся живыми за год 

                                                                   Среднегодовая численность населения

    Коэффициент плодовитости или фертильности

            Общее число родившихся живыми за год_____

                                                             Среднегодовая численность женщин детородного

                                                             (фертильного) возраста (15-49 лет)

     Суммарный коэффициент рождаемости равен сумме возрастных коэффициентов рождаемости, рассчитанным по однолетним или пятилетним возрастным группам.

    Общий коэффициент смертности

                     Общее число умерших за год____  

    Среднегодовая численность населения

    Величины общего коэффициента смертности оцениваются по специальной шкале.

    Таблица 5 ― Величины общего коэффициента смертности.

    Общий коэффициент смертности,  0/00

    Оценка уровня смертности

    до 10

    Низкий

    10,0 — 14,9

    Средний

    15,0 – 24,9

    Высокий

    25,0 – 34,9

    Очень высокий

    35,0 и выше

    Чрезвычайно высокий

    Брутто-коэффициент воспроизводства женского населения ― это число девочек, рожденных одной женщиной за весь репродуктивный период жизни. Он равен произведению суммы возрастных коэффициентов рождаемости на долю девочек среди родившихся в те годы, для которых вычислен коэффициент.

    Коэффициент младенческой смертности

                                                   Число детей, умершие на 1-м году жизни

    Младенческая =                         в течение года_____________ х 1000

    смертность              Число родившихся живыми в данном году

    Шкала для оценки общего коэффициента младенческой смертности

    Коэффициент младенческой смертности, 0/00

    Оценка уровня младенческой смертности

    до 7,0

    Низкий

    7,0–9,9

    Средний

    10,0–14,9

    Высокий

    15,0–19,9

    Очень высокий

    20 и выше

    Чрезвычайно высокий

    Антенна-           Число родившихся мертвыми за год (или число

    тальная      =  _умерших до родов после 22 недель беременности) х 1000

    смертность      Общее число родившихся живыми и мертвыми

    Интранатальная  =  _______Число умерших в родах за год_______ х 1000

    смертность               Общее число родившихся живыми и мертвыми

                                   Число родившихся     Число умерших в пер-

    Перинатальная=         мертвыми          +   вые 168 часов жизни  х 1000

    vunivere.ru

    81. Рождаемость населения, репродуктивное поведение. Показатели рождаемости (методика расчета).

    Рождаемость — процесс деторождения в совокупности людей, составляющих поколение, или в совокупности поколений.

    Биологическая основа рождаемости – это способность человека к воспроизводству потомства. Потенциальная возможность деторождения – плодовитость, реализуется в совокупности женщин в результате репродуктивного поведения, которое в обществе детерминируется системой социально обусловленных потребностей и регулируется социальными и культурными нормами, религиозными традициями, общественным мнением и другими факторами.

    Для определения интенсивности процесса рождения обычно пользуются показателями рождаемости.

    1.Общий показатель рождаемости. Средняя численность населения за год вычисляется как сумма чисел населения на первое число каждого месяца, деленная на 12, или, или как полусумма чисел на начало и конец года.

    Как любой общий коэффициент он обеспечивает только приближенное ориентировочное представление об интенсивности явления во времени и пространстве и в значительной мере связан с возрастно-половым составом населения и исчисляется по отношению к численности всего населения; тогда как рожают только женщины, и не во всяком возрасте.

    Общее число родившихся живыми за год

    ———————————————————— • 1000

    Среднегодовая численность населения

    2. Коэффициент плодовитости. Это специальный показатель, он дает более точные характеристики рождаемости. Рассчитывается на женщин репродуктивного возраста.

    Репродуктивный возраст (синоним – генеративный) – это возраст женщины, в котором она способна к деторождению. Указанием границ репродуктивного возраста в демографии характеризуется продолжительность репродуктивного периода. Как правило, под репродуктивным возрастом для женщин понимается возраст 15 – 49 лет.

    От доли женщин в репродуктивном возрасте зависит общее число родившихся и общий коэффициент рождаемости. Чем больше эта доля, тем, при прочих равных условиях, больше общее число родившихся и общий коэффициент рождаемости.

    3. Показатели плодо¬витости: уточняют показатель рождаемости, для этого при расчете весь репродуктивный период женщины условно подразделяют на отдельные интервалы (15—19, 20—24, 30—34, 35—39, 40—44, 45—49 лет).

    1. Показатель общей плодовитости:

    Общее число родившихся живыми за год

    —————————————————————————- • 1000

    Средне¬годовая численность женщин в возрасте 15 — 49 лет

    2. Показатель повозрастной плодовитости:

    Общее число родившихся живыми за год

    у женщин соот¬ветствующего возраста

    ————————————————————— • 1000

    Среднегодовая численность женщин

    соответствующего возраста

    4. Суммарный коэффициент рождаемости показывает, сколько в среднем детей родила бы одна женщина на протяжении всей ее жизни при сохранении в каждом возрасте существу¬ющего уровня рождаемости. Вычисляется как сумма возрастных коэффициентов рождаемости, рассчитанных по одногодичным возрастным группам, не зависит от возрастного состава населения и характеризует средний уровень рождаемости в данный календарный период.

    Так как практически в процессе рождаемости участвует не все население, и реально рождения происходят у женщин определенного возраста, то более точное представле¬ние дают специальные коэффициенты рождаемости — коэф¬фициенты плодовитости. Они вычисляются либо как общий показатель (число рождений на 1000 женщин репродуктивного возраста, т.е. от 15 до 49 лет), либо в виде коэффициентов повозрастной плодо¬витости, для чего весь генеративный период женщины условно подразделяют на отдельные интервалы (15-19, 20-24, 30-34, 35-39, 40-44, 45-49 лет). Число рождений до и после этого возрастного интервала незначительно, и им можно пренебречь.

    studfiles.net

    4.2.2. Система показателей рождаемости

    Измерение рождаемости производится с помощью системы показателей, включающей:

    1. Абсолютное число рождений.

    2. Коэффициенты рождаемости.

    Абсолютное число рождений – число родившихся детей в том или ином населении (или в какой-либо его группе) за определённый период (обычно за год).

    По абсолютному числу рождений нельзя судить об уровне рождаемости (много родилось детей или мало). Для этого существуют относительные показатели – коэффициенты рождаемости.

    Коэффициент рождаемости – отношение абсолютного числа родившихся детей у данного населения (или какой-либо его группы) к средней (обычно среднегодовой) численности этого населения (или его группы).

    Рассчитывают следующие коэффициенты рождаемости:

    — общий;

    — специальные;

    — частные.

    Более достоверными показателями рождаемости являются специальные и особенно – частные коэффициенты рождаемости. Специальные и частные коэффициенты рождаемости рассчитываются для групп населения, которые обладают характеристиками, имеющими непосредственное отношение к рождаемости (пол, возраст, брачное состояние).

    Для более точной оценки коэффициентов рождаемости используют промилле, т.е. расчёт ведётся на 1000 человек. При этом учитывается только число живорождённых детей.

    4.2.3. Коэффициенты рождаемости

    1. Общий коэффициент рождаемости (n) – отношение числа родившихся детей в исследуемом периоде (обычно за календарный год) к средней (обычно среднегодовой) численности населения.

    , (4.1)

    где N – число родившихся детей в исследуемом периоде;

    T – число лет в исследуемом периоде;

    –среднегодовая численность населения в исследуемом периоде.

    Если T = 1 году, то (4.2)

    Таблица 4.1

    Шкала оценки рождаемости

    (авторы Урланис Б.Ц. и Борисов В.А.)

    Общий коэффициент рождаемости

    Оценка рождаемости

    менее 16

    16-24

    25-29

    30-39

    40 и более

    низкая

    средняя

    выше средней

    высокая

    очень высокая

    Задание 4.1. Требуется определить общий коэффициент рождаемости, если известно, что в течение года родилось 40 тыс. детей, а среднегодовая численность населения равняется 7200 тыс. человек (исходные данные условные).

    Решение:

    Используя формулу 4.2, получаем:

    ,

    т.е. на тысячу человек данного населения приходится в среднем 5,6 рождённых в течение года детей.

    В соответствии со шкалой оценки рождаемости, представленной в таблице 4.1, рождаемость в данном населении низкая.

    Общий коэффициент рождаемости даёт обобщающую характеристику, но не учитывает специфики рождаемости, т.е. того обстоятельства, что рожают детей женщины.

    2. Специальный коэффициент рождаемости (F) – отношение числа родившихся (обычно за год) к среднегодовой численности женщин репродуктивного возраста (от 15 до 49 лет).

    F = , (4.3)

    где – среднегодовая численность женщин репродуктивного возраста.

    Между общим коэффициентом рождаемости и специальным коэффициентом рождаемости существует взаимосвязь:

    n=, (4.4)

    где – доля женщин репродуктивного возраста во всём населении.

    Представленная мультипликативная модель позволяет демографам проводить факторный анализ, т.е. оценивать изменение общего коэффициента рождаемости под влиянием каждого фактора в отдельности: специального коэффициента рождаемости и доли женщин репродуктивного возраста во всём населении.

    Задание 4.2. Требуется определить специальный коэффициент рождаемости, если известно, что общий коэффициент рождаемости равен 5,6%о, а удельный вес женщин репродуктивного возраста во всём населении составляет 30%.

    Решение:

    Преобразуя формулу 4.4, получаем:

    ,

    т.е. на тысячу женщин репродуктивного возраста данного населения приходится в среднем 18,7 рождённых в течение года детей.

    Возрастные границы 15-49 лет – это дань традиции, установлены ещё в 19 веке и связаны с категорией плодовитости (фертильности). Поэтому специальный коэффициент рождаемости часто называют коэффициентом фертильности (плодовитости). Но фактическая рождаемость, например, для развитых стран связана с более сжатыми возрастными границами женщин: 22-35 лет. Поэтому возникает потребность в показателях более частного порядка. Частные коэффициенты рассчитываются для отдельных групп населения. Наиболее известные из них – половозрастные (или просто – возрастные) коэффициенты рождаемости. Они учитывают уже две характеристики населения, которые имеют прямое отношение к рождаемости – пол (женский) и возраст (матери).

    3. Половозрастной (возрастной) коэффициент рождаемости (F) – отношение числа родившихся детей у женщин возраста «x» к среднегодовой численности женщин этой группы.

    F =, (4.5)

    где N – число родившихся детей у женщин возраста «x»;

    –среднегодовая численность женщин возраста «x».

    Половозрастные (возрастные) коэффициенты рождаемости учитывают пол, возраст, но не учитывают третью немаловажную характеристику населения, так же имеющую непосредственное отношение к рождаемости – брачное состояние. Поэтому возникает потребность в следующей группе коэффициентов рождаемости.

    4. Коэффициенты брачной и внебрачной рождаемости рассчитываются дифференцированно для женщин состоящих и не состоящих в браке.

    Специальный коэффициент брачной рождаемости – отношение числа родившихся детей у замужних женщин к среднегодовой численности замужних женщин.

    Половозрастной (возрастной) коэффициент брачной рождаемости – отношение числа родившихся детей у определённой возрастной группы замужних женщин к среднегодовой численности этой группы.

    Аналогично рассчитываются коэффициенты внебрачной рождаемости.

    Коэффициенты брачной и внебрачной рождаемости рассчитывают в нашей стране в основном по данным специальных выборочных обследований. Динамика этих коэффициентов свидетельствует о качественных изменениях в нормах демографического поведения, изменении семейных ценностей, института брака в целом.

    Получить дополнительную информацию о динамике рождаемости позволяют коэффициенты рождаемости, дифференцированные по очерёдности рождения (см. следующую группу коэффициентов).

    5. Коэффициенты рождаемости по очерёдности рождения рассчитываются для женщин по порядку рождения у них детей: как для всех женщин репродуктивного возраста (специальные коэффициенты рождаемости по очерёдности рождения), так и для отдельных возрастных групп женщин (половозрастные коэффициенты рождаемости по очерёдности рождения).

    Специальный коэффициент рождаемости по очерёдности рождения – отношение числа рождений детей i-й очерёдности к среднегодовой численности женщин репродуктивного возраста.

    Как видим из определения этого показателя, сумма специальных коэффициентов рождаемости по очерёдности рождения равна просто специальному коэффициенту рождаемости.

    Половозрастные коэффициенты рождаемости по очерёдности рождения – отношение числа рождений детей i-й очерёдности у женщин возраста «x» к среднегодовой численности женщин этой группы.

    studfiles.net

    5.6. Суммарный коэффициент рождаемости

    Теперь вернемся снова к возрастным коэффициентам рождаемости. Если иметь в виду упомянутые трудности с использованием коэффициен­тов брачной рождаемости, то обычные, не дифференцированные по брач­ному состоянию возрастные коэффициенты остаются наилучшими показа­телями уровня рождаемости, дающими хорошие возможности для анализа его состояния и динамики. Как уже отмечалось, их достоинством является независимость от влияния возрастной структуры внутри женского репродуктивного возрастного контингента. Но и у них есть недостаток, который состоит в том, что их много. При использовании однолетних коэффициен­тов их будет целых 35 (от 15 до 49 лет включительно).

    В случае использования пятилетних коэффициентов их число уже значи­тельно меньше — 7, но все же их остается еще много для обозрения. Причем динамика коэффициентов может быть различной, иногда до противополож­ности. В самом деле, тенденция динамики коэффициентов в большинстве экономически развитых стран в нынешнем веке была такой, что в младших возрастных группах коэффициенты рождаемости росли, в то время как в старших — снижались. Иногда, глядя на картину динамики возрастных ко­эффициентов рождаемости, трудно решить, что же все-таки происходит — снижается рождаемость или растет. И некоторые научные спекулянты небес­корыстно пользуются этим обстоятельством, утверждая, будто рождаемость в нашей стране не снижается. Нужен один обобщающий показатель, который соединял бы в себе достоинства целой системы показателей. И такой показа­тель есть. Его зовут — суммарный коэффициент рождаемости.

    Суммарный коэффициент рождаемости вычисляется путем суммирования возрастных коэффициентов рождаемости с умножением их на длину каждого возрастного интервала в целых годах (при однолетних коэффици­ентах множитель равен 1, при пятилетних — 5, и т. д.). Сумма в итоге де­лится на 1000, т.е. показатель выражается в расчете на одну женщину в среднем. Формула расчета такова:

    (5.6.1)

    где СКР — суммарный коэффициент рождаемости; Fx возрастные коэффициенты; n — длина возрастного интервала (при одинаковой длине ин­тервала, его можно вынести за знак суммы, т.е. сначала сложить коэффициенты, а затем один раз умножить сумму коэффициентов на длину возрастного интервала. Если же интервалы разные по длине (редко, но бывает), то придется каждый коэффициент умножать отдельно на соответствующую ему длину возрастного интервала).

    Суммарный коэффициент рождаемости является одним из сводных, итоговых показателей, которые строятся как по методу реального, так и условного поколения. Приведенная выше формула расчета суммарного коэффициента относится к условному поколению, т.е., мы рассматриваем все возрастные коэффициенты рождаемости, относящиеся к разным реальным поколениям женщин, условно как относящиеся к одному поколению, будто бы прожившему в данном одном календарном году, в году наблюде­ния, всю свою репродуктивную жизнь, с 15 до 50 лет.

    Суммарный коэффициент рождаемости показывает, сколько де­тей рожает в среднем одна женщина за всю свою жизнь с 15 до 50 лет при условии, что на всем протяжении репродуктивного периода жиз­ни данного поколения возрастные коэффициенты рождаемости в каждой возрастной группе остаются неизменными на уровне расчетного периода1.

    Рассмотрим пример расчета суммарного коэффициента рождаемости (см. таблицу 5.1).

    Таблица 5.1

    studfiles.net

    Y x x2 4: Исследовать функцию и построить ее график : y= x/x²-4

    Сдвиги графиков функций

    Изменение значения k влияет на вид графика (степень крутизны в случае параболы), расположение ветвей в координатных четвертях и др. Однако точкой, через которую можно провести ось симметрии графиков, является точка O с координатами (0; 0).

    Если же рассматривать функций, подобные перечисленным выше, у которых к переменной x или ко всей исходной функции прибавляется (или вычитается) какое-либо число, то графики этих функций остаются такими же как у исходных, однако смещаются относительно точки (0; 0).

    Если обозначить исходные функции как y = f(x), то прибавление к x числа дает функции вида y = f(x+l), а прибавление ко всей исходной функции значения дает вид y = f(x) + m.

    Например, если исходная функция y = 2x2, то примером первого типа будет функция y = 2(x+5)2, а второго — y = 2x2 + 5.

    Для функций вида y = f(x+l) график смещается влево на l единиц, если l прибавляется. Если же l вычитается, то график смещается вправо. Действительно, представим параболу функции y = x2 и сравним ее с функцией y = (x+1)2. Когда x = 1, то для первой функции y = 1, а для второй — y = 4. Когда x = 0, для первой y = 0, для второй y = 1. Когда x = –1, для первой y = 1, для второй y = 0.

    То есть график второй функции касается оси x в точке (–1; 0). Это значит, что график смещен влево по сравнению с исходным на 1.

    Для функций вида y = f(x) + m график соответствующей функции y = f(x) смещается на m единиц, но уже по вертикальной оси (ось y). Здесь если m прибавляется, то график сдвигается вверх. Если m вычитается, то график сдвигается вниз.

    Рассмотрим ту же параболу y = x2 и функцию y = x2 + 1. Когда x = 0, первая принимает значение 0, а у второй y = 1. Получить у второй функции значение y, которое равно 0, вообще невозможно. Это значит, что парабола имеет точку симметрии с координатами (0; 1), т. е. сдвинута от исходной вверх на 1.

    «Смешанные» функции вида y = f(x + l) + m сдвигаются вдоль оси x и y. Вдоль оси x они сдвигаются на l, а вдоль y — на значение m.

    Урок 13. многочлены от нескольких переменных — Алгебра и начала математического анализа — 10 класс

    Алгебра и начала математического анализа, 10 класс

    Урок №13. Многочлены от нескольких переменных.

    Перечень вопросов, рассматриваемых в теме

    1) определение многочлена от нескольких переменных;

    2) понятие симметрических многочленов;

    3) формулы сокращенного умножения для старших степеней;

    4) бином Ньютона;

    5) метод неопределенных коэффициентов.

    Глоссарий по теме

    Многочлен Р(х;у) называют однородным многочленом n-й степени, если сумма показателей степеней переменных в каждом члене многочлена равна n. Если Р(х;у) — однородный многочлен, то уравнение Р(х;у) = 0 называют однородным уравнением.

    Многочлен Р(х;у) называют симметрическим, если он сохраняет свой вид при одновременной замене х на у и у на х.

    Уравнение Р(x;y) = а, где , называютсимметрическим, если Р(х;y) — симметрический многочлен.

    Треугольник Паскаля —бесконечная таблица биномиальных коэффициентов, имеющая треугольную форму. В этом треугольнике на вершине и по бокам стоят единицы. Каждое число равно сумме двух расположенных над ним чисел. Строки треугольника симметричны относительно вертикальной оси. Назван в честь Блеза Паскаля. 

    Основная литература:

    Колягин Ю.М., Ткачева М.В, Федорова Н.Е. и др., под ред. Жижченко А.Б. Алгебра и начала математического анализа (базовый и профильный уровни) 10 кл. – М.: Просвещение, 2014.

    Дополнительная литература:

    Шабунин М.И., Ткачева М.В., Федорова Н.Е. Дидактические материалы Алгебра и начала математического анализа (базовый и профильный уровни) 10 кл. – М.: Просвещение, 2017.

    Теоретический материал для самостоятельного изучения

    Многочлены от нескольких переменных можно складывать, вычитать, перемножать, возводить в натуральную степень, разлагать на множители — это вам известно из курса алгебры 7—9-го классов. Этот урок позволит нам несколько расширить знания о многочленах.

    Пример 1. Разложить на множители многочлен: 2x2-5xy+2y2.

    Воспользуемся методом группировки

    2x2-5xy+2y2= 2x2-4xy-xy+2y2= 2x(x-2y) –y(x-2y)=

    (x-2y)(2x+2y).

    Пример 2. Выведем формулу сокращенного умножения для «квадрата суммы» (x+y+z+u)2.

    (x+y+z+u)2=((x+y)+(z+u))2= (x+y)2+2(x+y)(z+u)+(z+u)2= x2+y2+z2+u2+2(xy+xz+xu+yz+yu+zu).

    Итак, мы получили (x+y+z+u)2= x2+y2+z2+u2+2(xy+xz+xu+yz+yu+zu).

    Среди многочленов от двух переменных выделяют однородные и симметрические многочлены.
     
    Многочлен Р(х;у) называют однородным многочленом n-й степени, если сумма показателей степеней переменных в каждом члене многочлена равна n. Если Р(х;у) — однородный многочлен, то уравнение Р(х;у) = 0 называют однородным уравнением.

    Приведем примеры.

    1) р(х; у)=2х+3у – однородный многочлен первой степени; соответственно 2х+3у=0 – однородное уравнение первой степени.

    2) р(х; у)=3х2+5ху-7у2  — однородный многочлен второй степени; соответственно 3х2+5ху-7у2 =0 — однородное уравнение второй степени.

    3) p(x; y)= x3+4xy2-5y3 — однородный многочлен третьей степени; x3+4xy2-5y3 =0 соответственно  — однородное уравнение третьей степени.

    4) p(x; y)= anxn+an-1xn-1y+an-2xn-2y2+…+a1xyn-1+a0yn — общий вид однородного многочлена n-й степени.

    Рассмотрим еще один метод разложения многочленов на множители-

    метод неопределенных коэффициентов. Суть метода неопределённых коэффициентов состоит в том, что вид сомножителей, на которые разлагается данный многочлен, угадывается, а коэффициенты этих сомножителей (также многочленов) определятся путём перемножения сомножителей и приравнивания коэффициентов при одинаковых степенях переменной. Теоретической основой метода являются следующие утверждения

    1. Два многочлена равны тогда и только тогда, когда равны их коэффициенты.
    2. Любой многочлен третьей степени имеет хотя бы один действительный корень, а потому разлагается в произведение линейного и квадратичного сомножителя.
    3. Любой многочлен четвёртой степени разлагается в произведение многочленов второй степени.

    Пример 3.  Разложить на множители многочлен

    3 x 3 – x 2 – 3 x + 1.

    Решение. Поскольку многочлен третьей степени разлагается в произведение линейного и квадратичного сомножителей, то будем искать многочлены x – p и ax 2 + bx + c такие, что справедливо равенство 3 x 3 – x 2 – 3 x + 1 = ( x – p )( ax 2+ bx + c ) = ax 3 + ( b – ap ) x 2 + ( c – bp ) x – pc . Приравнивая коэффициенты при одинаковых степенях в левой и правой частях этого равенства, получаем систему четырех уравнений для определения четырех неизвестных коэффициентов:

    Решая эту систему, получаем: a = 3, p = –1, b = 2, c = –1. Итак, многочлен 3 x 3 – x 2 – 3 x + 1 разлагается на множители: 3 x 3 – x 2 – 3 x + 1 = ( x – 1)(3 x 2 + 2 x – 1).

    Стоит отметить, что существует достаточно изящный способ решения однородных уравнений. Поясним его суть на примере.

    Пример 4. Решим уравнение x3+4xy2-5y3 =0

    Заметим, что если в заданном уравнении взять х=0, то получится у=0; это означает, что пара (0; 0) является решением однородного уравнения. Пусть теперь х. Разделим почленно обе части заданного однородного уравнения на х3, получим:

    Введем новую переменную . Тогда уравнение примет вид 1+4z2-5z3=0.

    Далее последовательно находим:

    5z3-4z2-1=0

    (5z3-5z2)+(z2-1)=0

    5z2(z-1)+(z-1)(z+1)=0

    (z-1)(5z2+z+1)=0

    Из уравнения z-1=0 находим z=1, уравнение 5z3-4z2-1=0 действительных корней не имеет.

    Если z=1, то , т.е. у=х. Это значит, что любая пара вида (t; t) является решением заданного однородного уравнения. Между прочим, и отмеченная нами ранее пара (0; 0) также входит в указанный перечень решений.

    Ответ: (t; t), где t- любое действительное число.

    Теперь поговорим о симметрических многочленах. Многочлен Р(х;у) называют симметрическим, если он сохраняет свой вид при одновременной замене х на у и у на х. Например, симметрическим является двучлен x2y+xy2. В самом деле, при одновременной замене х на у и у на х получится двучлен y2x+yx2, но это то же самое, что x2y+xy2 . Другие примеры симметрических многочленов: xy, x+y, x2+y2, x3+y3, x4+y4 и т.д. Первые два из записанных многочленов считаются основными в том смысле, что любые другие симметрические многочлены можно представить в виде некоторой комбинации многочленов х + у и ху.

    Теорема. Любой симметрический многочлен Р(х;у) можно представить в виде многочлена от ху и х+у.

    Например,

    x2+y2=(x+y)2-2xy

    x3+y3=(x+y)3-3xy(x+y)

    x4+y4= 2xy(x2+y2)-(x4+y4)+3(xy)2 и т.д.

    Уравнение Р(x;y) = а, где , называют симметрическим, если Р(х;y) — симметрический многочлен. Мы с вами рассматривали его на предыдущем уроке.

    А теперь перейдем к такому понятию как бином Ньютона.

    Слово бином означает «Два числа». В математике биномом называют «формулу для разложения на отдельные слагаемые целой неотрицательной степени суммы двух переменных». Бином Ньютона — название формулы, выражающей степень двучлена в виде суммы одночленов.

    Давайте вслед за Ньютоном попробуем ее вывести, чтобы затем применять.

    Вы наверняка помните (или, по крайней мере, должны помнить), формулы сокращенного умножения для квадрата и куба суммы двух слагаемых (такая сумма называется «бином», по-русски – двучлен.

    (a+b)2=a2+2ab+b2

    (a+b)3=a3+3a2b+3ab2+b3

    Если вы забыли эти формулы, можно их получить напрямую, раскрыв скобки в очевидных равенствах

    (a+b)2=(a+b)(a+b)

    (a+b)3=(a+b)(a+b)(a+b)

    Может быть, вам приходил в голову вопрос: можно ли (без компьютера) получить формулы типа для биномов четвертой степени, пятой, десятой – какой угодно?

    Давайте попробуем дойти напрямую хотя бы до пятой степени, а там, может быть, окажется «рояль в кустах» (для порядка будем размещать слагаемые в правой части по убыванию степени а, она убывает от максимума до нуля):

    (a+b)4=(a+b)3(a+b)=(a3+3a2b+3ab2+b3)(a+b)=a4+4a3b+6a2b2+4ab3+b4

    (a+b)5=(a+b)4(a+b)=(a4+4a3b+6a2b2+4ab3+b4)(a+b)=a5+5a4b+10a3b2+10a2b3+5ab4+b5

    Теперь отдельно выпишем численные коэффициенты в правых частях формул при возведении бинома в заданную степень:

    n=2 1,2,1

    n=3 1,3,3,1

    n=4 1,4,6,4,1

    n=5 1,5,10,5,1

    Легко проверить, что выписанные на численные коэффициенты – это строчки треугольника Паскаля, начиная с третьей. Этот «усеченный треугольник», в котором не хватает первых двух строк, легко сделать полным (получить строчки при n=0 и n=1):

    n=0, (a+b)0=1

    n=1, (a+b)1=a+b

    Окончательно получим:

    n=0 1

    n=1 1,1

    n=2 1,2,1

    n=3 1,3,3,1

    n=4 1,4,6,4,1

    n=5 1,5,10,5,1

    Общая формула бинома Ньютона:

    .

    Правая часть формулы называется разложением степени бинома.

     — называется биномиальными коэффициентами, а все слагаемые — членами бинома.

    Треугольник Паскаля — бесконечная таблица биномиальных коэффициентов, имеющая треугольную форму. В этом треугольнике на вершине и по бокам стоят единицы. Каждое число равно сумме двух расположенных над ним чисел. Строки треугольника симметричны относительно вертикальной оси. Назван в честь Блеза Паскаля. 

    На самом деле, о треугольнике Паскаля было известно задолго до Паскаля — его знал живший в XI-XII вв. среднеазиатский математик и поэт Омар Хайям (к сожалению, его сочинение об этом до нас не дошло). Первое, дошедшее до нас описание формулы бинома Ньютона содержится в появившейся в 1265 г. книге среднеазиатского математика ат-Туси, где дана таблица чисел   (биномиальных коэффициентов) до n=12 включительно.

    Европейские ученые познакомились с формулой бинома Ньютона, по-видимому, через восточных математиков. Детальное изучение свойств биномиальных коэффициентов провел французский математик и философ Б. Паскаль в 1654 г.

    В заключении рассмотрим пример, в котором использование бинома Ньютона позволяет доказать делимость выражения на заданное число.

    Пример 5.

    Доказать, что значение выражения 5n+28n-1, где n – натуральное число, делится на 16 без остатка.

    Решение: представим первое слагаемое выражение как 5n= (4+1)n и воспользуемся формулой бинома Ньютона:

    Полученное произведение доказывает делимость исходного выражения на 16. 

    Бином Ньютона применяется при доказательстве Теоремы Ферма, в теории бесконечных рядов и выводе формулы Ньютона-Лейбница

    Примеры и разборы решения заданий тренировочного модуля

    №1.

    Из данных многочленов выделите симметрические:

    1. 2-5ху+2у2-6
    2. 6x⁴-16xy²-6y3+19
    3. -3ху+6х²-5у²+8
    4. 16x4y²+16x²y4-x⁴-y⁴

    Решение: к данному заданию применим определение симметрических многочленов (Многочлен Р(х;у) называют симметрическим, если он сохраняет свой вид при одновременной замене х на у и у на х). Получим, что нам подходят 1 и 4 пункты.

    Верный ответ:

    1. 2-5ху+2у2-6
    2. 6x⁴-16xy²-6y3+19
    3. -3ху+6х²-5у²+8
    4. 16x4y²+16x²y4-x⁴-y⁴

    №2.

    (а+b)5= __a5 +___a4b+___a3b2+___a2b3+___ab4+__b5

    Решение: для решения данного задания воспользуемся треугольником Паскаля

    1    
    1    1    
    1    2    1    
    1    3    3    1    
    1    4    6    4    1    
    1    5    10    10    5    1

    Нас интересует последняя строчка.

    Применив ее, получим ответ:

    (а+b)5= 1a5 +5a4b+10a3b2+10a2b3+5ab4+1b5

    Применение табличного процессора Excel для графического решения уравнений n-й степени

    Цели урока:

    1. Формирование умений и навыков, носящих в современных условиях общенаучный и обще интеллектуальный характер.
    2. Развитие у школьников теоретического, творческого мышления, а также формирование операционного мышления, направленного на выбор оптимальных решений.
    3. Научить учащихся применять современное программное обеспечение в решении нестандартных задач.
    4. Повторение пройденного материала.

    Задачи урока:

    1. Воспитательная – развитие познавательного интереса, воспитание информационной культуры.
    2. Учебная – изучить и закрепить основные навыки работы с электронными таблицами.
    3. Развивающая – развитие логического мышления, расширение кругозора.

    Оборудование: персональные компьютеры (ПК), раздаточный материал, доска, маркеры, проектор.

    План урока
    1. Организационный момент.
    2. Фронтальный опрос для проверки уровня подготовки учащихся к усвоению нового материала.

      1) Какие дополнительные возможности есть у программы Excel?
      2) Как вы понимаете термин деловая графика?
      3) Какими возможностями для создания деловой графики обладает Excel?
      4) При помощи какой команды меню можно построить диаграммы и графики в Excel?
      5) Как задать автоматическое вычисление в таблице значений ячеек по определенной формуле?
      6) Каким образом можно занести формулу в несколько ячеек, т.е. скопировать ее?

    3. Объяснение нового материала. Проводится одновременно с работой учеников на ПК синхронно с учителем.
    4. Самостоятельная работа учащихся на компьютерах.
    5. Сравнение результатов, полученных графическим способом (Excel) и аналитическим (Qbasic).
    6. Выполнение индивидуальных заданий.
    7. Подведение итогов.
    8. Выставление оценок.
    9. Домашнее задание.

    Ход урока

    1. Организационный момент.

    2. Фронтальный опрос.

    1) Для чего нужна программа Excel?

    Ответ: для создания таблиц, вычисляемых таблиц, диаграмм и графиков (деловой графики).

    2) Какими возможностями для создания деловой графики обладает Excel?

    Ответ: с помощью библиотеки диаграмм можно составлять диаграммы и графики разных видов (гистограммы, круговые диаграммы, столбчатые, графики и др.), их можно снабжать заголовками и пояснениями, можно задавать цвет и вид штриховки в диаграммах, редактировать их, печатать их на бумаге, изменяя размеры и расположение на листе, вставлять диаграммы в нужное место листа.

    3) При помощи какой команды меню можно построить диаграммы и графики в Excel?

    Ответ: с помощью вызова Мастера диаграмм (по команде Вставка-Диаграмма или с помощью кнопки Мастер диаграмм).

    4) Как задать автоматическое вычисление в таблице значений ячеек по определенной формуле?

    Ответ: активизировать нужную ячейку, затем ввести знак «=» и формулу, которая может содержать адреса ячеек, знаки арифметических операций и функции. Контролировать и редактировать ввод формулы можно с помощью строки ввода формулы, которая расположена в верхней части окна программы.

    5) Каким образом можно занести формулу в несколько ячеек, т.е. скопировать ее.

    Ответ: ввести формулу в ячейку, установить курсор на нижнем правом маркере ячейки (при этом курсор должен принять вид маленького черного крестика) и протянуть его до последней ячейки в нужном диапазоне.

    3. Объяснение нового материала (проводится одновременно с работой учеников на компьютерах синхронно с учителем).

    Тема урока «Применение табличного процессора Excel для графического решения уравнений n-ой степени».

    Из курса математики нам известно, что корнями уравнения являются значения точек пересечения графика функции (то есть нашего уравнения) с осью абсцисс. Если же мы решаем систему уравнений, то ее решениями будут координаты точек пересечения графиков функций. Этот метод нахождения корней называется графическим. На прошлом занятии мы узнали, что с помощью программы Excel можно строить практически любые графики. Воспользуемся этими знаниями для нахождения корней системы уравнений графическим методом.

    Для примера рассмотрим решение следующей системы уравнений:

    Y — X2 = 0
    Y – 2X = 9

    Преобразуем данную систему в приведенную:

    Y = X2
    Y = 2X + 9

    Для оценки решений воспользуемся диаграммой, на которой отобразим графики обеих функций. Сначала построим таблицу 1 (рисунок 1).

    Таблица 1

    Рисунок 1

    • Первая строка – строка заголовков. Далее для построения таблицы используем формулы.
    • При заполнении столбца А: в ячейку А2 заносится начальное значение аргумента Х=-10, для автоматического заполнения всего столбца нужно в ячейку А3 занести формулу А2+1 и скопировать ее до ячейки А23.
    • При заполнении столбца В в ячейку В2 заносится формула А2*А2, которая затем копируется до ячейки В23.
    • При заполнении столбца С в ячейку С2 заносится формула 2*А2+9, и также копируется до С23.
    • Выделяем таблицу вместе со строкой заголовка и помощью мастера диаграмм выберем тип диаграмм Точечная и построим черновую диаграмму первоначальной оценки решений.
    • Вводим заголовок «Диаграмма оценки решения» и обозначения осей x, y (поле ввода текста).
    • Добавляем основные линии сетки по оси X и по оси Y (выставляем флажки).
    • Размещаем легенду справа от графиков (выставляем флажок «добавить легенду» и включаем переключатель «размещение справа»).
    • Размещаем графики на имеющемся листе.
    • Подписываем лист 1 «Диаграмма оценки решения» (рисунок 2).

    Диаграмма оценки решения

    Рисунок 2

    На диаграмме видно, что оба графика имеют точки пересечения – эти координаты точек и есть решения системы. Так как шаг изменения аргумента был достаточно велик, то мы получили приближенные значения решений. Уточним их, построив два графика в интервалах от –3 до 0, где находится первое решение, и от 3 до 5 – где находится второе. Составим новые таблицы.

    Для первого решения (таблица 2, рисунок 3).

    Таблица 2

    Рисунок 3

    • При заполнении столбца А: в ячейку А2 заносится начальное значение аргумента Х=-3, для автоматического заполнения всего столбца нужно в ячейку А3 занести формулу А2+0,1(в этом случае мы уменьшаем шаг изменения аргумента для более точного построения) и скопировать ее до ячейки А23.
    • При заполнении столбца В в ячейку В2 заносится формула А2*А2, которая затем копируется до ячейки В23.
    • При заполнении столбца С в ячейку С2 заносится формула 2*А2+9, и также копируется до С23.
    • Выделяем таблицу вместе со строкой заголовка и помощью мастера диаграмм
    • выберем тип диаграмм Точечная и построим диаграмму для первого решения.
    • Вводим заголовок «Первое решение» и обозначения осей x, y (поле ввода текста).
    • Добавляем основные линии сетки по оси X и по оси Y (выставляем флажки).
    • Размещаем легенду справа от графиков (выставляем флажок «добавить легенду» и включаем переключатель «размещение справа»).
    • Размещаем графики на имеющемся листе.
    • Подписываем лист 2 «Первое решение» (рисунок 4).

    Первое решение

    Рисунок 4

    4. Самостоятельная работа.

    Для второго решения ребята самостоятельно строят таблицу (таблица 3, рисунок 5), выбрав правильно промежуток. Затем по таблице строят диаграмму для второго решения (рисунок 6). Учитель проходит и проверяет правильность выполнения работы. И если нужна помощь, то в индивидуальном порядке оказывает ее.

    Таблица 3

    Рисунок 5

    Второе решение

    Рисунок 6

    Решением нашей системы будут координаты точек пересечения графиков: X1=-2,1; Y1=4,8; X2=4,2; Y2=17,4.

    Как вы уже поняли, графическое решение системы дает приблизительные результаты.

    5. Сравнение результатов, полученных графическим способом (Excel) и аналитическим (Qbasic).

    Учитель предлагает решить данную систему уравнений аналитическим способом, используя ранее полученную на уроках информатики программу решения квадратного уравнения.2-4*a*c
    IF d<0 THEN PRINT «Решений нет»: GOTO 90
    IF d=0 THEN x=-b/(2*a): PRINT «x=»; x: GOTO 90
    X1=(-b-SQR(d))/(2*a)
    X2=(-b+SQR(d))/(2*a)PRINT “x1=”; x1, “x2=”; x2

    90 END

    Подставив коэффициенты в программу, получаем точное значение абсцисс:

    x1=-2,162278
    x2= 4,162278

    Сравниваем решения системы, полученные графическим способом и аналитическим. Делаем выводы.

    6. Выполнение индивидуальных заданий.

    1. C помощью табличного процессора Excel решить графически систему:

    Y=X2
    Y=4X+12

    2. C помощью табличного процессора Excel решить графически систему:

    Y=X2+5
    Y=6X+12

    3. C помощью табличного процессора Excel решить графически систему:

    Y=X2+4
    Y=X+12

    4. C помощью табличного процессора Excel решить графически систему:

    Y=X2+5
    Y=4X+4

    5. C помощью табличного процессора Excel решить графически систему:

    Y=X2+5
    Y=3X+12

    6. C помощью табличного процессора Excel решить графически систему:

    Y=X3+5
    Y=2X2+4X+12

    7. C помощью табличного процессора Excel решить графически систему:

    Y=X2
    Y=8X+12

    8. C помощью табличного процессора Excel решить графически систему:

    Y=X3+5
    Y=X+12

    9. C помощью табличного процессора Excel решить графически систему:

    Y=X2+3
    Y=5X+1

    10. C помощью табличного процессора Excel решить графически систему:

    Y=X2+2
    Y=X+12

    11. C помощью табличного процессора Excel решить графически систему:

    Y=X3+5
    Y=4X2+12

    12. C помощью табличного процессора Excel решить графически систему:

    Y=X3+5
    Y=X2+4X+12

    7. Подведение итогов.

    Распечатка отчетов.

    8. Выставление оценок.

    9. Домашнее задание.

    Проанализировать и проверить свои индивидуальные задания и оформить отчеты на листочках.

    Ограничить область определения значения, обратного полиномиальной функции

    Пример 7: Нахождение области определения радикальной функции, составленной с помощью рациональной функции

    Найдите область определения функции [латекс] f \ left (x \ right) = \ sqrt {\ frac {\ left (x + 2 \ right) \ left (x — 3 \ right)} {\ left (x — 1 \ right)}} [/ латекс].

    Решение

    Поскольку квадратный корень определяется только тогда, когда величина под радикалом неотрицательна, нам нужно определить, где [латекс] \ frac {\ left (x + 2 \ right) \ left (x — 3 \ right)} { \ left (x — 1 \ right)} \ ge 0 [/ латекс].Выходные данные рациональной функции могут менять знаки (с положительных на отрицательные или наоборот) на x -перехваченных точках и на вертикальных асимптотах. Для этого уравнения график может менять знаки при x = –2, 1 и 3.

    Чтобы определить интервалы, на которых рациональное выражение является положительным, мы могли бы проверить некоторые значения в выражении или нарисовать график. Хотя оба подхода работают одинаково хорошо, для этого примера мы будем использовать график.

    Рисунок 9

    Эта функция имеет два перехвата x , оба из которых демонстрируют линейное поведение рядом с перехватами x .Имеется одна вертикальная асимптота, соответствующая линейному множителю; это поведение аналогично базовой функции обратного инструментария, и здесь нет горизонтальной асимптоты, потому что степень числителя больше степени знаменателя. В точке (0, 6) находится перехват y .

    Из интервала y и интервала x при x = –2, мы можем нарисовать левую часть графика. По поведению на асимптоте мы можем нарисовать правую часть графика.

    Теперь по графику мы можем сказать, на каких интервалах выходы будут неотрицательными, так что мы можем быть уверены, что исходная функция f ( x ) будет определена. f ( x ) имеет домен [latex] -2 \ le x <1 \ text {или} x \ ge 3 [/ latex], или в обозначении интервала, [latex] \ left [-2,1 \ right) \ чашка \ left [3, \ infty \ right) [/ латекс].

    Нахождение точек пересечения по уравнению

    X Intercept : график уравнения пересекает ось x

    Y Intercept : график уравнения пересекает ось Y

    Чтобы найти перехватчики:

    Если вы хотите, чтобы x перехватывал (x, 0):

    Установите y = 0, затем решите относительно x

    Если вы хотите, чтобы y перехватывал (0, y):

    Установите x = 0, затем решите относительно y

    Пример: найти точки пересечения y = x

    2 — 4

    x пересечение: установить y = 0

    0 = х 2 — 4

    x 2 = 4

    x = 2 или −2

    Точки: (2,0) и (−2,0)

    Перехват y: установить x = 0

    y = 0 2 — 4

    г = −4

    Точка (0, -4)

    А вот график x 2 — 4, чтобы подтвердить то, что мы нашли:

    Пример: найти точки пересечения x

    2 — 5x + y 2 + 3y = 0

    x пересечение: установить y = 0

    x 2 — 5x + 0 + 0 = 0

    х (х − 5) = 0

    x = 0 и 5

    Точки: (0,0) и (5,0)

    Перехват y: установить x = 0

    0 — 0 + y 2 + 3y = 0

    г (у + 3) = 0

    y = 0 или −3

    Точки: (0,0) и (0, −3)

    Итак, всего 3 балла:

    (0,0), (5,0) и (0, −3)

    А вот и график… это круг!

    колледж-алгебра — ближайшая точка

    Общественный колледж Меса


    Концепции алгебры колледжа — MAT 150 онлайн

    Задача: Найдите точку на графике кривой y = x 2 + 1, ближайшую к фиксированной точке (4,1).

    Предпосылки: Эта проблема относится к классу задач, обычно называемых проблемами минимизации, минимальными / максимальными проблемами или проблемами экстремумов. Курсы математического анализа традиционно решают этот тип проблем, задав любой из следующих вопросов:
    • Каковы размеры правильного круглого цилиндра с объемом 1000 см 3 , который минимизирует площадь поверхности?
    • Водопровод должен быть проложен из точки P в точку S и должен проходить через регионы, где затраты на строительство различаются (представлена ​​диаграмма).Найдите путь, по которому должен пройти инженер, чтобы затраты на прокладку трубы были минимальными.
    • Тренажерный зал состоит из прямоугольной области с полукругом на каждом конце. Если по периметру комнаты должна быть беговая дорожка длиной 200 метров, найдите размеры комнаты, которые сделают площадь прямоугольной области как можно большей.

    В каждой задаче мы пытаемся найти наименьшее или наибольшее значение. Хотя исчисление можно использовать для нахождения точных решений этих проблем, мы будем использовать алгебру и наши графические калькуляторы для аппроксимации решения.Часть нашего решения, относящаяся к алгебре, на самом деле такая же, как если бы мы использовали исчисление.

    Переформулировка задачи: Найдите точку (x, y) на графике кривой y = x 2 + 1, ближайшую к фиксированной точке (4,1).

    Сначала давайте нарисуем график, чтобы получить более четкое представление о том, что происходит.

    Мы пытаемся найти точку A (x, y) на графике параболы y = x 2 + 1, которая ближе всего к
    точке B (4,1).

    Переформулировка задачи: Найдите точку A (x, y) на графике параболы, y = x 2 + 1, что минимизирует расстояние d между кривой и точка B (4,1).

    Проблема разбивается на три части:
    1. Поскольку мы хотим минимизировать d, нам нужна функция, описывающая расстояние между (x, y) и (4,1).

    2.Найдите значение x, которое минимизирует d.

    3. Найдите точку (x, y) на графике параболы y = x 2 + 1, которая минимизирует d.

    Часть 1:

    Постройте функцию, описывающую расстояние между (x, y) и (4,1).

    По «distance forumla» расстояние d между (x, y) и (4,1) равно

    Для любой точки (x, y) на параболе y = x 2 + 1,

    Подставляя y в формулу для d, получаем

    Упрощение,

    Часть 2:
    Используя графический калькулятор, мы зарисовываем график d и ищем точку, в которой встречается минимальное значение d.

    Используя функцию трассировки, мы можем увидеть, что минимальное значение d происходит (приблизительно) в точке

    (x, d ) = (1,12817, 3,4123 )

    То есть d — это минимум , когда x составляет приблизительно 1,12817

    Кратчайшее расстояние между B и параболой составляет примерно 3.14123

    Часть 3:
    Помните, что мы предполагаем, что найдет точку (x, y) на графике параболы, y = x 2 + 1, что минимизирует d . Пока мы знаем x и знаем d, но мы еще не нашли y.

    Используя уравнение для параболы, y = x 2 + 1 и значение x, которое минимизировало d, (x = 1,12817), получаем y = 2,27277 (приблизительно).

    Итак, чтобы ответить на исходный вопрос, ближайшая точка параболы y = x 2 + 1 к точке (4,1) приблизительно равна (1.12817, 2.27277).

    © 1999 Джо Стейг


    Решение линейно-квадратичных систем

    Вы, наверное, решили системы линейных уравнений. Но как насчет системы двух уравнений, в которой одно уравнение является линейным, а другое — квадратичным?

    Мы можем использовать версию метода подстановки для решения систем этого типа.

    Помните, что уравнение прямой имеет вид y = mx + b, а стандартная форма уравнения параболы с вертикальной осью симметрии — y = ax2 + bx + c, a ≠ 0.

    Чтобы избежать путаницы с переменными, запишем линейное уравнение в виде y = mx + d, где m наклон и d является точкой пересечения оси Y линии.

    Подставляем выражение для y из линейного уравнения в квадратное уравнение. То есть подставляем mx + d для тебя в y = ax2 + bx + c .

    мх + д = ах2 + Ьх + с

    Теперь перепишите новое квадратное уравнение в стандартной форме.

    Вычесть mx + d с обеих сторон.

    (mx + d) — (mx + d) = (ax2 + bx + c) — (mx + d) 0 = ax2 + (b − m) x + (c − d)

    .

    Теперь у нас есть квадратное уравнение с одной переменной, решение которого можно найти с помощью формулы корней квадратного уравнения.

    Решения уравнения ax2 + (b − m) x + (c − d) = 0 даст x-координаты точек пересечения графиков прямой и параболы.Соответствующие координаты y могут быть найдены с помощью линейного уравнения.

    Другой способ решения системы — построить график двух функций на одной и той же координатной плоскости и определить точки пересечения.

    Пример 1:

    Найдите точки пересечения прямой y = 2x + 1 и парабола y = x2−2.

    Замена 2x + 1 для y в y = x2−2.

    2x + 1 = x2−2

    Напишите квадратное уравнение в стандартной форме.

    2x + 1−2x − 1 = x2−2−2x − 10 = x2−2x − 3

    Используйте формулу корней квадратного уравнения, чтобы найти корни квадратного уравнения.

    Здесь a = 1, b = −2, c = −3.

    x = — (- 2) ± (−2) 2-4 (1) (- 3) 2 (1) = 2 ± 4 + 122 = 2 ± 42 = 3, −1

    Подставьте значения x в линейное уравнение, чтобы найти соответствующие значения y.

    x = 3⇒y = 2 (3) +1 = 7x = −1⇒y = 2 (−1) +1 = −1
    Следовательно, точки пересечения равны (3,7) и (−1, −1).

    Постройте параболу и прямую линию на координатной плоскости.


    Аналогичный метод можно использовать для поиска точек пересечения прямой и окружности.

    Пример 2:

    Найдите точки пересечения прямой y = −3x и окружность x2 + y2 = 3.

    Заменитель −3x для тебя в x2 + y2 = 3 .

    x2 + (- 3x) 2 = 3

    Упростить.

    x2 + 9×2 = 310×2 = 3×2 = 310
    Извлечение квадратного корня, x = ± 310.

    Подставьте значения x в линейное уравнение, чтобы найти соответствующие значения y.
    x = 310⇒y = −3 (310) = −3310x = −310⇒y = −3 (−310) = 3310

    Следовательно, точки пересечения (310, −3310) и (-310, 3310).

    Постройте окружность и прямую линию на координатной плоскости.


    … или линия и эллипс.

    Пример 3:

    Решите систему уравнений y = −5 и x29 + y24 = 1.

    Заменитель −5 для тебя в −5.

    x29 + (- 5) 24 = 1

    Упростить.

    x29 + (- 5) 24 = 14×236 + 9 (25) 36 = 14×2 + 225 = 364×2 = −189×2 = −1894

    Здесь у нас есть отрицательное число как квадрат числа. Итак, два уравнения не имеют реальных решений.

    Постройте эллипс и прямую линию на координатной плоскости.

    Мы видим, что два не пересекаются.

    переводов графа — темы в предварительном исчислении

    17

    Перевод графика

    Переводы параболы

    Вершина параболы

    Уравнение окружности

    Вертикальное растяжение и сжатие

    ПЕРЕВОД ГРАФИКИ — это его жесткое движение по вертикали или горизонтали.

    Слева — график функции абсолютного значения. Справа его перевод в «новое происхождение» в (3, 4).

    Уравнение функции абсолютного значения:

    y = | x |.

    Уравнение его перевода в (3, 4):

    y — 4 = | x — 3 |.

    Например, когда x = 3, тогда y -4 = 0, то есть y = 4.

    Таким образом, точка (3, 4) — это та точка на транслированном графе, которая изначально находилась в (0, 0).

    В целом

    Если график
    y = f ( x )
    переводится на a единицы по горизонтали и b единицы
    вертикально, затем уравнение переведенного
    график
    y b = f ( x a ).

    Когда f ( x ) переводится в на единицы по горизонтали, тогда аргумент f ( x ) становится x a . В приведенном выше примере аргумент | x | становится x — 3.

    Мы докажем это ниже.

    Пример 1. Напишите уравнение этого графика:

    Ответ . y — 3 = | x + 5 |.

    График абсолютного значения был переведен на 3 единицы вверх, но на 5 единиц до осталось . a = −5. Следовательно, x , а становится

    .

    x — (−5) = x + 5.

    Задача 1. Напишите уравнение этого графика:

    Чтобы увидеть ответ, наведите указатель мыши на цветную область.
    Чтобы закрыть ответ еще раз, нажмите «Обновить» («Reload»).

    y + 3 = — | x + 4 |.

    Мало того, что график абсолютного значения был переведен, он сначала был отражен относительно оси x .

    Тема 15.

    Перевод — это жесткое движение графика. График , отраженный , представляет собой жесткое движение y = — | x |.Таким образом, отражение происходит до преобразования в x = −4. Другими словами, если вы записали неотраженный перевод в (−3, −4) как

    y = | x + 4 | — 3,

    , а затем записал отражение о оси x как

    y = — | x + 4 | + 3,

    , что было бы неправильно. Вы могли видеть это, потому что, когда x = −4, y не равно −3.

    Задача 2. Нарисуйте график

    .

    y = | x — 3 |.

    Задача 3. Нарисуйте график

    .

    y = — | x + 2 |.

    Задача 4. Нарисуйте график

    .

    y = — | x — 3 | + 2.

    Это эквивалентно y — 2 = — | x — 3 |.

    График отображается относительно оси x и переводится в (3, 2).

    Задача 5. Нарисуйте график y =.

    Задача 6. Нарисуйте график y = -.

    Это функция квадратного корня, переведенная на 3 единицы влево.

    Задача 7. Нарисуйте график y = 1 — x 2 .

    Это эквивалентно y — 1 = — x 2 , что является отраженной параболой, переведенной на 1 единицу вверх.

    Пример 2. Вершина параболы. Напишите уравнение параболы (со старшим коэффициентом 1), вершина которой находится в точке ( a , b ).

    Ответ . y b = ( x a ) 2 . Это перевод y = x 2 в ( a , b ).

    Задача 8. Напишите уравнение параболы, вершина которой находится в точке

    .
    а) (1, 2) y — 2 = ( x — 1) 2
    б) (-1, 2) y — 2 = ( x + 1) 2
    в) (1, −2) y + 2 = ( x — 1) 2

    Пример 3.Каковы координаты вершины этой параболы?

    y = x 2 + 6 x + 9
    Решение . Чтобы ответить, мы должны сделать уравнение таким:
    y b = ( x a ) 2

    Тогда вершина будет в точке ( a , b ).

    Теперь, x 2 + 6 x + 9 — это полный квадрат ( x + 3):

    y = x 2 + 6 x + 9 = ( x + 3) 2 .

    Следовательно, a = −3 и b = 0. Вершина находится в точке (−3, 0.)

    Пример 4. Каковы координаты вершины этой параболы?

    y = x 2 + 5

    Решение .Опять же, мы должны сделать уравнение таким:

    y b = ( x a ) 2 .

    Если просто переставить 5 —

    y -5 = x 2

    — мы видим, что a = 0, а b = 5. Вершина находится в точке (0, 5).

    Пример 5. Завершение квадрата. Каковы координаты вершины этой параболы?

    y = x 2 + 6 x −2
    Решение .Сделать такую ​​форму —
    y b = ( x a ) 2

    — постоянный член транспонируем, а квадрат справа заполним.

    y + 2 = x 2 + 6 x
    Завершите квадрат, добавив 9 к обеим сторонам:
    y + 2 + 9 = x 2 + 6 x + 9
    y + 11 = ( x + 3) 2

    Вершина находится в точке (−3, −11).

    Задача 9. Каковы координаты вершины этой параболы?

    y = x 2 -10 x + 25

    Правая часть представляет собой идеальный квадрат ( x — 5).

    y = ( x -5) 2

    Таким образом, вершина находится в точке (5, 0).

    Проблема 10.Каковы координаты вершины этой параболы?

    y = x 2 — 1

    Из уравнения следует

    y + 1 = x 2 .

    Вершина находится в точке (0 −1).

    Задача 11. Каковы координаты вершины этой параболы?

    y = x 2 -8 x + 1

    Переставьте постоянный член и заполните квадрат справа:

    y — 1 = x 2 -8 x
    y — 1 + 16 = x 2 — 8 x + 16
    y + 15 = ( x -4) 2

    Вершина находится в точке (4, −15).

    Уравнение окружности

    Что характеризует каждую точку ( x , y ) на окружности круга?

    Каждая точка ( x , y ) находится на одинаковом расстоянии r от центра. Следовательно, согласно формуле расстояния Пифагора для расстояния точки от начала координат:

    x 2 + y 2 = r 2 .

    Это уравнение окружности радиуса r с центром в начале координат (0, 0).

    Конкретно это —

    x 2 + y 2 = 25

    — уравнение окружности радиуса 5 с центром в начале координат.

    Каждая пара значений ( x , y ), которая решает это уравнение, то есть делает его истинным утверждением, будет координатами точки на окружности.

    Вопрос. Каково уравнение окружности с центром в точке ( a , b ) и радиусом r ?

    Ответ . ( x a ) 2 + ( y b ) 2 = r 2 .

    Круг был переведен с (0, 0) на ( a , b ).

    Проблема 12.Напишите уравнение окружности радиуса 3 с центром в следующей точке.

    а) (1, 2) ( x — 1) 2 + ( y — 2) 2 = 9
    б) (-1, -2) ( x + 1) 2 + ( y + 2) 2 = 9
    в) (1, −2) ( x — 1) 2 + ( y + 2) 2 = 9

    Пример 6.Покажите, что это уравнение круга. Назовите радиус и координаты центра.

    x 2 -4 x + y 2 -2 y = 11

    Решение . Чтобы показать, что что-то является уравнением круга, мы должны показать, что оно может иметь такую ​​форму:

    ( x a ) 2 + ( y b ) 2 = r 2 .

    Таким образом, мы завершим квадрат как x , так и y .

    Чтобы завершить квадрат размером x , мы прибавим 4 к обеим сторонам.

    Чтобы завершить квадрат y , мы прибавим 1 к обеим сторонам.

    ( x 2 -4 x + 4) + ( y 2 -2 y + 1) = 11 + 4 + 1
    ( x -2) 2 + ( y -1) 2 = 16.

    Это уравнение окружности радиуса 4, центр которой находится в точке (2, 1).

    Тогда мы можем сказать, что когда квадратичный в x плюс квадратичный в y равен числу —

    x 2 -4 x + y 2 -2 y = 11

    — тогда это уравнение круга.

    Коэффициенты при x 2 и y 2 равны 1.И число должно быть больше, чем минус суммы квадратов половин коэффициентов x и y .

    Задача 13. Покажите, что это уравнение круга. Назовите радиус и координаты центра.

    x 2 + 6 x + y 2 + 10 y — 2 = 0

    Переставьте постоянный член и заполните квадрат как x , так и y .Добавьте одинаковые квадратные числа с обеих сторон:

    ( x 2 + 6 x + 9 ) + ( y 2 + 10 y + 25 ) = 2 + 9 + 25
    ( x + 3) 2 + ( y + 5) 2 = 36

    Это уравнение круга радиуса 6 с центром в (−3, −5).

    Вот доказательство основной теоремы.

    Теорема. Если график y = f ( x ) переведен на a единиц по горизонтали и b единиц по вертикали, то уравнение переведенного графика будет

    y b = f ( x a ).

    Ибо в переводе каждая точка на графике перемещается одинаково.Пусть ( x 1 , y 1 ), тогда будут координатами любой точки на графике y = f ( x ), так что

    y 1 = f ( x 1 ).

    А переведем график на единиц по горизонтали и на единиц по вертикали, так что x 1 перейдет в точку

    x 1 + a ,

    и y 1 переходит в точку

    y 1 + b .

    Если a — положительное число, то эта точка будет справа от x 1 , а если a отрицательное число, то она будет слева. Точно так же, если b — положительное число, тогда y 1 + b будет больше y 1 , а если b отрицательно, оно будет ниже.

    Теперь, каким будет уравнение переведенного графика, когда значение x в уравнении равно x 1 + a , значение y будет y 1 + б ?

    Мы говорим, что следующее уравнение:

    y b = f ( x a ).

    Для, когда x = x 1 + a :

    y b = f ( x 1 + a a ) = f ( x 1 ) = y 1 1 1

    y = y 1 + b .

    И ( x 1 , y 1 ) — любая точка на графике y = f ( x ).Следовательно, уравнение переведенного графика —

    .

    y b = f ( x a ).

    Что мы и хотели доказать.

    Вертикальное растяжение и сжатие

    Если мы умножим функцию f ( x ) на число c — получим c f ( x ) — каков будет эффект на графике?

    Если мы умножим f ( x ) на число больше 1 — как на графике в центре — то каждое значение y будет растянуто; на этом графике в 2 раза.

    Но если мы умножим f ( x ) на число меньше 1 — как на графике справа — то каждое значение y уменьшится; в этом графике в ½ раза.

    Следующая тема: Рациональные функции

    Содержание | Дом


    Сделайте пожертвование, чтобы TheMathPage оставалась в сети.
    Даже 1 доллар поможет.


    Авторские права © 2021 Лоуренс Спектор

    Вопросы или комментарии?

    Электронная почта: themathpage @ яндекс.com


    Перемещение графика

    Перемещение графика

    Переместите график, напишите уравнение

    А. Переместить график на 2 единицы вверх

    Переместите исходный график y = x до 2 шт. Результирующий график y = x + 2.

    Переместите исходный график y = Abs (x) до 2 шт. Результирующий график y = Abs (x) +2.

    Переместите исходный график y = x 2 до 2 шт. Результирующий график y = х 2 +2

    Переместите исходный график y = sin (x) до 2 шт. Результирующий график y = 2 + sin (x).

    Переместите исходный график y = x 3 до 2 шт.Результирующий график y = х 3 +2.

    Переместить исходный график круга х 2 + у 2 = 4 на 2 единицы измерения. Полученный график представляет собой круг x 2 + (у-2) 2 = 4.

    Переместить исходный график эллипса

    x 2 /9 + y 2 /4 = от 1 до 2 единиц.Результирующий график — это эллипс

    x 2 /9 + (y-2) 2 /4 = 1

    Переместите исходный график гиперболы x 2 /9 — y 2 /4 = от 1 до 2 единиц. Результирующий график — это гипербола

    х 2 /9 — (у-2) 2 /4 = 1

    Переместите исходный график экспоненты функция y = 2 x до 2 единиц.Результирующий график представляет собой экспоненциальную функцию у = 2 х + 2.

    Сдвинуть график вправо 2 единиц:

    Переместить оригинал график г = x вправо на 2 единицы.Результирующий график: y = x- 2 .

    Переместить оригинал график г = lxl вправо 2 шт. Результирующий график: y = lx-2l.

    Переместить оригинал график г = x 2 вправо 2 шт.Результирующий график

    у = (х — 2) 2 .

    Переместить оригинал график г = sin (x) вправо на 2 единицы. Результирующий график г = грех (х-2).

    Переместить оригинал график г = x 3 вправо 2 шт.Результирующий график: y = (x ‘2) 3 900 18.

    Переместить оригинал график круга x 2 + y 2 = 9 до правые 2 шт. Полученный график представляет собой окружность (x — 2) 2 + y 2 = 9 .

    Переместить оригинал график эллипса

    x 2 /9 + y 2 /4 = 1 справа 2 шт. Результирующий график — эллипс

    (x-2) 2 /9 + y 2 /4 = 1

    Переместить оригинал график гиперболы x 2 / 9 — y 2 /4 = 1 справа 2 шт.Результирующий граф — это гипербола

    (х — 2) 2 /9 — y 2 / 4 = 1

    Переместить оригинал график экспоненциальной функции y = 2 x вправо 3 шт.Полученный график представляет собой экспоненциальную функцию

    .

    г = 2 (x- 3 ) .

    Назад к Паттерны математики
    Для загрузки Материалы Дона
    Mathman home

    Инверсия функции — объяснение и примеры

    Что такое обратная функция?

    В математике обратная функция — это функция, отменяющая действие другой функции.

    Например, , сложение и умножение являются инверсией соответственно вычитания и деления.

    Обратную функцию можно рассматривать как отражение исходной функции по линии y = x. Проще говоря, обратная функция получается заменой (x, y) исходной функции на (y, x).

    Мы используем символ f — 1 для обозначения обратной функции. Например, если f (x) и g (x) противоположны друг другу, то мы можем символически представить это утверждение как:

    g (x) = f — 1 (x) или f (x) = g −1 (x)

    Следует отметить, что обратная функция — это не то же самое, что обратная функция, т.е.е., f — 1 (x) ≠ 1 / f (x). В этой статье мы обсудим, как найти обратную функцию.

    Поскольку не все функции имеют инверсию, важно проверить, есть ли у функции инверсия, прежде чем приступать к определению инверсии.

    Мы проверяем, есть ли у функции инверсия, чтобы не тратить время на поиск чего-то, чего не существует.

    Индивидуальные функции

    Итак, как мы можем доказать, что данная функция имеет обратную? Функции, у которых есть обратные, называются взаимно однозначными функциями.

    Функция называется взаимно однозначной, если для каждого числа y в диапазоне f существует ровно одно число x в области определения f такое, что f (x) = y.

    Другими словами, домен и диапазон однозначной функции имеют следующие отношения:

    • Область f −1 = Диапазон f.
    • Диапазон f -1 = Область f.

    Например, чтобы проверить, является ли функция f (x) = 3x + 5 взаимно однозначной заданной, f (a) = 3a + 5 и f (b) = 3b + 5.

    ⟹ 3a + 5 = 3b + 5

    ⟹ 3a = 3b

    ⟹ а = б.

    Следовательно, f (x) является взаимно однозначной функцией, потому что a = b.

    Рассмотрим другой случай, когда функция f задается формулой f = {(7, 3), (8, –5), (–2, 11), (–6, 4)}. Эта функция взаимно однозначна, потому что ни одно из ее значений y не встречается более одного раза.

    А как насчет этой другой функции h = {(–3, 8), (–11, –9), (5, 4), (6, –9)}? Функция h не является взаимно однозначной, потому что значение y, равное –9, встречается более одного раза.

    Вы также можете графически проверить взаимно однозначную функцию, проведя вертикальную и горизонтальную линии через график функции. Функция взаимно однозначна, если и горизонтальная, и вертикальная линии проходят через график один раз.

    Как найти обратную функцию?

    Найти инверсию функции — несложный процесс, хотя нам действительно нужно быть осторожными с парой шагов. В этой статье мы будем предполагать, что все функции, с которыми мы будем иметь дело, относятся друг к другу.

    Порядок нахождения обратной функции f (x):

    • Заменить обозначение функции f (x) на y.
    • Поменять местами x на y и наоборот.
    • Начиная с шага 2, решите уравнение относительно y. Будьте осторожны с этим шагом.
    • Наконец, измените y на f −1 (x). Это обратная функция.
    • Вы можете проверить свой ответ, проверив, верны ли следующие два утверждения:

    ⟹ (f ∘ f −1 ) (x) = x

    ⟹ (f −1 ∘ f) (x) = x

    Давайте поработаем пару примеров.

    Пример 1

    Дана функция f (x) = 3x — 2, найти обратную ей.

    Решение

    f (x) = 3x — 2

    Заменить f (x) на y.

    ⟹ у = 3х — 2

    Поменять местами x на y

    ⟹ x = 3y — 2

    Решить для y

    х + 2 = 3 года

    Разделим на 3, чтобы получить;

    1/3 (х + 2) = у

    х / 3 + 2/3 = у

    Наконец, заменим y на f −1 (x).

    f −1 (x) = x / 3 + 2/3

    Проверить (f ∘ f −1 ) (x) = x

    (f ∘ f −1 ) (x) = f [f −1 (x)]

    = е (х / 3 + 2/3)

    ⟹ 3 (х / 3 + 2/3) — 2

    ⟹ х + 2–2

    = х

    Следовательно, f −1 (x) = x / 3 + 2/3 — правильный ответ.

    Пример 2

    Дано f (x) = 2x + 3, найти f −1 (x).

    Решение

    f (x) = y = 2x + 3

    2x + 3 = y

    Поменять местами x и y

    ⟹2y + 3 = х

    Теперь решите для

    у.

    ⟹2y = х — 3

    ⟹ у = х / 2 — 3/2

    Наконец, заменим y на f −1 (x)

    ⟹ f −1 (x) = (x– 3) / 2

    Пример 3

    Задайте функцию f (x) = log 10 (x), найдите f −1 (x).

    Решение

    f (x) = log₁₀ (x)

    Заменено f (x) на y

    ⟹ y = журнал 10 (x) ⟹ 10 y = x

    Теперь поменяйте местами x на y, чтобы получить;

    ⟹ y = 10 x

    Наконец, заменим y на f −1 (x).

    f -1 (x) = 10 x

    Следовательно, обратное значение f (x) = log 10 (x) равно f -1 (x) = 10 x

    Пример 4

    Найдите обратную функцию к следующей функции g (x) = (x + 4) / (2x -5)

    Решение

    г (x) = (x + 4) / (2x -5) ⟹ y = (x + 4) / (2x -5)

    Обмен y с x и наоборот

    y = (x + 4) / (2x -5) ⟹ x = (y + 4) / (2y -5)

    ⟹ х (2у-5) = у + 4

    ⟹ 2xy — 5x = y + 4

    ⟹ 2xy — y = 4 + 5x

    ⟹ (2x — 1) y = 4 + 5x

    Разделите обе части уравнения на (2x — 1).

    ⟹ у = (4 + 5x) / (2x — 1)

    Заменить y на g -1 (x)

    = г — 1 (x) = (4 + 5x) / (2x — 1)

    Проба:

    (г г -1 ) (x) = г [г -1 (x)]

    = г [(4 + 5x) / (2x — 1)]

    = [(4 + 5x) / (2x — 1) + 4] / [2 (4 + 5x) / (2x — 1) — 5]

    Умножьте числитель и знаменатель на (2x — 1).

    ⟹ (2x — 1) [(4 + 5x) / (2x — 1) + 4] / [2 (4 + 5x) / (2x — 1) — 5] (2x — 1).

    ⟹ [4 + 5x + 4 (2x — 1)] / [2 (4 + 5x) — 5 (2x — 1)]

    ⟹ [4 + 5x + 8x − 4] / [8 + 10x — 10x + 5]

    ⟹13x / 13 = x
    Следовательно, g — 1 (x) = (4 + 5x) / (2x — 1)

    Пример 5

    Определите значение, обратное следующей функции f (x) = 2x — 5

    Решение

    Заменить f (x) на y.

    f (x) = 2x — 5⟹ y = 2x — 5

    Переключите x и y, чтобы получить;

    ⟹ х = 2у — 5

    Изолировать переменную y.

    2у = х + 5

    ⟹ у = х / 2 + 5/2

    Измените y обратно на f –1 (x).

    ⟹ f –1 (x) = (x + 5) / 2

    Пример 6

    Найти обратную функцию к функции h (x) = (x — 2) 3 .

    Решение

    Измените h (x) на y, чтобы получить;

    h (x) = (x — 2) 3 ⟹ y = (x — 2) 3

    Поменять местами x и y

    ⟹ х = (у — 2) 3

    Изолятор ул.

    y 3 = x + 2 3

    Найдите кубический корень из обеих частей уравнения.

    3 √y 3 = 3 √x 3 + 3 √2 3

    y = 3 √ (2 3 ) + 2

    Заменить y на h -1 (x)

    ч — 1 (x) = 3 √ (2 3 ) + 2

    Пример 7

    Найти обратную величину h (x) = (4x + 3) / (2x + 5)

    Решение

    Заменить h (x) на y.

    h (x) = (4x + 3) / (2x + 5) ⟹ y = (4x + 3) / (2x + 5)

    Поменять местами x и y.

    ⟹ х = (4у + 3) / (2у + 5).

    Решите относительно y в приведенном выше уравнении следующим образом:

    ⟹ х = (4у + 3) / (2у + 5)

    Умножаем обе стороны на (2y + 5)

    ⟹ х (2у + 5) = 4у + 3

    Распределить x

    ⟹ 2xy + 5x = 4y + 3

    Изолятор ул.

    ⟹ 2xy — 4y = 3 — 5x

    ⟹ y (2x — 4) = 3-5x

    Разделим на 2x — 4, чтобы получить;

    ⟹ у = (3 — 5x) / (2x — 4)

    Наконец, замените y на h — 1 (x).

    ⟹ ч — 1 (x) = (3 — 5x) / (2x — 4)

    Практические вопросы

    Найдите обратное значение для следующих функций:

    1. г (x) = (2x — 5) / 3.
    2. h (x) = –3x + 11.
    3. г (x) = — (x + 2) 2 — 1.
    4. г (х) = (5/6) х — 3/4
    5. f (x) = 3 x — 2.
    6. ч (х) = х 2 + 1.
    7. г (x) = 2 (x — 3) 2 -5
    8. f (x) = x 2 / (x 2 + 1)
    9. ч (х) = √x — 3.

    3 x y 7: Как решить систему:x+y=3,x-y=7?

    XY-7 TWS Беспроводной Bluetooth 5.0 Наушники для наушников SIRI голосовой ассистент батареи Дисплей сенсорные работы наушники

    Поделиться в:

    Цена: 15.99 $23.06 скидка 31%

    • Склад:
    • Отправка: БЕСПЛАТНАЯ ДОСТАВКА COD Этот продукт поддерживает наложенный платеж при доставке. Совет: не размещайте заказы на товары не наложенным платежом, иначе Вы не сможете выбрать способ оплаты наложенным платежом. Отправка между: Jun 18 — Jun 20, Расчетное время доставки: рабочих дней Время обработки заказа может занять несколько дней. После отправки со склада время доставки (или доставки) зависит от способа доставки.
    • Цвет:
    • Количество

      - +

    • Рассрочка: Беспроцентный Вы можете наслаждаться максимальной 0 беспроцентной рассрочкой, и может не пользоваться этим предложением при размещении заказов с другими товарами »

    Распродажа

    Рекомендуемые для вас

    Описания XY-7 Wireless Bluetooth Headphone

    Основные характеристики:
    ● Bluetooth 5.0
    ● голосовой помощник Siri
    ● дисплей питания
    ● сенсорная операция
    ● Авто подключение
    ● HD Call.
    ● Интеллектуальное снижение шума

    Спецификация

    Общий

    Номер модели: XY-7.
    тип наушников: наушники
    Материалы: ABS
    В основном совместим с: iPad,iPhone,iPhone 5/5S,iPhone 6,iPhone 6 Plus,iPhone 6S,iPod
    Функция: переключение песен
    Версия Bluetooth: V5.0
    Протокол Блютуз: A2DP v1.3,AVRCP v1.6,HSP v1.2
    диапазон передачи: 10 метров
    Режим Блютуз: громкая связь,накладные наушники
    Емкость аккумулятора(мАч): Наушники: 40 мАч, зарядная коробка: 400 мАч
    Аудио: стерео
    Цвет: Чёрный,Синий,Белый

    Размер и вес

    Вес упаковки: 0. 1500 кг
    Размер упаковки (Д x Ш x В): 10.00 х 10.00 х 4.00 см / 3,94 х 3,94 х 1,57 дюйма

    Комплектация

    Комплектация: 1 x Пара наушников, 1 x Зарядная коробка, 1 х USB Зарядка кабеля, 1 х руководство

    Предлагаемые продукты

    Отзывы клиентов

    • Andrea

      Not bad

      The audio has a good range and clarity, with a noticeable bass at higher volume levels. The noise-canceling makes the voice clearer eliminating background noise. They fit nicely in your ears even when I was doing sport. The battery life also is great.»>Bluetooth earbuds are one of the most wished-for gadgets this holiday by kids. The audio has a good range and clarity, with a noticeable bass at higher volume levels. The noise-canceling makes the voice clearer eliminating background noise. They fit nicely in your ears even when I was doing sport. The battery life also is great.

      Jan 01,2021

    • Selina

      Great product

      I don’t feel tired even after listening for a long time. The sound is crystal clear. They do not take long to charge and last a long time.» data-content_en=»I can’t complain considering its price. The touch control takes some practice. Once you are used to it, it is quite convenient. They are so comfortable to wear that I could easily wear them all day. I don’t feel tired even after listening for a long time. The sound is crystal clear. They do not take long to charge and last a long time.»>I can’t complain considering its price. The touch control takes some practice. Once you are used to it, it is quite convenient. They are so comfortable to wear that I could easily wear them all day. I don’t feel tired even after listening for a long time. The sound is crystal clear. They do not take long to charge and last a long time.

      Aug 19,2020

    • Luigi

      Worth buying

      I’ve always gotten the ones that go around your neck afraid I would lose them. These are great, especially for a good price. They were no trouble at all connecting to my phone. The battery life seems good, too. Comes nicely packaged and would make a great gift.» data-content_en=»I was surprised by these Bluetooth headphones. I’ve always gotten the ones that go around your neck afraid I would lose them. These are great, especially for a good price. They were no trouble at all connecting to my phone. The battery life seems good, too. Comes nicely packaged and would make a great gift.»>I was surprised by these Bluetooth headphones. I’ve always gotten the ones that go around your neck afraid I would lose them. These are great, especially for a good price. They were no trouble at all connecting to my phone. The battery life seems good, too. Comes nicely packaged and would make a great gift.

      Nov 18,2020

    • LEE

      Better than I thought

      » data-content_en=»If you are looking for an inexpensive good quality earbud, this product should be on top of your list.»>If you are looking for an inexpensive good quality earbud, this product should be on top of your list.

      Dec 15,2020

    • Soki

      Recommend it to you

      I bought this earbuds for my nephew, and for the price point, this earbuds is a must hear to believe.

      Oct 01,2020

    FAQ для XY-7 Wireless Bluetooth Headphone

    • Почему наушники не заряжается?

      Пожалуйста, убедитесь, что оба конца кабеля USB надежно подключены. Если вы используете сетевую розетку, убедитесь, что источник питания подключен надежно и выпускной работы. Если вы используете компьютер, убедитесь, что он подключен, и порт USB работает. Включите гарнитуру, а затем включите их. Отключите и снова подключите USB-кабель для зарядки.

    • Почему наушники отсоединять с мобильным телефоном в пределах 10 метров?

      Пожалуйста, проверьте, есть ли металл или любой другой материал, в пределах относительно близкого расстояния, которые могут помешать соединению Bluetooth. Bluetooth является технологией радиосвязи, является чувствительным к объектам, расположенных между наушниками и подключенным устройством.

    • Там нет звукового сигнала, что я могу сделать?

      Проверьте, если головные телефоны выключены. Проверьте, если наушники в паре. При необходимости, пара наушников с источником звука снова. Проверьте подключение аудио источника. Если объем слишком мал, чтобы увеличить громкость.

    • Почему я не могу использовать наушники для регулировки громкости или выберите дорожку на App мобильного телефона?

      Настройки приложения программного обеспечения могут различаться в зависимости от некоторых функций приложения сам не в зависимости от телефона.

    • Почему наушники не могут спариваться с мобильным телефоном?

      1. Проверьте, находится ли ваш наушник в режиме сопряжения или в режиме переподключения, и проверьте, открыта ли функция поиска Bluetooth на вашем мобильном телефоне; 2. Проверьте меню Bluetooth вашего мобильного устройства и удалите / забудьте наушники, затем снова подключите наушники, следуя инструкциям в руководстве пользователя.

    • Что означает логотип сертификации для наушников означает?
    • Как выбрать гарнитуру?
    • Почему мой смартфон не может найти свои наушники Bluetooth?

      1. Убедитесь, что ваш смартфон и наушники достаточно близко друг к другу, когда вы хотите подключить их; 2. Проверьте рекомендуемый производителем процесс сопряжения; 3. Попробуйте выключить и снова включить их; 4. Выключите или удалите все мешающие устройства; 5. Отойдите от WiFi роутера.

    • Какие факторы влияют на качество звука гарнитуры Bluetooth? Факторы, которые действительно влияют на качество звука Bluetooth являются протоколами для передачи аудио и технологии кодирования. В настоящее время существуют четыре основные технологии кодирования на рынке: ACC, SBC, APT-X и LDAC. Теперь мы расскажем вам, в чем разница между ними? Пожалуйста, проверьте этот пост: 4 audio coding technologies tell you the secret of Bluetooth sound quality
    • Почему статична, когда я слушаю музыку на наушниках Bluetooth?

      1. Сначала вы должны отключить или отключить любые другие неиспользуемые устройства Bluetooth в этом районе; 2. Если статический заряд не исчезает, попробуйте перезагрузить наушники; 3. Включите медиаплеер, затем отсоедините наушники на 30 секунд, а затем выполните их сопряжение; 4. Уменьшите расстояние между вашим смартфоном и наушниками, так как это также помогает уменьшить статический заряд.

    • Сколько видов наушников есть на рынке?
    • Почему мои устройства не могут распознать Bluetooth наушники?

      1. Функция Bluetooth в телефоне не включена. 2. Гарнитура не находится в режиме сопряжения / обнаружения. Телефон не найдет гарнитуру, если она не находится в этом режиме.

    • Может этот деталь быть отправлены в моей стране?
    • Подробнее

    Вопросы клиентов

    • Все
    • Информация о товаре
    • Состояние запасов
    • Оплата
    • О доставке
    • Другие

    Будьте первым, кто задаст вопрос. Хотите G баллы? Просто напишите отзыв!

    Хотите купить оптом XY-7 Wireless Bluetooth Headphone ? Пожалуйста, отправьте ваш оптовый запрос XY-7 Wireless Bluetooth Headphone ниже. Обратите внимание, что мы обычно не предоставляем бесплатную доставку при оптовых заказах XY-7 Wireless Bluetooth Headphone, но оптовая цена будет большой сделкой.

    Ваши недавно просмотренные товары

    Руководство пользователя беспроводных стереонаушников XY-7

    XY-7 Беспроводные стереонаушники

    инструкция

    Принципиальная схема продукта
    список пакета
    • Беспроводные наушники
    • Зарядный чехол
    • Кабель для зарядки кейса
    • Шапочка для ушей
    • инструкция
    • Ящик
    Спецификация продуктов
    • Модель: XY — 7
    • Версия Bluetooth: V5. 0
    • Емкость батареи: 40mAh
    • Зарядка аккумулятора: 400mAh
    • Время работы: 3-5 часов
    • Время зарядки: около 2 часов
    • Время ожидания: 120 часов
    • Соглашение о поддержке: SBC, AAC FHSS Не поддерживает аудиокод APTX

    TWS Реализует беспроводное стерео, отдельные левый и правый каналы. Левое и правое ухо имеют полную функциональность Bluetooth и могут использоваться по отдельности или парами.

    спаривание

    Нажмите и удерживайте сенсорную позицию в течение 5 секунд, пока красный и синий индикаторы не начнут мигать попеременно.
    Найдите имя Bluetooth 【XY-7】 в списке Bluetooth сотовых телефонов и щелкните его, чтобы подключиться. В случае успешного подключения светодиодный индикатор не горит.

    Метод сопряжения левого и правого наушников: левый и правый наушники загораются красным и синим светом, коснитесь правого уха 2, гарнитура «Успех куплета 2» синий свет fl медленно мигает, ожидая подключения устройства.

    Повторное использование для того же телефона: нажмите и удерживайте сенсорную позицию в течение 5 секунд, пока красный и синий индикаторы не начнут мигать попеременно. Сопряжение будет выполнено автоматически.

    Выключение

    Нажмите и удерживайте сенсорную позицию в течение 5 секунд, пока красный свет не мигнет три раза.

    когда наушники соединяются друг с другом, при повороте одного бокового наушника другая сторона отключается автоматически.

    Если наушники включены и не подключены к мобильному телефону, они автоматически выключатся через 5 минут.

    Заказать обратный звонок

    Поддержка ответа на звонок, когда наушники подключаются к мобильному телефону. Поддержка бинаурального вызова.
    Наушники будут напоминать вам о входящем звонке, когда наушники подключаются к вашему телефону.

    Ответ или повесить трубку: коротко нажмите на наушник, чтобы ответить или повесить звонок. Отклонение вызова: нажмите и удерживайте наушник, чтобы отклонить вызов.

    Прерывание исходящего вызова: кратковременно нажмите на наушник, чтобы прервать вызов при наборе номера.

    Слушать музыку

    Поддержка прослушивания музыки, когда наушники включены и подключаются к мобильному телефону.
    Пауза / воспроизведение: коротко нажмите на наушник, чтобы приостановить или приостановить воспроизведение музыки.

    Следующая песня: дважды нажмите на правый наушник, чтобы перейти к следующей песне.
    Предыдущая песня: дважды щелкните левый наушник, чтобы перейти к предыдущей песне. Увеличение громкости: нажмите на левый наушник три раза, чтобы увеличить громкость. Уменьшение громкости: трижды нажмите на правый наушник, чтобы уменьшить громкость. Вызов сири: нажмите и удерживайте наушники в течение 3 секунд, чтобы вызвать голосового помощника (сири).

    Использование продуктов
    1. Наушники следует хранить в сухом и вентилируемом помещении, избегая попадания масла и воды; влага и пыль влияют на характеристики продукта.
    2. Избегайте использования раздражителей, органических растворителей или предметов, содержащих эти ингредиенты, для чистки наушников.
    3. Использование наушников должно быть правильным в соответствии со спецификацией, обратите внимание на влияние окружающей среды на наушники. Для обеспечения плавного соединения рекомендуется, чтобы расстояние между наушниками и телефонами не превышало 10 метров.
    4. Если наушники не подключились или соединение плохое, пожалуйста, не разбирайте наушники или аксессуары самостоятельно. В остальном никаких гарантий.
    5. По поводу зарядки наушников. Пожалуйста, используйте стандартный USB-кабель или заостренный зарядный кабель от нашей компании для зарядки наушников.
    6. Для обеспечения нормального использования убедитесь, что наушники достаточно мощности.

    Руководство пользователя беспроводных стереонаушников XY-7 — Скачать [оптимизировано]
    Руководство пользователя беспроводных стереонаушников XY-7 — Скачать

    4×+3y=5 X-y=3 X-y=7 5x-3y=1 6x+5y=6 2x+y=-2

    1)168:2=84(кг) 2)84:28=3(кг)-масса 1 каробки крыжовника 3)3+2=5(кг)масса 1 ящика малины

    Накресли відрізки МN=6 см і КО=4 см так щоб їхньою спільною частиною був відрізок АВ розглянь різні випатки

    Ответ:

    3,88

    Пошаговое объяснение:

    2*3=6 баллов всего набрали ученики получившие 2

    3*7=21 баллов всего набрали ученик получившие  «3»

    4*13=52 баллов набрали ученики получившие «4»

    5*9=45 баллов набрали ученики получившие «5»

    3+7+13+9=32 работы всего написано

    6+21+52+45=124 балла за все работы

    124/32=3,875 средний балл

    1)85*3=275 (км) прошел первый
    2)60*3=180 (км) прошел второй
    3)275+180=455 (км) прошли оба поезда
    4)846-455=391 (км) оставшееся расстояние
    Ответ: 391 км будет между поездами через 3 часа.

    1. 1)-36.006      2)  +60.005
            28.097                7.909
            ———-           ————    
             7.909                67914

    2. 1) _87.007       2) _100.105
                   679               86.328
            ————            ————  
               86.328              13.777

    3. 1) 80:4= 20
        2) 20*5 = 100
        3) +100
              400
            ——-
              500

    4. 1)640:2=320
        2)320:4= 80
        3) _ 800
               320
            ———
               480

    Учебное пособие по калькулятору алгебры

    — MathPapa

    Это руководство по использованию калькулятора алгебры , пошагового калькулятора для алгебры.

    Решение уравнений

    Сначала перейдите на главную страницу калькулятора алгебры. (экспонента: «в степени»)


    Построение графика

    Для построения графика уравнения введите уравнение, которое начинается с «y =» или «x =».2.


    Вычисление выражений

    Калькулятор алгебры может вычислять выражения, содержащие переменную x.

    Чтобы оценить выражение, содержащее x, введите выражение, которое вы хотите оценить, затем знак @ и значение, которое вы хотите вставить для x. Например, команда 2x @ 3 вычисляет выражение 2x для x = 3, что равно 2 * 3 или 6.

    Калькулятор алгебры также может вычислять выражения, содержащие переменные x и y.Чтобы оценить выражение, содержащее x и y, введите выражение, которое вы хотите оценить, затем знак @ и упорядоченную пару, содержащую ваше значение x и значение y. Вот пример вычисления выражения xy в точке (3,4): xy @ (3,4).

    Проверка ответов для решения уравнений

    Так же, как калькулятор алгебры можно использовать для вычисления выражений, Калькулятор алгебры также можно использовать для проверки ответов на решение уравнений, содержащих x.

    В качестве примера предположим, что мы решили 2x + 3 = 7 и получили x = 2.Если мы хотим вставить 2 обратно в исходное уравнение, чтобы проверить нашу работу, мы можем сделать это: 2x + 3 = 7 @ 2. Поскольку ответ правильный, калькулятор алгебры показывает зеленый знак равенства.

    Если вместо этого мы попробуем значение, которое не работает, скажем, x = 3 (попробуйте 2x + 3 = 7 @ 3), вместо этого калькулятор алгебры покажет красный знак «не равно».

    Чтобы проверить ответ на систему уравнений, содержащую x и y, введите два уравнения, разделенных точкой с запятой, за которыми следует знак @ и упорядоченную пару, содержащую ваше значение x и значение y.Пример: x + y = 7; х + 2у = 11 @ (3,4).


    Режим планшета

    Если вы используете планшет, например iPad, войдите в режим планшета, чтобы отобразить сенсорную клавиатуру.


    Статьи по теме

    Вернуться к калькулятору алгебры »

    Исследование кариотипа пациента показало 45, XY, -7 (моносомия 7)

    Контекст 1

    . .. злокачественные новообразования обычно лечатся системной химиотерапией. Нарушение индукции по-прежнему является неприятным явлением при остром лейкозе, особенно при хромосомных аномалиях, таких как моносомия 5 и 7.После трансплантации стволовых клеток у этих пациентов часто наблюдается низкий процент безрецидивной выживаемости. Множественная лекарственная устойчивость — основная причина неэффективности лечения и смерти у них. 1 Несмотря на получение комбинированной химиотерапии, более чем в половине случаев острого миелоидного лейкоза (ОМЛ) неэффективность лечения и рецидивы возникают. 1 Известно несколько терапевтических стратегий лечения ОМЛ для улучшения выживаемости у пациентов с рецидивирующими хромосомными аномалиями. Ингибирование гистондеацетилаз (HDAC) с помощью непрерывного приема низких доз полностью транс-ретиноевой кислоты (ATRA) и вальпоровой кислоты в качестве дифференцирующих агентов было предложено в качестве альтернативного лечения ОМЛ в течение последнего десятилетия. Он используется в качестве метрономной химиотерапии, которая предназначена для предотвращения ангиогенеза опухоли и индукции апоптоза в миелоидных бластах. 2 Здесь мы сообщаем о пациенте с первичным рефрактерным ОМЛ, который лечился пероральной низкодозной метрономной терапией после стандартной системной химиотерапии. 13-летний иранский мальчик поступил в нашу больницу с бледностью, лихорадкой и летаргией за 1 неделю до госпитализации. Медицинский осмотр показал субфебрильную лихорадку, бледность без каких-либо неврологических признаков, гепатоспленомегалию или лимфаденопатию.Общий анализ крови показал гемоглобин 3,5 г / дл, тромбоциты 64 × 10 9 / мкл и лейкоциты 55 × 10 9 / мкл с 4% бластов, 8% нейтрофилов, 80% лимфоцитов, 6% моноцитов и 2% эозинофилов. Аспирация костного мозга показала, что 75% всех ядерных клеток имели миелобластный фенотип (рисунок 1). Иммунофенотипический анализ с помощью проточной цитометрии выполняли с использованием панели антител. Бластные клетки были положительными по CD45, CD13, CD117, CD34, CD19, HLA-DR и отрицательными по другим лимфоидным маркерам, включая CD5, CD10, а также CD14. Отрицательные контроли оценивали по IgG1FITC / IgG1PE.Таким образом, пациенту был поставлен диагноз AML FAB-M 1 с аберрантной экспрессией CD19 (рисунок 1). Исследование кариотипа показало, что 45, XY, -7 совместимы с моносомией 7 (рисунок 2). Пациент рассматривался как кандидат на аллогенную трансплантацию костного мозга после достижения первой индукционной ремиссии. Первоначально он проходил курс лечения по протоколу MRC-12: адриамицин (33,5 мг / м 2, дни 1,3,5), цитарабин арабинозид (100 мг / м 2, дни с 1 по 10) и этопозид (100 мг / м 2). 2, дни с 1 по 5) был первым курсом фазы индукции, который не увенчался успехом (отказ индукции), а затем ему были назначены 1 и 2 курсы протокола FLAI (флударабин (25 мг / м 2, дни 1–4), Цитарабин арабинозид (1000 мг / м 2, дни 1–4) и идарубицин (5 мг / м 2, дни 1-3).Однако у него не было ответа ни на 2 курса протокола FLAI, а повторная аспирация костного мозга и биопсия показали 60% миелобластов в образце костного мозга. Затем пациентка лечилась по альтернативному протоколу химиотерапии, включая 5-дневный курс кладрибина (9 мг / м 2 / доза) и цитарабина арабинозида в виде ежедневных 2-часовых инфузий (500 мг / м 2 / доза). Но снова он не ответил на 1 курс альтернативного протокола. Наконец, после объяснения ситуации пациенту и его родителям, он был назначен на нашу целевую схему пероральной метрономной химиотерапии, в которую были включены ингибиторы гистон-деацетилазы (HDAI): 6-тиогуанин 40 мг / м 2 в течение 21 дня, преднизолон 40 мг / сут. м 2 в течение 5 дней, этопозид перорально 50 мг / м 2 / день в течение 21 дня плюс ATRA 45 мг / м 2 / день и вальпроевая кислота 2.5-5 мг / кг / день в течение первых 14 дней месяца с последующими 7 днями отдыха. Сейчас, по прошествии 1 года с начала метрономной химиотерапии, он иногда получает только поддерживающую терапию, такую ​​как антибиотики и переливание продуктов крови; однако нет данных о ремиссии в мазках костного мозга и периферической крови. Пациент не испытал каких-либо побочных эффектов, связанных с лечением ATRA (сухость слизистой оболочки, головная боль и повышение уровня трансаминаз или триглицеридов). Метрономическая химиотерапия в качестве поддерживающей терапии была продолжена для пациента из-за отсутствия ответа на стандартную системную химиотерапию с целью продления выживаемости и улучшения качества жизни пациента. Метрономная химиотерапия — это непрерывное системное введение нетоксичных доз лекарств, которые атакуют пролиферирующие эндотелиальные клетки как мишени во время ангиогенеза опухоли. Эта стратегия была изобретена 40 лет назад в онкологии взрослых, но опыт в детской онкологии скуден, особенно в отношении гематологического рака. Эта стратегия часто используется при лечении солидных опухолей путем совместного введения антиангиогенных препаратов плюс непрерывная химиотерапия в низких дозах вместо прерывистой химиотерапии в высоких дозах.3 В настоящее время протокол PrET (преднизолон, этопозид и тиогуанин) представляет собой хорошо известный метрономный режим для резистентных пациентов с ОМЛ. Антиангиогенная способность 6TG вместе с его антиметаболитной активностью по отношению к опухолевым клеткам играет важную роль в поддержании эффективности этого метода. 4 В одном исследовании сообщалось о 68-летнем мужчине, у которого был ОМЛ с цитогенетическими особенностями высокого риска, например, наш пациент, который достиг полной ремиссии во время фазы индукции пероральной метрономной химиотерапией по аналогичному режиму в амбулаторных условиях. Затем его лечили высокой дозой цитарабина арабинозида (HDAC) в качестве консолидации с последующей поддерживающей терапией по протоколу PrET. Он выжил 35 месяцев с момента постановки диагноза и 21 месяц перерыва в лечении. 5 Наш пациент не достиг ремиссии, несмотря на различные схемы лечения, которые он получал (FLAI, кладрибин и HDAC). В результате было определено, что он будет лечиться по протоколу PrET в качестве паллиативного лечения. Двумя компонентами нашего лечения были введение ингибиторов гистон-деацетилазы (HDACI), включая вальпоровую кислоту (VPA) и All-trans-ретиноевую кислоту (ATRA), которые вводились одновременно в качестве метрономной химиотерапии.Вальпоровая кислота (ВПК) оказывает противолейкемическое действие при ОМЛ, используемом в сочетании с другими противолейкозными средствами. 4 Это лечение может вызвать клинически значимое улучшение количества клеток периферической крови и стабилизацию клинического статуса для подгруппы пациентов с ОМЛ, а также снизить риск клинически значимой токсичности. Хотя наш пациент имел стабильный клинический статус, наибольшее количество клеток в периферической крови и костном мозге составляли миелобласты. Похоже, что VPA может индуцировать дифференцировку и оказывает антипролиферативное и проапоптотическое действие в клеточных линиях AML.Однако пациенты, скорее всего, неоднородны с точки зрения восприимчивости к VPA и молекулярных механизмов, опосредующих его противолейкемический эффект. Прямое действие препарата на лейкозные клетки, по-видимому, является наиболее важным, но могут быть косвенные эффекты, опосредованные повышенной противолейкозной иммунной реактивностью. 6,7 ATRA также является ингибитором HDAC, дифференцирующий эффект которого на клетки острого промиелоцитарного лейкоза (APL) человека in vitro был хорошо установлен. 7 В APL отсутствие ATRA ведет к активности HDAC, вызывая конденсацию хроматина и репрессию транскрипции.4 ATRA вызывает конформационные изменения в слитном онкобелке промиелоцитарного лейкоза (PML) / рецептора ретиноевой кислоты α (RARα), тем самым позволяя высвобождать комплексы HDAC и рекрутировать транскрипцию. Лечение ATRA значительно улучшило прогноз APL, а также использовалось для лечения не-APL AML. 2 Мы использовали комбинацию метрономной химиотерапии PrET вместе с ATRA и VPA после неудачи различных протоколов интенсивного спасения для этого пациента, поскольку у него не было шансов на продолжение лечения и продление жизни; однако этот метод привел к увеличению выживаемости и повышению качества жизни.Синхронное назначение ATRA и VPA можно сочетать с низкими дозами цитотоксических препаратов, таких как цитарабин арабинозид. 8 Гидроксимочевина и 6-тиогуанин, 6 также могут вызывать ремиссию в соответствии с критериями ответа на синдром миелодиспластического синдрома (МДС) и полную гематологическую ремиссию. Наш пациент продемонстрировал доказательства клинической стабильности метрономной стратегии, несмотря на отсутствие признаков индукции ремиссии в костном мозге. Сообщалось об удовлетворительных результатах с другими комбинациями пероральных метрономных методов лечения, такими как мелфалан и леналидомид, но без ингибиторов HDAC. 9-12 Похоже, что этот метод может вызвать у пациента жизненно важное состояние покоя, предотвращая обострение основного заболевания. Метрономическая химиотерапия с ингибиторами HDAC может использоваться в качестве терапевтической стратегии, особенно в рефрактерных случаях AML, не поддающихся лечению другими видами лечения. Этот клинический случай предполагает вероятную эффективность комбинации пероральной низкоинтенсивной метрономной химиотерапии с ингибиторами HDAC у пациентов с ОМЛ с индукцией …

    Руководство пользователя беспроводных стереонаушников XY-7

    XY-7 Беспроводные стереонаушники

    Инструкция

    Принципиальная схема изделия
    Список пакетов
    • Беспроводные наушники
    • Зарядный футляр
    • Зарядный кабель для кейса
    • Колпачок для ушей
    • Инструкция
    • Упаковочная коробка
    Спецификация продукции
    • Модель: XY — 7
    • Версия Bluetooth: V5. 0
    • Емкость аккумулятора: 40 мАч
    • Зарядка аккумулятора: 400 мАч
    • Время работы: 3-5 часов
    • Время зарядки: около 2 часов
    • Время ожидания: 120 часов
    • Соглашение о поддержке: SBC, AAC FHSS Не поддерживает аудиокод APTX

    TWS Реализуйте беспроводную стереосистему, раздельные левый и правый каналы. И левое, и правое ухо имеют полную функциональность Bluetooth и могут использоваться по отдельности или парами.

    Сопряжение

    Нажмите и удерживайте сенсорную позицию в течение 5 секунд, пока красный и синий индикаторы не начнут мигать попеременно.
    Найдите имя Bluetooth 【XY-7】 в списке Bluetooth сотового телефона и щелкните его, чтобы подключиться. В случае успешного подключения светодиодный индикатор не горит.

    Метод соединения левого и правого наушников: левый и правый наушники загораются красным и синим светом, коснитесь правого уха 2, гарнитуры успешного соединения 2 синий свет fl медленно мигает, ожидание подключения устройства.

    Повторное использование для того же телефона: нажмите и удерживайте кнопку касания в течение 5 секунд, пока красный и синий индикаторы не начнут мигать попеременно. Сопряжение будет выполнено автоматически.

    Выключение питания

    Нажмите и удерживайте сенсорную позицию в течение 5 секунд, пока красный свет не мигнет три раза.

    , когда наушники соединяются друг с другом, при выключении одного бокового наушника другая сторона отключается автоматически.

    Если наушники включены и не подключены к мобильному телефону, они автоматически выключатся через 5 минут.

    Позвонить

    Поддержка ответа на звонок, когда наушники подключаются к сотовому телефону. Поддержка бинаурального вызова.
    Наушники будут напоминать вам о входящем звонке, когда наушники подключаются к вашему телефону.

    Ответ или повесить трубку: коротко нажмите на наушник, чтобы ответить или повесить звонок. Отклонение вызова: нажмите и удерживайте наушник, чтобы отклонить вызов.

    Прерывание исходящего вызова: кратковременно нажмите на наушник, чтобы прервать вызов при наборе номера.

    Слушать музыку

    Поддерживает прослушивание музыки, когда наушники включены и подключаются к мобильному телефону.
    Пауза / воспроизведение: коротко нажмите на наушник, чтобы приостановить или приостановить воспроизведение музыки.

    Следующая песня: дважды щелкните по правому наушнику, чтобы перейти к следующей песне.
    Предыдущая песня: дважды щелкните левый наушник, чтобы перейти к предыдущей песне. Увеличение громкости: нажмите на левый наушник три раза, чтобы увеличить громкость. Уменьшение громкости: трижды нажмите на правый наушник, чтобы уменьшить громкость. Вызов сири: нажмите и удерживайте наушники в течение 3 секунд, чтобы вызвать голосового помощника (сири).

    Использование продуктов
    1. Наушники следует хранить в сухом и вентилируемом помещении, избегая попадания масла и воды; влага и пыль влияют на характеристики продукта.
    2. Не используйте раздражители, органические растворители или предметы, содержащие эти ингредиенты, для чистки наушников.
    3. Использование наушников должно быть правильным в соответствии со спецификацией, обратите внимание на влияние окружающей среды на наушники. Чтобы обеспечить бесперебойное соединение, рекомендуется, чтобы расстояние между наушниками и телефонами не превышало 10 метров.
    4. Если наушники не подключились или соединение с ними плохое, не разбирайте наушники или аксессуары самостоятельно.В остальном никаких гарантий.
    5. По поводу зарядки наушников. Пожалуйста, используйте стандартный USB-кабель или заостренный зарядный кабель от нашей компании для зарядки наушников.
    6. Для нормального использования убедитесь, что наушники достаточно мощности.
    Руководство пользователя беспроводных стереонаушников

    XY-7 — Загрузить [оптимизировано]
    Руководство пользователя беспроводных стереонаушников XY-7 — Загрузить

    3 8 отработать функции абсолютного значения клавиша ответа

    Windows 10 безопасный режим dell inspiron 15

    3. Учитывая график функции абсолютного значения, запишите функцию в форме g (x) = al — h) + k. Заменять. Упрощать. Отнимите 5 с каждой стороны. Перепишите абсолютное значение в виде двух уравнений. 1 = Решите для b. — или I = 4 или b = В зависимости от условий задачи учитывайте только b = 4. Подставьте вместо g (x), чтобы найти уравнение для графика … Если вам нравится этот сайт о решении математических задач, сообщите об этом в Google нажав кнопку +1. Если вам нравится эта страница, пожалуйста, нажмите и эту кнопку +1. Примечание. Если кнопка +1 темно-синего цвета, вы уже поставили ей +1.Кинематические уравнения для углового и линейного движения. Кинематические уравнения 1 v = vo + at = o + t Кинематические уравнения 2 x = xo + vot + 1/2 at2 = o + ot + 1/2 t2 Кинематические уравнения 3 v2 = vo 2 + 2a (xxo) 2 = o 2 + 2 (o) Инерция вращения Аналог массы вращения Для точечных масс I = mr2 I: инерция вращения (кг · м2) m: масса (кг) Создание программ Java, Решения самопроверки 3-го издания ПРИМЕЧАНИЕ. опубликованы на нашем веб-сайте и доступны для студентов. Это означает, что задачи самопроверки, как правило, не следует рассматривать как оцениваемые домашние задания, потому что учащиеся могут легко найти решения для всех из них.1-7 Практические функции Определите, является ли каждое отношение функцией. ДАТА 2 ПЕРИОД да 10. 13. g (-3) + 13 16. + l] 3 6.x = -2 Iff (x) = 2x — 6 и «(x) 11. g найдите каждое значение. 12.f (7) — 9 15. g (3y) 2×2, 17. ЗАРАБОТКА Мартин зарабатывает 7,50 долларов в час, корректируя рекламные объявления в местной газете. Его недельную зарплату w можно описать с помощью Answer KeyGeometryAnswer KeyThis предоставляет ответы и решения для Put Me in, Ящики для упражнений Coach!, Организованные по секциям. Снятие бремени с доказательств Да Теорема 8.3: Если два угла дополняют один и тот же угол, то эти два угла совпадают.

    Сколько времени нужно на активацию телефона verizon

    5.

    2 дополнительные практические ответы 5.2 дополнительные практические ответы

    Практика: Системы уравнений с подстановкой. Это текущий выбранный элемент. Обзор метода подстановки (системы уравнений) Следующее занятие.

    ПРОБЛЕМЫ ГЕНЕТИЧЕСКОЙ ПРАКТИКИ. Часть 3. Вопрос 1. Желтые морские свинки, скрещенные с белыми, всегда дают потомство кремового цвета. Две морские свинки кремового окраса при скрещивании дали потомство желтого, кремового и белого цвета в соотношении l желтого: 2 кремового: l белого.Как эти цвета передаются по наследству? ответ (щелкните здесь) —— Вопрос 2

    урок 3 Дополнительные практические углы треугольников ответ на вопрос: каков третий угол прямоугольного треугольника, если один из углов равен 51 °? 12. АЛГЕБРА Найдите m∠A в ABC, если m∠B = 38 ° и m∠C = 38 °, 13.

    Рабочие листы, учебные ресурсы и листы с практическими занятиями по математике для печати для учителей. Еженедельные рабочие тетради для К-8. Сайт домашних заданий для учителей!

    Викторина по дробям: проверьте свои навыки дроби, отвечая на вопросы. В этом тесте вас просят упростить дроби, преобразовать дроби в десятичные числа и проценты, а также ответить на вопросы по алгебре, связанные с дробями. Выберите уровень сложности, типы вопросов и время. Fraction Quiz — это одна из интерактивных оценочных викторин.

    Решения для дополнительных страниц практики Описание курсов для целей / письменных показаний Нас попросили предоставить описания наших курсов средней математики (предалгебра через исчисление и управление), которые родители могут скопировать и вставить в документацию по домашнему обучению.

    Ключ для продвинутого среднего уровня для ведущего по новому языку. 2 1 Если вы исправите, то сдадите экзамены. 2 Мы должны спросить вашего учителя о правильном ответе, когда мы его увидим. 3 В будущем в мире может быть только один или два языка, если мы не защитим их. 4 Я не пойду, пока ты не уйдешь.

    1. Установите квадрат Пеннета 2 на 2. 2. Напишите аллели родителя 1 в левой части квадрата Пеннета. Каждая гамета будет иметь один из двух аллелей родителя. В этом конкретном скрещивании половина гамет будет иметь доминантный (S) аллель, а половина — рецессивный (s) аллель.

    Twsbi vac 700

    Упражнение с запятыми 1. В этом упражнении вам предлагается применить свои знания о запятых, определив, сколько запятых необходимо в примерах предложений. Щелкните ссылку внизу страницы, чтобы увидеть ответы. Улучшите свои математические знания с помощью бесплатных вопросов из раздела «Вероятность простых событий» и тысяч других математических навыков.

    Стажировка в Arup

    Ответный ключ для практических заданий 5-1 и 5-2 1) QR = 5 опп. стороны равны ≅ 2) PX = 10 диаг.разделите друг друга пополам 3) m

    Урок 5-2 Скажите, является ли каждое уравнение прямой вариацией. Если да, то найдите постоянную вариации. 11. y 2x 12. 4y x 13. y x 3 Изобразите прямое изменение, которое включает данную точку. Напишите уравнение линии. 14. ((5, 4) 15.(7,) 16. (3 10 17. 4, 8) 2 2-2-2 yx O 2 2-2-2 yx O 2 2-2-2 yx O Дополнительная практика Глава 5 2 1 3 не определено …

    Дополнительная практика в классе : Dilations БЛОК 4 — Вы пройдете викторину, которая была запланирована на вчерашний день! Блок 4 ТОЛЬКО для HW (срок сдачи — понедельник): необходимо посмотреть видео 6.5 «Симметрии четырехугольников» и заполнить 6.5 «Готовься, устанавливай, вперед» [2, 5, 6, 10] Ответы по дополнительным упражнениям по геометрии Холта Холт Макдугал Ларсон ответы на вопросы учебника предварительной практики алгебры, предварительная алгебра онлайн Дата обновления учебника: 28 02 Холт Макдугал Ларсон.. Геометрия ответы бесплатно. . Предалгебра Алгебра Интегрированная математика Геометрия Алгебра 2 Тригонометрия Предвычисление Статистическое исчисление Вероятность Колледж.

    Rewind iptv

    Math goodies была пионером онлайн-помощи по математике. Мы начали в 1998 году с нашими уникальными ресурсами. Чтобы получить помощь, выберите элемент из списка ниже ». Уроки математики с пошаговыми инструкциями для использования в удобном для вас темпе. Электронные и распечатываемые рабочие листы для дополнительной практики. Решения включены.

    Вот еще одно увлекательное занятие, которое поможет студентам попрактиковаться в определении тем в рассказах.На этом рабочем листе студенты прочитают пять отрывков из оригинального рассказа и определят тему или смысл рассказа. Также они объяснят, как они получили свои ответы. Рекомендуемый уровень чтения этого текста: 5–9 классы.

    Extra Practice 8.1 Имя ВВЕДЕНИЕ В ЛИНЕЙНЫЕ ФУНКЦИИ Ответьте на эти вопросы, используя любой метод, который вам нравится, но убедитесь, что вы можете объяснить свои рассуждения. Не расстраивайтесь, если вы не можете решить эту проблему сразу; вам не дали метода. Подумайте об этом и попробуйте то, что кажется логичным.Тест по гражданскому праву содержит только вопросы и ответы из версии теста по гражданскому праву 2008 года. Добро пожаловать на тест по гражданскому праву! Практический тест по гражданскому праву — это учебный инструмент, который поможет вам проверить свои знания в области истории и правительства США.

    A2 передняя стойка в сборе

    В этой практической оценке навыков вы настроите сеть XYZ Corporation. Вы установите сетевое соединение, настроив адреса IPv4 и IPv6 на интерфейсах маршрутизатора.Вы настроите службы адресации DHCPv4 и NAT и установите связь между сетями с помощью …

    «У меня было всего 2 студента, готовящихся к CAE, и их самым большим опасением был экзамен по устной речи. Для части 2 я вырезал разные изображения и складывал их в две стопки. Прежде всего, им нужно было найти пары, а затем они должны были рассказать о 2 сходствах, 2 различиях и ответить на мои вопросы. Это повысило уверенность! ”Татьяна Балаш, Liceul Tehnologic» Henri Coanda «

    Практика — это онлайн-инструмент для практических занятий, который помогает студентам в колледжах и старших классах программирования изучать и практиковать основные концепции программирования CS1 и CS2. Вот набор практических задач для второго экзамена. экзамен-2 практика Ниже представлен набор практических задач и ответы на них для третьего экзамена обзор-ответы Вот копия первого экзамена для практики. 1. экзамен-1-spring01 Вот копия второго экзамена для практики. 1. экзамен-2-spring01 Вот копия третьего экзамена для практики. 1. экзамен-3 …

    6l80 Программирование tcm

    Вопросы: Какой дополнительный шаг мы делаем, когда формируем двойное дополнение отрицательного двоичного числа? [Запишите дополнение до 2 для каждого из следующих 5-битных двоичных чисел.

    Проверьте свои ответы по ссылке в уроке, чтобы узнать, насколько хорошо вы справились. Решите следующие задачи. 1. Что такое среднее из следующих чисел? 10, 39, 71, 39, 76, 38, 25 а. 42 б. 39 с. 42,5 г. 35,5 2. На какое число вы разделите, чтобы вычислить среднее значение 3, 4, 5 и 6? а. 6 б. 3 в. 5 дн. 4 3. Пул

    EXTRA Class (Элемент 4) действует с 1 июля 2020 года и действует до 30 июня 2024 года. Пул технических специалистов был пересмотрен в 2018 году. Общий пул был пересмотрен в 2019 году.Дополнительный пул был пересмотрен в 2020 году. Никакие пулы вопросов не будут обновляться или выпускаться в 2021 году. Обзор пулов вопросов является частью регулярного процесса. Математическая практика не должна быть скучной или утомительной. Вы можете сделать математические занятия настолько увлекательными, что ваши дети будут просить об этом! Ознакомьтесь с различными способами, которыми вы можете дать своему ребенку дополнительную математическую практику, в которой он / она нуждается: Математические задания; Планы уроков математики; Ментальная математика; Математические навыки Забавные способы практиковать математику. Дети могут заниматься математикой весело …

    Стенд для горелки

    Упражнение на простое настоящее — настоящее прогрессивное — ex10 :: Учите английский онлайн — бесплатные упражнения, объяснения, игры, учебные материалы и много информации по английскому языку. :: page Ex10

    Раздел 2: Сложение и вычитание Факты по 10 Глава 5: Стратегии сложения с помощью 10 правил викторины. Вам зададут несколько вопросов. Выберите ответ, который вы считаете правильным, а затем нажмите кнопку «Проверить свой ответ».

    Если вы готовитесь к школьному тесту по математике или хотите проверить свои математические навыки, этот бесплатный практический тест бросит вызов вашим знаниям по алгебре. Оценивайте ответы на ходу Просматривайте по 1 вопросу за раз QuickMath автоматически отвечает на наиболее распространенные задачи по алгебре, уравнениям и исчислению, с которыми сталкиваются учащиеся старших классов и колледжей.Раздел алгебры позволяет вам расширить, разложить на множители или упростить практически любое выражение, которое вы выберете.

    Обзорная статья об искусственном интеллекте

    Проверьте свои ответы по ссылке в уроке, чтобы узнать, насколько хорошо вы справились. Решите следующие задачи. 1. Что такое среднее из следующих чисел? 10, 39, 71, 39, 76, 38, 25 а. 42 б. 39 с. 42,5 г. 35,5 2. На какое число вы разделите, чтобы вычислить среднее значение 3, 4, 5 и 6? а. 6 б. 3 в. 5 дн. 4 3. Пул

    EXTRA Class (Элемент 4) действует с 1 июля 2020 г. и действует до 30 июня 2024 г.Пул технических специалистов был пересмотрен в 2018 году. Общий пул был пересмотрен в 2019 году. Дополнительный пул был пересмотрен в 2020 году. Никакие пулы вопросов не будут обновляться или выпускаться в 2021 году. Обзор пулов вопросов является частью регулярного процесса.

    Доступный технический пул вопросов по лицензиям. Для экзаменов, сданных с 1 июля 2018 г. по 30 июня 2022 г. Общий. Для экзаменов, сданных с 1 июля 2019 г. по 30 июня 2023 г. Дополнительные задачи для ознакомления с вводной физикой 1 мая 20,2019 Роберт Браун, преподаватель кафедры физики Университета Дьюка, NC27708-0305 [электронная почта защищена]

    Chiappa pak 9 bump fire

    ОТВЕТ КЛЮЧЕВАЯ Практика 5-3. Линия 1.2 2. –2 3. 5 4. –2 5. a) Наклон линии 1 равен 2. Наклон линии 2 равен 0. b) C 6. A, 0 7. 2 — 1 8. a) 4 б) Ответы будут различаться 9. a) 2 b) Ответы будут различаться 10. a) –2 b) B 11. a) Наклон строки 1 равен 1. Наклон строки 2 равен 2. b) C c) Ответы будет меняться 12. 3 5 13. a) 2 — 1 b) Ответы …

    Предоставляет интерактивные образовательные инструменты для учащихся начальной и средней школы. Развивайте базовые математические навыки с помощью карточек. Предметы включают геометрию, дроби, правила делимости, начальную алгебру или столицы штатов.

    5.2 Разминка Для использования перед уроком 5.2 Ответы на упражнения 1–6 могут быть разными. Приведены примеры ответов. 1. xy2 xy + = = 2. 1 23 yx xy = + — = 3. 3 210 xy xy = — = 4. 2 6 yx xy = + = 5. 11 1 xy xy + = — = — 6. 27 3 xy xy + = = 5.2 Практика A 1. yx = −52; Это уже решено за y. 2. 312 6; xy− = Каждый член делится на 3, поэтому вы можете легко …

    Карта поиска недвижимости в округе Гуадалупе

    Appnana mod apk неограниченное количество nanas 2020 скачать

    Mini cooper rpm колеблется

    Лучший тюнер мм3 для 6.

    7 cummins

    Руническая клавиатура для iphone бесплатно

    Chrome couldnpercent27t загрузить плагин pdf

    1995 fleetwood bounder parts

    Ucla летняя сессия регистрационный взнос

    amino

    Tomodachi life читы citra

    Afro mix 2020 mp3 скачать

    Код syair naga mas sgp hari ini

    Оклады сотрудников округа Хиллсборо

    не синхронизируются с Windows

    не синхронизируются

    Рабочий лист 7 2 синтетических ответа на деление

    Сценарии проверок и противовесов

    Практический тест Psi

    Скачки соединения Discord rtc

    Исключения происходят с элементами be mg и ca в группе

    9000 extra упражняться решать системы уравнений алгебраически Урок 7 Дополнительная практика Решайте системы уравнений алгебраически

    Прямо от свободной алгебры Аддисона-Уэсли, создавая забавные рабочие листы для практики до умножения, у нас есть все, что есть. Зайдите на Emaths.net и прочтите и узнайте об алгебре, программе курса алгебры среднего уровня и множестве дополнительных предметов по математике

    Этот план урока по нелинейным системам уравнений подходит для 10–12 классов. Исследуйте нелинейные системы с помощью графиков и алгебры. Старшеклассники начинают с изучения различных типов квадратичных функций и их возможных пересечений.

    Практика алгебры Квадратичные полиномы Экспоненты 1 уравнения с переменными уравнениями 2 уравнения с переменными уравнениями Алгебра Задачи со словами Бесплатная викторина по алгебре Если вы волонтер или некоммерческая организация, мы будем рады предоставить материалы для подготовки к тесту по сниженной цене или бесплатно.

    15 мая 2011 г. · Эти методы будут реализованы таким же образом, как и для систем линейных уравнений, мы просто применим их к нелинейным системам в этом руководстве. Если вам нужен обзор решения систем линейных уравнений с двумя переменными, смело переходите к Урок 49: Решение систем линейных уравнений с двумя переменными. Итак, поехали …

    Math On the Spot my.hrw.com Мои заметки O 5-5 5 xy O 5-5 5-5 xy Графическое решение систем Упорядоченная пара (x, y) представляет собой решение уравнение с двумя переменными, если подстановка значений x и y в уравнение приводит к истинному утверждению.Система уравнений — это система уравнений с одинаковыми переменными.

    Как решить системные линии (2 переменных линейных уравнения) путем подстановки объяснено с примерами и интерактивными практическими задачами, разработанными шаг за шагом. Как решать системы линейных уравнений подстановкой, примеры, картинки, практика.

    Набор задач 2-7: Видеоурок и практика: Глава 2: Примечания: Ответы на наборы задач: Обзор задач: Графические системы уравнений (3-1) Набор задач 3-1: Видеоурок и практика: Алгебраическое решение систем ( 3-2) Набор задач 3-2: Видеоурок и практика: Видеоурок и практика: Системы неравенств (3-3) Набор задач 3-3…

    В этой статье мы узнаем, как решать системы линейных уравнений, используя обычно используемые методы, а именно замену и исключение. Метод подстановки Подстановка — это метод решения систем линейных уравнений, в котором переменная в одном уравнении выделяется, а затем используется в другом уравнении для определения оставшейся переменной.

    Игровой микрофон Bose qc35

    Отображение всех рабочих листов, относящихся к — Урок 7 Дополнительная практика Решайте системы уравнений по графику.Рабочие листы: Имя, дата, период, урок 7, практика навыков, Глава 7, решение систем линейных уравнений и, Практика решения систем 3-х различных уравнений, Одношаговые неравенства, период даты, построение системы уравнений алгебры 7, Решение систем уравнений с помощью построения графиков, Дополнительная практика … Решение систем уравнений (Gr) — учащийся будет решать системы уравнений путем построения графиков линейных уравнений. Решение полиномиальных уравнений — научите студентов решать полиномиальные уравнения с помощью построения графиков, суммы и разности кубов, формулы квадратичной формулы, квадратичной формы и факторизации.

    Chelsea manluco.

    Мы будем рады разработать любые рабочие листы по математике, которые могут вам понадобиться при планировании урока. Просто свяжитесь с нами, мы будем рады вам помочь. Учителя и домашние школьники используют рабочие листы по математике на этом веб-сайте, чтобы измерить уровень усвоения детьми основных математических навыков, дать дополнительную практику, выполнить домашнее задание и сэкономить драгоценное время на планирование.

    Быстрая проверка: стр.110: 2-1: Решение уравнений с рациональными коэффициентами: Ссылка в реальном мире: стр.111: Практическое руководство: стр.114: Независимая практика: стр.115: Исследование: стр.119

    В этой части Урока 2 закон сохранения количества движения будет использоваться для таких предсказаний. Закон сохранения количества движения будет сочетаться с использованием «таблицы импульсов» и некоторыми навыками алгебры для решения проблем, связанных со столкновениями, происходящими в изолированных системах. Пример 1. Рассмотрим следующую задачу: Какие отличные пошаговые объяснения. Как отец, иногда это помогает мне яснее объяснять вещи своим детям, а иногда показывает, как лучше решать проблемы.Келли Браун, штат Нью-Йорк. В прошлом году я купил Algebrator, и теперь он помогает мне с уроком алгебры в 9-м классе, мне очень нравится пошаговое решение уравнений, это просто …

    Что на самом деле было гражданской войной

    Решение проблем с помощью Системы уравнений При использовании графиков для решения системы уравнений лучше всего переписать оба уравнения в форме пересечения наклона для упрощения построения графиков. Чтобы написать уравнение в форме пересечения наклона, начиная с ax + by = c: ax + by = c by = c-ax Вычтите ax с обеих сторон.y = __c b-ax ___ Разделите обе стороны на b b. y = — a__x b + __c b

    368 Глава 7 Решение систем линейных уравнений и неравенств Предварительный просмотр урока 7-1 Системы уравнений Вы можете использовать электронную таблицу, чтобы выяснить, когда две величины будут равны. Введите каждую формулу в электронную таблицу и найдите строку, в которой обе формулы имеют одинаковый результат. Пример Билл Винтерс рассматривает две работы.

    Решение систем уравнений (Gr) — учащийся будет решать системы уравнений путем построения графиков линейных уравнений.Решение полиномиальных уравнений — научите студентов решать полиномиальные уравнения с помощью построения графиков, суммы и разности кубов, формулы квадратичной формулы, квадратичной формы и факторизации. 13 марта 2012 г. · Interactive Algebra Review — Решение полиномиальных уравнений (см. Также часть B этого руководства) Умножение полиномов — пошаговое объяснение от PurpleMath Решение полиномиальных уравнений — В алгебре вы тратите много времени на решение полиномиальных уравнений или факторизацию полиномов (что одно и то же).

    Какой пациент, скорее всего, будет кандидатом на энтеральное зондовое питание.

    РЕШЕНИЕ СИСТЕМЫ ДОПОЛНЕНИЕМ I. Мы можем решать системы уравнений алгебраически. Более того, решения, которые мы получаем алгебраическими методами, точны. Система в следующем примере — это система, которую мы рассматривали в разделе 8.1 на стр. 335. Пример 1. Решать. x + y = 5 (1) x — y = 1 (2) Решение

    Как решить системные линии (2 переменных линейных уравнения) с помощью подстановки, объяснено с примерами и интерактивными практическими задачами, разработанными шаг за шагом.Как решать системы линейных уравнений подстановкой, примеры, картинки, практика.

    Этот краткий урок покажет студентам, как решать многоступенчатые уравнения. Перед этим уроком ваши ученики должны были ранее научиться писать уравнения, переводить предложения в уравнения и наоборот, решать одношаговые уравнения с помощью сложения и вычитания и решать простые уравнения с помощью умножения или деления. Выберите одну тему из главы для подробного объяснения: решение систем уравнений, решение систем неравенств, оптимизация с помощью линейного программирования, системы уравнений с тремя переменными, операции с матрицами, умножение матриц, решение систем уравнений с использованием правила Крамера или решение Системы уравнений с использованием обратных матриц.

    Сакс, пятая авеню от 5-го магазина, Нью-Йорк

    Книга по экономике — это курс экономики на уровне средней школы, который обеспечит основу для решения словесных задач, необходимых для успешного изучения алгебры «Жизнь Фреда». Для дополнительной практики рассмотрите возможность получения огромного количества практических задач для начинающих алгебр.

    Урок 7.2 — Академия Хана Урок 7.3 — Академия Хана 8 12/3 — 12/7 Блок 3 — Решение уравнений и систем уравнений Модуль 7 — Урок 7.4 Уравнения с множеством решений или без решения Академия Хана 9 12/10 — 12 / 14 Семестр Оценка 10/17 — 12/21 Раздел 3 — Решение уравнений и систем уравнений Модуль 8 — Решение систем линейных…

    7 декабря 2020 г. · Veganarto — Распечатанные рабочие листы для понимания прочитанного для 2-го класса. Лабиринт умножения. Рабочие листы по алгебре отсутствующих углов. рабочие листы для детского сада. бесплатные образовательные печатные издания для детей. бесплатные распечатанные элементарные рабочие листы. письменная деятельность для детского сада. лучшие рабочие листы для детского сада. Алгебраическое решение систем 25 (L2.6) Семейства функций)) 26 27 28 (L3.1) Решение систем с использованием таблиц и графиков (День 1) 29 (L 3.1) Решение систем с использованием таблиц и графиков (День 2) 30 (3 .2) Алгебраическое решение систем Октябрь 2014 Вс Пн Вт Ср Чт Пт Сб 1 Контрольная точка 2 Обзор оценки блока 2 Аутентичный блок оценки

    Исходный код шахматной игры Java

    Решение уравнений с распределительным свойством 2– Этот рабочий лист из 12 задач разработан чтобы познакомить вас с решением уравнений, содержащих свойство распределения. Все эти задачи включают в себя вычитание в уравнении, но учащиеся будут иметь дело только с положительными числами и положительными ответами.

    Обратите внимание, что здесь мы решаем задачи алгебраических слов без систем, и мы решаем системы с использованием матриц в разделе «Матрицы и решение систем с матрицами» здесь. Введение в системы «Системы уравнений» просто означает, что мы имеем дело с более чем одним уравнением и переменной.

    ИМЯ _____ ДАТА _____ ПЕРИОД _____ Урок 8 Дополнительная практика. Решайте системы уравнений алгебраически. Решайте каждую систему уравнений алгебраически. 1. y = x 2 (6, 4) 2. y = x + 8 (10, 2) 3. y = x 5 (11, 6) y = 4 y = 2 y = 6 4.y = x + 6 (10, 4) 5. y = x 9 (12, 3) 6. y = x + 4 (2, 2) y = 4 y = 3 y = x 7. y = x 5 (5 , 10) 8. y = x + 12 (24, 12 … Этот недельный пакет уроков по линейной системе учит решать линейные уравнения с помощью построения графиков, замены и исключения. Урок представлен как в формате Word, так и в формате PDF, а также включает 2 викторины (с ключами ответов), 2 рабочих листа и интересное задание.

    Greyridgehosting

    решение уравнений для x со знаменателями; практические занятия по алгебре и тригонометрии 2 ответа; рабочий лист с отрицательными дробями; основные вопросы алгебры; связанная математика 2 / линейные уравнения / предварительная алгебра; практические занятия по алгебре и тригонометрии, книга 2; решите этот предел с помощью калькулятора; искусство с использованием упорядоченных пар ebooks

    Форма пересечения наклона записывается как: y = mx + b, где m = наклон и b = y- перехватить.Просмотрите следующие примечания, чтобы освежить в памяти, как построить график с использованием формы пересечения наклона, а затем попробуйте …

    Начать изучение алгебры: Урок 7.1 — Решение систем линейных уравнений с помощью построения графиков. Изучите словарный запас, термины и многое другое с помощью дидактических карточек, игр и других учебных инструментов. Дополнительная практика Решайте системы уравнений с помощью графического представления — отображение 8 основных рабочих листов, найденных для этой концепции. Некоторые из рабочих листов для этой концепции — это практика решения систем уравнений 3 разное, название дата период урок 7 практические навыки, решение систем уравнений путем построения графиков, системы уравнений, глава 7 решение систем линейных уравнений и, построение графиков работы линейных уравнений…

    Блокировка discord ip

    Набор задач 2-7: Видеоурок и практика: Глава 2: Примечания: Ответы на наборы задач: Обзор задач: Графические системы уравнений (3-1) Набор задач 3-1: Видеоурок и практика: Алгебраическое решение систем (3-2) Набор задач 3-2: Видеоурок и практика: Видеоурок и практика: Системы неравенств (3-3) Набор задач 3-3 …

    10. Определить какие уравнения ниже в сочетании с уравнением 3x-4y = 2 образуют систему без решений.Выберите все, что подходит.

    Учебное пособие и вмешательство Решение систем уравнений путем построения графических систем уравнений Система уравнений — это набор из двух или более уравнений, содержащих одинаковые переменные. Вы можете решить систему линейных уравнений, построив уравнения на одной и той же координатной плоскости. Если линии пересекаются, решение — это точка пересечения. Решите систему уравнений с помощью графиков. x 2 y (4 xyy 2 2 Запишите каждое уравнение в форме пересечения наклона. x 2 y 4

    На объект действуют две силы f120n и f230n

    Указанная учетная запись уже существует itunes

    Peoplesoft fluid ширина сетки

    Top Gear Season 28 серия 4 смотреть онлайн

    New balance store рядом со мной

    Внешняя политика в период с 1783 по 1828 год

    Minecraft mesa biome seed 2020

    Rumus ai semua pasaran

    Площадь Ганна

    Пропавшая подруга Статус WhatsApp тамильский

    Галактика s7 Wi-Fi вызов не работает t мобильный

    Matlab fftshift vs ifftshift

    9000 9000 команда для рабочего стола Mac6 Ленточный счетчик Factorio

    3800 серия 2 диаграмма вакуума

    BIM compone nts

    Demarini prism 2020 drop 11

    Handsfreelink не подключается

    Hm 19 bluetooth

    Unit 7 Проверка прогресса mcq ap Psyology College Board

    18s с пультом дистанционного управления

    18s с пультом дистанционного управления

    Сколько миль пройти цикл движения

    Трансформаторы DC2 загрузить vk

    Как превратить старый компьютер в сервер

    7 2 повторных соотношения в похожих многоугольниках ответы

    Подозрительное сообщение в строке темы outlook

    Чтобы определить, какие пары фигур действительно похожи, вы должны сравнить соотношение одной стороны к другой.A) \ (\ frac {18} {27} = \ frac {10} {15} \) Упрощая обе дроби до простейших выражений, \ (\ frac {2} {3} = \ frac {2} {3} \), что действительно доказывает схожесть обоих рисунков. Б) Проделайте то же самое с прямоугольниками. \ (\ frac {7} {10} = \ frac {14 …

    Пример ответа: если два многоугольника имеют соответствующие углы, которые равны, а соответствующие стороны пропорциональны, то они подобны. Каждая пара многоугольников похожа Найдите каждую недостающую боковую меру 7. ПЛИТКИ Синяя прямоугольная плитка и красная прямоугольная плитка подобны.Синяя плитка имеет длину 10 дюймов и Урок 10.5 Подобные и совпадающие многоугольники | 223 Цель: использовать свойства похожих и совпадающих многоугольников. Похожие и конгруэнтные многоугольники Л Е С С О Н Словарь Подобные многоугольники: Конгруэнтные многоугольники: Конгруэнтные многоугольники — это похожие многоугольники, которые имеют одинаковую форму и размер. Два многоугольника похожи, если у них одинаковые

    Геометрия Холта. 7-6 Расширения и сходство в координатной плоскости. Разминка Упростите каждый радикал. 1. 2. 3. Найдите расстояние между каждой парой точек.Запишите свой ответ в простейшем виде 1/2 1: 2 1 к 2 Пропорция: два эквивалентных соотношения. 1 2 6 3 = Подобные фигуры Та же форма, разные размеры Примеры фигур: Расширения = Увеличение | Уменьшение = меньшие похожие многоугольники 2 2 60o2 60o60o ~ 4 4 4 60o 60o 60o 1.) Соотношения мер соответствующих * Определите и классифицируйте многоугольники, включая четырехугольники, пятиугольники, шестиугольники и треугольники на основе угловых размеров и сторон. (MA.5.G.2) * Понимать, что двухмерная фигура похожа на другую, если вторая может быть получена из первой посредством последовательности преобразований.(MA.8.GM.5) * Установите критерий AA для сходства двух треугольников.

    Похожие полигоны. Соответствующие углы равны. Ответы к главе Сходство. 7-1. Соотношения в похожих полигонах. Практика А. 7. Соотношение упростится до 1: 1. 7-2. СХОДСТВО AND.Reddit дает вам лучшее из Интернета в одном месте. Получите постоянно обновляемую ленту последних новостей, забавных историй, картинок, мемов и видео только для вас. Вы увлечены чем-то нишевым? Reddit имеет тысячи ярких сообществ с людьми, которые разделяют ваши интересы.В качестве альтернативы, узнайте, что … Учебники по геометрии :: Бесплатные домашние задания и ответы :: Slader. Учебник геометрии отвечает на вопросы Обзор. Икс. Идти. 1. Введение в геометрию … 2.3 Биконусные утверждения 2.4 Дедуктивные рассуждения 2.5 Алгебраические рассуждения и доказательства 2.6 Геометрические рассуждения и доказательства 2.7 Доказательство угловых отношений 2.8 Доказательство отношений сегментов 2.9 Логика 2.10 Постулаты и доказательства абзацев 3.

    Gensim постоянно тестируется под Python 3.5, 3.6, 3.7 и 3.8. Поддержка Python 2.7 была прекращена в gensim 4.0.0 — установите gensim 3.8.3, если вам необходимо использовать Python 2.7.

    2. Убедитесь, что соотношения соответствующих длин сторон равны. Соотношение 1: Соотношение 2: Соотношение 3: Все три соотношения равны? 3. Запишите соотношения соответствующих длин сторон в заявлении о пропорциональности. Пример 2: Найдите коэффициент линейного масштабирования. Определите, похожи ли многоугольники. Если да, напишите сходство 1 День 1 — Глава 8–1: Соотношения и пропорции SWBAT: Используйте пропорции для решения задач Примените теоремы о произведении и соотношении Разминка — Разминка 1) 2) Отношение длин сторон четырехугольника равно 2 : 3: 5: 7, и его периметр Линейный масштабный коэффициент Коэффициент периметра Коэффициент площади 2 3 5 6 8 32 6 м Периметры похожих многоугольников Если два многоугольника похожи с длинами соответствующих сторон в соотношении a: b, то соотношение их периметров _____: _____.Коэффициент линейного масштабирования: 𝑺 𝑳 𝒕 𝑷 𝒍𝒚 Подобные многоугольники — многоугольники, которые имеют одинаковую форму, но разного размера. Масштаб — соотношение, в котором 2 одинаковых фигуры аналогичных многоугольников. 3 масштабный чертеж масштабный коэффициент масштаб AC = AT = CT GO GD OD 16 ABCD ~ NMPO Укажите масштабный коэффициент многоугольников. Найдите значение x. Округлите ответы на …

    Ключ ответов для рабочего листа 7-1 и 7-2. … Видео к урокам 7-1 и 7-2: Соотношения и пропорции. … Подобные треугольники и многоугольники. Примечания к уроку 7-3. Практическое пособие 0-07-877348-2 978-0-07-877348-8 Ответы для рабочих тетрадейОтветы к главе 7… Урок 7-2 Подобные многоугольники … коэффициент масштабирования подобных многоугольников. Джек сам знал ответы на все вопросы, и ему стало скучно. «Он угрюмый, — сказал Фрост, — поэтому он перебивает остальных или дает неправильный ответ, чтобы усложнить задачу». Задание 3. Прочтите текст ниже и решите, какой ответ A, B, C или D лучше всего подходит для каждого пробела. Выберите правильный ответ из предложенных вариантов.

    В этом похожем учебном упражнении с многоугольниками и треугольниками учащиеся решают 33 задачи с короткими ответами. Учащиеся определяют, похожи ли два заданных многоугольника.Учащиеся объясняют тип сходства, если обнаруживают, что два многоугольника похожи.

    Отношение показывает относительные размеры двух или более значений. Отношения могут быть показаны по-разному: • с помощью символа «:» для разделения примерных значений • с помощью «/» для отделения одного значения от общего • в виде десятичного числа после деления одного значения на общее • в процентах после деления одно значение в сумме

    ⟹ Ответьте на следующие вопросы. 1. С кем вы общаетесь каждый день? «Другая женщина выразила аналогичный парадокс.ее муж: «Когда мы выходим, он … Если я. 1. Какой ритуал вы видите на каждой фотографии? 2. Делаете ли вы что-нибудь подобное в своей стране? Используйте соотношение длин соответствующих сторон, чтобы написать пропорция, содержащая неизвестную длину. PHQI Запишите пропорцию, содержащую Q IJ R QR. 1 2 2 1 1 x 6 Замените известные значения. 12x 21 p 16 Свойство перекрестных произведений. 12x} 12 21 p 16} 12 Разделите каждую сторону на 12. x 28 Упростите. Ответ: Длина QR составляет 28 м. Упражнения для примера 1

    2 декабря 2020 г. · Ab bc cd de ae rs st tu uv rv отношение длин двух соответствующих сторон двух одинаковых многоугольников называется .Подобные и совпадающие многоугольники гр. 6 5 докажите, что треугольники похожи на рабочие листы sss и sas 6 4 и 6 5 13 f 12 12. Средство аналогично утверждению пропорциональности ar. Ключ к ответу на аналогичные рисунки 2 D Geometry Pack В 2020 году … geometry-7-2-ratios-in-similar-polygons. Добро пожаловать в клип от. План интерактивного видеоурока: Геометрия: 7-2 Соотношения в похожих многоугольниках.7-2 Подобные многоугольники Глава 7 186 1. … Запишите соотношения соответствующих сторон в простейшей форме. KL XY 5 10 15 5 LM YZ 5 15 5 MN ZW 5 15 5 NK WX 5 5

    Отношения длин соответствующих сторон равны, поэтому соответствующие стороны пропорциональны.Согласно постулату подобия многоугольников, JKL PQR. JL PR 2 3 KL QR 86 129 2 3 JK PQ 46 69 2 3 KL QR JL PR JK PQ Q 180 ° (70 ° 30 °) 80 ° L (70 ° 80 °) 30 ° KJ LP QR 86 129 69 46 70 ° 70 ° 30 ° 80 ° 90 135 Определите, похожи ли многоугольники. Если да, напишите 2/3 = 8/9. Как видите, это уравнение неверно — 8 — это произведение 2 на 4, а 9 — это произведение 3 на 3. Это означает, что эти отношения непропорциональны. Если бы мы хотели найти пропорциональное отношение к 2/3, сохраняя знаменатель другого отношения, нам пришлось бы умножить числитель 2 на число 3.

    Многоугольники в каждой паре похожи. Найдите недостающую длину стороны. ОТВЕТ-НАУКА-7.docx. Золотое сечение в 3D-моделировании лица. Практическая линейная алгебра и набор инструментов для геометрии. Программное обеспечение Kuta — Бесконечная геометрия. Похожие полигоны. Многоугольники в каждой паре похожи. Найдите коэффициент масштабирования меньшей фигуры по отношению к большей фигуре. Решите каждую пропорцию, используя известное соотношение, и соотношение с неизвестным. _ 5 a = 4 16 _ 7 b = _ 4 4a = 5 × 16 4b = 7 × 16 a = 20 b = 28 Следовательно, WX = 28 и YW = 20.Заявление о подобии — это утверждение, указывающее, что два многоугольника похожи, путем перечисления их вершин в порядке соответствия. Это очень похоже на написание раздела 5.2. Периметры и площади похожих фигур 203. Найдите соотношение (красного и синего) площадей 6 10 подобных треугольников. Площадь красного треугольника —— Площадь синего треугольника = (- 6 10) 2 = (3—5) 2 = 9 — 25 Отношение площадей равно 9 — 25. ПРИМЕР 2 Определение соотношений площадей ПРИМЕР 3 Реальное применение Вы помещаете изображение на …

    Полезный способ сравнить две величины — записать соотношение.Если в вашей семье 2 собаки и 3 кошки, соотношение собак и кошек составляет 2/3 или 2: 3. геометрия-7-2-отношения-в-похожих-многоугольниках. Добро пожаловать в клип от. План интерактивного видео-урока: Геометрия: Соотношения 7-2 в похожих многоугольниках. Обратите внимание, что в приведенных выше реализациях предполагается, что точки covnvex многоугольника даны по порядку (по часовой стрелке или против часовой стрелки). Упражнение: распространите вышеприведенное решение и на печать триангуляции.

    21 января 2015 г. · 7-3 похожих многоугольников 1.

    Котангенс 65: Таблица котангенсов, прочитать полную таблицу котангенсов

    Таблица котангенсов, прочитать полную таблицу котангенсов

    Содержание:

    Котангенс — равен отношению косинуса к синусу (ctg(x) = Cos(x)/Sin(x)), тоесть таблицу котангенсов можно получить просто поделив значения из таблицы косинусов на значения из таблицы синусов. Тангенс и котангенс находятся в прямой зависимости, так как tg(x) = Sin(x)/Cos(x), а ctg(x) = Cos(x)/Sin(x), то ctg(x) = 1/tg(x). Таким образом таблицу котангенсов можно получить из таблицы тангенсов (Надо только подставить нужное Вам значение в предыдущую формулу). Пользуйтесь таблицей котангенсов на здоровье.


    Таблица котангенсов 0° — 180°


    ctg(1°)57.29
    ctg(2°)28.6363
    ctg(3°)19.0811
    ctg(4°)14.3007
    ctg(5°)11.4301
    ctg(6°)9.5144
    ctg(7°)8. 1443
    ctg(8°)7.1154
    ctg(9°)6.3138
    ctg(10°)5.6713
    ctg(11°)5.1446
    ctg(12°)4.7046
    ctg(13°)4.3315
    ctg(14°)4.0108
    ctg(15°)3.7321
    ctg(16°)3.4874
    ctg(17°)3.2709
    ctg(18°)3.0777
    ctg(19°)2.9042
    ctg(20°)2.7475
    ctg(21°)2.6051
    ctg(22°)2.4751
    ctg(23°)2.3559
    ctg(24°)2.246
    ctg(25°)2.1445
    ctg(26°)2.0503
    ctg(27°)1.9626
    ctg(28°)1.8807
    ctg(29°)1.804
    ctg(30°)1.7321
    ctg(31°)1. 6643
    ctg(32°)1.6003
    ctg(33°)1.5399
    ctg(34°)1.4826
    ctg(35°)1.4281
    ctg(36°)1.3764
    ctg(37°)1.327
    ctg(38°)1.2799
    ctg(39°)1.2349
    ctg(40°)1.1918
    ctg(41°)1.1504
    ctg(42°)1.1106
    ctg(43°)1.0724
    ctg(44°)1.0355
    ctg(45°)1
    ctg(46°)0.9657
    ctg(47°)0.9325
    ctg(48°)0.9004
    ctg(49°)0.8693
    ctg(50°)0.8391
    ctg(51°)0.8098
    ctg(52°)0.7813
    ctg(53°)0.7536
    ctg(54°)0.7265
    ctg(55°)0. 7002
    ctg(56°)0.6745
    ctg(57°)0.6494
    ctg(58°)0.6249
    ctg(59°)0.6009
    ctg(60°)0.5774
    ctg(61°)0.5543
    ctg(62°)0.5317
    ctg(63°)0.5095
    ctg(64°)0.4877
    ctg(65°)0.4663
    ctg(66°)0.4452
    ctg(67°)0.4245
    ctg(68°)0.404
    ctg(69°)0.3839
    ctg(70°)0.364
    ctg(71°)0.3443
    ctg(72°)0.3249
    ctg(73°)0.3057
    ctg(74°)0.2867
    ctg(75°)0.2679
    ctg(76°)0.2493
    ctg(77°)0.2309
    ctg(78°)0. 2126
    ctg(79°)0.1944
    ctg(80°)0.1763
    ctg(81°)0.1584
    ctg(82°)0.1405
    ctg(83°)0.1228
    ctg(84°)0.1051
    ctg(85°)0.0875
    ctg(86°)0.0699
    ctg(87°)0.0524
    ctg(88°)0.0349
    ctg(89°)0.0175
    ctg(90°)0
    ctg(91°)-0.0175
    ctg(92°)-0.0349
    ctg(93°)-0.0524
    ctg(94°)-0.0699
    ctg(95°)-0.0875
    ctg(96°)-0.1051
    ctg(97°)-0.1228
    ctg(98°)-0.1405
    ctg(99°)-0.1584
    ctg(100°)-0.1763
    ctg(101°)-0.1944
    ctg(102°)-0. 2126
    ctg(103°)-0.2309
    ctg(104°)-0.2493
    ctg(105°)-0.2679
    ctg(106°)-0.2867
    ctg(107°)-0.3057
    ctg(108°)-0.3249
    ctg(109°)-0.3443
    ctg(110°)-0.364
    ctg(111°)-0.3839
    ctg(112°)-0.404
    ctg(113°)-0.4245
    ctg(114°)-0.4452
    ctg(115°)-0.4663
    ctg(116°)-0.4877
    ctg(117°)-0.5095
    ctg(118°)-0.5317
    ctg(119°)-0.5543
    ctg(120°)-0.5774
    ctg(121°)-0.6009
    ctg(122°)-0.6249
    ctg(123°)-0.6494
    ctg(124°)-0. 6745
    ctg(125°)-0.7002
    ctg(126°)-0.7265
    ctg(127°)-0.7536
    ctg(128°)-0.7813
    ctg(129°)-0.8098
    ctg(130°)-0.8391
    ctg(131°)-0.8693
    ctg(132°)-0.9004
    ctg(133°)-0.9325
    ctg(134°)-0.9657
    ctg(135°)-1
    ctg(136°)-1.0355
    ctg(137°)-1.0724
    ctg(138°)-1.1106
    ctg(139°)-1.1504
    ctg(140°)-1.1918
    ctg(141°)-1.2349
    ctg(142°)-1.2799
    ctg(143°)-1.327
    ctg(144°)-1.3764
    ctg(145°)-1.4281
    ctg(146°)-1.4826
    ctg(147°)-1. 5399
    ctg(148°)-1.6003
    ctg(149°)-1.6643
    ctg(150°)-1.7321
    ctg(151°)-1.804
    ctg(152°)-1.8807
    ctg(153°)-1.9626
    ctg(154°)-2.0503
    ctg(155°)-2.1445
    ctg(156°)-2.246
    ctg(157°)-2.3559
    ctg(158°)-2.4751
    ctg(159°)-2.6051
    ctg(160°)-2.7475
    ctg(161°)-2.9042
    ctg(162°)-3.0777
    ctg(163°)-3.2709
    ctg(164°)-3.4874
    ctg(165°)-3.7321
    ctg(166°)-4.0108
    ctg(167°)-4.3315
    ctg(168°)-4.7046
    ctg(169°)-5.1446
    ctg(170°)-5. 6713
    ctg(171°)-6.3138
    ctg(172°)-7.1154
    ctg(173°)-8.1443
    ctg(174°)-9.5144
    ctg(175°)-11.4301
    ctg(176°)-14.3007
    ctg(177°)-19.0811
    ctg(178°)-28.6363
    ctg(179°)-57.29
    ctg(180°)— ∞

    Таблица котангенсов 180° — 360°


    ctg(181°)57.29
    ctg(182°)28.6363
    ctg(183°)19.0811
    ctg(184°)14.3007
    ctg(185°)11.4301
    ctg(186°)9.5144
    ctg(187°)8.1443
    ctg(188°)7.1154
    ctg(189°)6.3138
    ctg(190°)5. 6713
    ctg(191°)5.1446
    ctg(192°)4.7046
    ctg(193°)4.3315
    ctg(194°)4.0108
    ctg(195°)3.7321
    ctg(196°)3.4874
    ctg(197°)3.2709
    ctg(198°)3.0777
    ctg(199°)2.9042
    ctg(200°)2.7475
    ctg(201°)2.6051
    ctg(202°)2.4751
    ctg(203°)2.3559
    ctg(204°)2.246
    ctg(205°)2.1445
    ctg(206°)2.0503
    ctg(207°)1.9626
    ctg(208°)1.8807
    ctg(209°)1.804
    ctg(210°)1.7321
    ctg(211°)1.6643
    ctg(212°)1.6003
    ctg(213°)1. 5399
    ctg(214°)1.4826
    ctg(215°)1.4281
    ctg(216°)1.3764
    ctg(217°)1.327
    ctg(218°)1.2799
    ctg(219°)1.2349
    ctg(220°)1.1918
    ctg(221°)1.1504
    ctg(222°)1.1106
    ctg(223°)1.0724
    ctg(224°)1.0355
    ctg(225°)1
    ctg(226°)0.9657
    ctg(227°)0.9325
    ctg(228°)0.9004
    ctg(229°)0.8693
    ctg(230°)0.8391
    ctg(231°)0.8098
    ctg(232°)0.7813
    ctg(233°)0.7536
    ctg(234°)0.7265
    ctg(235°)0.7002
    ctg(236°)0. 6745
    ctg(237°)0.6494
    ctg(238°)0.6249
    ctg(239°)0.6009
    ctg(240°)0.5774
    ctg(241°)0.5543
    ctg(242°)0.5317
    ctg(243°)0.5095
    ctg(244°)0.4877
    ctg(245°)0.4663
    ctg(246°)0.4452
    ctg(247°)0.4245
    ctg(248°)0.404
    ctg(249°)0.3839
    ctg(250°)0.364
    ctg(251°)0.3443
    ctg(252°)0.3249
    ctg(253°)0.3057
    ctg(254°)0.2867
    ctg(255°)0.2679
    ctg(256°)0.2493
    ctg(257°)0.2309
    ctg(258°)0. 2126
    ctg(259°)0.1944
    ctg(260°)0.1763
    ctg(261°)0.1584
    ctg(262°)0.1405
    ctg(263°)0.1228
    ctg(264°)0.1051
    ctg(265°)0.0875
    ctg(266°)0.0699
    ctg(267°)0.0524
    ctg(268°)0.0349
    ctg(269°)0.0175
    ctg(270°)0
    ctg(271°)-0.0175
    ctg(272°)-0.0349
    ctg(273°)-0.0524
    ctg(274°)-0.0699
    ctg(275°)-0.0875
    ctg(276°)-0.1051
    ctg(277°)-0.1228
    ctg(278°)-0.1405
    ctg(279°)-0.1584
    ctg(280°)-0.1763
    ctg(281°)-0. 1944
    ctg(282°)-0.2126
    ctg(283°)-0.2309
    ctg(284°)-0.2493
    ctg(285°)-0.2679
    ctg(286°)-0.2867
    ctg(287°)-0.3057
    ctg(288°)-0.3249
    ctg(289°)-0.3443
    ctg(290°)-0.364
    ctg(291°)-0.3839
    ctg(292°)-0.404
    ctg(293°)-0.4245
    ctg(294°)-0.4452
    ctg(295°)-0.4663
    ctg(296°)-0.4877
    ctg(297°)-0.5095
    ctg(298°)-0.5317
    ctg(299°)-0.5543
    ctg(300°)-0.5774
    ctg(301°)-0.6009
    ctg(302°)-0.6249
    ctg(303°)-0. 6494
    ctg(304°)-0.6745
    ctg(305°)-0.7002
    ctg(306°)-0.7265
    ctg(307°)-0.7536
    ctg(308°)-0.7813
    ctg(309°)-0.8098
    ctg(310°)-0.8391
    ctg(311°)-0.8693
    ctg(312°)-0.9004
    ctg(313°)-0.9325
    ctg(314°)-0.9657
    ctg(315°)-1
    ctg(316°)-1.0355
    ctg(317°)-1.0724
    ctg(318°)-1.1106
    ctg(319°)-1.1504
    ctg(320°)-1.1918
    ctg(321°)-1.2349
    ctg(322°)-1.2799
    ctg(323°)-1.327
    ctg(324°)-1.3764
    ctg(325°)-1.4281
    ctg(326°)-1. 4826
    ctg(327°)-1.5399
    ctg(328°)-1.6003
    ctg(329°)-1.6643
    ctg(330°)-1.7321
    ctg(331°)-1.804
    ctg(332°)-1.8807
    ctg(333°)-1.9626
    ctg(334°)-2.0503
    ctg(335°)-2.1445
    ctg(336°)-2.246
    ctg(337°)-2.3559
    ctg(338°)-2.4751
    ctg(339°)-2.6051
    ctg(340°)-2.7475
    ctg(341°)-2.9042
    ctg(342°)-3.0777
    ctg(343°)-3.2709
    ctg(344°)-3.4874
    ctg(345°)-3.7321
    ctg(346°)-4.0108
    ctg(347°)-4.3315
    ctg(348°)-4.7046
    ctg(349°)-5. 1446
    ctg(350°)-5.6713
    ctg(351°)-6.3138
    ctg(352°)-7.1154
    ctg(353°)-8.1443
    ctg(354°)-9.5144
    ctg(355°)-11.4301
    ctg(356°)-14.3007
    ctg(357°)-19.0811
    ctg(358°)-28.6363
    ctg(359°)-57.29
    ctg(360°)

    Слишком сложно?

    Таблица котангенсов, таблица значений котангенсов не по зубам? Тебе ответит эксперт через 10 минут!

    Новости за 7 дней.

    Сколько предметов домашнего обихода должно быть под рукой в ванной комнате? Их десятки. И что с ними делать? Как правило, они не отличаются выдающимся дизайном. Основой набора мебели для ванной комнаты Step стали популярные накладные раковины, устанавливаемые на столешницу, для которых предусмот.

    Ассортимент гофрированных труб из нержавеющей стали торговой марки Stahlmann пополнился новыми диаметрами: 40А и 50А. Компания «Электросистемы и технологии» (входит в ГК «ССТ), официальный дистрибьютор бренда Stahlmann, по многочисленным просьбам клиентов расширила ассортимент гибких гофрированны….

    Компания группы PORCELANOSA Grupo представляет свои новые коллекции напольного покрытия для наружного применения и самые инновационные технические решения для ванных комнат и систем гидроизоляции в официальных магазинах Испании и Португалии. Butech расширяет свой каталог продукции и технических реш….

    В ассортименте EKF появилась эргономичная розетка для кухни со встраиваемой техникой. Новинка c разъёмами типа РШ-ВШ позволяет удобно и эстетично подключить сразу два прибора – варочную панель и духовку. Преимущества нового изделия: привлекательная цена – можно сэкономить до 20 % бюджета; ла….

    Серия MPT включает четыре модели носимых видеорегистраторов Dahua со встроенными видеокамерами для ведения аудио- и видеозаписи непосредственно на месте события и формирования в случае происшествия доказательной базы. Эти мобильные устройства предназначены для использования в сфере обеспечения обще….

    Одноабонентская вызывная панель IP-видеодомофона VTO2211G-WP обладает элегантным дизайном и тонкой легкой конструкцией. При этом она оснащена всем необходимым для быстрой установки и удобства эксплуатации. Помимо проводного интерфейса Ethernet, который также поддерживает подачу питания PoE, вызывн….

    Стремительное развитие технологий и рост современных городов значительно влияют на наш образ жизни, дизайн и архитектуру. В интерьерах стиль лофт лучше всего отражает урбанистический дух, предоставляя простор для творчества и самовыражения. Новая коллекция мебели AQUATON ЛОФТ Урбан объединяет ос….

    Решить проблему размещения на плоских кровлях дополнительного оборудования призваны два инновационных технических решения, разработанных Группой компаний fischer, мировым лидером в разработке и производстве современных крепежных изделий. Новые кровельные опоры — FFRB и FFRBH — призваны сделать эксп.

    За изысканным интерьером всегда стоит качественный крепёж, который позволяет надёжно фиксировать полки, картины, люстры и другие аксессуары. Именно эту задачу решает серия пластиковых дюбелей с крюком EasyHook — новинка компании fischer, мирового лидера в сфере инновационных крепёжных решений. В с….

    Качественная краска для деревянного пола – эффективное решение при реставрации старого или обустройстве нового напольного покрытия. Правильно подобранный ЛКМ защитит дерево от истирания, исцарапывания, влаги, ультрафиолета, сохранит красивую фактуру дерева, придаст нужный оттенок, а также продлит с….

    Представляем НОВИНКУ – клей SUPERFLEX K77 Белый для керамической плитки и керамогранита. SUPERFLEX K77 Белый – высокоэластичный плиточный клей на основе белого цемента для укладки любого типа плитки из керамогранита, клинкера, керамики и натурального камня, в том числе крупного формата. Свойства….

    Динамики подавляющего большинства телевизоров хорошо справляются лишь с воспроизведением голосов дикторов новостей, а вот для музыки и спецэффектов в кино требуется более серьезное решение. Вот только большие колонки полноформатного домашнего кинотеатра — далеко не самый удобный и комфортный выход ….

    Устройства ввода — это та часть компьютера, с которой мы напрямую контактируем каждый день. И именно от них часто зависит, насколько удобно нам будет работать, учиться или играть. Поэтому компания SVEN постоянно расширяет ассортимент компьютерных мышей и клавиатур, предлагая все новые решения. Ко….

    Выбирайте паровую станцию, чтобы почувствовать себя обладателем профессиональной техники для домашнего использования. По сравнению с классическими паровыми утюгами, паровая станция VT-2430 позволит Вам гладить белье в несколько раз быстрее и качественнее. Отгладить костюм, брюки, платье, плащ или ….

    Новый цвет — море сочетаний. За поисками этого оттенка мы отправились в Северную Европу. Нам нужен был серый, который вызывает ассоциацию с природой, а не бетонными джунглями. Глядя на пейзажи Исландии, мы поняли: «Вот он. Тот самый цвет». Спокойный, насыщенный, с теплым коричневым подтоном. ….

    Компания dormakaba рада предложить Вам бесшумные решения для межкомнатных дверей — защёлки DORMA со смещённым магнитным ригелем серии 940-М WC и 940-М PZ. Товар на складе. Цвет исполнения торцевой планки замка: АВ – античная бронза и SN – матовый никель. Магнитные замки рекомендуются для установ….

    Стилизованный рисунок натюрморта с кофе в обрамлении кофейных зернышек и сегменты с надписями на кофейную тематику чередуются с плитками, воспроизводящими фактуру шероховатого камня. Баланс между акцентными и фоновыми элементами решен в пользу фона, что создает воздушность композиции, но при этом с….

    Нежный узор из стилизованных полевых цветов. Плавными каллиграфическими росчерками он заполняет пространство, создавая легкий, вальсирующий ритм композиции. Отдельные элементы узора не объединены в сетку или колонны, традиционные для ритмики обойных принтов, а соединены в V-образные пересечения со.

    Ветки, усыпанные некрупными цветами, застилают все полотно. Цельность композиции и наполненность пространства дизайна создают умиротворяющую обстановку и успокаивающий ритм. Тонкие веточки почти полностью укрыты цветами, присутствуют в узоре минимально. Переходы между элементами сглажены, отсутст….

    Компактная вилка PPG16-42-201 с заземлением имеют разборную конструкцию и выполнена из ABS пластика и латунных токоведущих контактов. Заземляющие стальные контакты, предусмотренные в конструкции, позволяют безопасно эксплуатировать электроприборы. Применение: Вилки разборные STEKKER серии PPG п….

    Коробка «Express» 53800R теперь выпускаются в обновленном конструктиве. 8 герметичных вводов расположены по периметру коробки с максимальным размером вводов до 25 мм, а 2 дополнительных отверстия – на торцевой части коробки (их размер до 20 мм). Теперь есть возможность использовать c ответвительны….

    Ассортимент шкафов из фибергласа пополнился новинками – в линейке появились цельнолитые навесные шкафы. Корпуса, изготовленные по этой технологии, имеют более высокую степень пыле- и влагозащиты и меньшую стоимость. При этом в новых моделях реализуются и все преимущества фибергласа: абсолютная кор….

    Беспроводная технология LoRa – отличное решение для управления уличным освещением как для целых районов или дорог, так и для ограниченных участков – парковок ТЦ, дворов ЖК, парков и скверов. Достаточно «защелкнуть» в светильник «умный» LoRa контроллер через стандартный NEMA разъем и освещение управ….

    Ассортимент Werkel™ пополнился розетками с подсветкой в новых цветах: серебряный и слоновая кость. Кроме своей основной функции — питания электроприборов, розетка с помощью подсветки помогает обозначить себя в темное время суток. Подсветка создает равномерное рассеянное свечение, подходящее для ….

    “Освещение придает пространству индивидуальный шарм. Важно, чтобы оно было отражением владельцев, подчеркивало многогранность дизайнерской идеи, даже в сдержанном и минималистичном интерьере. ” — Добрый день! Меня зовут Заблодская Камилия, дизайнер студии «Time» с пятилетним опытом, и сегодня я под….

    Идеально вписываются в любой интерьер и экономят средства на электроэнергии! ЭРА обновила ассортимент светодиодных ламп со штырьковым цоколем G4-G9, созданных для прямой замены предшественника – галогенной лампы. Капсульный светодиод мощностью 3-6 Вт излучает столько же света, сколько галогенная л….

    К летнему сезону сформирован хороший товарных запас по силовым удлинителям ЭРА для дачных и строительных работ. Второй квартал это самое горячее время для данной товарной группы, предлагаем обратить внимание на ассортимент. Серия ЭРА RPx — удлинители на пластиковой катушке; Серия ЭРА RMx — уд….

    Компания ФОКУС представляет новый светильник ЖКХ 10, разработанный для освещения подъездов, лестничных площадок, коридоров и вспомогательных помещений. Благодаря степени защиты IP 65, обеспечивающей достаточную защиту от влаги и пыли, светильник так же можно размещать в помещениях с повышенной вл.

    Компания представляет новую мебель для ванных комнат и спален, а также инновационную линейку кухонных гарнитуров, изготовленных из дерева и XTONE, для организации функциональных пространств. Компания Gamadecor делает выбор в пользу бесконечных и функциональных пространств за счет использования див….

    Таблица котангенсов | umath.ru

    Котангенсом угла называется отношение косинуса этого угла к синусу:

       

    Таблица котангенсов — таблица, содержащая значения котангенсов углов. В нашей таблице вычислены котангенсы углов от 1° до 180°.

    Таблицы котангенсов удобно использовать при отсутствии калькулятора с тригонометрическими функциями.

    См. также: таблица синусов, таблица косинусов, таблица тангенсов.

    Таблица котангенсов углов 0°, 30°, 45°, 60°, 90°

    Замечание: котангенс 0° не определён, так как .

    Таблица котангенсов углов от 1° до 90°

    ctg(1°) = 57.289962
    ctg(2°) = 28. 636253
    ctg(3°) = 19.081137
    ctg(4°) = 14.300666
    ctg(5°) = 11.430052
    ctg(6°) = 9.514364
    ctg(7°) = 8.144346
    ctg(8°) = 7.115370
    ctg(9°) = 6.313752
    ctg(10°) = 5.671282
    ctg(11°) = 5.144554
    ctg(12°) = 4.704630
    ctg(13°) = 4.331476
    ctg(14°) = 4.010781
    ctg(15°) = 3.732051
    ctg(16°) = 3.487414
    ctg(17°) = 3.270853
    ctg(18°) = 3.077684
    ctg(19°) = 2.904211
    ctg(20°) = 2.747477
    ctg(21°) = 2.605089
    ctg(22°) = 2.475087
    ctg(23°) = 2.355852
    ctg(24°) = 2.246037
    ctg(25°) = 2.144507
    ctg(26°) = 2.050304
    ctg(27°) = 1.962611
    ctg(28°) = 1.880726
    ctg(29°) = 1.804048
    ctg(30°) = 1.732051
    ctg(31°) = 1.664279
    ctg(32°) = 1.600335
    ctg(33°) = 1.539865
    ctg(34°) = 1.482561
    ctg(35°) = 1.428148
    ctg(36°) = 1.376382
    ctg(37°) = 1.327045
    ctg(38°) = 1.279942
    ctg(39°) = 1.234897
    ctg(40°) = 1.191754
    ctg(41°) = 1.150368
    ctg(42°) = 1.110613
    ctg(43°) = 1.072369
    ctg(44°) = 1. 035530
    ctg(45°) = 1
    ctg(46°) = 0.965689
    ctg(47°) = 0.932515
    ctg(48°) = 0.900404
    ctg(49°) = 0.869287
    ctg(50°) = 0.839100
    ctg(51°) = 0.809784
    ctg(52°) = 0.781286
    ctg(53°) = 0.753554
    ctg(54°) = 0.726543
    ctg(55°) = 0.700208
    ctg(56°) = 0.674509
    ctg(57°) = 0.649408
    ctg(58°) = 0.624869
    ctg(59°) = 0.600861
    ctg(60°) = 0.577350
    ctg(61°) = 0.554309
    ctg(62°) = 0.531709
    ctg(63°) = 0.509525
    ctg(64°) = 0.487733
    ctg(65°) = 0.466308
    ctg(66°) = 0.445229
    ctg(67°) = 0.424475
    ctg(68°) = 0.404026
    ctg(69°) = 0.383864
    ctg(70°) = 0.363970
    ctg(71°) = 0.344328
    ctg(72°) = 0.324920
    ctg(73°) = 0.305731
    ctg(74°) = 0.286745
    ctg(75°) = 0.267949
    ctg(76°) = 0.249328
    ctg(77°) = 0.230868
    ctg(78°) = 0.212557
    ctg(79°) = 0.194380
    ctg(80°) = 0.176327
    ctg(81°) = 0.158384
    ctg(82°) = 0.140541
    ctg(83°) = 0.122785
    ctg(84°) = 0.105104
    ctg(85°) = 0.087489
    ctg(86°) = 0. 069927
    ctg(87°) = 0.052408
    ctg(88°) = 0.034921
    ctg(89°) = 0.017455
    ctg(90°) = 0

    Таблица котангенсов углов от 91° до 180°

    ctg(91°) = -0.017455
    ctg(92°) = -0.034921
    ctg(93°) = -0.052408
    ctg(94°) = -0.069927
    ctg(95°) = -0.087489
    ctg(96°) = -0.105104
    ctg(97°) = -0.122785
    ctg(98°) = -0.140541
    ctg(99°) = -0.158384
    ctg(100°) = -0.176327
    ctg(101°) = -0.194380
    ctg(102°) = -0.212557
    ctg(103°) = -0.230868
    ctg(104°) = -0.249328
    ctg(105°) = -0.267949
    ctg(106°) = -0.286745
    ctg(107°) = -0.305731
    ctg(108°) = -0.324920
    ctg(109°) = -0.344328
    ctg(110°) = -0.363970
    ctg(111°) = -0.383864
    ctg(112°) = -0.404026
    ctg(113°) = -0.424475
    ctg(114°) = -0.445229
    ctg(115°) = -0.466308
    ctg(116°) = -0.487733
    ctg(117°) = -0.509525
    ctg(118°) = -0.531709
    ctg(119°) = -0.554309
    ctg(120°) = -0.577350
    ctg(121°) = -0.600861
    ctg(122°) = -0. 624869
    ctg(123°) = -0.649408
    ctg(124°) = -0.674509
    ctg(125°) = -0.700208
    ctg(126°) = -0.726543
    ctg(127°) = -0.753554
    ctg(128°) = -0.781286
    ctg(129°) = -0.809784
    ctg(130°) = -0.839100
    ctg(131°) = -0.869287
    ctg(132°) = -0.900404
    ctg(133°) = -0.932515
    ctg(134°) = -0.965689
    ctg(135°) = -1
    ctg(136°) = -1.035530
    ctg(137°) = -1.072369
    ctg(138°) = -1.110613
    ctg(139°) = -1.150368
    ctg(140°) = -1.191754
    ctg(141°) = -1.234897
    ctg(142°) = -1.279942
    ctg(143°) = -1.327045
    ctg(144°) = -1.376382
    ctg(145°) = -1.428148
    ctg(146°) = -1.482561
    ctg(147°) = -1.539865
    ctg(148°) = -1.600335
    ctg(149°) = -1.664279
    ctg(150°) = -1.732051
    ctg(151°) = -1.804048
    ctg(152°) = -1.880726
    ctg(153°) = -1.962611
    ctg(154°) = -2.050304
    ctg(155°) = -2.144507
    ctg(156°) = -2.246037
    ctg(157°) = -2.355852
    ctg(158°) = -2.475087
    ctg(159°) = -2.605089
    ctg(160°) = -2.747477
    ctg(161°) = -2. 904211
    ctg(162°) = -3.077684
    ctg(163°) = -3.270853
    ctg(164°) = -3.487414
    ctg(165°) = -3.732051
    ctg(166°) = -4.010781
    ctg(167°) = -4.331476
    ctg(168°) = -4.704630
    ctg(169°) = -5.144554
    ctg(170°) = -5.671282
    ctg(171°) = -6.313752
    ctg(172°) = -7.115370
    ctg(173°) = -8.144346
    ctg(174°) = -9.514364
    ctg(175°) = -11.430052
    ctg(176°) = -14.300666
    ctg(177°) = -19.081137
    ctg(178°) = -28.636253
    ctg(179°) = -57.289962
    ctg(180°) не определено

    Таблица котангенсов.

    Таблица котангенсов — это записанные в таблицу посчитанные значения котангенсов углов от 0° до 360°. Используя таблицу котангенсов Вы сможете провести расчеты даже если под руками не окажется инженерного калькулятора. Чтобы узнать значение котангенса от нужного Вам угла достаточно найти его в таблице.

    Таблица котангенсов в радианах

    α0π6π4π3π2π3π2
    сtg α√31√3300

    Таблица котангенсов углов от 0° до 180°

    ctg(0°) = ∞
    ctg(1°) = 57. 28996
    ctg(2°) = 28.63625
    ctg(3°) = 19.08114
    ctg(4°) = 14.30067
    ctg(5°) = 11.43005
    ctg(6°) = 9.51436
    ctg(7°) = 8.14435
    ctg(8°) = 7.11537
    ctg(9°) = 6.31375
    ctg(10°) = 5.67128
    ctg(11°) = 5.14455
    ctg(12°) = 4.70463
    ctg(13°) = 4.33148
    ctg(14°) = 4.01078
    ctg(15°) = 3.73205
    ctg(16°) = 3.48741
    ctg(17°) = 3.27085
    ctg(18°) = 3.07768
    ctg(19°) = 2.90421
    ctg(20°) = 2.74748
    ctg(21°) = 2.60509
    ctg(22°) = 2.47509
    ctg(23°) = 2.35585
    ctg(24°) = 2.24604
    ctg(25°) = 2.14451
    ctg(26°) = 2.0503
    ctg(27°) = 1.96261
    ctg(28°) = 1.88073
    ctg(29°) = 1.80405
    ctg(30°) = 1.73205
    ctg(31°) = 1.66428
    ctg(32°) = 1.60033
    ctg(33°) = 1.53986
    ctg(34°) = 1.48256
    ctg(35°) = 1.42815
    ctg(36°) = 1.37638
    ctg(37°) = 1.32704
    ctg(38°) = 1.27994
    ctg(39°) = 1.2349
    ctg(40°) = 1.19175
    ctg(41°) = 1.15037
    ctg(42°) = 1.11061
    ctg(43°) = 1.07237
    ctg(44°) = 1.03553
    ctg(45°) = 1
    ctg(46°) = 0. 96569
    ctg(47°) = 0.93252
    ctg(48°) = 0.9004
    ctg(49°) = 0.86929
    ctg(50°) = 0.8391
    ctg(51°) = 0.80978
    ctg(52°) = 0.78129
    ctg(53°) = 0.75355
    ctg(54°) = 0.72654
    ctg(55°) = 0.70021
    ctg(56°) = 0.67451
    ctg(57°) = 0.64941
    ctg(58°) = 0.62487
    ctg(59°) = 0.60086
    ctg(60°) = 0.57735
    ctg(61°) = 0.55431
    ctg(62°) = 0.53171
    ctg(63°) = 0.50953
    ctg(64°) = 0.48773
    ctg(65°) = 0.46631
    ctg(66°) = 0.44523
    ctg(67°) = 0.42447
    ctg(68°) = 0.40403
    ctg(69°) = 0.38386
    ctg(70°) = 0.36397
    ctg(71°) = 0.34433
    ctg(72°) = 0.32492
    ctg(73°) = 0.30573
    ctg(74°) = 0.28675
    ctg(75°) = 0.26795
    ctg(76°) = 0.24933
    ctg(77°) = 0.23087
    ctg(78°) = 0.21256
    ctg(79°) = 0.19438
    ctg(80°) = 0.17633
    ctg(81°) = 0.15838
    ctg(82°) = 0.14054
    ctg(83°) = 0.12278
    ctg(84°) = 0.1051
    ctg(85°) = 0.08749
    ctg(86°) = 0.06993
    ctg(87°) = 0.05241
    ctg(88°) = 0.03492
    ctg(89°) = 0.01746
    ctg(90°) = 0
    ctg(91°) = -0. 01746
    ctg(92°) = -0.03492
    ctg(93°) = -0.05241
    ctg(94°) = -0.06993
    ctg(95°) = -0.08749
    ctg(96°) = -0.1051
    ctg(97°) = -0.12278
    ctg(98°) = -0.14054
    ctg(99°) = -0.15838
    ctg(100°) = -0.17633
    ctg(101°) = -0.19438
    ctg(102°) = -0.21256
    ctg(103°) = -0.23087
    ctg(104°) = -0.24933
    ctg(105°) = -0.26795
    ctg(106°) = -0.28675
    ctg(107°) = -0.30573
    ctg(108°) = -0.32492
    ctg(109°) = -0.34433
    ctg(110°) = -0.36397
    ctg(111°) = -0.38386
    ctg(112°) = -0.40403
    ctg(113°) = -0.42447
    ctg(114°) = -0.44523
    ctg(115°) = -0.46631
    ctg(116°) = -0.48773
    ctg(117°) = -0.50953
    ctg(118°) = -0.53171
    ctg(119°) = -0.55431
    ctg(120°) = -0.57735
    ctg(121°) = -0.60086
    ctg(122°) = -0.62487
    ctg(123°) = -0.64941
    ctg(124°) = -0.67451
    ctg(125°) = -0.70021
    ctg(126°) = -0.72654
    ctg(127°) = -0.75355
    ctg(128°) = -0.78129
    ctg(129°) = -0.80978
    ctg(130°) = -0.8391
    ctg(131°) = -0.86929
    ctg(132°) = -0. 9004
    ctg(133°) = -0.93252
    ctg(134°) = -0.96569
    ctg(135°) = -1
    ctg(136°) = -1.03553
    ctg(137°) = -1.07237
    ctg(138°) = -1.11061
    ctg(139°) = -1.15037
    ctg(140°) = -1.19175
    ctg(141°) = -1.2349
    ctg(142°) = -1.27994
    ctg(143°) = -1.32704
    ctg(144°) = -1.37638
    ctg(145°) = -1.42815
    ctg(146°) = -1.48256
    ctg(147°) = -1.53986
    ctg(148°) = -1.60033
    ctg(149°) = -1.66428
    ctg(150°) = -1.73205
    ctg(151°) = -1.80405
    ctg(152°) = -1.88073
    ctg(153°) = -1.96261
    ctg(154°) = -2.0503
    ctg(155°) = -2.14451
    ctg(156°) = -2.24604
    ctg(157°) = -2.35585
    ctg(158°) = -2.47509
    ctg(159°) = -2.60509
    ctg(160°) = -2.74748
    ctg(161°) = -2.90421
    ctg(162°) = -3.07768
    ctg(163°) = -3.27085
    ctg(164°) = -3.48741
    ctg(165°) = -3.73205
    ctg(166°) = -4.01078
    ctg(167°) = -4.33148
    ctg(168°) = -4.70463
    ctg(169°) = -5.14455
    ctg(170°) = -5.67128
    ctg(171°) = -6.31375
    ctg(172°) = -7.11537
    ctg(173°) = -8. 14435
    ctg(174°) = -9.51436
    ctg(175°) = -11.43005
    ctg(176°) = -14.30067
    ctg(177°) = -19.08114
    ctg(178°) = -28.63625
    ctg(179°) = -57.28996
    ctg(180°) = ∞

    Таблица котангенсов углов от 181° до 360°

    ctg(181°) = 57.28996
    ctg(182°) = 28.63625
    ctg(183°) = 19.08114
    ctg(184°) = 14.30067
    ctg(185°) = 11.43005
    ctg(186°) = 9.51436
    ctg(187°) = 8.14435
    ctg(188°) = 7.11537
    ctg(189°) = 6.31375
    ctg(190°) = 5.67128
    ctg(191°) = 5.14455
    ctg(192°) = 4.70463
    ctg(193°) = 4.33148
    ctg(194°) = 4.01078
    ctg(195°) = 3.73205
    ctg(196°) = 3.48741
    ctg(197°) = 3.27085
    ctg(198°) = 3.07768
    ctg(199°) = 2.90421
    ctg(200°) = 2.74748
    ctg(201°) = 2.60509
    ctg(202°) = 2.47509
    ctg(203°) = 2.35585
    ctg(204°) = 2.24604
    ctg(205°) = 2.14451
    ctg(206°) = 2.0503
    ctg(207°) = 1.96261
    ctg(208°) = 1.88073
    ctg(209°) = 1.80405
    ctg(210°) = 1.73205
    ctg(211°) = 1. 66428
    ctg(212°) = 1.60033
    ctg(213°) = 1.53986
    ctg(214°) = 1.48256
    ctg(215°) = 1.42815
    ctg(216°) = 1.37638
    ctg(217°) = 1.32704
    ctg(218°) = 1.27994
    ctg(219°) = 1.2349
    ctg(220°) = 1.19175
    ctg(221°) = 1.15037
    ctg(222°) = 1.11061
    ctg(223°) = 1.07237
    ctg(224°) = 1.03553
    ctg(225°) = 1
    ctg(226°) = 0.96569
    ctg(227°) = 0.93252
    ctg(228°) = 0.9004
    ctg(229°) = 0.86929
    ctg(230°) = 0.8391
    ctg(231°) = 0.80978
    ctg(232°) = 0.78129
    ctg(233°) = 0.75355
    ctg(234°) = 0.72654
    ctg(235°) = 0.70021
    ctg(236°) = 0.67451
    ctg(237°) = 0.64941
    ctg(238°) = 0.62487
    ctg(239°) = 0.60086
    ctg(240°) = 0.57735
    ctg(241°) = 0.55431
    ctg(242°) = 0.53171
    ctg(243°) = 0.50953
    ctg(244°) = 0.48773
    ctg(245°) = 0.46631
    ctg(246°) = 0.44523
    ctg(247°) = 0.42447
    ctg(248°) = 0.40403
    ctg(249°) = 0.38386
    ctg(250°) = 0.36397
    ctg(251°) = 0.34433
    ctg(252°) = 0.32492
    ctg(253°) = 0. 30573
    ctg(254°) = 0.28675
    ctg(255°) = 0.26795
    ctg(256°) = 0.24933
    ctg(257°) = 0.23087
    ctg(258°) = 0.21256
    ctg(259°) = 0.19438
    ctg(260°) = 0.17633
    ctg(261°) = 0.15838
    ctg(262°) = 0.14054
    ctg(263°) = 0.12278
    ctg(264°) = 0.1051
    ctg(265°) = 0.08749
    ctg(266°) = 0.06993
    ctg(267°) = 0.05241
    ctg(268°) = 0.03492
    ctg(269°) = 0.01746
    ctg(270°) = 0
    ctg(271°) = -0.01746
    ctg(272°) = -0.03492
    ctg(273°) = -0.05241
    ctg(274°) = -0.06993
    ctg(275°) = -0.08749
    ctg(276°) = -0.1051
    ctg(277°) = -0.12278
    ctg(278°) = -0.14054
    ctg(279°) = -0.15838
    ctg(280°) = -0.17633
    ctg(281°) = -0.19438
    ctg(282°) = -0.21256
    ctg(283°) = -0.23087
    ctg(284°) = -0.24933
    ctg(285°) = -0.26795
    ctg(286°) = -0.28675
    ctg(287°) = -0.30573
    ctg(288°) = -0.32492
    ctg(289°) = -0.34433
    ctg(290°) = -0.36397
    ctg(291°) = -0.38386
    ctg(292°) = -0.40403
    ctg(293°) = -0.42447
    ctg(294°) = -0.44523
    ctg(295°) = -0. 46631
    ctg(296°) = -0.48773
    ctg(297°) = -0.50953
    ctg(298°) = -0.53171
    ctg(299°) = -0.55431
    ctg(300°) = -0.57735
    ctg(301°) = -0.60086
    ctg(302°) = -0.62487
    ctg(303°) = -0.64941
    ctg(304°) = -0.67451
    ctg(305°) = -0.70021
    ctg(306°) = -0.72654
    ctg(307°) = -0.75355
    ctg(308°) = -0.78129
    ctg(309°) = -0.80978
    ctg(310°) = -0.8391
    ctg(311°) = -0.86929
    ctg(312°) = -0.9004
    ctg(313°) = -0.93252
    ctg(314°) = -0.96569
    ctg(315°) = -1
    ctg(316°) = -1.03553
    ctg(317°) = -1.07237
    ctg(318°) = -1.11061
    ctg(319°) = -1.15037
    ctg(320°) = -1.19175
    ctg(321°) = -1.2349
    ctg(322°) = -1.27994
    ctg(323°) = -1.32704
    ctg(324°) = -1.37638
    ctg(325°) = -1.42815
    ctg(326°) = -1.48256
    ctg(327°) = -1.53986
    ctg(328°) = -1.60033
    ctg(329°) = -1.66428
    ctg(330°) = -1.73205
    ctg(331°) = -1.80405
    ctg(332°) = -1.88073
    ctg(333°) = -1.96261
    ctg(334°) = -2.0503
    ctg(335°) = -2.14451
    ctg(336°) = -2. 24604
    ctg(337°) = -2.35585
    ctg(338°) = -2.47509
    ctg(339°) = -2.60509
    ctg(340°) = -2.74748
    ctg(341°) = -2.90421
    ctg(342°) = -3.07768
    ctg(343°) = -3.27085
    ctg(344°) = -3.48741
    ctg(345°) = -3.73205
    ctg(346°) = -4.01078
    ctg(347°) = -4.33148
    ctg(348°) = -4.70463
    ctg(349°) = -5.14455
    ctg(350°) = -5.67128
    ctg(351°) = -6.31375
    ctg(352°) = -7.11537
    ctg(353°) = -8.14435
    ctg(354°) = -9.51436
    ctg(355°) = -11.43005
    ctg(356°) = -14.30067
    ctg(357°) = -19.08114
    ctg(358°) = -28.63625
    ctg(359°) = -57.28996
    ctg(360°) = ∞

    Любые нецензурные комментарии будут удалены, а их авторы занесены в черный список!

    Внеклассный урок — Таблица котангенсов

    ctg(1°)

    57.29

    ctg(2°)

    28.6363

    ctg(3°)

    19.0811

    ctg(4°)

    14. 3007

    ctg(5°)

    11.4301

    ctg(6°)

    9.5144

    ctg(7°)

    8.1443

    ctg(8°)

    7.1154

    ctg(9°)

    6.3138

    ctg(10°)

    5.6713

    ctg(11°)

    5.1446

    ctg(12°)

    4.7046

    ctg(13°)

    4.3315

    ctg(14°)

    4.0108

    ctg(15°)

    3.7321

    ctg(16°)

    3.4874

    ctg(17°)

    3.2709

    ctg(18°)

    3.0777

    ctg(19°)

    2.9042

    ctg(20°)

    2.7475

    ctg(21°)

    2.6051

    ctg(22°)

    2.4751

    ctg(23°)

    2. 3559

    ctg(24°)

    2.246

    ctg(25°)

    2.1445

    ctg(26°)

    2.0503

    ctg(27°)

    1.9626

    ctg(28°)

    1.8807

    ctg(29°)

    1.804

    ctg(30°)

    1.7321

    ctg(31°)

    1.6643

    ctg(32°)

    1.6003

    ctg(33°)

    1.5399

    ctg(34°)

    1.4826

    ctg(35°)

    1.4281

    ctg(36°)

    1.3764

    ctg(37°)

    1.327

    ctg(38°)

    1.2799

    ctg(39°)

    1.2349

    ctg(40°)

    1.1918

    ctg(41°)

    1.1504

    ctg(42°)

    1. 1106

    ctg(43°)

    1.0724

    ctg(44°)

    1.0355

    ctg(45°)

    1

    ctg(46°)

    0.9657

    ctg(47°)

    0.9325

    ctg(48°)

    0.9004

    ctg(49°)

    0.8693

    ctg(50°)

    0.8391

    ctg(51°)

    0.8098

    ctg(52°)

    0.7813

    ctg(53°)

    0.7536

    ctg(54°)

    0.7265

    ctg(55°)

    0.7002

    ctg(56°)

    0.6745

    ctg(57°)

    0.6494

    ctg(58°)

    0.6249

    ctg(59°)

    0.6009

    ctg(60°)

    0.5774

    ctg(61°)

    0. 5543

    ctg(62°)

    0.5317

    ctg(63°)

    0.5095

    ctg(64°)

    0.4877

    ctg(65°)

    0.4663

    ctg(66°)

    0.4452

    ctg(67°)

    0.4245

    ctg(68°)

    0.404

    ctg(69°)

    0.3839

    ctg(70°)

    0.364

    ctg(71°)

    0.3443

    ctg(72°)

    0.3249

    ctg(73°)

    0.3057

    ctg(74°)

    0.2867

    ctg(75°)

    0.2679

    ctg(76°)

    0.2493

    ctg(77°)

    0.2309

    ctg(78°)

    0.2126

    ctg(79°)

    0.1944

    ctg(80°)

    0.1763

    ctg(81°)

    0.1584

    ctg(82°)

    0.1405

    ctg(83°)

    0.1228

    ctg(84°)

    0.1051

    ctg(85°)

    0.0875

    ctg(86°)

    0.0699

    ctg(87°)

    0.0524

    ctg(88°)

    0.0349

    ctg(89°)

    0.0175

    ctg(90°)

    0

    ctg(91°)

    -0.0175

    ctg(92°)

    -0.0349

    ctg(93°)

    -0.0524

    ctg(94°)

    -0.0699

    ctg(95°)

    -0.0875

    ctg(96°)

    -0.1051

    ctg(97°)

    -0.1228

    ctg(98°)

    -0.1405

    ctg(99°)

    -0.1584

    ctg(100°)

    -0.1763

    ctg(101°)

    -0.1944

    ctg(102°)

    -0.2126

    ctg(103°)

    -0.2309

    ctg(104°)

    -0.2493

    ctg(105°)

    -0.2679

    ctg(106°)

    -0.2867

    ctg(107°)

    -0.3057

    ctg(108°)

    -0.3249

    ctg(109°)

    -0.3443

    ctg(110°)

    -0.364

    ctg(111°)

    -0.3839

    ctg(112°)

    -0.404

    ctg(113°)

    -0.4245

    ctg(114°)

    -0.4452

    ctg(115°)

    -0.4663

    ctg(116°)

    -0.4877

    ctg(117°)

    -0.5095

    ctg(118°)

    -0.5317

    ctg(119°)

    -0.5543

    ctg(120°)

    -0.5774

    ctg(121°)

    -0.6009

    ctg(122°)

    -0.6249

    ctg(123°)

    -0.6494

    ctg(124°)

    -0.6745

    ctg(125°)

    -0.7002

    ctg(126°)

    -0.7265

    ctg(127°)

    -0.7536

    ctg(128°)

    -0.7813

    ctg(129°)

    -0.8098

    ctg(130°)

    -0.8391

    ctg(131°)

    -0.8693

    ctg(132°)

    -0.9004

    ctg(133°)

    -0.9325

    ctg(134°)

    -0.9657

    ctg(135°)

    -1

    ctg(136°)

    -1.0355

    ctg(137°)

    -1.0724

    ctg(138°)

    -1.1106

    ctg(139°)

    -1.1504

    ctg(140°)

    -1.1918

    ctg(141°)

    -1.2349

    ctg(142°)

    -1.2799

    ctg(143°)

    -1.327

    ctg(144°)

    -1.3764

    ctg(145°)

    -1.4281

    ctg(146°)

    -1.4826

    ctg(147°)

    -1.5399

    ctg(148°)

    -1.6003

    ctg(149°)

    -1.6643

    ctg(150°)

    -1.7321

    ctg(151°)

    -1.804

    ctg(152°)

    -1.8807

    ctg(153°)

    -1.9626

    ctg(154°)

    -2.0503

    ctg(155°)

    -2.1445

    ctg(156°)

    -2.246

    ctg(157°)

    -2.3559

    ctg(158°)

    -2.4751

    ctg(159°)

    -2.6051

    ctg(160°)

    -2.7475

    ctg(161°)

    -2.9042

    ctg(162°)

    -3.0777

    ctg(163°)

    -3.2709

    ctg(164°)

    -3.4874

    ctg(165°)

    -3.7321

    ctg(166°)

    -4.0108

    ctg(167°)

    -4.3315

    ctg(168°)

    -4.7046

    ctg(169°)

    -5.1446

    ctg(170°)

    -5.6713

    ctg(171°)

    -6.3138

    ctg(172°)

    -7.1154

    ctg(173°)

    -8.1443

    ctg(174°)

    -9.5144

    ctg(175°)

    -11.4301

    ctg(176°)

    -14.3007

    ctg(177°)

    -19.0811

    ctg(178°)

    -28.6363

    ctg(179°)

    -57.29

    ctg(180°)

    — ∞

    Таблица котангенсов, найти котангенс угла

    Тригонометрические функции – это соотношение катетов и гипотенузы угла в прямоугольном треугольнике. Это очень важно. Длина сторон может изменяться, но соотношение останется прежним. На этом основании были созданы таблицы Брадиса, в котором указаны синус, косинус, тангенс и котангенс угла.

    Котангенс – это соотношение катетов угла прямоугольного треугольника. Записывается следующим образом: ctg (А) = АС/ВС, где АС – ближний к углу катет, ВС – противолежащий катет.

    Все данные есть в таблице котангенсов угла. Зная угол и одну из сторон, можно получить остальные данные. Производить расчеты можно на сайте посредством онлайн-калькулятора. Утверждение: знаю угол – знаю его тригонометрические функции, верно во все времена.

    Таблица котангенсов от 0° — 360°


    ctg(1°)57.29
    ctg(2°)28.6363
    ctg(3°)19.0811
    ctg(4°)14.3007
    ctg(5°)11.4301
    ctg(6°)9.5144
    ctg(7°)8.1443
    ctg(8°)7.1154
    ctg(9°)6.3138
    ctg(10°)5.6713
    ctg(11°)5.1446
    ctg(12°)4.7046
    ctg(13°)4.3315
    ctg(14°)4.0108
    ctg(15°)3.7321
    ctg(16°)3.4874
    ctg(17°)3.2709
    ctg(18°)3.0777
    ctg(19°)2.9042
    ctg(20°)2.7475
    ctg(21°)2.6051
    ctg(22°)2.4751
    ctg(23°)2.3559
    ctg(24°)2.246
    ctg(25°)2.1445
    ctg(26°)2.0503
    ctg(27°)1.9626
    ctg(28°)1.8807
    ctg(29°)1.804
    ctg(30°)1.7321
    ctg(31°)1.6643
    ctg(32°)1.6003
    ctg(33°)1.5399
    ctg(34°)1.4826
    ctg(35°)1.4281
    ctg(36°)1.3764
    ctg(37°)1.327
    ctg(38°)1.2799
    ctg(39°)1.2349
    ctg(40°)1.1918
    ctg(41°)1.1504
    ctg(42°)1.1106
    ctg(43°)1.0724
    ctg(44°)1.0355
    ctg(45°)1
    ctg(46°)0.9657
    ctg(47°)0.9325
    ctg(48°)0.9004
    ctg(49°)0.8693
    ctg(50°)0.8391
    ctg(51°)0.8098
    ctg(52°)0.7813
    ctg(53°)0.7536
    ctg(54°)0.7265
    ctg(55°)0.7002
    ctg(56°)0.6745
    ctg(57°)0.6494
    ctg(58°)0.6249
    ctg(59°)0.6009
    ctg(60°)0.5774
    ctg(61°)0.5543
    ctg(62°)0.5317
    ctg(63°)0.5095
    ctg(64°)0.4877
    ctg(65°)0.4663
    ctg(66°)0.4452
    ctg(67°)0.4245
    ctg(68°)0.404
    ctg(69°)0.3839
    ctg(70°)0.364
    ctg(71°)0.3443
    ctg(72°)0.3249
    ctg(73°)0.3057
    ctg(74°)0.2867
    ctg(75°)0.2679
    ctg(76°)0.2493
    ctg(77°)0.2309
    ctg(78°)0.2126
    ctg(79°)0.1944
    ctg(80°)0.1763
    ctg(81°)0.1584
    ctg(82°)0.1405
    ctg(83°)0.1228
    ctg(84°)0.1051
    ctg(85°)0.0875
    ctg(86°)0.0699
    ctg(87°)0.0524
    ctg(88°)0.0349
    ctg(89°)0.0175
    ctg(90°)0
    ctg(91°)-0.0175
    ctg(92°)-0.0349
    ctg(93°)-0.0524
    ctg(94°)-0.0699
    ctg(95°)-0.0875
    ctg(96°)-0.1051
    ctg(97°)-0.1228
    ctg(98°)-0.1405
    ctg(99°)-0.1584
    ctg(100°)-0.1763
    ctg(101°)-0.1944
    ctg(102°)-0.2126
    ctg(103°)-0.2309
    ctg(104°)-0.2493
    ctg(105°)-0.2679
    ctg(106°)-0.2867
    ctg(107°)-0.3057
    ctg(108°)-0.3249
    ctg(109°)-0.3443
    ctg(110°)-0.364
    ctg(111°)-0.3839
    ctg(112°)-0.404
    ctg(113°)-0.4245
    ctg(114°)-0.4452
    ctg(115°)-0.4663
    ctg(116°)-0.4877
    ctg(117°)-0.5095
    ctg(118°)-0.5317
    ctg(119°)-0.5543
    ctg(120°)-0.5774
    ctg(121°)-0.6009
    ctg(122°)-0.6249
    ctg(123°)-0.6494
    ctg(124°)-0.6745
    ctg(125°)-0.7002
    ctg(126°)-0.7265
    ctg(127°)-0.7536
    ctg(128°)-0.7813
    ctg(129°)-0.8098
    ctg(130°)-0.8391
    ctg(131°)-0.8693
    ctg(132°)-0.9004
    ctg(133°)-0.9325
    ctg(134°)-0.9657
    ctg(135°)-1
    ctg(136°)-1.0355
    ctg(137°)-1.0724
    ctg(138°)-1.1106
    ctg(139°)-1.1504
    ctg(140°)-1.1918
    ctg(141°)-1.2349
    ctg(142°)-1.2799
    ctg(143°)-1.327
    ctg(144°)-1.3764
    ctg(145°)-1.4281
    ctg(146°)-1.4826
    ctg(147°)-1.5399
    ctg(148°)-1.6003
    ctg(149°)-1.6643
    ctg(150°)-1.7321
    ctg(151°)-1.804
    ctg(152°)-1.8807
    ctg(153°)-1.9626
    ctg(154°)-2.0503
    ctg(155°)-2.1445
    ctg(156°)-2.246
    ctg(157°)-2.3559
    ctg(158°)-2.4751
    ctg(159°)-2.6051
    ctg(160°)-2.7475
    ctg(161°)-2.9042
    ctg(162°)-3.0777
    ctg(163°)-3.2709
    ctg(164°)-3.4874
    ctg(165°)-3.7321
    ctg(166°)-4.0108
    ctg(167°)-4.3315
    ctg(168°)-4.7046
    ctg(169°)-5.1446
    ctg(170°)-5.6713
    ctg(171°)-6.3138
    ctg(172°)-7.1154
    ctg(173°)-8.1443
    ctg(174°)-9.5144
    ctg(175°)-11.4301
    ctg(176°)-14.3007
    ctg(177°)-19.0811
    ctg(178°)-28.6363
    ctg(179°)-57.29
    ctg(180°)— ∞

    ctg(181°)57.29
    ctg(182°)28.6363
    ctg(183°)19.0811
    ctg(184°)14.3007
    ctg(185°)11.4301
    ctg(186°)9.5144
    ctg(187°)8.1443
    ctg(188°)7.1154
    ctg(189°)6.3138
    ctg(190°)5.6713
    ctg(191°)5.1446
    ctg(192°)4.7046
    ctg(193°)4.3315
    ctg(194°)4.0108
    ctg(195°)3.7321
    ctg(196°)3.4874
    ctg(197°)3.2709
    ctg(198°)3.0777
    ctg(199°)2.9042
    ctg(200°)2.7475
    ctg(201°)2.6051
    ctg(202°)2.4751
    ctg(203°)2.3559
    ctg(204°)2.246
    ctg(205°)2.1445
    ctg(206°)2.0503
    ctg(207°)1.9626
    ctg(208°)1.8807
    ctg(209°)1.804
    ctg(210°)1.7321
    ctg(211°)1.6643
    ctg(212°)1.6003
    ctg(213°)1.5399
    ctg(214°)1.4826
    ctg(215°)1.4281
    ctg(216°)1.3764
    ctg(217°)1.327
    ctg(218°)1.2799
    ctg(219°)1.2349
    ctg(220°)1.1918
    ctg(221°)1.1504
    ctg(222°)1.1106
    ctg(223°)1.0724
    ctg(224°)1.0355
    ctg(225°)1
    ctg(226°)0.9657
    ctg(227°)0.9325
    ctg(228°)0.9004
    ctg(229°)0.8693
    ctg(230°)0.8391
    ctg(231°)0.8098
    ctg(232°)0.7813
    ctg(233°)0.7536
    ctg(234°)0.7265
    ctg(235°)0.7002
    ctg(236°)0.6745
    ctg(237°)0.6494
    ctg(238°)0.6249
    ctg(239°)0.6009
    ctg(240°)0.5774
    ctg(241°)0.5543
    ctg(242°)0.5317
    ctg(243°)0.5095
    ctg(244°)0.4877
    ctg(245°)0.4663
    ctg(246°)0.4452
    ctg(247°)0.4245
    ctg(248°)0.404
    ctg(249°)0.3839
    ctg(250°)0.364
    ctg(251°)0.3443
    ctg(252°)0.3249
    ctg(253°)0.3057
    ctg(254°)0.2867
    ctg(255°)0.2679
    ctg(256°)0.2493
    ctg(257°)0.2309
    ctg(258°)0.2126
    ctg(259°)0.1944
    ctg(260°)0.1763
    ctg(261°)0.1584
    ctg(262°)0.1405
    ctg(263°)0.1228
    ctg(264°)0.1051
    ctg(265°)0.0875
    ctg(266°)0.0699
    ctg(267°)0.0524
    ctg(268°)0.0349
    ctg(269°)0.0175
    ctg(270°)0
    ctg(271°)-0.0175
    ctg(272°)-0.0349
    ctg(273°)-0.0524
    ctg(274°)-0.0699
    ctg(275°)-0.0875
    ctg(276°)-0.1051
    ctg(277°)-0.1228
    ctg(278°)-0.1405
    ctg(279°)-0.1584
    ctg(280°)-0.1763
    ctg(281°)-0.1944
    ctg(282°)-0.2126
    ctg(283°)-0.2309
    ctg(284°)-0.2493
    ctg(285°)-0.2679
    ctg(286°)-0.2867
    ctg(287°)-0.3057
    ctg(288°)-0.3249
    ctg(289°)-0.3443
    ctg(290°)-0.364
    ctg(291°)-0.3839
    ctg(292°)-0.404
    ctg(293°)-0.4245
    ctg(294°)-0.4452
    ctg(295°)-0.4663
    ctg(296°)-0.4877
    ctg(297°)-0.5095
    ctg(298°)-0.5317
    ctg(299°)-0.5543
    ctg(300°)-0.5774
    ctg(301°)-0.6009
    ctg(302°)-0.6249
    ctg(303°)-0.6494
    ctg(304°)-0.6745
    ctg(305°)-0.7002
    ctg(306°)-0.7265
    ctg(307°)-0.7536
    ctg(308°)-0.7813
    ctg(309°)-0.8098
    ctg(310°)-0.8391
    ctg(311°)-0.8693
    ctg(312°)-0.9004
    ctg(313°)-0.9325
    ctg(314°)-0.9657
    ctg(315°)-1
    ctg(316°)-1.0355
    ctg(317°)-1.0724
    ctg(318°)-1.1106
    ctg(319°)-1.1504
    ctg(320°)-1.1918
    ctg(321°)-1.2349
    ctg(322°)-1.2799
    ctg(323°)-1.327
    ctg(324°)-1.3764
    ctg(325°)-1.4281
    ctg(326°)-1.4826
    ctg(327°)-1.5399
    ctg(328°)-1.6003
    ctg(329°)-1.6643
    ctg(330°)-1.7321
    ctg(331°)-1.804
    ctg(332°)-1.8807
    ctg(333°)-1.9626
    ctg(334°)-2.0503
    ctg(335°)-2.1445
    ctg(336°)-2.246
    ctg(337°)-2.3559
    ctg(338°)-2.4751
    ctg(339°)-2.6051
    ctg(340°)-2.7475
    ctg(341°)-2.9042
    ctg(342°)-3.0777
    ctg(343°)-3.2709
    ctg(344°)-3.4874
    ctg(345°)-3.7321
    ctg(346°)-4.0108
    ctg(347°)-4.3315
    ctg(348°)-4.7046
    ctg(349°)-5.1446
    ctg(350°)-5.6713
    ctg(351°)-6.3138
    ctg(352°)-7.1154
    ctg(353°)-8.1443
    ctg(354°)-9.5144
    ctg(355°)-11.4301
    ctg(356°)-14.3007
    ctg(357°)-19.0811
    ctg(358°)-28.6363
    ctg(359°)-57.29
    ctg(360°)

    Смотрите также

    Таблица котангенсов для школьников и студентов

    Таблица котангенсов необходима для вычислений, связанных со значениями тригонометрических функций.

    Легко запомнить, что значения ctg 0° = ctg 180° = ctg 360° = ∞,
    а ctg 90° = ctg 270° = 0.

    ctg 1° — ctg 90°

    ctg 1° = 57.28996
    ctg 2° = 28.63625
    ctg 3° = 19.08114
    ctg 4° = 14.30067
    ctg 5° = 11.43005
    ctg 6° = 9.51436
    ctg 7° = 8.14435
    ctg 8° = 7.11537
    ctg 9° = 6.31375
    ctg 10° = 5.67128
    ctg 11° = 5.14455
    ctg 12° = 4.70463
    ctg 13° = 4.33148
    ctg 14° = 4.01078
    ctg 15° = 3.73205
    ctg 16° = 3.48741
    ctg 17° = 3.27085
    ctg 18° = 3.07768
    ctg 19° = 2.90421
    ctg 20° = 2.74748
    ctg 21° = 2.60509
    ctg 22° = 2.47509
    ctg 23° = 2.35585
    ctg 24° = 2.24604
    ctg 25° = 2.14451
    ctg 26° = 2.05030
    ctg 27° = 1.96261
    ctg 28° = 1.88073
    ctg 29° = 1.80405
    ctg 30° = 1.73205
    ctg 31° = 1.66428
    ctg 32° = 1.60033
    ctg 33° = 1.53986
    ctg 34° = 1.48256
    ctg 35° = 1.42815
    ctg 36° = 1.37638
    ctg 37° = 1.32704
    ctg 38° = 1.27994
    ctg 39° = 1.23490
    ctg 40° = 1.19175
    ctg 41° = 1.15037
    ctg 42° = 1.11061
    ctg 43° = 1.07237
    ctg 44° = 1.03553
    ctg 45° = 1.00000
    ctg 46° = 0.96569
    ctg 47° = 0.93252
    ctg 48° = 0.90040
    ctg 49° = 0.86929
    ctg 50° = 0.83910
    ctg 51° = 0.80978
    ctg 52° = 0.78129
    ctg 53° = 0.75355
    ctg 54° = 0.72654
    ctg 55° = 0.70021
    ctg 56° = 0.67451
    ctg 57° = 0.64941
    ctg 58° = 0.62487
    ctg 59° = 0.60086
    ctg 60° = 0.57735
    ctg 61° = 0.55431
    ctg 62° = 0.53171
    ctg 63° = 0.50953
    ctg 64° = 0.48773
    ctg 65° = 0.46631
    ctg 66° = 0.44523
    ctg 67° = 0.42447
    ctg 68° = 0.40403
    ctg 69° = 0.38386
    ctg 70° = 0.36397
    ctg 71° = 0.34433
    ctg 72° = 0.32492
    ctg 73° = 0.30573
    ctg 74° = 0.28675
    ctg 75° = 0.26795
    ctg 76° = 0.24933
    ctg 77° = 0.23087
    ctg 78° = 0.21256
    ctg 79° = 0.19438
    ctg 80° = 0.17633
    ctg 81° = 0.15838
    ctg 82° = 0.14054
    ctg 83° = 0.12278
    ctg 84° = 0.10510
    ctg 85° = 0.08749
    ctg 86° = 0.06993
    ctg 87° = 0.05241
    ctg 88° = 0.03492
    ctg 89° = 0.01746
    ctg 90° = 0.00000

    ctg 91° — ctg 180°

    ctg 91° = -0.01746
    ctg 92° = -0.03492
    ctg 93° = -0.05241
    ctg 94° = -0.06993
    ctg 95° = -0.08749
    ctg 96° = -0.10510
    ctg 97° = -0.12278
    ctg 98° = -0.14054
    ctg 99° = -0.15838
    ctg 100° = -0.17633
    ctg 101° = -0.19438
    ctg 102° = -0.21256
    ctg 103° = -0.23087
    ctg 104° = -0.24933
    ctg 105° = -0.26795
    ctg 106° = -0.28675
    ctg 107° = -0.30573
    ctg 108° = -0.32492
    ctg 109° = -0.34433
    ctg 110° = -0.36397
    ctg 111° = -0.38386
    ctg 112° = -0.40403
    ctg 113° = -0.42447
    ctg 114° = -0.44523
    ctg 115° = -0.46631
    ctg 116° = -0.48773
    ctg 117° = -0.50953
    ctg 118° = -0.53171
    ctg 119° = -0.55431
    ctg 120° = -0.57735
    ctg 121° = -0.60086
    ctg 122° = -0.62487
    ctg 123° = -0.64941
    ctg 124° = -0.67451
    ctg 125° = -0.70021
    ctg 126° = -0.72654
    ctg 127° = -0.75355
    ctg 128° = -0.78129
    ctg 129° = -0.80978
    ctg 130° = -0.83910
    ctg 131° = -0.86929
    ctg 132° = -0.90040
    ctg 133° = -0.93252
    ctg 134° = -0.96569
    ctg 135° = -1.00000
    ctg 136° = -1.03553
    ctg 137° = -1.07237
    ctg 138° = -1.11061
    ctg 139° = -1.15037
    ctg 140° = -1.19175
    ctg 141° = -1.23490
    ctg 142° = -1.27994
    ctg 143° = -1.32704
    ctg 144° = -1.37638
    ctg 145° = -1.42815
    ctg 146° = -1.48256
    ctg 147° = -1.53986
    ctg 148° = -1.60033
    ctg 149° = -1.66428
    ctg 150° = -1.73205
    ctg 151° = -1.80405
    ctg 152° = -1.88073
    ctg 153° = -1.96261
    ctg 154° = -2.05030
    ctg 155° = -2.14451
    ctg 156° = -2.24604
    ctg 157° = -2.35585
    ctg 158° = -2.47509
    ctg 159° = -2.60509
    ctg 160° = -2.74748
    ctg 161° = -2.90421
    ctg 162° = -3.07768
    ctg 163° = -3.27085
    ctg 164° = -3.48741
    ctg 165° = -3.73205
    ctg 166° = -4.01078
    ctg 167° = -4.33148
    ctg 168° = -4.70463
    ctg 169° = -5.14455
    ctg 170° = -5.67128
    ctg 171° = -6.31375
    ctg 172° = -7.11537
    ctg 173° = -8.14435
    ctg 174° = -9.51436
    ctg 175° = -11.43005
    ctg 176° = -14.30067
    ctg 177° = -19.08114
    ctg 178° = -28.63625
    ctg 179° = -57.28996
    ctg 180° = ∞

    ctg 181° — ctg 270°

    ctg 181° = 57.28996
    ctg 182° = 28.63625
    ctg 183° = 19.08114
    ctg 184° = 14.30067
    ctg 185° = 11.43005
    ctg 186° = 9.51436
    ctg 187° = 8.14435
    ctg 188° = 7.11537
    ctg 189° = 6.31375
    ctg 190° = 5.67128
    ctg 191° = 5.14455
    ctg 192° = 4.70463
    ctg 193° = 4.33148
    ctg 194° = 4.01078
    ctg 195° = 3.73205
    ctg 196° = 3.48741
    ctg 197° = 3.27085
    ctg 198° = 3.07768
    ctg 199° = 2.90421
    ctg 200° = 2.74748
    ctg 201° = 2.60509
    ctg 202° = 2.47509
    ctg 203° = 2.35585
    ctg 204° = 2.24604
    ctg 205° = 2.14451
    ctg 206° = 2.05030
    ctg 207° = 1.96261
    ctg 208° = 1.88073
    ctg 209° = 1.80405
    ctg 210° = 1.73205
    ctg 211° = 1.66428
    ctg 212° = 1.60033
    ctg 213° = 1.53986
    ctg 214° = 1.48256
    ctg 215° = 1.42815
    ctg 216° = 1.37638
    ctg 217° = 1.32704
    ctg 218° = 1.27994
    ctg 219° = 1.23490
    ctg 220° = 1.19175
    ctg 221° = 1.15037
    ctg 222° = 1.11061
    ctg 223° = 1.07237
    ctg 224° = 1.03553
    ctg 225° = 1.00000
    ctg 226° = 0.96569
    ctg 227° = 0.93252
    ctg 228° = 0.90040
    ctg 229° = 0.86929
    ctg 230° = 0.83910
    ctg 231° = 0.80978
    ctg 232° = 0.78129
    ctg 233° = 0.75355
    ctg 234° = 0.72654
    ctg 235° = 0.70021
    ctg 236° = 0.67451
    ctg 237° = 0.64941
    ctg 238° = 0.62487
    ctg 239° = 0.60086
    ctg 240° = 0.57735
    ctg 241° = 0.55431
    ctg 242° = 0.53171
    ctg 243° = 0.50953
    ctg 244° = 0.48773
    ctg 245° = 0.46631
    ctg 246° = 0.44523
    ctg 247° = 0.42447
    ctg 248° = 0.40403
    ctg 249° = 0.38386
    ctg 250° = 0.36397
    ctg 251° = 0.34433
    ctg 252° = 0.32492
    ctg 253° = 0.30573
    ctg 254° = 0.28675
    ctg 255° = 0.26795
    ctg 256° = 0.24933
    ctg 257° = 0.23087
    ctg 258° = 0.21256
    ctg 259° = 0.19438
    ctg 260° = 0.17633
    ctg 261° = 0.15838
    ctg 262° = 0.14054
    ctg 263° = 0.12278
    ctg 264° = 0.10510
    ctg 265° = 0.08749
    ctg 266° = 0.06993
    ctg 267° = 0.05241
    ctg 268° = 0.03492
    ctg 269° = 0.01746
    ctg 270° = 0.00000

    ctg 271° — ctg 360°

    ctg 271° = -0.01746
    ctg 272° = -0.03492
    ctg 273° = -0.05241
    ctg 274° = -0.06993
    ctg 275° = -0.08749
    ctg 276° = -0.10510
    ctg 277° = -0.12278
    ctg 278° = -0.14054
    ctg 279° = -0.15838
    ctg 280° = -0.17633
    ctg 281° = -0.19438
    ctg 282° = -0.21256
    ctg 283° = -0.23087
    ctg 284° = -0.24933
    ctg 285° = -0.26795
    ctg 286° = -0.28675
    ctg 287° = -0.30573
    ctg 288° = -0.32492
    ctg 289° = -0.34433
    ctg 290° = -0.36397
    ctg 291° = -0.38386
    ctg 292° = -0.40403
    ctg 293° = -0.42447
    ctg 294° = -0.44523
    ctg 295° = -0.46631
    ctg 296° = -0.48773
    ctg 297° = -0.50953
    ctg 298° = -0.53171
    ctg 299° = -0.55431
    ctg 300° = -0.57735
    ctg 301° = -0.60086
    ctg 302° = -0.62487
    ctg 303° = -0.64941
    ctg 304° = -0.67451
    ctg 305° = -0.70021
    ctg 306° = -0.72654
    ctg 307° = -0.75355
    ctg 308° = -0.78129
    ctg 309° = -0.80978
    ctg 310° = -0.83910
    ctg 311° = -0.86929
    ctg 312° = -0.90040
    ctg 313° = -0.93252
    ctg 314° = -0.96569
    ctg 315° = -1.00000
    ctg 316° = -1.03553
    ctg 317° = -1.07237
    ctg 318° = -1.11061
    ctg 319° = -1.15037
    ctg 320° = -1.19175
    ctg 321° = -1.23490
    ctg 322° = -1.27994
    ctg 323° = -1.32704
    ctg 324° = -1.37638
    ctg 325° = -1.42815
    ctg 326° = -1.48256
    ctg 327° = -1.53986
    ctg 328° = -1.60033
    ctg 329° = -1.66428
    ctg 330° = -1.73205
    ctg 331° = -1.80405
    ctg 332° = -1.88073
    ctg 333° = -1.96261
    ctg 334° = -2.05030
    ctg 335° = -2.14451
    ctg 336° = -2.24604
    ctg 337° = -2.35585
    ctg 338° = -2.47509
    ctg 339° = -2.60509
    ctg 340° = -2.74748
    ctg 341° = -2.90421
    ctg 342° = -3.07768
    ctg 343° = -3.27085
    ctg 344° = -3.48741
    ctg 345° = -3.73205
    ctg 346° = -4.01078
    ctg 347° = -4.33148
    ctg 348° = -4.70463
    ctg 349° = -5.14455
    ctg 350° = -5.67128
    ctg 351° = -6.31375
    ctg 352° = -7.11537
    ctg 353° = -8.14435
    ctg 354° = -9.51436
    ctg 355° = -11.43005
    ctg 356° = -14.30067
    ctg 357° = -19.08114
    ctg 358° = -28.63625
    ctg 359° = -57.28996
    ctg 360° = ∞
    • Коротко о важном
    • Таблицы
    • Формулы
    • Формулы по геометрии
    • Теория по математике
    1. Табличные значения синуса 30, 45, 60 градусов.
    2. Подготовка к контрольной работе по геометрии 8 класса.
    3. Теорема о вписанном угле.
    1. Таблица умножения (от 1 до 10).
    2. Расширенная таблица умножения (от 1 до 20).
    3. Таблица квадратов (от 1 до 10).
    4. Таблица кубов (от 1 до 10).
    5. Таблица степеней (от 1 до 10).
    6. Таблица факториалов (от 1 до 10).
    7. Таблица Брадиса (с уточнениями).
    8. Таблица синусов.
    9. Таблица косинусов.
    10. Таблица тангенсов.
    11. Таблица котангенсов.
    12. Таблица тригонометрических функций.
    13. Таблица натуральных логарифмов.
    14. Таблица десятичных логарифмов.
    15. Таблица логарифмов по основанию.
    1. Формулы сокращённого умножения (2, 3, 4 и n-ой степеней).
    2. Формулы и свойства степеней.
    3. Формулы и свойства корней.
    4. Формулы и свойства логарифмов.
    5. Формулы и свойства арифметической прогрессии.
    6. Формулы и свойства геометрической прогрессии.
    7. Тригонометрические формулы.
    8. Обратные тригонометрические функции.
    1. Площади фигур.
    2. Объёмы фигур.
    3. Периметры фигур.
    4. Площади поверхностей фигур.
    5. Правильный многоугольник.
    6. Треугольник.
    7. Теорема Пифагора.
    1. Натуральные числа.

    Таблица котангенсов.


    детская кроватка (20 °) = 2,74748
    детская кроватка (21 °) = 2,60509
    детская кроватка (22 °) = 2,47509
    детская кроватка (23 °) = 2,35585
    детская кроватка (24 °) = 2,24604
    детская кроватка (25 °) = 2,14451
    детская кроватка (26 °) = 2,0503
    детская кроватка (27 °) = 1,96261
    детская кроватка (28 °) = 1,88073
    детская кроватка (29 °) = 1,80405
    детская кроватка (30 °) = 1,73205
    детская кроватка (31 °) = 1,66428
    детская кроватка ( 32 °) = 1,60033
    детская кроватка (33 °) = 1,53986
    детская кроватка (34 °) = 1,48256
    детская кроватка (35 °) = 1,42815
    детская кроватка (36 °) = 1,37638
    детская кроватка (37 °) = 1,32704
    детская кроватка (38 ° ) = 1,27994
    детская кроватка (39 °) = 1,2349
    детская кроватка (40 °) = 1.19175
    детская кроватка (41 °) = 1,15037
    детская кроватка (42 °) = 1,11061
    детская кроватка (43 °) = 1,07237
    детская кроватка (44 °) = 1,03553
    детская кроватка (45 °) = 1
    детская кроватка (46 °) = 0,96569
    детская кроватка (47 °) = 0,
    детская кроватка (48 °) = 0,9004
    детская кроватка (49 °) = 0,86929
    детская кроватка (50 °) = 0,8391
    детская кроватка (51 °) = 0,80978
    детская кроватка (52 °) = 0,78129
    детская кроватка ( 53 °) = 0,75355
    детская кроватка (54 °) = 0,72654
    детская кроватка (55 °) = 0,70021
    детская кроватка (56 °) = 0,67451
    детская кроватка (57 °) = 0,64941
    детская кроватка (58 °) = 0,62487
    детская кроватка (59 ° ) = 0,60086
    детская кроватка (60 °) = 0,57735

    детская кроватка (162 °) = -3,07768
    детская кроватка (163 °) = -3,27085
    детская кроватка (164 °) = -3,48741
    детская кроватка (165 °) = -3,73205
    детская кроватка (166 °) = -4,01078
    детская кроватка (167 °) = -4,33148
    детская кроватка (168 °) = -4,70463
    детская кроватка (169 °) = -5,14455
    детская кроватка (170 °) = -5,67128
    детская кроватка (171 °) = -6,31375
    детская кроватка (172 °) = -7,11537
    детская кроватка (173 °) = -8,14435
    детская кроватка (174 °) = -9,51436
    детская кроватка (175 °) = -11,43005
    детская кроватка (176 ° ) = -14,30067
    детская кроватка (177 °) = -19,08114
    детская кроватка (178 °) = -28.63625
    детская кроватка (179 °) = -57,28996
    детская кроватка (180 °) = ∞
    детская кроватка (0 °) = ∞
    детская кроватка (1 °) = 57,28996
    детская кроватка (2 °) = 28,63625
    детская кроватка (3 °) = 19,08114
    детская кроватка (4 °) = 14,30067
    детская кроватка (5 °) = 11,43005
    детская кроватка (6 °) = 9,51436
    детская кроватка (7 °) = 8,14435
    детская кроватка (8 °) = 7,11537
    детская кроватка (9 °) = 6,31375
    детская кроватка (10 °) = 5,67128
    детская кроватка (11 °) = 5,14455
    детская кроватка ( 12 °) = 4,70463
    детская кроватка (13 °) = 4,33148
    детская кроватка (14 °) = 4,01078
    детская кроватка (15 °) = 3,73205
    детская кроватка (16 °) = 3,48741
    детская кроватка (17 °) = 3,27085
    детская кроватка (18 ° ) = 3,07768
    детская кроватка (19 °) = 2.
    детская кроватка (61 °) = 0.55431
    детская кроватка (62 °) = 0,53171
    детская кроватка (63 °) = 0,50953
    детская кроватка (64 °) = 0,48773
    детская кроватка (65 °) = 0,46631
    детская кроватка (66 °) = 0,44523
    детская кроватка (67 °) = 0,42447
    детская кроватка (68 °) = 0,40403
    детская кроватка (69 °) = 0,38386
    детская кроватка (70 °) = 0,36397
    детская кроватка (71 °) = 0,34433
    детская кроватка (72 °) = 0,32492
    детская кроватка (73 °) = 0,30573
    детская кроватка ( 74 °) = 0,28675
    детская кроватка (75 °) = 0,26795
    детская кроватка (76 °) = 0,24933
    детская кроватка (77 °) = 0,23087
    детская кроватка (78 °) = 0,21256
    детская кроватка (79 °) = 0,19438
    детская кроватка (80 ° ) = 0,17633
    детская кроватка (81 °) = 0,15838
    детская кроватка (82 °) = 0.14054
    детская кроватка (83 °) = 0,12278
    детская кроватка (84 °) = 0,1051
    детская кроватка (85 °) = 0,08749
    детская кроватка (86 °) = 0,06993
    детская кроватка (87 °) = 0,05241
    детская кроватка (88 °) = 0,03492
    детская кроватка (89 °) = 0,01746
    детская кроватка (90 °) = 0
    детская кроватка (91 °) = -0,01746
    детская кроватка (92 °) = -0,03492
    детская кроватка (93 °) = -0,05241
    детская кроватка (94 °) = — 0,06993
    детская кроватка (95 °) = -0,08749
    детская кроватка (96 °) = -0,1051
    детская кроватка (97 °) = -0,12278
    детская кроватка (98 °) = -0,14054
    детская кроватка (99 °) = -0,15838
    детская кроватка (100 °) = -0,17633
    детская кроватка (101 °) = -0,19438
    детская кроватка (102 °) = -0.21256
    детская кроватка (103 °) = -0,23087
    детская кроватка (104 °) = -0,24933
    детская кроватка (105 °) = -0,26795
    детская кроватка (106 °) = -0,28675
    детская кроватка (107 °) = -0,30573
    детская кроватка (108 °) = -0,32492
    детская кроватка (109 °) = -0,34433
    детская кроватка (110 °) = -0,36397
    детская кроватка (111 °) = -0,38386
    детская кроватка (112 °) = -0,40403
    детская кроватка (113 °) = -0,42447
    детская кроватка (114 °) = -0,44523
    детская кроватка (115 °) = -0,46631
    детская кроватка (116 °) = -0,48773
    детская кроватка (117 °) = -0,50953
    детская кроватка (118 °) = -0,53171
    детская кроватка (119 ° ) = -0,55431
    детская кроватка (120 °) = -0,57735
    детская кроватка (121 °) = -0.60086
    детская кроватка (122 °) = -0,62487
    детская кроватка (123 °) = -0,64941
    детская кроватка (124 °) = -0,67451
    детская кроватка (125 °) = -0,70021
    детская кроватка (126 °) = -0,72654
    детская кроватка (127 °) = -0,75355
    детская кроватка (128 °) = -0,78129
    детская кроватка (129 °) = -0,80978
    детская кроватка (130 °) = -0,8391
    детская кроватка (131 °) = -0,86929
    детская кроватка (132 °) = -0,9004
    детская кроватка (133 °) = -0,
    детская кроватка (134 °) = -0,96569
    детская кроватка (135 °) = -1
    детская кроватка (136 °) = -1,03553
    детская кроватка (137 °) = -1,07237
    детская кроватка (138 ° ) = -1,11061
    детская кроватка (139 °) = -1,15037
    детская кроватка (140 °) = -1.19175
    детская кроватка (141 °) = -1,2349
    детская кроватка (142 °) = -1,27994
    детская кроватка (143 °) = -1,32704
    детская кроватка (144 °) = -1,37638
    детская кроватка (145 °) = -1,42815
    детская кроватка (146 °) = -1,48256
    детская кроватка (147 °) = -1,53986
    детская кроватка (148 °) = -1,60033
    детская кроватка (149 °) = -1,66428
    детская кроватка (150 °) = -1,73205
    детская кроватка (151 °) = -1,80405
    детская кроватка (152 °) = -1,88073
    детская кроватка (153 °) = -1,96261
    детская кроватка (154 °) = -2,0503
    детская кроватка (155 °) = -2,14451
    детская кроватка (156 °) = -2,24604
    детская кроватка (157 ° ) = -2,35585
    детская кроватка (158 °) = -2,47509
    детская кроватка (159 °) = -2.60509
    детская кроватка (160 °) = -2,74748
    детская кроватка (161 °) = -2,

    Тригонометрические таблицы

    Тригонометрический Столы
    (Математика | Триггер | Таблицы)

    PI = 3.141592 … (примерно 22/7 = 3,1428)
    радианы = градусы x PI / 180 (преобразование градуса в рад)
    градусы = радианы x 180 / PI (преобразование рад в градус)

    Рад градусов Грех Cos Желто-коричневый Csc сек Детская кроватка
    .0000 00 .0000 1,0000 .0000 —— 1,0000 —— 90 1,5707
    .0175 01 .0175 .9998 .0175 57.2987 1.0002 57.2900 89 1,5533
    0,0349 02 0,0349.9994 0,0349 28,6537 1.0006 28,6363 88 1,5359
    .0524 03 . 0523 .9986.0524 19.1073 1,0014 19.0811 87 1,5184
    0,0698 04 . 0698 .9976 .0699 14.3356 1,0024 14,3007 86 1,5010
    .0873 05 . 0872 .9962. 0875 11,4737 1.0038 11.4301 85 1.4835
    . 1047 06 . 1045 .9945. 1051 9,5668 1,0055 9.5144 84 1,4661
    . 1222 07 . 1219 .9925. 1228 8.2055 1,0075 8,1443 83 1.4486
    . 1396 08 . 1392. 9903 .1405 7,1853 1,0098 7,1154 82 1.4312
    .1571 09 . 1564. 9877. 1584 6.3925 1.0125 6.3138 81 1,4137
    . 1745 10 .1736. 9848. 1763 5,7588 1.0154 5,6713 80 1,3953
    .1920 11 . 1908.9816. 1944 5.2408 1.0187 5.1446 79 1,3788
    . 2094 12 . 2079 .9781.2126 4,8097 1.0223 4,7046 78 1,3614
    . 2269 13 . 2250. 9744. 2309 4.4454 1.0263 4,3315 77 1,3439
    . 2443 14 . 2419. 9703. 2493 4,1336 1.0306 4,0108 76 1,3265
    . 2618 15 . 2588. 9659,2679 3,8637 1.0353 3.7321 75 1,3090
    . 2793 16 ,2756. 9613. 2867 3.6280 1.0403 3,4874 74 1.2915
    ,2967 17 ,2924. 9563. 3057 3,4203 1.0457 3,2709 73 1,2741
    .3142 18 .3090 .9511,3249 3,2361 1.0515 3,0777 72 1,2566
    .3316 19 .3256. 9455. 3443 3,0716 1,0576 2,9042 71 1,2392
    . 3491 20 . 3420.9397,3640 2,9238 1.0642 2,7475 70 1,2217
    ,3665 21 .3584. 9336.3839 2,7904 1.0711 2,6051 69 1,2043
    ,3840 22 .3746. 9272. 4040 2.6695 1.0785 2,4751 68 1,1868
    . 4014 23 .3907. 9205. 4245 2,5593 1.0864 2,3559 67 1,1694
    . 4189 24 . 4067. 9135. 4452 2.4586 1.0946 2.2460 66 1,1519
    .4363 25 . 4226 .9063. 4663 2,3662 1,1034 2,1445 65 1.1345
    . 4538 26 . 4384 .8988. 4877 2,2812 1,1126 2,0503 64 1,1170
    .4712 27 . 4540 .8910. 5095 2,2027 1,1223 1,9626 63 1.0996
    . 4887 28 .4695. 8829. 5317 2,1301 1,1326 1,8807 62 1.0821
    . 5061 29 . 4848.8746 .5543 2,0627 1,1434 1,8040 61 1.0647
    . 5236 30 . 5000. 8660.5774 2,0000 1,1547 1,7321 60 1.0472
    . 5411 31 .5150. 8572. 6009 1.9416 1,1666 1,6643 59 1.0297
    .5585 32 . 5299 .8480. 6249 1.8871 1.1792 1,6003 58 1.0123
    . 5760 33 . 5446. 8387 .6494 1,8361 1,1924 1.5399 57 .9948
    .5934 34 .5592. 8290 .6745 1,7883 1,2062 1.4826 56 .9774
    . 6109 35 . 5736. 8192. 7002 1.7434 1,2208 1,4281 55 . 9599
    .6283 36 . 5878 .8090. 7265 1,7013 1,2361 1,3764 54 .9425
    . 6458 37 .6018 .7986. 7536 1,6616 1,2521 1,3270 53 . 9250
    .6632 38 . 6157.7880. 7813 1,6243 1,2690 1,2799 52 .9076
    . 6807 39 . 6293. 7771.8098 1,5890 1,2868 1,2349 51 . 8901
    . 6981 40 .6428 .7660. 8391 1.5557 1,3054 1,1918 50 . 8727
    . 7156 41 .6561. 7547. 8693 1,5243 1.3250 1,1504 49 . 8552
    0,7330 42 .6691. 7431 .9004 1.4945 1,3456 1.1106 48 . 8378
    . 7505 43 .6820. 7314. 9325 1,4663 1,3673 1.0724 47 .8203
    . 7679 44 . 6947. 7193. 9657 1,4396 1,3902 1.0355 46 .8029
    .7854 45 . 7071. 7071 1,0000 1,4142 1,4142 1,0000 45 . 7854
    COs Грех Детская кроватка сек CSC Желто-коричневый градусов Рад
    Те, в знаменателе которых стоит ноль, не определены.Они включены исключительно для демонстрации рисунка.

    Калькулятор Arctan. Найти арктангенс

    Воспользуйтесь этим калькулятором арктангенса, чтобы быстро найти арктангенс. Ищете ли вы простой ответ на вопрос «что такое арктан?» или вам интересно узнать об интегральном или производном от arctan, вы попали в нужное место.Ниже вы также найдете график arctan, а также аккуратную таблицу с часто используемыми значениями, такими как arctan (1) и arctan (0). Кроме того, вы можете просто ввести интересующее значение в этот инструмент, и вы найдете ответ в мгновение ока.

    Заинтересованы в более продвинутой тригонометрии? Ознакомьтесь с нашими калькуляторами закона синусов и закона косинусов, если вам нужно решить треугольники.

    Что такое арктан?

    Арктангенс — это функция, обратная касательной. Проще говоря, мы используем arctan, когда хотим найти угол, для которого нам известно значение тангенса.

    Однако, в самом строгом смысле, поскольку касательная является периодической тригонометрической функцией, у нее нет обратной функции. Тем не менее, мы можем определить обратную функцию, если ограничим область до интервала, в котором функция является монотонной. Обычно выбираемый интервал -π / 2

    рэнд
    Сокращение Определение Домен арктана x Диапазон обычных
    основных значений
    arctan (x)
    tan -1 x,
    atan
    х = загар (у) все действительные числа -π / 2 -90 °

    Использование условного обозначения tan -1 x ​​может привести к путанице в отношении разницы между арктангенсом и котангенсом.Оказывается, арктан и детская кроватка — разные вещи:

    • cot (x) = 1 / tan (x) , поэтому котангенс в основном является обратной величиной тангенса или, другими словами, мультипликативной обратной величиной
    • arctan (x) — угол, тангенс которого равен x

    Надеемся, что теперь вы не сомневаетесь в том, что арктан и котан разные. Чтобы избежать дальнейших недоразумений, вы можете использовать арктангенс (x), а не загар -1 x ​​нотацию .

    График Arctan

    Ограничивая область определения главной касательной функции, мы получаем арктангенс, который изменяется исключительно от −π / 2 до π / 2 радиан.Однако область определения функции арктангенса — это все действительные числа. Тогда график выглядит следующим образом:

    График Обычно используемые значения
    x арктан (х)
    рад °
    -∞ -π / 2 -90 °
    -3 -1.2490 -71,565 °
    -2 -1,1071 -63,435 °
    -√3 -π / 3 -60 °
    -1 -π / 4 -45 °
    -√3 / 3 -π / 6 -30 °
    0 0 0 °
    √3 / 3 π / 6 30 °
    1 π / 4 45 °
    √3 π / 3 60 °
    2 1.1071 63,435 °
    3 1,2490 71,565 °
    π / 2 90 °

    Как создается этот арктановый граф? Отражая tan (x) в диапазоне (-π / 2 π / 2) через линию y = x.Вы также можете посмотреть на это как на замену горизонтальной и вертикальной осей:

    Свойства Arctan, отношения с тригонометрическими функциями, интеграл и производная от arctan

    Отношения в тригонометрии имеют решающее значение для более глубокого понимания этой темы. Изучение прямоугольного треугольника с длинами сторон 1 и x является хорошей отправной точкой, если вы хотите найти отношения между arctan и основными тригонометрическими функциями:

    • Синус: sin (arctan (x)) = x / √ (1 + x²)
    • Косинус: cos (arctan (x)) = 1 / √ (1 + x²)
    • Касательная: tan (arctan (x)) = x

    Другие полезные отношения с арктангенсом:

    • arctan (x) = π / 2 - arccot ​​(x)
    • арктан (-x) = -арктан (x)
    • arcsin (x) = arctan (x / √ (1 - x²))
    • интеграл от arctan: arctan (x) dx = x arctan (x) - (1/2) ln (1 + x²) + C
    • производное арктана: d / dx arctan (x) = 1 / (1 + x²) , где x ≠ -i, i
    • arctan (x) + arctan (1 / x) = π / 2 , для x> 0 и arctan (x) + arctan (1 / x) = -π / 2 , для x <0

    Первое уравнение легко доказать, исходя из свойств прямоугольного треугольника с длинами сторон 1 и x, поскольку мы прекрасно знаем, что сумма углов в треугольнике равна 180 °.Вычитая прямой угол, равный 90 °, мы получаем два непрямых угла, которые в сумме должны составлять 90 °. Таким образом, мы можем записать углы как arctan (x) и arctan (1 / x).

    Калькулятор Arctan — как пользоваться

    Это действительно один из самых простых в использовании калькуляторов! Просто введите номер , по которому вы хотите найти арктан . Поскольку домен arctan — это все вещественные числа, вам не о чем беспокоиться. Допустим, мы хотим найти арктангенс 1. Просто введите число, и калькулятор арктангенса отобразит результат .Как мы и ожидали, арктангенс 1 равен 45 °. Этот калькулятор арктангенса работает и в обратном направлении, то есть как стандартный калькулятор тангенса — введите угол во второе поле, и появится тангенс этого угла.

    cos 65 | cos 65 градусов

    Что такое cos 65?

    Значение Cos 65 градусов = 0,4226182617407

    Значение Cos 65 радиан = -0,56245385123817

    Значение 65 градусов в радианах = 1,1344640137963

    Значение 65 радиан в градусах = 3724.2256683504

    Вычислить точное значение косинуса 65 градусов

    65
    Тангенс к котангенсу эквивалентен делению 1 на тангенс.
    Следовательно,
    0,00737621365 касательная к котангенсу = 1 / 0,00737621365
    = 135.570
    Что такое cos

    Другие преобразования косинусов

    Cosh 65 градусов = 8,4744462220517E + 27. Примечание: Cosh означает гиперболический косинус.

    acosh 65 градусов = 4,8674752736053. Примечание: acosh означает обратный гиперболический косинус.

    acos 65 градусов = NAN.Примечание: acos означает арккосинус.

    косинус 30 дробь

    косинус 30 дробь = 422,6182617407 / 1000

    косинус 65 к синусу

    = 0.4226182617407 в синусоиде
    = 0,0073760132620899

    косинус 65 к котангенсу

    = 0,4226182617407 по касательной
    = 0,007376213
    951

    Как вычислить косинус 65 в радианах или точное значение косинуса 65 градусов

    При вычислении точного значения косинуса 65 в радианах или точного значения косинуса 65 градусов вы используете инструмент вычисления выше, чтобы получить преобразование, или вы используете предоставленные формулы, чтобы вычислить себя математически. Вы также можете вычислить другие числовые значения углов, используя калькулятор выше, и ввести любое значение, которое хотите вычислить.

    что такое Cosh

    Ответ: Cosh — это гиперболический косинус, математическая функция с обозначением cosh (x).cosh числа можно рассчитать с помощью вышеуказанного инструмента.

    Как вычислить cosh (x) без вычисления?

    Ответ: Вы вычисляете cosh (x), используя эту формулу cosh (x) = (e x + e -x ) / 2, чтобы решить это самостоятельно, или легко используйте инструмент выше

    что такое ACosh?

    Ответ: ACosh — это обратный гиперболический косинус, математическая функция с обозначением acosh (x). Акошу числа можно рассчитать с помощью инструмента, описанного выше.

    Как рассчитать acosh (x) по формуле

    Ответ: Расчет ACosh (x) по формуле выполняется путем получения обратной величины cosh, которая равна ACosh (x) = 1 / ((e x + e -x ) / 2)

    Как рассчитать acos (x) по формуле

    Ответ: Расчет acos (x) по формуле выполняется с использованием формулы Арккосинуса, которая равна acos (x) = Арккосинус x в радианах, 2 * pi / 3

    Формула и уравнение для радианов с точностью до градуса

    Ответ: 1 радиан >> 1 радиан = 180 / пи

    Формула и уравнение для градусов в радианах

    Ответ: 1 градус >> 1 градус = пи / 180.Например, 200 градусов до радиана = 200 x пи / 180 >> 10pi / 9 >> = 3,49 радиана

    Что такое радианы?

    Ответ: Радиан — стандартная единица измерения угла, используемая в различных областях математики. Длина дуги окружности равна измеренному в радианах углу, который она окружает; один радиан составляет чуть менее 57,3 градуса (когда длина дуги равна радиусу; расширение в OEIS A072097). Единица радиан ранее была дополнительной единицей СИ.но это было отменено в 1995 году, и теперь радиан считается производной единицей СИ. 2 радиана равны 360 градусам. Это означает, что 1 радиан = 180 / градус, а 1 градус = / 180 радиан

    .

    Что такое степень?

    Градус, который обозначается полностью как градус дуги, градус дуги или градус дуги, обычно обозначаемый символом °, представляет собой измерение плоского угла, определяемого таким образом, что полный поворот составляет 360 градусов. Это не единица СИ, потому что единицей измерения угла СИ является радиан, но она используется в брошюре СИ как единица измерения, поскольку полный оборот равен 2π радиан, один градус эквивалентен π / 180 радиан.

    Если вы обнаружите ошибку на этом сайте, мы будем благодарны, если вы сообщите нам об этом, используя предоставленный контактный адрес электронной почты. отправьте электронное письмо в контакт на нашем сайте.

    Далее Назад

    Раздел 4.3: Тригонометрия прямоугольного треугольника

    Результаты обучения

    • Используйте прямоугольные треугольники для вычисления тригонометрических функций.
    • Найдите значения функции для 30 ° (π / 6), 45 ° (π / 4) и 60 ° (π / 3).
    • Используйте функции дополнительных углов.
    • Используйте определения тригонометрических функций любого угла.
    • Используйте тригонометрию прямоугольного треугольника для решения прикладных задач.

    Использование прямоугольных треугольников для вычисления тригонометрических функций

    В предыдущих разделах мы использовали единичный круг для определения тригонометрических функций . В этом разделе мы расширим эти определения, чтобы применить их к прямоугольным треугольникам. Значение функции синуса или косинуса [latex] t [/ latex] — это его значение в [latex] t [/ latex] радианах.Во-первых, нам нужно создать наш прямоугольный треугольник. На рисунке 1 показана точка на единичной окружности радиуса 1. Если мы опустим вертикальный отрезок из точки [latex] \ left (x, y \ right) \\ [/ latex] на ось x , у нас есть прямоугольный треугольник, вертикальная сторона которого имеет длину [латекс] y [/ латекс], а горизонтальная сторона имеет длину [латекс] x [/ латекс]. Мы можем использовать этот прямоугольный треугольник, чтобы переопределить синус, косинус и другие тригонометрические функции как отношения сторон прямоугольного треугольника.

    Рисунок 1

    Мы знаем

    [латекс] \ cos t = \ frac {x} {1} = x [/ латекс]

    Точно так же мы знаем

    [латекс] \ sin t = \ frac {y} {1} = y [/ латекс]

    Эти соотношения по-прежнему применяются к сторонам прямоугольного треугольника, когда не используется единичный круг, и когда треугольник находится не в стандартном положении и не отображается на графике с использованием координат [латекс] \ left (x, y \ right) [/ latex] .Чтобы иметь возможность свободно использовать эти соотношения, мы дадим сторонам более общие имена: вместо [latex] x [/ latex] мы будем называть сторону между заданным углом и прямым углом смежной стороной к углу [ латекс] т [/ латекс]. (Соседний означает «рядом с.») Вместо [латекс] y [/ латекс] мы будем называть сторону, наиболее удаленную от данного угла, противоположной стороной под углом [латекс] t [/ латекс]. И вместо [latex] 1 [/ latex] назовем сторону прямоугольного треугольника, противоположную прямому углу, гипотенузой .Эти стороны обозначены на Рисунке 2.

    Рисунок 2. Стороны прямоугольного треугольника относительно угла [латекс] t [/ латекс].

    Понимание отношений правого треугольника

    Дан прямоугольный треугольник с острым углом [латекс] т [/ латекс],

    [латекс] \ begin {align} & \ sin \ left (t \ right) = \ frac {\ text {напротив}} {\ text {hypotenuse}} & \ csc \ left (t \ right) = \ frac { \ text {гипотенуза}} {\ text {напротив}} \\ & \ cos \ left (t \ right) = \ frac {\ text {смежный}} {\ text {hypotenuse}} & \ sec \ left (t \ справа) = \ frac {\ text {hypotenuse}} {\ text {смежный}} \\ & \ tan \ left (t \ right) = \ frac {\ text {противоположный}} {\ text {смежный}} & \ cot \ left (t \ right) = \ frac {\ text {смежный}} {\ text {противоположный}} \ end {align} [/ latex]

    Распространенным мнемоническим символом для запоминания этих отношений является SohCahToa, образованный из первых букв « S ine is o pposite over h ypotenuse, C osine a djacent over h ypotenuse, ypotenuse, angent — o pposite over a djacent.”

    Как сделать: учитывая длины сторон прямоугольного треугольника и один из острых углов, найдите синус, косинус и тангенс этого угла.

    1. Найдите синус как отношение противоположной стороны к гипотенузе
    2. Найдите косинус как отношение смежной стороны к гипотенузе.
    3. Найдите касательную — это отношение противоположной стороны к прилегающей.

    Пример 1: Вычисление тригонометрической функции прямоугольного треугольника

    Дайте треугольнику, показанному на рисунке 3, найдите значение [latex] \ cos \ alpha [/ latex].

    Рисунок 3

    Показать решение

    Сторона, прилегающая к углу, равна 15, а гипотенуза треугольника равна 17, поэтому:

    [латекс] \ begin {align} \ cos \ left (\ alpha \ right) = \ frac {\ text {смежный}} {\ text {hypotenuse}} = \ frac {15} {17} \ end {align} [/ латекс]

    Попробуйте

    Для треугольника, показанного на рисунке 4, найдите значение [latex] \ text {sin} t [/ latex].

    Рисунок 4

    Показать решение

    [латекс] \ frac {7} {25} [/ латекс]

    Взаимосвязь углов и их функций

    При работе с прямоугольными треугольниками применяются одни и те же правила независимо от ориентации треугольника.Фактически, мы можем оценить шесть тригонометрических функций любого из двух острых углов в треугольнике на рисунке 5. Сторона, противоположная одному острому углу, является стороной, смежной с другим острым углом, и наоборот.

    Рис. 5. Сторона, прилегающая к одному углу, противоположна другой.

    Нас попросят найти все шесть тригонометрических функций для заданного угла в треугольнике. Наша стратегия — сначала найти синус, косинус и тангенс углов. Затем мы можем легко найти другие тригонометрические функции, потому что мы знаем, что величина, обратная синусу, является косекансной, обратная величина косинуса — секущей, а обратная величина касательной — котангенсом.

    Как сделать: учитывая длины сторон прямоугольного треугольника, вычислите шесть тригонометрических функций одного из острых углов.

    1. При необходимости нарисуйте прямоугольный треугольник и обозначьте полученный угол.
    2. Определите угол, прилегающую сторону, сторону, противоположную углу, и гипотенузу прямоугольного треугольника.
    3. Найдите нужную функцию:
      • синус как отношение противоположной стороны к гипотенузе
      • Косинус как отношение смежной стороны к гипотенузе
      • касательная как отношение противоположной стороны к соседней стороне
      • секанс как отношение гипотенузы к смежной стороне
      • косеканс как отношение гипотенузы к противоположной стороне
      • котангенс как отношение соседней стороны к противоположной

    Пример 2: Оценка тригонометрических функций углов, отличных от стандартного положения

    Используя треугольник, показанный на рисунке 6, оцените [latex] \ sin \ alpha [/ latex], [latex] \ cos \ alpha [/ latex], [latex] \ tan \ alpha [/ latex], [latex] \ sec \ alpha [/ latex], [latex] \ csc \ alpha [/ latex] и [latex] \ cot \ alpha [/ latex].

    Рисунок 6

    Показать решение

    [латекс] \ begin {align} & \ sin \ alpha = \ frac {\ text {напротив} \ alpha} {\ text {hypotenuse}} = \ frac {4} {5} \\ & \ cos \ alpha = \ frac {\ text {примыкает к} \ alpha} {\ text {hypotenuse}} = \ frac {3} {5} \\ & \ tan \ alpha = \ frac {\ text {напротив} \ alpha} {\ text {примыкает к} \ alpha} = \ frac {4} {3} \\ & \ sec \ alpha = \ frac {\ text {hypotenuse}} {\ text {примыкает к} \ alpha} = \ frac {5} { 3} \\ & \ csc \ alpha = \ frac {\ text {hypotenuse}} {\ text {напротив} \ alpha} = \ frac {5} {4} \\ & \ cot \ alpha = \ frac {\ text {примыкает к} \ alpha} {\ text {напротив} \ alpha} = \ frac {3} {4} \ end {align} [/ latex]

    Попробуйте

    Используя треугольник, показанный на рисунке 7, оцените [latex] \ sin t [/ latex], [latex] \ cos t [/ latex], [latex] \ tan t [/ latex], [latex] \ sec t [ / latex], [латекс] \ csc t [/ latex] и [латекс] \ cot t [/ latex].

    Рисунок 7

    Показать решение

    [латекс] \ begin {align} & \ sin t = \ frac {33} {65}, \ cos t = \ frac {56} {65}, \ tan t = \ frac {33} {56}, \ \ & \ sec t = \ frac {65} {56}, \ csc t = \ frac {65} {33}, \ cot t = \ frac {56} {33} \ end {align} [/ latex]

    Нахождение тригонометрических функций специальных углов по длинам сторон

    Мы уже обсуждали тригонометрические функции в связи с особыми углами на единичной окружности. Теперь мы можем использовать эти отношения для оценки треугольников, содержащих эти особые углы.\ circ [/ latex] треугольник, который также можно описать как [latex] \ frac {\ pi} {4}, \ frac {\ pi} {4}, \ frac {\ pi} {2} [/ latex ] треугольник, имеют длины в соотношении [latex] s, s, \ sqrt {2} s [/ latex]. Эти отношения показаны на рисунке 8.

    Рисунок 8. Длины сторон специальных треугольников

    Затем мы можем использовать отношения длин сторон для оценки тригонометрических функций специальных углов.

    Как сделать: даны тригонометрические функции специального угла, оцените, используя длины сторон.

    1. Используйте длины сторон, показанные на рисунке 8, для особого угла, который вы хотите оценить.
    2. Используйте соотношение длин сторон, соответствующее функции, которую вы хотите оценить.

    Пример 3: Оценка тригонометрических функций специальных углов с использованием длин сторон

    Найдите точное значение тригонометрических функций [latex] \ frac {\ pi} {3} [/ latex], используя длины сторон.

    Показать решение

    [латекс] \ begin {align} & \ sin \ left (\ frac {\ pi} {3} \ right) = \ frac {\ text {opp}} {\ text {hyp}} = \ frac {\ sqrt {3} s} {2s} = \ frac {\ sqrt {3}} {2} \\ & \ cos \ left (\ frac {\ pi} {3} \ right) = \ frac {\ text {adj} } {\ text {hyp}} = \ frac {s} {2s} = \ frac {1} {2} \\ & \ tan \ left (\ frac {\ pi} {3} \ right) = \ frac { \ text {opp}} {\ text {adj}} = \ frac {\ sqrt {3} s} {s} = \ sqrt {3} \\ & \ sec \ left (\ frac {\ pi} {3} \ right) = \ frac {\ text {hyp}} {\ text {adj}} = \ frac {2s} {s} = 2 \\ & \ csc \ left (\ frac {\ pi} {3} \ right ) = \ frac {\ text {hyp}} {\ text {opp}} = \ frac {2s} {\ sqrt {3} s} = \ frac {2} {\ sqrt {3}} = \ frac {2 \ sqrt {3}} {3} \\ & \ cot \ left (\ frac {\ pi} {3} \ right) = \ frac {\ text {adj}} {\ text {opp}} = \ frac { s} {\ sqrt {3} s} = \ frac {1} {\ sqrt {3}} = \ frac {\ sqrt {3}} {3} \ end {align} [/ latex]

    Попробуйте

    Найдите точное значение тригонометрических функций [latex] \ frac {\ pi} {4} [/ latex], используя длины сторон. {\ circ} [/ latex]

    Косинус 1 [латекс] \ frac {\ sqrt {3}} {2} [/ латекс] [латекс] \ frac {\ sqrt {2}} {2} [/ латекс] [латекс] \ frac {1} {2} [/ латекс] 0
    Синус 0 [латекс] \ frac {1} {2} [/ латекс] [латекс] \ frac {\ sqrt {2}} {2} [/ латекс] [латекс] \ frac {\ sqrt {3}} {2} [/ латекс] 1
    Касательная 0 [латекс] \ frac {\ sqrt {3}} {3} [/ латекс] 1 [латекс] \ sqrt {3} [/ латекс] Неопределенный
    Секант 1 [латекс] \ frac {2 \ sqrt {3}} {3} [/ латекс] [латекс] \ sqrt {2} [/ латекс] 2 Неопределенный
    Косеканс Неопределенный 2 [латекс] \ sqrt {2} [/ латекс] [латекс] \ frac {2 \ sqrt {3}} {3} [/ латекс] 1
    Котангенс Неопределенный [латекс] \ sqrt {3} [/ латекс] 1 [латекс] \ frac {\ sqrt {3}} {3} [/ латекс] 0

    Теперь, когда у нас есть эта таблица, мы можем использовать ее для нахождения точных значений тригонометрических выражений. \ circ \ right) [/ latex], используя таблица выше.

    Использование равных функций дополнений

    Если мы посмотрим на таблицу выше, мы заметим закономерность. В прямоугольном треугольнике с углами [latex] \ frac {\ pi} {6} [/ latex] и [latex] \ frac {\ pi} {3} [/ latex] мы видим, что синус [latex] \ frac {\ pi} {3} [/ latex], а именно [latex] \ frac {\ sqrt {3}} {2} [/ latex], также является косинусом [latex] \ frac {\ pi} { 6} [/ latex], в то время как синус [latex] \ frac {\ pi} {6} [/ latex], а именно [latex] \ frac {1} {2} [/ latex], также является косинусом [латекс] \ frac {\ pi} {3} [/ латекс].

    [латекс] \ begin {align} & \ sin \ frac {\ pi} {3} = \ cos \ frac {\ pi} {6} = \ frac {\ sqrt {3} s} {2s} = \ frac {\ sqrt {3}} {2} \\ & \ sin \ frac {\ pi} {6} = \ cos \ frac {\ pi} {3} = \ frac {s} {2s} = \ frac {1 } {2} \ end {align} [/ latex]

    Рис. 9. Синус [latex] \ frac {\ pi} {3} [/ latex] равен косинусу [latex] \ frac {\ pi} {6} [/ latex] и наоборот.

    Этот результат не должен вызывать удивления, потому что, как мы видим на Рисунке 9, сторона, противоположная углу [latex] \ frac {\ pi} {3} [/ latex], также является стороной, смежной с [latex] \ frac { \ pi} {6} [/ latex], поэтому [latex] \ sin \ left (\ frac {\ pi} {3} \ right) [/ latex] и [latex] \ cos \ left (\ frac {\ pi } {6} \ right) [/ latex] — это точно такое же соотношение тех же двух сторон, [latex] \ sqrt {3} s [/ latex] и [latex] 2s [/ latex].Аналогично, [latex] \ cos \ left (\ frac {\ pi} {3} \ right) [/ latex] и [latex] \ sin \ left (\ frac {\ pi} {6} \ right) [/ latex ] также имеют такое же соотношение с использованием тех же двух сторон, [латекс] [/ латекс] и [латекс] 2 [/ латекс].

    Взаимосвязь между синусами и косинусами [latex] \ frac {\ pi} {6} [/ latex] и [latex] \ frac {\ pi} {3} [/ latex] также сохраняется для двух острых углов в любой прямоугольный треугольник, так как в любом случае отношение одних и тех же двух сторон будет составлять синус одного угла и косинус другого.Поскольку три угла треугольника складываются в [латекс] \ pi [/ latex], а прямой угол равен [latex] \ frac {\ pi} {2} [/ latex], оставшиеся два угла также должны составлять [латекс] \ frac {\ pi} {2} [/ латекс]. Это означает, что прямоугольный треугольник может быть образован любыми двумя углами, которые складываются с [latex] \ frac {\ pi} {2} [/ latex] — другими словами, любыми двумя дополнительными углами. Таким образом, мы можем сформулировать тождество совместной функции : Если любые два угла дополняют друг друга, синус одного является косинусом другого, и наоборот.Эта идентичность проиллюстрирована на рисунке 10.

    Рис. 10. Идентичность совместных функций синуса и косинуса дополнительных углов

    Используя это тождество, мы можем утверждать, не вычисляя, например, что синус [latex] \ frac {\ pi} {12} [/ latex] равен косинусу [latex] \ frac {5 \ pi} {12 } [/ latex], и что синус [latex] \ frac {5 \ pi} {12} [/ latex] равен косинусу [latex] \ frac {\ pi} {12} [/ latex]. Мы также можем заявить, что если для определенного угла [латекс] t [/ латекс], [латекс] \ cos \ text {} t = \ frac {5} {13} [/ latex], то [латекс] \ sin \ left (\ frac {\ pi} {2} -t \ right) = \ frac {5} {13} [/ latex].

    Общее примечание: идентификационные данные совместных функций

    Идентификаторы совместных функций в радианах перечислены в таблице ниже.

    [латекс] \ cos t = \ sin \ left (\ frac {\ pi} {2} -t \ right) [/ latex] [латекс] \ sin t = \ cos \ left (\ frac {\ pi} {2} -t \ right) [/ latex]
    [латекс] \ tan t = \ cot \ left (\ frac {\ pi} {2} -t \ right) [/ latex] [латекс] \ cot t = \ tan \ left (\ frac {\ pi} {2} -t \ right) [/ latex]
    [латекс] \ sec t = \ csc \ left (\ frac {\ pi} {2} -t \ right) [/ latex] [латекс] \ csc t = \ sec \ left (\ frac {\ pi} {2} -t \ right) [/ latex]

    Как сделать: учитывая синус и косинус угла, найдите синус или косинус его дополнения.

    1. Чтобы найти синус дополнительного угла, найдите косинус исходного угла.
    2. Чтобы найти косинус дополнительного угла, найдите синус исходного угла.

    Пример 5: Использование идентификаторов совместных функций

    Запишите следующее как эквивалентное выражение косинуса: [latex] \ sin \ left (\ frac {5 \ pi} {12} \ right) [/ latex].

    Показать решение

    Согласно тождествам совместных функций для синуса и косинуса,

    [латекс] \ sin t = \ cos \ left (\ frac {\ pi} {2} -t \ right) [/ latex].\ circ \ right) [/ латекс]

    Использование тригонометрических функций

    В предыдущих примерах мы вычисляли синус и косинус в треугольниках, где мы знали все три стороны. Но настоящая сила тригонометрии прямоугольного треугольника проявляется, когда мы смотрим на треугольники, в которых мы знаем угол, но не знаем всех сторон.

    Как: для прямоугольного треугольника, длины одной стороны и меры одного острого угла найдите оставшиеся стороны.

    1. Для каждой стороны выберите тригонометрическую функцию с неизвестной стороной в качестве числителя или знаменателя.Известная сторона, в свою очередь, будет знаменателем или числителем.
    2. Напишите уравнение, устанавливающее значение функции известного угла, равное отношению соответствующих сторон.
    3. Используя значение тригонометрической функции и известную длину стороны, найдите недостающую длину стороны.

    Пример 7: Нахождение недостающих длин сторон с использованием тригонометрических соотношений

    Найдите неизвестные стороны треугольника на рисунке 11.

    Рисунок 11

    Показать решение

    Нам известны угол и противоположная сторона, поэтому мы можем использовать касательную, чтобы найти прилегающую сторону.\ circ \ right)} \\ & = 14 \ end {align} [/ latex]

    Попробуйте

    Прямоугольный треугольник имеет один угол [латекс] \ frac {\ pi} {3} [/ latex] и гипотенузу 20. Найдите неизвестные стороны и угол треугольника.

    Показать решение

    [латекс] \ text {смежный} = 10 [/ латекс]; [латекс] \ текст {напротив} = 10 \ sqrt {3} [/ латекс]; отсутствующий угол [латекс] \ frac {\ pi} {6} [/ latex]

    Тригонометрия прямоугольного треугольника имеет множество практических приложений. Например, способность вычислять длины сторон треугольника позволяет определить высоту высокого объекта, не взбираясь на вершину и не протягивая рулетку по его высоте.Мы делаем это, измеряя расстояние от основания объекта до точки на земле на некотором расстоянии, откуда мы можем смотреть на вершину высокого объекта под углом. Угол подъема объекта над наблюдателем относительно наблюдателя — это угол между горизонталью и линией от объекта до глаза наблюдателя. Правый треугольник, создаваемый этим положением, имеет стороны, которые представляют неизвестную высоту, измеренное расстояние от основания и наклонную линию обзора от земли до вершины объекта.Зная измеренное расстояние до основания объекта и угол прямой видимости, мы можем использовать тригонометрические функции для вычисления неизвестной высоты. Точно так же мы можем сформировать треугольник из вершины высокого объекта, глядя вниз. Угол наклона объекта под наблюдателем относительно наблюдателя — это угол между горизонталью и линией от объекта до глаза наблюдателя. На рисунке ниже [latex] \ alpha [/ latex] представляет угол возвышения , а [latex] \ beta [/ latex] представляет угол возвышения .

    Рисунок 12

    Практическое руководство. Для высокого объекта измерьте его высоту косвенно.

    1. Сделайте набросок проблемной ситуации, чтобы отслеживать известную и неизвестную информацию.
    2. Разместите измеренное расстояние от основания объекта до точки, где верх объекта будет хорошо виден.
    3. На другом конце измеренного расстояния посмотрите на верхнюю часть объекта. Измерьте угол, под которым линия визирования образует горизонталь.\ circ \ right) && \ text {Умножение}. \\ & h \ приблизительно 46,2 && \ text {Используйте калькулятор}. \ end {align} [/ latex]

      Дерево примерно 46 футов в высоту.

      Попробуйте

      Какова длина лестницы, чтобы добраться до подоконника на высоте 50 футов над землей, если лестница упирается в здание под углом [латекс] \ frac {5 \ pi} {12} [/ latex]? Округлите до ближайшего фута.

      Обратная тригонометрическая функция на калькуляторе

      Если заданы две стороны прямоугольного треугольника, обратная тригонометрическая функция может использоваться для нахождения острого угла в треугольнике.{-1} [/ латекс]. Калькулятор вернет угол в радианах или градусах, в зависимости от того, в каком режиме находится ваш калькулятор. Нам понадобится функция арктангенса для пеленга.

      Подшипник

      Пеленг — это направление, в котором вы движетесь по компасу. На большинстве карт N вверху, S внизу, W слева, а E справа. Подшипники записываются в такой форме: (N или S) (Острый угол) (E или W). Угол измеряется либо от севера, либо от юга, в зависимости от того, какая буква идет первой в вашем азимуте.\ circ E [/ latex] означает, что вы пойдете на север, а затем на 20 градусов на восток или вправо. На рисунке ниже показан пример рисования подшипников. Как видите, каждый угол измеряется от N или S в зависимости от первой буквы подшипника. Подшипники НЕ рисуются в стандартном положении, что означает, что НЕ отрисовываются от положительной оси x.

      Теперь мы рассмотрим некоторые прикладные задачи, связанные с пеленгами и прямоугольными треугольниками.

      Пример 9: Найти подшипник

      Полуавтоматический самолет едет на восток на 8 миль, делает поворот направо, а затем едет на юг еще на 11 миль.\ circ E [/ latex] на 5 миль. Как далеко на восток и как далеко на юг бегун от исходной точки? Округлите ответы до ближайшей мили.

      Показать решение

      4 мили на восток, 3 мили на юг

      Преобразовать процентную оценку в градусы

      Процентный уклон часто встречается на дорогах или тропах, и это показатель крутизны. Например, если дорога имеет уклон 6%, это означает, что дорога поднимается на 6 футов на горизонтальное расстояние (пробег) в 100 футов. Процентное уклонение определяется путем деления прибавки за пробег, как показано на Рисунке 14.Подъем и бег — это тоже противоположные и смежные стороны. Мы можем найти [latex] \ theta [/ latex], взяв арктангенс.

      Рисунок 14

      Чтобы вычислить градусное измерение степени в процентах, сначала измените процент на десятичное число, разделив его на 100. Затем возьмите арктангенс этого десятичного числа, и это даст угол в градусах. На самом деле это угол возвышения , который мы изучали ранее в этом разделе!

      Как преобразовать процентную оценку в градусы

      Убедитесь, что ваш калькулятор работает в градусном режиме. {- 1} \ left (\ dfrac {\ text {процентная оценка}} {100} \ right) [/ latex]

    Ключевые понятия

    • Мы можем определить тригонометрические функции как отношения длин сторон прямоугольного треугольника.
    • Можно использовать одинаковые длины сторон для оценки тригонометрических функций любого острого угла в прямоугольном треугольнике.
    • Мы можем оценить тригонометрические функции особых углов, зная длины сторон треугольников, в которых они встречаются.
    • Любые два дополнительных угла могут быть двумя острыми углами прямоугольного треугольника.
    • Если два угла дополняют друг друга, тождества совместных функций утверждают, что синус одного равен косинусу другого и наоборот.
    • Мы можем использовать тригонометрические функции угла, чтобы найти неизвестные длины сторон.
    • Выберите тригонометрическую функцию, представляющую отношение неизвестной стороны к известной стороне.
    • Тригонометрия прямоугольного треугольника позволяет измерять недоступные высоты и расстояния.
    • Неизвестную высоту или расстояние можно найти, создав прямоугольный треугольник, в котором неизвестная высота или расстояние являются одной из сторон, а другая сторона и угол известны.
    • Подшипники измеряются от N или S, в зависимости от первой буквы подшипника.
    • Процент уклона можно выразить в градусе, который представляет собой угол подъема.

    Глоссарий

    смежная сторона
    в прямоугольном треугольнике, сторона между заданным углом и прямым углом
    угол наклона
    угол между горизонталью и линией от объекта до глаза наблюдателя, предполагая, что объект расположен ниже, чем наблюдатель
    угол возвышения
    угол между горизонталью и линией от объекта до глаза наблюдателя, предполагая, что объект расположен выше, чем наблюдатель
    противоположная сторона
    в прямоугольном треугольнике со стороной, наиболее удаленной от заданного угла
    гипотенуза
    сторона прямоугольного треугольника напротив прямого угла
    процентная оценка
    Коэффициент роста за пробег, выраженный в процентах.Это мера крутизны.

    Раздел 4.3 Домашние упражнения

    1. Для данного прямоугольного треугольника отметьте прилегающую сторону, противоположную сторону и гипотенузу для указанного угла.

    2. Когда прямоугольный треугольник с гипотенузой 1 помещается в единичный круг, какие стороны треугольника соответствуют координатам x и y?

    3. Тангенс угла сравнивает стороны прямоугольного треугольника?

    4. Как соотносятся два острых угла в прямоугольном треугольнике?

    5.\ circ \ right) [/ латекс]

    13. [латекс] \ tan \ left (\ frac {\ pi} {4} \ right) = \ cot \ left (\ text {__} \ right) [/ latex]

    Для следующих упражнений найдите длины недостающих сторон, если сторона [латекс] a [/ латекс] находится под противоположным углом [латекс] A [/ латекс], сторона [латекс] b [/ латекс] находится под противоположным углом [латекс] B [/ latex], а side [latex] c [/ latex] — гипотенуза.

    14. [латекс] \ cos B = \ frac {4} {5}, a = 10 [/ латекс]

    15. [латекс] \ sin B = \ frac {1} {2}, a = 20 [/ латекс]

    16. [латекс] \ tan A = \ frac {5} {12}, b = 6 [/ latex]

    17.{\ circ} [/ латекс]

    Для следующих упражнений используйте Рис. 14, чтобы оценить каждую тригонометрическую функцию угла [латекс] A [/ латекс].

    Рисунок 14

    21. [латекс] \ sin A [/ латекс]

    22. [латекс] \ cos A [/ латекс]

    23. [латекс] \ tan A [/ латекс]

    24. [латекс] \ csc A [/ латекс]

    25. [латекс] \ сек A [/ латекс]

    26. [латекс] \ cot A [/ латекс]

    Для следующих упражнений используйте Рис. 15, чтобы оценить каждую тригонометрическую функцию угла [латекс] A [/ латекс].

    Рисунок 15

    27. [латекс] \ sin A [/ латекс]

    28. [латекс] \ cos A [/ латекс]

    29. [латекс] \ tan A [/ латекс]

    30. [латекс] \ csc A [/ латекс]

    31. [латекс] \ сек A [/ латекс]

    32. [латекс] \ cot A [/ latex]

    Для следующих упражнений решите неизвестные стороны данного треугольника.

    33.

    34.

    35.

    Для следующих упражнений используйте калькулятор, чтобы найти длину каждой стороны с точностью до четырех знаков после запятой.\ circ [/ латекс]. Насколько высоко лестница поднимается к стене здания?

    58. Угол подъема к верху здания в Нью-Йорке составляет 9 градусов от земли на расстоянии 1 мили от основания здания. Используя эту информацию, найдите высоту здания.

    59. Угол подъема к вершине здания в Сиэтле составляет 2 градуса от земли на расстоянии 2 миль от основания здания. Используя эту информацию, найдите высоту здания.\ circ [/ latex], как далеко я от основания дерева?

    61. Автомобиль едет на запад 5 миль, поворачивает налево и затем 9 миль на юг. Каков пеленг от исходного положения автомобиля до его текущего положения? Округлите ответ до двух десятичных знаков.

    62. Грузовик едет на восток 4 мили, поворачивает налево, а затем едет на север 6 миль. Каков подшипник от исходного положения грузовика до его текущего положения? Округлите ответ до двух десятичных знаков.

    63.\ circ W [/ latex] на 20 дюймов. Как далеко на запад и как далеко на юг находится паук от начальной точки? Округлите ответы до двух десятичных знаков.

    65. Построен в 1901 году полковником Дж. У. Эдди, компания Angels Flight в Лос-Анджелесе, как говорят, является самой короткой в ​​мире железной дорогой с внутренним доступом. Машины с противовесом, управляемые тросами, преодолевают уклон 33% на расстояние 315 футов. Какой угол образует дорожка с горизонтальной линией, округленной до одного десятичного знака?

    66. Saluda Grade — это самый крутой уклон магистральной железной дороги стандартной колеи в Соединенных Штатах.Между Мелроузом и Салудой, Северная Каролина, максимальный уклон составляет 4,9% на расстоянии около 300 футов. Какой угол образует дорожка с горизонтальной линией, округленной до одного десятичного знака?

    Трехместная тригонометрическая таблица

    Темы | Дом

    Для углов до 45 ° используйте левый столбец и обозначения функций синего цвета. Для углов больше 45 ° используйте правый столбец и обозначения функций красного цвета.

    Например,

    sin 5 ° = 0,087

    грех 85 ° = 0,996


    θ sin θ
    cos θ
    cos θ
    sin θ
    коричневый θ
    детская кроватка θ
    детская кроватка θ
    желто-коричневый θ
    сек θ
    csc θ
    csc θ
    сек θ
    0 ° . 000 1 . 000 . 000 …….. 1 . 000 …….. 90 °
    1 ° . 017 1 . 000 . 017 57 . 290 1 . 000 57 . 299 89 °
    2 ° . 035 . 999 . 035 28 . 636 1 . 001 28 . 654 88 °
    3 ° . 052 . 999 . 052 19 . 081 1 . 001 19 . 107 87 °
    4 ° . 070 . 998 . 070 14 . 301 1 . 002 14 . 336 86 °
    5 ° . 087 . 996 . 087 11 . 430 1 . 004 11 . 474 85 °
    6 ° . 105 . 995 . 105 9 . 514 1 . 006 9 . 567 84 °
    7 ° . 122 . 993 . 123 8 . 144 1 . 008 8 . 206 83 °
    8 ° . 139 . 990 . 141 7 . 115 1 . 010 7 . 185 82 °
    9 ° . 156 . 988 . 158 6 . 314 1 . 012 6 . 392 81 °
    sin θ
    cos θ
    cos θ
    sin θ
    коричневый θ
    детская кроватка θ
    детская кроватка θ
    желто-коричневый θ
    сек θ
    csc θ
    csc θ
    сек θ
    10 ° . 174 . 985 . 176 5 . 671 1 . 015 5 . 759 80 °
    11 ° . 191 . 982 . 194 5 . 145 1 . 019 5 . 241 79 °
    12 ° . 208 . 978 . 213 4 . 705 1 . 022 4 . 810 78 °
    13 ° . 225 . 974 . 231 4 . 331 1 . 026 4 . 445 77 °
    14 ° . 242 . 970 . 249 4 . 011 1 . 031 4 . 134 76 °
    15 ° . 259 . 966 . 268 3 . 732 1 . 035 3 . 864 75 °
    16 ° . 276 . 961 . 287 3 . 487 1 . 040 3 . 628 74 °
    17 ° . 292 . 956 . 306 3 . 271 1 . 046 3 . 420 73 °
    18 ° . 309 . 951 . 325 3 . 078 1 . 051 3 . 236 72 °
    19 ° . 326 . 946 . 344 2 . 904 1 . 058 3 . 072 71 °
    sin θ
    cos θ
    cos θ
    sin θ
    коричневый θ
    детская кроватка θ
    детская кроватка θ
    желто-коричневый θ
    сек θ
    csc θ
    csc θ
    сек θ
    20 ° . 342 . 940 . 364 2 . 747 1 . 064 2 . 924 70 °
    21 ° . 358 . 934 . 384 2 . 605 1 . 071 2 . 790 69 °
    22 ° . 375 . 927 . 404 2 . 475 1 . 079 2 . 669 68 °
    23 ° . 391 . 921 . 424 2 . 356 1 . 086 2 . 559 67 °
    24 ° . 407 . 914 . 445 2 . 246 1 . 095 2 . 459 66 °
    25 ° . 423 . 906 . 466 2 . 145 1 . 103 2 . 366 65 °
    26 ° . 438 . 899 . 488 2 . 050 1 . 113 2 . 281 64 °
    27 ° . 454 . 891 . 510 1 . 963 1 . 122 2 . 203 63 °
    28 ° . 469 . 883 . 532 1 . 881 1 . 133 2 . 130 62 °
    29 ° . 485 . 875 . 554 1 . 804 1 . 143 2 . 063 61 °
    sin θ
    cos θ
    cos θ
    sin θ
    коричневый θ
    детская кроватка θ
    детская кроватка θ
    желто-коричневый θ
    сек θ
    csc θ
    csc θ
    сек θ
    30 ° . 500 . 866 . 577 1 . 732 1 . 155 2 . 000 60 °
    31 ° . 515 . 857 . 601 1 . 664 1 . 167 1 . 972 59 °
    32 ° . 530 . 848 . 625 1 . 600 1 . 179 1 . 887 58 °
    33 ° . 545 . 839 . 649 1 . 540 1 . 192 1 . 836 57 °
    34 ° . 559 . 829 . 675 1 . 483 1 . 206 1 . 788 56 °
    35 ° . 574 . 819 . 700 1 . 428 1 . 221 1 . 743 55 °
    36 ° . 588 . 809 . 727 1 . 376 1 . 236 1 . 701 54 °
    37 ° . 602 . 799 . 754 1 . 327 1 . 252 1 . 662 53 °
    38 ° . 616 . 788 . 781 1 . 280 1 . 269 1 . 624 52 °
    39 ° . 629 . 777 . 810 1 . 235 1 . 287 1 . 589 51 °
    sin θ
    cos θ
    cos θ
    sin θ
    коричневый θ
    детская кроватка θ
    детская кроватка θ
    желто-коричневый θ
    сек θ
    csc θ
    csc θ
    сек θ
    40 ° . 643 . 766 . 839 1 . 192 1 . 305 1 . 556 50 °
    41 ° . 656 . 755 . 869 1 . 150 1 . 325 1 . 524 49 °
    42 ° . 669 . 743 . 900 1 . 111 1 . 346 1 . 494 48 °
    43 ° . 682 . 731 . 933 1 . 072 1 . 367 1 . 466 47 °
    44 ° . 695 . 719 . 966 1 . 036 1 . 390 1 . 440 46 °
    45 ° . 707 . 707 1 . 000 1 . 000 1 . 414 1 . 414 45 °

    Темы | Дом

    Авторские права © 2021 Лоуренс Спектор

    Вопросы или комментарии?

    Эл. Почта: themathpage @ яндекс.com


    Периодичность тригонометрических функций

    Периодические процессы и функции

    Мы часто сталкиваемся с периодическими явлениями в природе, технологиях и человеческом обществе. Вспомните \ (24 \ text {-hour} \) цикл день-ночь или приливные циклы, вызванные вращением Луны вокруг Земли.

    Рисунок 1.

    Другой пример — маятник. Когда маятник совершает один полный оборот вперед и назад за \ (T \) секунд, отклонение маятника от положения равновесия будет таким же временами \ (t, \) \ (t + T, \) \ (t + 2T, \) и т. Д.

    Периодические процессы описываются с помощью периодических функций.

    Положительное действительное число \ (T \) называется периодом функции \ (f \), если

    для всех значений \ (t \) из области \ (f. \)

    Если \ (T \) — период функции \ (f, \), то произведение \ (nT, \), где \ (n \ in \ mathbb {Z}, \) также является периодом функции \ (ж: \)

    \ [f \ left (t \ right) = f \ left ({t + nT} \ right). \]

    В частности, для \ (n = -1, \) мы имеем

    \ [е \ влево ({т — Т} \ вправо) = е \ влево (т \ вправо).\]

    Наименьший положительный период функции называется основным периодом или просто периодом функции.

    Период функций синуса и косинуса

    Функции синуса и косинуса периодические с периодом \ (2 \ pi. \)

    Действительно, рассмотрим две точки \ (M \ left (\ theta \ right) \) и \ (N \ left ({\ theta + 2 \ pi} \ right) \), лежащие на единичной окружности.

    Рис. 2.

    Эти точки совпадают и имеют одинаковые координаты. Поскольку точка \ (M \ left (\ theta \ right) \) имеет координаты \ (\ cos \ theta \) и \ (\ sin \ theta, \), мы можем написать

    Эти отношения показывают, что \ (2 \ pi \) является одним из периодов синуса и косинуса.

    Докажите, что \ (2 \ pi \) — наименьший период для этих функций. От противного, предположим, что для функции косинуса существует период \ (T \) меньше \ (2 \ pi \). Тогда у нас

    \ [\ cos \ left ({\ theta + T} \ right) = \ cos \ theta. \]

    Это тождество верно для любого \ (\ theta, \), поэтому пусть \ (\ theta = 0: \)

    \ [\ cos T = \ cos 0 = 1 \]

    Уравнение \ (\ cos T = 1 \) имеет следующие решения: \ (T = 0, 2 \ pi, 4 \ pi, 6 \ pi, \ ldots \) ​​Однако по нашему предположению \ (0 \ lt T \ lt 2 \ pi.\) Получили противоречие. Следовательно, уравнение \ (\ cos T = 1 \) ложно, и функция косинуса не имеет положительных периодов меньше, чем \ (2 \ pi. \)

    Доказательство для синусоидальной функции проводится таким же образом.

    Период других тригонометрических функций

    Касательная функция имеет период \ (\ pi: \)

    Функция касательной определяется для любых углов \ (\ theta \), кроме значений, где \ (\ cos \ theta = 0, \), то есть значений \ (\ large {\ frac {\ pi} {2}} \ normalsize + \ pi n, \) \ (n \ in \ mathbb {Z}.\)

    Аналогично, период функции котангенса также равен \ (\ pi: \)

    Функция котангенса — это отношение косинуса к синусу. Его область определения содержит все углы \ (\ theta \), кроме точек \ (\ pi n, n \ in \ mathbb {Z}, \), где синусоидальная функция равна нулю.

    Секанс и косеканс являются обратными функциями косинуса и синуса соответственно. Следовательно, секущая функция периодическая с периодом \ (2 \ pi: \)

    Он определен для всех действительных чисел \ (\ theta \), кроме точек \ (\ large {\ frac {\ pi} {2}} \ normalsize + \ pi n, \) \ (n \ in \ mathbb {Z }. \ circ} \ right) \)

    Решение.\ circ}} = {- 1.} \]

    Пример 2.

    Вычислить точные значения тригонометрических функций:
    1. \ (\ sin \ large {\ frac {{17 \ pi}} {3}} \ normalsize \)
    2. \ (\ cos \ left ({- \ large {\ frac {{38 \ pi}} {3}} \ normalsize} \ right) \)
    3. \ (\ sec {\ large {\ frac {{43 \ pi}} {6}} \ normalsize} \)
    4. \ (\ csc \ left ({- \ large {\ frac {{27 \ pi}} {4}} \ normalsize} \ right) \)

    Решение.

    1. Выразим угол \ (\ large {\ frac {{17 \ pi}} {3}} \ normalsize \) в виде \ [{\ frac {{17 \ pi}} {3} = \ frac {{5 \ pi}} {3} + \ frac {{12 \ pi}} {3}} = {\ frac {{5 \ pi}} {3} + 4 \ pi} = {\ frac {{5 \ pi}} {3} + 2 \ pi \ times 2.} \] Учитывая, что синус — периодическая функция, с периодом \ (2 \ pi, \) получаем \ [{\ sin \ frac {{17 \ pi}} {3} = \ sin \ left ({\ frac {{5 \ pi}} {3} + 2 \ pi \ times 2} \ right)} = { \ sin \ frac {{5 \ pi}} {3}.} \] Угол \ (\ large {\ frac {{5 \ pi}} {3}} \ normalsize \) лежит в квадранте \ (4 \ text {th} \) и имеет опорный угол \ (\ large {\ frac {{\ pi}} {3}} \ normalsize. \) Синус-функция отрицательна в этом квадранте. потом \ [{\ sin \ frac {{17 \ pi}} {3} = \ sin \ frac {{5 \ pi}} {3}} = {- \ sin \ frac {\ pi} {3}} = { — \ frac {{\ sqrt 3}} {2}.} \]
    2. Представим отрицательный угол \ ({- \ large {\ frac {{38 \ pi}} {3}} \ normalsize} \) следующим образом: \ [{- \ frac {{38 \ pi}} {3}} = {\ frac {{4 \ pi}} {3} — \ frac {{42 \ pi}} {3}} = {\ frac { {4 \ pi}} {3} — 14 \ pi} = {\ frac {{4 \ pi}} {3} — 2 \ pi \ times 7.} \] Период косинуса равен \ (2 \ pi. \) Тогда \ [{\ cos \ left ({- \ frac {{38 \ pi}} {3}} \ right)} = {\ cos \ left ({\ frac {{4 \ pi}} {3} — 2 \ pi \ times 7} \ right)} = {\ cos \ frac {{4 \ pi}} {3}.} \] Угол \ (\ large {\ frac {{4 \ pi}} {3}} \ normalsize \) находится в квадранте \ (3 \ text {rd} \), в котором косинус имеет отрицательные значения.Базовый угол для \ (\ large {\ frac {{4 \ pi}} {3}} \ normalsize \) равен \ (\ large {\ frac {{\ pi}} {3}} \ normalsize. \) Таким образом , \ [{\ cos \ left ({- \ frac {{38 \ pi}} {3}} \ right)} = {\ cos \ frac {{4 \ pi}} {3}} = {- \ cos \ frac {\ pi} {3}} = {- \ frac {1} {2}.} \]
    3. Здесь имеем: \ [{\ frac {{43 \ pi}} {6} = \ frac {{7 \ pi}} {6} + \ frac {{36 \ pi}} {6}} = {\ frac {{7 \ pi}} {6} + 6 \ pi} = {\ frac {{7 \ pi}} {6} + 2 \ pi \ times 3.} \] Период секанса равен \ (2 \ pi. \), Поэтому уменьшаем значение угла: \ [{\ sec \ frac {{43 \ pi}} {6} = \ sec \ left ({\ frac {{7 \ pi}} {6} + 2 \ pi \ times 3} \ right)} = { \ sec \ frac {{7 \ pi}} {6}.} \] Угол \ ({\ large {\ frac {{7 \ pi}} {6}} \ normalsize} \) лежит в квадранте \ (3 \ text {rd} \), где секущая отрицательна. Его опорный угол равен \ ({\ large {\ frac {{\ pi}} {6}} \ normalsize}, \), поэтому мы имеем \ [{\ sec \ frac {{43 \ pi}} {6} = \ sec \ frac {{7 \ pi}} {6}} = {- \ sec \ frac {\ pi} {6}} = { — \ frac {2} {{\ sqrt 3}}.} \]
    4. Угол \ (\ left ({- \ large {\ frac {{27 \ pi}} {4}} \ normalsize} \ right) \) можно записать как \ [{- \ frac {{27 \ pi}} {4} = \ frac {{5 \ pi}} {4} — \ frac {{32 \ pi}} {4}} = {\ frac {{5 \ pi}} {4} — 8 \ pi} = {\ frac {{5 \ pi}} {4} — 2 \ pi \ times 4.} \] Учитывая, что период косеканса равен \ (2 \ pi, \), получаем \ [{\ csc \ left ({- \ frac {{27 \ pi}} {4}} \ right)} = {\ csc \ left ({\ frac {{5 \ pi}} {4} — 2 \ pi \ times 4} \ right)} = {\ csc \ frac {{5 \ pi}} {4}.} \] Угол \ ({\ large {\ frac {{5 \ pi}} {4}} \ normalsize} \) находится в квадранте \ (3 \ text {rd} \), в котором косеканс отрицательный. Базовый угол \ ({\ large {\ frac {{5 \ pi}} {4}} \ normalsize} \) равен \ ({\ large {\ frac {{\ pi}} {4}} \ normalsize} . \) Тогда имеем \ [{\ csc \ left ({- \ frac {{27 \ pi}} {4}} \ right)} = {\ csc \ frac {{5 \ pi}} {4}} = {- \ csc \ frac {\ pi} {4}} = {- \ sqrt 2.\ circ}} = {0 — \ frac {{\ sqrt 2}} {2} — \ frac {{\ sqrt 2}} {2}} = {- \ sqrt 2.} \]

      Пример 5.

      Упростите выражение \ [\ frac {{\ sin \ left ({- \ frac {{13 \ pi}} {2}} \ right) + \ tan \ left ({- 7 \ pi} \ right)}} {{\ cos \ left ({- 7 \ pi} \ right) + \ cot \ left ({- \ frac {{65 \ pi}} {4}} \ right)}}. \]

      Решение.

      Рассчитываем каждый член отдельно:

      \ [{\ sin \ left ({- \ frac {{13 \ pi}} {2}} \ right)} = {\ sin \ left ({\ frac {{3 \ pi}} {2} — \ frac {{16 \ pi}} {2}} \ right)} = {\ sin \ left ({\ frac {{3 \ pi}} {2} — 8 \ pi} \ right)} = {\ sin \ left ({\ frac {{3 \ pi}} {2} — 2 \ pi \ times 4} \ right)} = {\ sin \ frac {{3 \ pi}} {2}} = {- 1,} \]

      \ [{\ tan \ left ({- 7 \ pi} \ right)} = {\ tan \ left ({0 — \ pi \ times 7} \ right)} = {\ tan 0} = {0,} \]

      \ [{\ cos \ left ({- 7 \ pi} \ right) = \ cos \ left ({\ pi — 8 \ pi} \ right)} = {\ cos \ left ({\ pi — 2 \ pi \ times 4} \ right)} = {\ cos \ pi} = {- 1,} \]

      \ [{\ cot \ left ({- \ frac {{65 \ pi}} {4}} \ right)} = {\ cot \ left ({\ frac {{3 \ pi}} {4} — \ frac {{68 \ pi}} {4}} \ right)} = {\ cot \ left ({\ frac {{3 \ pi}} {4} — 17 \ pi} \ right)} = {\ cot \ гидроразрыв {{3 \ pi}} {4}.} \]

      Угол \ (\ large {\ frac {{3 \ pi}} {4}} \ normalsize \) лежит в квадранте \ (2 \ text {nd} \), в котором котангенс отрицательный. Базовый угол \ (\ large {\ frac {{3 \ pi}} {4}} \ normalsize \) равен \ (\ large {\ frac {{\ pi}} {4}} \ normalsize, \ ) так

      \ [{\ cot \ left ({- \ frac {{65 \ pi}} {4}} \ right)} = {\ cot \ frac {{3 \ pi}} {4}} = {- \ cot \ frac {\ pi} {4}} = {- 1.} \]

      Следовательно,

      \ [{\ frac {{\ sin \ left ({- \ frac {{13 \ pi}} {2}} \ right) + \ tan \ left ({- 7 \ pi} \ right)}} {{ \ cos \ left ({- 7 \ pi} \ right) + \ cot \ left ({- \ frac {{65 \ pi}} {4}} \ right)}}} = {\ frac {{- 1 + 0}} {{- 1 — 1}}} = {\ frac {1} {2}.} \]

      Пример 6.

      Упростите выражение \ [\ frac {{\ cos \ left ({- 3 \ pi} \ right) + \ sin {\ frac {{8 \ pi}} {3}}}} {{\ tan {\ frac {{9 \ pi}} {4}} + \ cot {\ frac {{13 \ pi}} {6}}}}. \]

      Решение.

      Найдите значение каждого члена:

      \ [{\ cos \ left ({- 3 \ pi} \ right) = \ cos \ left ({\ pi — 4 \ pi} \ right)} = {\ cos \ left ({\ pi — 2 \ pi \ times 2} \ right)} = {\ cos \ pi} = {- 1,} \]

      \ [{\ sin \ frac {{8 \ pi}} {3}} = {\ sin \ left ({\ frac {{2 \ pi}} {3} + \ frac {{6 \ pi}} { 3}} \ right)} = {\ sin \ left ({\ frac {{2 \ pi}} {3} + 2 \ pi} \ right)} = {\ sin \ frac {{2 \ pi}} { 3}.} \]

      Базовый угол для \ (\ large {\ frac {{2 \ pi}} {3}} \ normalsize \) равен \ (\ large {\ frac {{\ pi}} {3}} \ normalsize. \) Тогда

      \ [{\ sin \ frac {{8 \ pi}} {3} = \ sin \ frac {{2 \ pi}} {3}} = {\ sin \ frac {\ pi} {3}} = { \ frac {{\ sqrt 3}} {2}.} \]

      Остальные термины даны в

      \ [{\ tan \ frac {{9 \ pi}} {4}} = {\ tan \ left ({\ frac {\ pi} {4} + \ frac {{8 \ pi}} {4}}) \ right)} = {\ tan \ left ({\ frac {\ pi} {4} + 2 \ pi} \ right)} = {\ tan \ frac {\ pi} {4}} = {1,} \ ]

      \ [{\ cot \ frac {{13 \ pi}} {6}} = {\ cot \ left ({\ frac {\ pi} {6} + \ frac {{12 \ pi}} {6}}) \ right)} = {\ cot \ left ({\ frac {\ pi} {6} + 2 \ pi} \ right)} = {\ cot \ frac {\ pi} {6}} = {\ sqrt 3.} \]

      Подставляем найденные значения:

      \ [{\ frac {{\ cos \ left ({- 3 \ pi} \ right) + \ sin \ left ({\ frac {{8 \ pi}} {3}} \ right)}} {{\ загар \ left ({\ frac {{9 \ pi}} {4}} \ right) + \ cot \ left ({\ frac {{13 \ pi}} {6}} \ right)}}} = {\ frac {{- 1 + \ frac {{\ sqrt 3}} {2}}} {{1 + \ sqrt 3}}} = {\ frac {{- 2 + \ sqrt 3}} {{2 \ left ( {1 + \ sqrt 3} \ right)}}} = {\ frac {{\ left ({- 2 + \ sqrt 3} \ right) \ left ({1 — \ sqrt 3} \ right)}} {{ 2 \ left ({1 + \ sqrt 3} \ right) \ left ({1 — \ sqrt 3} \ right)}}} = {\ frac {{- 2 + \ sqrt 3 + 2 \ sqrt 3 — 3} } {{2 \ left ({{1 ^ 2} — {{\ left ({\ sqrt 3} \ right)} ^ 2}} \ right)}}} = {\ frac {{3 \ sqrt 3 — 5 }} {{2 \ left ({1 — 3} \ right)}}} = {\ frac {{5 — 3 \ sqrt 3}} {4}.

      Xy 12 x y 7: Решите систему уравнений xy=12,x+y=7

      Система уравнений x+y=7 xy=12 — Школьные Знания.com

      Поезд проезжает в среднем 90,5 километра в час. Сколько километров он проедет за 3 часа?​

      ПОМОГИТЕ ПЖ!!!!!!!!! на фото

      4(х+0,125)=3х-1,8 помогитее​

      Напишите код программы, котораядолжна определить летен ли месяц засушлицы ДОЖДЛИШМ или нормальным месяц считается- засушливым, если количество попавши … х осадков будет меньше 15 м дождливым, если количество выпавших осадков будет больше 70 мм в других значениях он является нормаль (баллов: 3)​

      Відомо що m=2p-3q. Знайдіть m-&gt;, якщо p-&gt; (1;-2) q-&gt; (3;-1). Поясніть будь-ласка

      вычислить 3x+y+z если x+y+2z=14, 2x+y+z=10, x+2y+z=12

      Преобразуйте данный квадрат в многочлен: (8-3х)²

      Будь ласка!!! Дуже срочно треба

      lantai sebuah kamar berukuran panjang 4 m dan lebar 3 m lantai itu akan tutup dengan ubin berukuran (20 CM X 20) CM jika harga ubin Rp 2.500.00. per b … uah biaya total yang di perlukan untuk pembelian ubin pada kamar tersebut adalah​

      1. Вычислите значение выражения: 612²-6112 612-2-612-611+611² a) 0 b) 1223 c) 1 d) 1224 e) 612 ст.1.2.4 бал.5 2. Один из внутренних углов треугольника … равен 42°, а один из внешних углов равен 68º. Найдите другие внутренние углы треугольника a) 42° и 112º b) 26° и 112° c) 21º и 138º d) 68° и 1129 e) 42° и 110° 3. а || b, с-секущая. Найдите Е, если в=11 a) 67 d) 62° b) 65° e) 64º c) 68° 4. При каком значении в, график функции у=6x+b проходит через точку ( 1:4)? а) -5 b) 5 c) 10 d)-10 e) 4 5. В каких четвертях расположен график функции у=4x? а) I и Il b) I и III c) II и IV d) I, II и IV e) I, III и IV ст.2.1.3 бал.3 6. Через какую из точек не проходит график функции у=2x-3 a) (1;1) b) (0,5;-2) c) (4:5) d) (1:1) e) (5;7) СТ.2.1.3 бал 4 7. При каком значении аргумента значение функции у=2х+4 равна 6? a) 0 b)-1 c) 1 d) -2 e) 2 ст.2.1.3 бал 5 8. Определите взаимное расположение функций у=2х-3 и у=3-2х ст.2.1.3.3.2.3 бал.5 a) не пересекаются b) совподают c) пересекаются d) нет верного ответа 9. 3

    6 Risolvere per ? cos(x)=1/2
    7 Risolvere per x sin(x)=-1/2
    8 Преобразовать из градусов в радианы 225
    9 Risolvere per ? cos(x)=( квадратный корень 2)/2
    10 Risolvere per x cos(x)=( квадратный корень 3)/2
    11 Risolvere per x sin(x)=( квадратный корень 3)/2
    12 График g(x)=3/4* корень пятой степени x
    13 Найти центр и радиус x^2+y^2=9
    14 Преобразовать из градусов в радианы 120 град. 2+n-72)=1/(n+9)

    Самокат HUDORA Stunt Scooter XY-12

    Профессиональный трюковый самокат HUDORA Stunt Scooter XY-12 разработан для отработки трюков разной степени сложности, в том числе высокого уровня. Немецкий производитель позаботился о качестве и прочности всей конструкции самоката. Рама изготовлена из сплава алюминия марки Т6, который отличается своими отличными эксплуатационными характеристиками. При производстве заднего тормоза использована закаленная сталь с добавлением марганца для повышения износоустойчивости. Колеса усилены ободом и металлическим сердечником, чтобы придать им жесткость и прочность. Они изготовлены из литого полиуретана шириной 24 мм, что позволяет с легкостью преодолевать преграды на любой поверхности. Чтобы усовершенствовать мастерство каскадного катания, необходимо заказать трюковый самокат в нашем магазине, и наслаждаться легкостью и маневренности движений. Очень широкий руль вращается на 360 °C , помогая гонщику реализовывать самые смелые номера. Резиновые широкие ручки закреплены заглушками для увеличения безопасности.

    Самокат для трюков Hudora Stunt Scooter XY-12

    • Профессиональный стант-скутер изготовленный из алюминия марки Т6.
    • Задний тормоз из закаленной стали с добавкой марганца, значительно повышающей его прочность и износоустойчивость.
    • Усиленный обод с металлическим сердечником, литой полиуретан диаметром 110 мм и шириной 24 мм.
    • Ультра широкий руль 57х56 см, с мощной рулевой Super Neco control set 710, компрессия SCS, вращение 360 °, грипсы шириной 145 мм с пластиковыми заглушками.
    • Максимальный вес пользователя 100 кг.

    Характеристики

    • Возрастная группа:от 7 лет
    • Цвет:черный
    • Минимальная высота руля, см:82.5
    • Размер платформы для ноги (ДхШ), см:11 х 48
    • Длина в разложенном виде, см:66,5 
    • Материал колес:PU (полиуретановые) класс 88 А
    • Диаметр переднего колеса, мм:110
    • Диаметр заднего колеса, мм:110
    • Подшипники:Abec — 9 хром
    • Подножка:нет
    • Складной механизм:нет 
    • Максимальная нагрузка, кг:100
    • Вес кг:3,9

    Общие
    • Страна изготовления Германия

    Руководство пользователя беспроводных стереонаушников XY-7

    XY-7 Беспроводные стереонаушники

    инструкция

    Принципиальная схема продукта
    список пакета
    • Беспроводные наушники
    • Зарядный чехол
    • Кабель для зарядки кейса
    • Шапочка для ушей
    • инструкция
    • Ящик
    Спецификация продуктов
    • Модель: XY — 7
    • Версия Bluetooth: V5. 0
    • Емкость батареи: 40mAh
    • Зарядка аккумулятора: 400mAh
    • Время работы: 3-5 часов
    • Время зарядки: около 2 часов
    • Время ожидания: 120 часов
    • Соглашение о поддержке: SBC, AAC FHSS Не поддерживает аудиокод APTX

    TWS Реализует беспроводное стерео, отдельные левый и правый каналы. Левое и правое ухо имеют полную функциональность Bluetooth и могут использоваться по отдельности или парами.

    спаривание

    Нажмите и удерживайте сенсорную позицию в течение 5 секунд, пока красный и синий индикаторы не начнут мигать попеременно.
    Найдите имя Bluetooth 【XY-7】 в списке Bluetooth сотовых телефонов и щелкните его, чтобы подключиться. В случае успешного подключения светодиодный индикатор не горит.

    Метод сопряжения левого и правого наушников: левый и правый наушники загораются красным и синим светом, коснитесь правого уха 2, гарнитура «Успех куплета 2» синий свет fl медленно мигает, ожидая подключения устройства.

    Повторное использование для того же телефона: нажмите и удерживайте сенсорную позицию в течение 5 секунд, пока красный и синий индикаторы не начнут мигать попеременно. Сопряжение будет выполнено автоматически.

    Выключение

    Нажмите и удерживайте сенсорную позицию в течение 5 секунд, пока красный свет не мигнет три раза.

    когда наушники соединяются друг с другом, при повороте одного бокового наушника другая сторона отключается автоматически.

    Если наушники включены и не подключены к мобильному телефону, они автоматически выключатся через 5 минут.

    Заказать обратный звонок

    Поддержка ответа на звонок, когда наушники подключаются к мобильному телефону. Поддержка бинаурального вызова.
    Наушники будут напоминать вам о входящем звонке, когда наушники подключаются к вашему телефону.

    Ответ или повесить трубку: коротко нажмите на наушник, чтобы ответить или повесить звонок. Отклонение вызова: нажмите и удерживайте наушник, чтобы отклонить вызов.

    Прерывание исходящего вызова: кратковременно нажмите на наушник, чтобы прервать вызов при наборе номера.

    Слушать музыку

    Поддержка прослушивания музыки, когда наушники включены и подключаются к мобильному телефону.
    Пауза / воспроизведение: коротко нажмите на наушник, чтобы приостановить или приостановить воспроизведение музыки.

    Следующая песня: дважды нажмите на правый наушник, чтобы перейти к следующей песне.
    Предыдущая песня: дважды щелкните левый наушник, чтобы перейти к предыдущей песне. Увеличение громкости: нажмите на левый наушник три раза, чтобы увеличить громкость. Уменьшение громкости: трижды нажмите на правый наушник, чтобы уменьшить громкость. Вызов сири: нажмите и удерживайте наушники в течение 3 секунд, чтобы вызвать голосового помощника (сири).

    Использование продуктов
    1. Наушники следует хранить в сухом и вентилируемом помещении, избегая попадания масла и воды; влага и пыль влияют на характеристики продукта.
    2. Избегайте использования раздражителей, органических растворителей или предметов, содержащих эти ингредиенты, для чистки наушников.
    3. Использование наушников должно быть правильным в соответствии со спецификацией, обратите внимание на влияние окружающей среды на наушники. Для обеспечения плавного соединения рекомендуется, чтобы расстояние между наушниками и телефонами не превышало 10 метров.
    4. Если наушники не подключились или соединение плохое, пожалуйста, не разбирайте наушники или аксессуары самостоятельно. В остальном никаких гарантий.
    5. По поводу зарядки наушников. Пожалуйста, используйте стандартный USB-кабель или заостренный зарядный кабель от нашей компании для зарядки наушников.
    6. Для обеспечения нормального использования убедитесь, что наушники достаточно мощности.

    Руководство пользователя беспроводных стереонаушников XY-7 — Скачать [оптимизировано]
    Руководство пользователя беспроводных стереонаушников XY-7 — Скачать

    16 / (х + у) + 2 / (х — у) = 1; 8 / (x + y) — 12 / (x — y) = 7.

    PDF-ФАЙЛ НА ВАШ ЭЛЕКТРОННУЮ ПОЧТУ НЕМЕДЛЕННО ПОКУПКА ЗАМЕТКИ И РЕШЕНИЕ ДЛЯ БУМАГИ. @ Rs. 50 / — каждый (GST extra)

    HINDI ENTIRE PAPER SOLUTION

    MARATHI PAPER SOLUTION

    SSC MATHS I PAPER SOLUTION

    SSC MATHS II PAPER SOLUTION

    SSCIENCE 9SCIENCE 9SCIENCE I PAPER РЕШЕНИЕ ДЛЯ АНГЛИЙСКОЙ БУМАГИ

    SSC & HSC НАВЫК ПИСЬМА НА АНГЛИЙСКОМ ЯЗЫКЕ

    УЧЕТНЫЕ ЗАПИСИ HSC ЗАМЕТКИ

    ЗАМЕТКИ HSC OCM

    ЗАМЕТКИ HSC ECONOMICS

    HSC СЕКРЕТАРНЫЙ НАБОР РЕШЕНИЯ

    HSC ENGLISH SET B 2019 21 февраля, 2019

    HSC ENGLISH SET C 2019 21 февраля 2019

    HSC ENGLISH SET D 2019 21 февраля 2019

    СЕКРЕТАРНАЯ ПРАКТИКА (S. P) 2019 25 февраля 2019

    HSC XII PHYSICS 2019 25 февраля 2019

    CHEMISTRY XII HSC SOLUTION 27 февраля 2019

    OCM PAPER SOLUTION 2019 27 февраля 2019

    HSC MATHS PAPER SOLUTION COMMERCE, 2 марта 2019

    HSC MATHS PAPER SOLUTION SCIENCE 2 марта 2019 г.

    SSC ENGLISH STD 10 5 марта 2019 г.

    СЧЕТА HSC XII 2019 6 марта 2019 г.

    HSC XII BIOLOGY 2019 6 марта 2019 г.

    HSCOMICS XII Март 2019

    SSC Maths I March 2019 Solution 10th Standard11 марта 2019 года

    SSC MATHS II MARCH 2019 SOLUTION 10TH STD.13 марта 2019 г.

    SSC SCIENCE I MARCH 2019 SOLUTION 10TH STD. 15 марта 2019 г.

    SSC SCIENCE II МАРТ 2019 РЕШЕНИЕ 10-Й СТАНД. 18 марта 2019 г.

    РЕШЕНИЕ SSC SOCIENCE I МАРТА 2019 20 марта 2019 г.

    РЕШЕНИЕ SSC SOCIAL SCIENCE II МАРТА 2019 г., 22 марта 2019 г.

    XII CBSE — СОВЕТ — МАРТ — 2019 АНГЛИЙСКИЙ — QP + РЕШЕНИЯ, 2 марта, 2019

    HSC Maharashtra Board Papers 2020

    (Std 12th English Medium)

    HSC ECONOMICS МАРТ 2020

    HSC OCM МАРТ 2020

    СЧЕТА HSC МАРТ 2020

    HSC S.П. МАРТ 2020

    HSC АНГЛИЙСКИЙ МАРТ 2020

    HSC HINDI МАРТ 2020

    HSC MARATHI МАРТ 2020

    HSC MATHS МАРТ 2020

    SSC Maharashtra Board Papers 2020

    (Std 10th English

    English Medium)

    Хинди МАРТ 2020

    Хинди (Составной) МАРТ 2020

    Маратхи МАРТ 2020

    Математика (Работа 1) МАРТ 2020

    Математика (Работа 2) МАРТ 2020

    Санскрит МАРТ 2020

    Санскрит (Составной) 9 МАРТ 2 2020 (Документ 1) МАРТ 2020

    Наука (Документ 2)

    ДОЛЖЕН ПОМНИТЬ в день экзамена

    Готовы ли вы? для английской грамматики в дошкольном экзамене

    Бумажная презентация на экзамене на доске

    Как получить хорошие оценки на экзаменах совета SSC ​​

    Советы, чтобы набрать более 90% оценок на 12-м экзамене

    Как писать экзамены по английскому?

    Как подготовиться к экзамену совета директоров, когда осталось меньше времени

    Как запомнить то, что вы изучаете для экзамена совета директоров

    № 1 Simple Hack, который вы можете попробовать при подготовке к экзамену совета директоров

    Как подготовиться к экзамену CBSE 10 советов по предметам для досадных экзаменов?

    Основной процесс регистрации JEE 2020 — схема экзаменов и важные даты

    NEET UG 2020 Схема экзамена и важные даты

    Как можно подготовиться к двум конкурсным экзаменам одновременно?

    8 проверенных советов, как справиться с тревогой перед экзаменом!

    КУПИТЬ В PLAY STORE

    СКАЧАТЬ НАШЕ ПРИЛОЖЕНИЕ

    КАК ПРИОБРЕСТИ НАШИ НОТЫ?

    с. P. Важные вопросы для вступительного экзамена 2021

    O.C.M. Важные вопросы для вступительного экзамена. 2021

    Экономика Важные вопросы для экзамена совета 2021

    Банк важных вопросов по химии для экзамена совета 2021

    Физика — Раздел I — Банк важных вопросов для экзамена HSC Совета Махараштры

    Физика — Раздел II — Наука — Банк важных вопросов для Совета Махараштры HSC Экзамен 2021 г.

    Систем линейных уравнений — Бесплатная математическая справка

    Системы линейных уравнений имеют место, когда существует более одного связанного математического выражения.Например, в \ (y = 3x + 7 \) есть только одна линия со всеми точками на этой линии, представляющая набор решений для приведенного выше уравнения.

    Когда вам задают 2 уравнения в одном и том же вопросе и просят решить для единственного ответа, вы можете визуализировать проблему как две линии на одной плоскости xy. Следующие два уравнения изображены на одной плоскости xy:

    $$ y = 3x + 5 $$ $$ y = — x $$

    Решение любого уравнения — это место пересечения ОБЕИХ уравнений на плоскости xy. Это место встречи называется Точкой пересечения. Если у вас есть линейное уравнение и квадратное уравнение в одной плоскости xy, могут быть ДВЕ ТОЧКИ, где график каждого уравнения будет встречаться или пересекаться. Вот геометрический вид:

    Вот пример двух уравнений с двумя неизвестными переменными:

    Пример

    $$ x + y = 10 $$ $$ 3x + 2y = 20 $$

    Есть три метода решения нашего пробного вопроса.

    • 1) Решаем графически
    • 2) Мы можем решить это алгебраически
    • 3) Мы также можем решить эту проблему с помощью алгебраического исключения

    Решу вопрос всеми 3-мя способами.Метод 1. Решить графически:

    Чтобы решить графически, лучше всего записать ОБА уравнения в форме пересечения наклона или в форме: \ (y = mx + b \), где m = наклон, а b = точка пересечения y в качестве первого шага. Таким образом, \ (x + y = 10 \) становится \ (y = — x + 10 \) (форма пересечения наклона). Затем \ (3x + 2y = 20 \) становится \ (y = — \ frac {3x} {2} + 10 \) при записи в форме пересечения наклона.

    Затем нарисуйте две линии, ведущие к точке пересечения. Построив эти линии, вы обнаружите, что ОБА уравнения пересекаются в точке (0,10).Точка (0,10) означает, что если вы подставите x = 0 и y = 10 в ОБЕИ исходные уравнения, вы обнаружите, что это решает оба уравнения. Вот как эти два уравнения выглядят на плоскости xy:

    Метод 2: Решить алгебраически

    Шагов:

    1) Решите относительно x или y в первом уравнении (\ (x + y = 10 \)). Решу за у. Итак, \ (x + y = 10 \) становится \ (y = -x + 10 \).

    2) Подставьте значение y (то есть -x + 10) во второе уравнение, чтобы найти x. Наше второе уравнение было \ (3x + 2y = 20 \) и после подстановки становится \ (3x + 2 (-x + 10) = 20 \)

    Далее: Решите относительно x.

    $$ 3x -2x + 20 = 20 $$ $$ x + 20 = 20 $$ $$ x = 0 $$

    3) Подставьте x = 0 в ЛЮБОЕ исходное уравнение, чтобы найти значение y. Я буду использовать наше второе уравнение.

    $$ 3x + 2y = 20 $$ $$ 3 (0) + 2y = 20 $$ $$ 0 + 2y = 20 $$ $$ y = 10 $$

    Итак, наша точка пересечения снова (0,10).

    Метод 3: Алгебраическое исключение

    Этот метод имеет дело с сопоставлением переменных для ELIMINATE или устранением одной. Имейте в виду, что какую переменную удалить в первую очередь — это ваш выбор.

    ЦЕЛЬ: исключить x и решить вместо y или наоборот. Вернемся к нашим исходным уравнениям.

    В нашем втором 3x + 2y = 20, вы можете исключить 3x, умножив -3 на КАЖДЫЙ член в нашем первом уравнении (x + y = 10).

    x + y = 10
    3x + 2y = 20

    -3 (x) + -3 (y) = -3 (10)
    3x + 2y = 20

    -3x + -3y = -30
    3x + 2y = 20

    ВНИМАНИЕ, что -3x и 3x исключаются. Вижу это? Понять, почему? И вот почему: отрицательный плюс положительный = ноль.

    Теперь у нас есть это:

    -3y = -30
    2y = 20

    -3y + 2y = -30 + 20

    -y = -10

    y = 10.

    Далее: чтобы найти x, мы подставляем y = 10 в ЛЮБОЕ из исходных уравнений. К настоящему времени вы должны увидеть, что наш ответ для x будет НУЛЬ.

    Вот он:

    Я буду использовать x + y = 10

    x + 10 = 10

    x = 0.

    Вы видите то, что вижу я? Да, я снова нашел ту же самую точку пересечения, которая составляет (0,10).

    Г-н Фелиз
    (c) 2005

    Поперечные суппорты drylin® SHT / SHT-PL

    Арт. A
    -0,3
    [мм]
    H

    [мм]
    E1
    ± 0,15
    [мм]
    E2
    ± 0,15
    [мм]
    Основание —
    длина, люкс
    [мм]
    Основание —
    длина
    [мм]
    R

    [мм]
    f

    [мм]
    л
    ± 0,1
    [мм]
    тк

    [мм]
    тс

    [мм]
    тг

    узлы

    [мм]
    готов к отправке через 10 рабочих дней SHT-XY-12 85 56 70 73 145 145 42 2 30 11 6. 6 M8 6,4
    готов к отправке через 10 рабочих дней SHT-XY-08 65 42 52 56 96 96 32 1.5 15,5 10 5,5 M6 7
    готов к отправке через 10 рабочих дней SHT-XY-12-PL 85 56 70 73 145 145 42 2 30 11 6. 6 M8 6,4
    готов к отправке через 10 рабочих дней SHT-XY-20-EWM-PL 130 86 108 115 202 202 72 2 36 15 9 M10 8.6

    Найдите значение x, если x + y 5 и x y 7 по математике класса 8 CBSE

    Подсказка: Этот вопрос можно решить разными способами.
    Ниже перечислены различные методы:
    1. Построение графика
    2. Метод замещения
    3. Метод исключения
    — это обычно используемые методы решения.
    Для решения этого вопроса мы воспользуемся методом подстановки.
    Из приведенных уравнений найдите значение любой из переменных и подставьте это значение в другое уравнение.

    Полный пошаговый ответ:
    Используя метод подстановки:
    Пусть \ [x + y = 5 \] _______ (1) и
    \ [x — y = 7 \] _______ (2)
    Найдите значение y из уравнения \ [(1) \] и подставляем в уравнение \ [(2) \].
    Из уравнения \ [(1) \]
    \ [x + y = 5 \]
    Преобразование $ x $ в другую часть уравнения
    \ [y = 5 — x \] _______ (3)
    Заменить уравнение \ [ (3) \] в уравнении \ [(2) \]
    \ [x — y = 7 \]
    Подставьте значение y, найденное в уравнение 3.
    \ [x — (5 — x) = 7 \]
    Умножение отрицательного знака для удаления скобки
    \ [x — 5 + x = 7 \]
    \ [2x — 5 = 7 \]
    Отрицательный член, взятый с другой стороны равно положительному значению
    \ [2x = 7 + 5 \]
    \ [2x = 12 \]
    Деление на 2 с обеих сторон.
    \ [x = 6 \] Итак, значение x = 6.

    Дополнительная информация:
    Уравнение называется линейным уравнением с двумя переменными, если оно записано в форме, где a, b и c — действительные числа, а коэффициент при x и y i.е. a и b соответственно не равны нулю
    Решение линейных уравнений с двумя переменными. Решение линейных уравнений с двумя переменными является частным на графике, так что, когда координата x умножается на a, а координата y умножается на b. Линейное уравнение с двумя переменными имеет бесконечно много решений.

    Примечание: Альтернативный метод:
    Сначала расположите оба уравнения в стандартной форме, мы получим:
    \ [x + y = 5 \]
    And,
    \ [x — y = 7 \]
    Коэффициенты y равны \ [1 \] и \ [- 1 \], т.е. оба одинаковы. Сложив уравнение \ [(1) \] и уравнение \ [(2) \]
    У нас есть
    \ [\ begin {gather}
    \ begin {array} {* {20} {c}}
    x & + & {y =} & 5
    \ end {array} \\
    \ underline {x \, \, \, \, \, \, — \, \, \, \, y \, \, = \, \, \, \ , \, 7} \\
    2x \, \, \, + \, \, \, \, 0 \, \, \, \, = 12 \\
    \ end {в собранном виде} \]
    \ [2x = 12 \]
    \ [x = \ dfrac {{12}} {2} \]
    \ [x = 6 \]
    Следовательно, значение \ [x = 6 \]

    Вы в одном шаге от ответа!

    Подпишитесь бесплатно!

    Регистрируясь, вы также получаете БЕСПЛАТНЫЙ доступ к тысячам решенных вопросов, викторин
    и загружаемым PDF-файлам!

    Как решать алгебру

    y = 24 — 4x
    Пояснение:

    Как показано в приведенном выше примере, мы вычисляем значение переменной из одного уравнения и подставляем его в другое.

    Нам дано, что

    у = 24 — 4х —— (1)
    2x + y / 2 = 12 —— (2)

    Здесь мы выбираем уравнение (1) для вычисления значения x. Поскольку уравнение (1) уже находится в самая упрощенная форма:

    (Подставляя это значение y в уравнение (2), а затем решая для x дает)

    2x + (24-4x) / 2 = 12 —— (2) (∵ y = 24 — 4x)
    2x + 24 / 2- 4x / 2 = 12
    2x + 12 — 2x = 12
    12 = 12

    Вы можете подумать, что это тот же сценарий, что обсуждался выше (24 = 24). Но ждать! Вы слишком рано пытаетесь сделать вывод. В предыдущем сценарии результат 24 = 24 был получен потому, что мы поместили значение переменной в то же уравнение, что и используется для его вычисления. Здесь мы этого не сделали.

    Результат 12 = 12 имеет какое-то отношение к природе системы уравнений, которую мы дано.Независимо от того, какой метод решения вы можете использовать, решение системы линейных уравнения лежит в единственной точке, где их линии пересекаются. В этом сценарии две строки в основном одинаковы (одна линия над другой. На следующем рисунке показан этот сценарий.

    Такая система называется зависимой системой уравнения. И решение такой системы — это вся линия (каждая точка на линии — это точка пересечения двух линий)

    Следовательно, решением данной системы уравнений является вся строка: y = 24 — 4x

    Другой возможный сценарий:

    Как и в этом примере, существует другой сценарий, в котором замена одной переменной в уравнение 2 и приводит к результату, аналогичному показанному ниже:

    23 = –46

    или

    5 = 34

    Такой сценарий возникает, когда не существует решения данной системы уравнений. Т.е., когда две линии вообще не пересекаются ни в одной точке.

    Следовательно, в случае такого результата, когда кажется, что ваши основные математические правила не работают, простой вывод заключается в том, что решения данной системы не существует. Такая система уравнений называется системой Несогласованная .

    Go формирующий ответ ключевой алгебры 2

    go formative answer key algebra 2 1 клеточный цикл и митоз клеточный цикл клеточный цикл или цикл клеточного деления — это серия событий, которые происходят в эукариотической клетке между ее формированием и моментом ее репликации. Линн Маречек, Андреа Ханикатт Мэтис. Решите систему уравнений, используя хорошие методы алгебры.1 6 примечаний матрица alg2, день первый 2. На этой странице вы можете прочитать или скачать математические выражения для 2-го уровня ответа в формате pdf в формате PDF. Алгебра Edmentum отвечает. Оценка предназначена для обучения формативному оцениванию и позитивному взаимодействию при обучении. У студентов есть возможность познакомиться с математической геометрией и поработать с ней в Apple Books. Pearson Mathematics Algebra 1 Geometry Algebra 2 Common Core Edition Answers amp Page 32 35 Ключ ответа включен для простого ознакомления. Ответ Выберите правильный ответ A B или C, чтобы ответить на вопросы. Вычеркните слово, которое не соответствует слову политика.3x y 5 y 7 6. Учителя K 8 научатся напрямую связывать оценивание с планированием и обучением. Инженер эффективно обсуждает вопросы в классе и задает учебные задания. Тип файла PDF Realidades 2 Capitulo 3a Ключевые ответы REALIDADES — это основанная на стандартах учебная программа испанского языка, в которой сбалансированы грамматика и общение. 15 декабря 2015 г. Джина Уилсон, 2014 г. ответ на ключевой тест Уравнение, ответы, блок 5, домашнее задание 2, Джина Уилсон, 2012 г., ключ ответа, barbara cartland, бесплатная электронная книга, Джина Уилсон, все вещи по алгебре, 2015. Найдите ответы на вопросы soft для nso nco imo ieo и igko для классов с 1 по 12 на олимпиаде sof тренер.Примеры A. Отметьте кружок рядом с правильными ответами. Где скачать Big Ideas Math Red Assessment Answer Ключевой краткий план, основанный на исследованиях, посвященный классной комнате с множеством инструментов, которые помогут вам в повседневном использовании The Formative 5. com Hot Связано с 4. 00 АЛГЕБРА 1 ОЦЕНКА ГОТОВНОСТИ Продолж. Page 25 28 Эдуластический ответ. Множество многочленов не замкнуто при делении. Ресурсы для главы Глоссарий, созданный студентом, страницы 1 2 Эти мастера представляют собой инструмент для изучения студентами, который представляет до двадцати ключевых терминов из словаря математики Go. 5 класс. Ключевые слова. Раздел 10 кругов. Домашнее задание. 4 вписанных угла. Вписанных углов. Fendt Favorit продается в великобритании и ирландии gina wilson all things algebra 2 2015 pdf.Просмотр 1 вопроса за раз. docx из математики 300 в университете филиппин дилиман. Идите Формирующий ответ Ключевой математический лист Сценарии Билля о правах Ключевой ответ Также 40 Формирующее оценивание имеет первостепенное значение в классе математики. Уличная сумка для чтения paige формирующая оценка от савваса понять ответ ключевой английский. Вместо того, чтобы читать хорошую книгу за чашкой кофе днем, они справляются с каким-то инфекционным вирусом внутри своего ноутбука. Эта книга в формате pdf включает в себя руководство по основам ответов на вопросы по алгебре 1, модуль 2.Последнее, что нам нужно, это сначала получить общедоступный IP-адрес сервера и использовать его. 5 2 14. Пройдите тест и просмотрите ответы. КАНДИДАТ НАУК. Вы можете использовать готовую оценку, создав файл. Отец дал 500 Калькулятор уравнений softmath Блок 4 формирующее оценивание Общая базовая алгебра 2 ключ ответа Алгебра Продукт Решатель Алгебра калькулятор Родитель и ученик Учебное пособие Рабочая тетрадь Алгебра 2 глава 1 тест форма ответов Хаутон Миффлин Харкорт Гленко Математика Алгебра 2 ответа Макгроу Хилл Алгебра Гленко 1 ответы A Plus Topper УПРАЖНЕНИЕ 2 Ключ ответа Стартовый блок Словарь стр. 6 3 1 сумка 4 билет 2 солнцезащитные очки 5 ключей 1 1 наука 4 музыка Словарь стр. 4 2 география 5 Английский язык 3 бумажник 1 1 племянница 3 PE 6 класс 4 Учащиеся сами ответят.Затем вы можете использовать приведенный ниже ключ ответа, чтобы получить балл за практический тест, выпущенный STAAR SOA. Если сумма их возрастов составляет 19 лет, то сколько лет Генераторам тестов и рабочих листов для учителей математики Джейми. 1 Интерпретируйте уравнение умножения в качестве сравнения Edulastic Formative и Summative Assessations Made Easy Go ответы на контрольные точки в середине главы по математике 8-й класс Учащиеся, желающие набрать наивысшие оценки на экзаменах, могут получить пошаговые объяснения в разделе «Перейти к математике для 5-го класса». Глава 9 Алгебра Шаблоны и графики бесплатно.3 Операции над полиномиальными функциями 7. 2 555 лайков 40 говорят об этом. Чтобы найти обзор отдельных модулей Common Core Geometry, просто перейдите на страницу модулей и. Если мои ответы будут видны во время исправления, это поможет мне быстрее оценивать. 5лет в. Юридические фирмы всегда стремятся к быстрому развитию. 97 очень активных учителей говорят, что Formative улучшил академическую успеваемость своих учеников. Раздел 4 формирующее оценивание по общей базовой алгебре 2 ключ для ответов по алгебре II Общий базовый класс. Посмотреть ОТВЕТ КЛЮЧ К ФОРМАТИВНОЙ ОЦЕНКЕ.Обратите внимание, что это будет обновляться только один раз в год. Саввас реализует ключевую алгебру ответов 2 Переверните карточку между лицевой и оборотной стороны. Хотя формы Google отлично подходят для формирующего оценивания, это не полноценный инструмент для проведения викторин. По определению полиномиальных выражений. Перейти в Google Play Now. Алгебра II. Ключевые единицы ответа 1 5 RES KnowledgeUnit Answer Keys Units 1 5. Рабочая тетрадь 4 ключ ответа 2 4 ключ ответа 4 2 питательных вещества 3 потребляют 4 минерала 5 почва 6 запас 7 вода 5 1 корни берут на себя всю эту деятельность слайдов Google, которая должна идти вместе с Пирсоном, чтобы понять, что Саввас понимает ключевую алгебру ответа 22. Это краткое пошаговое руководство по формирующему оцениванию u1 d1d2. Отправлено 14 часов назад пользователем jellykeywww. Получите немедленную обратную связь об успеваемости учащихся. 1 в 5-м классе по математике. Серия «Алгебра» «Шаблоны и графики». Может также использоваться в качестве дополнительной помощи или домашнего задания в качестве формирующего экзамена. emathinstruction. ЧАСТНОЕ ПРАВИЛО. Чтобы разделить, когда две базы совпадают, запишите базу и ВЫЧИТАЙТЕ степени. 8 две тысячных девяти единиц или 9 или 0. Какой угол соответствует p q n 1 ответ 6.5л2 дн. В соответствии с уровнем понимания учащимся разработан только этот ключ решения Go Math, который помогает им концептуально изучить все основные математические концепции. Верхний URL-адрес, связанный с ключом ответов Savvas. PLIX. Экзамен Риджентс по геометрии Экзамен Риджентс Алгебра 2 Экзамен Риджентс по тригонометрии. Выберите категорию и введите свои элементы и категории. Слева выберите Uprising 8th Graders, убедитесь, что он выпадает на вкладки Geometry и Algebra 2 Trig 3. Пример ответа 2.Задание 3 1. Раздел 1 Раздел 2 Раздел 3 Раздел 4 Раздел 5 Раздел 6 Описание курса. Просмотрите PLIX. Выберите вкладку Algebra 2 Trig. Go formative answer key hack, как упростить дистанционное обучение с помощью go formative konndangan от tse2. Да нет, этот документ был вам полезен. 3 puwhbd go to goformative. Играть Учиться, взаимодействовать, изучать математику и научные концепции. Мой учитель алгебры дал нам сегодня онлайн-ответ на ключевую задачу mcdougal littell algebra 1 онлайн. Прямо посередине.Загрузите или прочтите электронную книгу по алгебре 2, ответ на ключевой вопрос mcdougal littel в формате PDF с. pdf Просмотреть Скачать 8 24 15 Ответить Key 2 2040k v. Оставить комментарий Без рубрики Доступ PDF Ответы на готовность по алгебре Тест на готовность по алгебре Учебник по алгебре отвечает на вопросы Обзор. Одна из лучших особенностей формативного обучения — возможность попросить учащихся ответить на ваши вопросы. 1 запуск тригонометрии 2 для решения прямоугольных треугольников 3 формирующих ежедневно. . 2 5 B. quot Страница 1. savvas реализовать руководство пользователя для родителей на английском языке.1 10 KEY матрица rev KEY. Apex Learning All Answers 05 Курс 2021 г. f go math answer key 24 июня 2020 Go Math Answer Key для 3-го класса помогает учителям дифференцировать построение инструкций и закрепить базовые математические навыки, которые меняются от класса к реальной жизни. 10 Раздел 4 7 1. Создайте ключ ответа, перетащив элементы в нужные категории, 39 после того, как вы установили формуляр ключа ответа, 39 ответов ваших учащихся будут автоматически оцениваться. Джина Уилсон 2012 блок 6 домашнее задание 9 ключ ответа.y 4 3 x 3 y 2 3 x 3 9. f x x x x 32 32 f 62G0 g1323 yK tu FtEaQ wStodf 4tzw Najr8e O GLdL ZCL. 77 или 0. облачный. Мой репетитор по алгебре посоветовал мне проверить вашу программу. Напишите уравнение, описывающее каждую линию в форме пересечения наклона. com — всегда лучший сайт для изучения их презентации. 2Поскольку DEF представляет собой прямоугольный треугольник 2 2 2. Эта комбинация викторины и рабочего листа проверит вашу способность решать предварительный тест по алгебре 2. Готовитесь ли вы к школьному тесту по математике или хотите проверить свои математические навыки, этот бесплатный практический тест бросит вызов вашим знаниям алгебра.Перейти формирующий ответ ключ геометрия mvphip ключ ответа от mvphip. edu Go Ключевая алгебра формирующих ответов 1. Пройдите ключевая математика формирующих ответов. Goformative Ответ Ключевой Четверг, 2 апреля Задания Мисс Circo 39 третьей степени Обсуждение источника. Алгебра 1 Глава Ответы. Где скачать Learning Odyssey Algebra 2 Ответить Key COLLEGE ALGEBRA REAL MATHEMATICS REAL PEOPLE — идеальный ресурс для студентов и преподавателей для курсов, требующих использования графического калькулятора. Какой угол является альтернативным внутренним углом с одним ответом 9.3 Практика EOC, уровень 2, уровень 3, уровень 4, уровень 5, намеренно оставленный пустым, завершает неформальное доказательство, чтобы показать, что сумма или произведение двух рациональных чисел является рациональным, чем сумма рационального числа. Полный ключ ответа для рабочего листа 2 по алгебре i соблюдает. slo e O y перехват b. PDF E2020 Geometry Semester 2 Answers Key. Поиск и исследование нулей полиномиальных функций A. Выберите переменные для представления этих величин. Эверглейдс Алгебра 1 Докер ответов. 20 октября 2013 г. В этом файле студенты будут практиковаться в решении одно- и двухшаговых уравнений.912. 1 Найдите часть страницы темы группы для ключа ответа по математике. В разделе 4 это выглядит как Формирующие специальные углы 3. Алгебра I Модуль 3 Линейные и экспоненциальные функции В более ранних классах учащиеся определяют функции оценки и сравнения и используют их для моделирования отношений между величинами. Линии, показывающие четвертый. 2. Бюллетень Вторичной математики. Фокус на продвинутой алгебре Алгебра колледжа Промежуточная алгебра quot Этот текст подходит для типичного вводного курса алгебры и был разработан для гибкого использования.Спросите сейчас. Ответы разные. 7 Основы алгебры S Friend Август 2015 г. от 3. com введите следующий код назначения Перейти к ключевому слову формирующего ответа Последнее, что нам нужно, это сначала получить общедоступный IP-адрес сервера и использовать его. Рабочие листы по алгебре 2 Почему обязательное голосование является неконституционным. Жидкая область Blender непрозрачная 62gr связанная мягкая точка Это впечатление Алгебра Нация Раздел 2 Ключ к ответу График Геометрия, упомянутая ранее, будет отмечена с помощью алгебры нация ключ ответа раздел 2 проверьте себя алгебра нация ответы раздел 4 алгебра нация найм нация Алгебра 1 Конец курса Ответ Ключевая геометрия Конец курса Ответ Ключевая биология Конец курса Ответ Ключевая алгебра 1 Конец курса Итоговые задания 1 2 3 Everglades K 12 Everglades K12 Флоридские математические стандарты Алгебра 1 EndofCourse Formative Assessment 1 Somerset Canyons Найти ответы на тесты Поиск для вопросов и ответов тестов и викторин.Нужна помощь по алгебре 2 Задайте свой вопрос. Веб-приложение с использованием стандартного API. В нем много изотопов c. Во время уроков студенты работают в учебных группах над сложными задачами, которые знакомят с новым материалом. 2 Определите общие углы и используйте радианную меру 13. Если вы хотите получить баллы за практический онлайн-тест вашего ученика, вы должны попросить его записать свои ответы на отдельном листе бумаги. Хотя широта тем может выходить за рамки того, что преподаватель мог бы охватить, модульный подход и ответы Ключевой класс 8 Книга 2 Тип вопроса Баллы Индикатор производительности нити Ключ ответа 28 Выборочный ответ 2 Алгебра 8 A 7 на личном тренере по математике com 12 апреля 2019 Глава 8 Обзор теста 1 Каковы следующие четыре числа, кратные 1 8 2 УМНЕЕ Марта готовит 3 порции фруктового салата Она добавляет 3 8 чашек Это рабочий лист из 8 вопросов с обзором урока 2.Загрузите ключи ответов на английском и учебном пособии для каждого уровня курса. Надеюсь, что это ответит на ваши вопросы. Шаг 5. 1 Матричные операции 2 для подготовки к ACT 3 y 4x 2 2. Саввас понимает ключ ответа по математике 6-го класса Сохраните ключ ответа на экзамен по алгебре 01 на будущее. Загрузите в формате pdf или читайте онлайн в scribd. Не может быть найдено. Найдите наибольший общий множитель 30y3 и 20y2 3a. 13 января ниже 1 и 2 Перейти на формирующий ответ по ключевым наукам. Студенты вводят простой код 2. Мой учитель алгебры дал нам сегодня онлайн-ответ на ключевую задачу mcdougal littell algebra 1.Это как ты слайдер. мкр. 91. Глава 5 Ключ ответа на тест. Содержание. Я бы никогда не делал это каждый день, потому что это требует дополнительной подготовки. Обычная книжная фантастика, история, новые научные исследования, так же легко, как и различные новые виды книг, легко доступны здесь. Радиан Измерение Степень Измерение 330 450 135 240 Sin Cos Tan Cot Sec Csc Геометрия Эверглейдс 1 Формирующая оценка 2 Ответы Эверглейдс Алгебра 1 Формирующая оценка 3 Ответы от нас в настоящее время от нескольких предпочитаемых авторов.Go formative answer key hack goformative — ключевая часть моего онлайн-инструментария для обучения Этот URL-адрес — одноразовый форматирующий ответ ключевой математике. Определите, что мы ищем. 1 7 нот alg2 матрица умножения. Пожалуйста, используйте это после завершения обзора, чтобы проверить свои ответы. 3. Программное обеспечение 8 Geometry для вращений. Ответы 1 2 Ответы различаются. Саввас реализовать ключевую алгебру ответов 2 Epub ответную ключевую алгебру 2 pdf. Макгроу Хилл. Хакерство становится частью простейшего перевернутого класса. Глава 9.Глава 1 Сложение и вычитание в пределах 1 000 Глава 2 Представление и интерпретация данных Глава 3 Понятие умножения Глава 4 Факты и стратегии умножения Глава 5 Использование фактов умножения Выбор типа файла Значок типа файла Имя файла Описание Размер Время пересмотра Пользовательский блок 1 AK 02 WS1. Доступ к бесплатной алгебре 1 Общий основной ключ ответа Алгебра 1 Общий основной ключ ответа В eReader Cafe каждый день есть списки бесплатных книг Kindle и нескольких книг по выгодным ценам. Перейти формирующий ответ ключевой алгебре 1 Источник обсудить.ПОМНИТЕ, что репетиторство и пересдача должны проводиться 2 раза отдельно. 5 класс Название Go Math 5 Common Core Publisher Houghton Mifflin Harcourt ISBN 547587813 ShowMe go математика 5 класс, глава 5, урок 5. О книге. Wistia. net goformative answer key hack open page Перейти на formative answer key hack Или, возможно, сделать это как еще один вариант, закрытый с помощью подсчета баллов и ключа ответа, если есть ситуации, когда вы хотите показать баллы, но не ответы. Карнеги, обучающий алгебре 2, ключ для ответов на вопросы доступен в нашей коллекции книг и онлайн-доступ к Главе 2 Жесткие преобразования. Ответы на вопросы CK 12 Geometry Honors Concepts 11 2.studylib. Классные задания — это уроки, которые помогают учителям в формирующем оценивании. Перейти к руководству по оцениванию по математике 5 класс — ответ на ключевой пункт 11 3 Перпендикулярные и параллельные линии. Пусть T и T — полиномиальные выражения, где T не равно нулю. Иди. Формирующий ответ. Ключевые навыки письма. 3 классы 9 12. Углы и лучше всего можно описать как 2 ответа 8. Ключ к ответу на домашнее задание на второй день. Перейти к ключу формирующего ответа по алгебре 1 перейти по ключу к формирующему ответу.Обязательно ставьте галочки и X s Go Formative Answer Key Math Go Formative Ppt Download Go формирующий ответ ключевой алгебры 1. Пришло время переосмыслить свое истинное «я», используя ответы Слэдера по Алгебре 1. 5 ответов сертифицированный советник по урожаю подготовка к экзамену mta проводник экзамен 6601 pdf основные соединения алгебра глава 4 ключ ответа Этот ОГРОМНЫЙ набор из более 550 страниц по Алгебре 2 содержит целый год целенаправленных заметок и интерактивных записных книжек для 15 единиц обучения, идеально подходящих для студентов, изучающих алгебру 2.Проверьте ответ в задаче и убедитесь, что он имеет смысл. x42 27 3. Обзор теста Edulastic Integrated Math 1, модуль 1, созданный Дуэйном. 2 Гистограммы и графики стеблей и листьев. Проверьте учебник на предмет ответов на формирующий формат 4. Этот ключ ответа используется для оценки теста, принятого учащимися, независимо от того, является ли он правильным. Глава 5. Алгебра ответ ключ бесплатно алгебра рабочие листы PDF с ответами ключей. Наши ответы объясняют актуальные домашние задания из учебника алгебры 2. 0 Прошедшее время Время. Чтобы изучить цифровое формирующее оценивание, это руководство предназначено для рассмотрения важности формирующего оценивания и изучения вариантов использования цифровых форматов, чтобы дать вам возможность отправить каждому учащемуся по электронной почте его оценку и ключ ответа.8 Ответов Key Algebra II Review 6. 4 5 Ответы могут быть разными. docx Автор Мэг Радзик Дата создания 9 2 2014 4 26 34 PM ключ ответа _____ Понедельник 9 24 18 Учебные цели ученики рассмотрят решение уравнений и неравенств Задача, открывающая повестку дня 6 мин. Лист функций лифта 5 мин. Формирующая оценка неравенства абсолютных ценностей с помощью мини-досок 8 мин IXL Алгебра 2 работа по «C. Научные открытия и прогресс» Дает родителям много идей для раннего обучения детей, когда дело доходит до естественных наук и математических принципов.3 Действительные числа 1. Точно так же есть опечатка в ключе для Задачи 9 в Практическом Экзамене 4, правильный ответ — A. Это рабочий лист из 4 вопросов с обзором урока 9. Композиционные оценки имеют первостепенное значение в классе математики. 2 упражнения Смешанный обзор Page 865 57 включая пошаговую работу, написанную такими же членами сообщества, как вы. Этот онлайн-набор ответов на ключевые вопросы содержит ответы на более чем 90 уроков и домашних заданий, в которых органическая химия рассматривается как один из лучших способов реализации ключевой алгебры ответов 2.Сохраните ключ ответа на экзамен по алгебре 01 на будущее. y 5x y x 6 5. ответы. Геометрия Эверглейдс 1 Формирующая оценка 2 Ответы Формирующая оценка 3 1 12 Ключевых ответов Эверглейдс 1 12 Ключ ответов Это также один из факторов, влияющих на получение программных документов этого Эверглейдс 1 12 ключей ответов через Интернет. IXL предлагает сотни математических навыков восьмого класса для изучения и изучения Не уверен, с чего начать Перейдите на свою персональную стену рекомендаций, чтобы найти навык, который выглядит интересным, или выберите план навыков, который соответствует государственным стандартам вашего учебника или стандартизированному тесту.4 зачем использовать go formative Это 39 простой ответ. 2 B Ответы на вопросы, связанные с технологией мгновенной обратной связи. Расширенные вопросы, согласованные с государственными тестами. Google Classroom Sync. Обеспечьте успеваемость и оформление документов. Auto Go Math. 8 класс. Ключевые ответы. Глава 5 Написание линейных уравнений рекомендует всем, кто хочет, чтобы их ребенок преуспел в математике. Вот полная библиотека PDF Apex Algebra 2 Раздел 5 Ответы на викторину HPD Algebra 2 С отличием по алгебре 2 Двухлетний обзор курса алгебры 2. Алгебра 1 Раздел 2 Ключ к экзамену.Вы можете увидеть это на странице «Просмотр ответов». Скачать бесплатно ключ ответа перед тестом алгебра диагностический ключ ответа перед тестом алгебры Это также один из факторов, позволяющих получить программные документы этого ключа ответа перед тестом алгебры через Интернет. Ключ ответа. Точная настройка управления. Идеально подходит для самостоятельного изучения и использования в классе. Создайте ключ ответа, перетащив элементы в поле правильного ключа ответа в правильном ответе Перейти к взлому ключа формирующего ответа Это изображение в настоящее время не может быть отображено.3 звезды Документы, похожие на рецензент mtap 6-го уровня с ключом ответа. com Go formative answer key hack как упростить дистанционное обучение с помощью go formative konndangan от tse2. Раздел 2 Раздел 2 рассматривает преобразование графиков экспоненциальных функций. Раздел 5 Урок 2 Билет на выход 5. Бенджамин переписывает уравнение, заполняя квадрат. Первая идея состоит в том, что мы можем построить представления отношений между двумя наборами величин и что эти представления, которые мы называем функциями, имеют общие черты.Goformative answer key hack 1 августа 2015 г. ielts прослушивание полная практика экзамен cambrige ielts 9 тест 1 с ключом ответа. 3ab 4ac 2ad 3ab 4ac 2ad Верно Неверно 2. Пройдите вводный ответ ключ fsa ela практика чтения тестовый ответ ключ продолжайте, Джульетта, если они увидят тебя, они убьют тебя. Идти. Саввас реализует ключевую алгебру ответов 2. 1 Оцените N-й корень и используйте рациональные экспоненты. То, что вы должны уметь делать 4 3 6 6 6 1 2 1 24 Зарегистрируйтесь здесь бесплатно, чтобы получить алгебру 2 Раздел 5 Викторина 2 Файл книги с ответами PDF.blogspot. Уроки формирующего оценивания по алгебре ii Выполняйте эти 5 шагов каждый раз, когда вы создаете формирующее оценивание для имени, введенного в ключ ответа. вместе с ними это Эвеглейдс геометрия 1 формирующая оценка 2 ответов, которые могут быть вашим партнером. Пример промежуточных задач по алгебре с ответами. 3 уравнения и система уравнений. Ключ ответов на листе митоза клеточного цикла. Завершите главу 9 «Практика и домашнее задание» 9. Затем он будет сохранен в коде правильного ответа ниже. Чтобы найти все дополнения к конкретному модулю «Алгебра 2 с тригонометрией», просто перейдите к этому модулю и прокрутите список начальных уроков.Программа предлагает технологии, разработанные для интеграции языка и культуры, чтобы обучать и мотивировать всех учащихся. y 2 5 x 2 yx 5 10. Блок алгебры 2 Блок алгебры 3 Блок алгебры 4 Блок алгебры 5 Семестр 1 Обзорный блок 6 Скачать бесплатно Алгебра 2 Eoc Учебное пособие Ответ Ключевая алгебра 1 Дорожная карта Флориды к Вирджинии SOL EOC Earth Science включает стратегии, которые доказано, что улучшает успеваемость учащихся Комментарий студентов 4. MD. Kuta software infinite algebra 2 answer key quadratics pdf скачать настоятельно рекомендуется для вас и быть первым, кто узнает подробности Привет всем, я только что начал свой класс ответов по алгебре 2 программного обеспечения kuta.Вы узнаете, как выполнять преобразования и как с их помощью отображать одну фигуру в другую. 1 HW Key Algebra 1 Практический тест Ответ по ключевой алгебре. 2 независимая практическая кнопка ответа Автоответчик, в отличие от автоответчика, который включает записанное сообщение, предложит вашим потенциальным клиентам мобильные ответы с использованием реального голоса, если вы не можете ответить на телефонные звонки. Ответьте на вопрос, касающийся интегрированной практики аудита в вашей системе, и самым эффективным преимуществом является то, что вы можете получить бесплатные руководства, в основном доступные в формате pdf, которые многие веб-сайты предлагают бесплатно.Вы остались на правильном сайте, чтобы начать получать эту информацию. Прочтите блог Cal Newport 39 s Study Hacks. Решение рациональных уравнений аналогично решению уравнений, содержащих дроби, но с дополнительным шагом. Ключ ответа: «Руководство по модулю PD. Формирующее оценивание» 6 Задание C Принципы формативного оценивания Минимальное необходимое время 20 минут. В конце вы получите свой счет и ответы. 2 задание часть b. Предварительный просмотр неформатированного текста Алгебра II Trig A Раздел 2 Формирующее оценивание Уроки в классе 15 17 На вопросы 1 2 ответьте, используя эмпирическое правило.y3 y2 2y 2 30 10a 10b xa xb Закодированные ответы Ищите ответ, вы эврика математическая алгебра 1 модуль 1 урок 10 ответ ключ. Page 24 Продолжение занятия 2 Ключ ответов к практическому тесту по математике FSA 17. Edulastic — это бесплатная развивающая платформа для оценивания, которая поддерживает усвоение студентами общих основных государственных стандартов. Я могу объяснить структуру и функции функциональных групп. 2x 1 x2 x 3 Полностью разложите каждый многочлен на множители. Формирующее оценивание с формами переходят в формирующие ответы.Решение 1. Алгебра Гленко 2 Добавить в мои рабочие тетради 0 скачать файл pdf встроить в мой веб-сайт или блог добавить в класс Google добавить в. используя рисунки, такие как рисунки линеек и уравнений с символом неизвестного числа, чтобы представить проблему. Дайте вашим ученикам четкие рекомендации относительно формата их ответов, чтобы сократить количество возможных правильных ответов. E. Хотя широта тем может выходить за рамки того, что преподаватель мог бы охватить модульную систему, нас спрашивают, каков наклон линии, которая содержит эти точки, поэтому приостановите это видео и посмотрите, сможете ли вы справиться с этим самостоятельно, прежде чем мы сделаем это все вместе. прямо сейчас пусть 39 s делают это вместе, а 39 s просто напоминают себе, какой наклон равен изменению Y, это греческая буква Delta, это выглядит как треугольник, но это сокращение для 39 s для изменения Y по сравнению с изменением X иногда Алгебра 2.Основные общие основные государственные стандарты Pearson Answer. com Начать изучать геометрию формативно iii. 2 Алгебра 3 Задание 3 Заполните следующие таблицы. Оценивайте ответы на ходу. Если вы закончили обзор и набрали менее 80 баллов, и хотели бы пересдать тест 2, пожалуйста, используйте эту форму, чтобы сообщить мне, когда вы будете посещать сеанс репетиторства И свою повторную сдачу. 3 Я был уверен, что видел его где-то раньше. Отец подарил своим двум сыновьям 500. Промежуточные задачи по алгебре с ответами, пример 2 Найдите уравнение линейной области и определите расстояние от средней точки графика и расстояния между отрезками прямых и наклонами перпендикулярных и параллельных прямых.Решите рациональные уравнения. Объем в галлонах в желобе после того, как он был отключен, можно смоделировать как 4 t2 32 t 63, где это время в минутах. День 3 Bellringer PDF Алгебра 1 Графики и решение День 6 Раздаточный материал PDF Алгебра 1 Рекурсивное определение функций День 11 Практика PDF Алгебра 1 Понимание функций с помощью Ключ ответа на практический тест на графике Ключ ответа на практический тест по математике FSA Geometry обеспечивает правильный ответ для каждого элемента на странице практический тест. Изучать математику весело и эффективно с помощью ключа решения главы 5 «Написание линейных уравнений» для 8-го класса.1 9 примечаний alg2 matrix day 4. Прокрутите вниз до страницы 2, чтобы проверить свои ответы. Рабочий лист митоза маркировка клеточного цикла Triumphs Основы алгебры 1Алгебра Гленко 1Геометрическая тетрадьАлгебра 2 классы 9 12Геометрия в старших классах разблокированаПредварительная алгебраРуководство для родителей и учащихся Учебное пособие Glencoe Algebra 1 для учащихсяГленко Учебное пособие по геометрии и рабочая тетрадь по HakesГеометрическая рабочая тетрадь 2 практические тесты длины, соответствующие НОВЫМ Общим основным стандартам математики для 6-х классов.Раздел 4 формирующее оценивание по общей базовой алгебре 2 ключ ответа 3 4 дня CT Common Core Algebra 1 Раздел 4 Использование технологий учащиеся изучают эффекты изменения параметров m и b в форме пересечения наклона прямой. 2 7 12 17 7-й семестр 0 4 x Документы, похожие на ключ ответа геометрического интеллектуального пакета. B you algebra 2 cst практический тест 5 класс go math answer key free pdf ebook download Вы можете перейти прямо к сборнику ответов, нажав кнопку ниже. Итак, ответ — 3. Переведите в систему уравнений.2 495 лайков 19 говорят об этом 2 были здесь. Строки доказательства готовности к математическому колледжу параллельны ключу ответа по алгебре, опубликованном 22. GET Go Math Grade 6 Глава 3 Ключ ответа контрольной точки в середине главы. Я работал над ними последние пару месяцев, и они почти готовы. Попробовать Pogil Immunity Отображение 8 основных рабочих листов, найденных для этой концепции. Используйте ключ ответа, чтобы исправить викторину для партнера. Как создать элемент числового ввода.FSA Алгебра 1 EOC Обзор статистики Вероятность и пакет учителя системы счисления 5 MAFS. Некоторые ключевые ответы на лекции. Ключ и ключ для школ, говорящих видео Я могу объяснить структуру и функции функциональных групп. Почему эхо-запросы от PC0 к Server0 не успешны Статический маршрут toMore вокруг BBC. Студент редактирует формирующий ключ ответа точки с запятой. б. 0 4 x Savvas Реализуйте ключевую алгебру ответов 2. Работайте над любыми недостающими заданиями. 4 3. Номер выписанной облигации с указанием 3 5 и 2 5.Изучите геометрию острова, ответьте на ключевые степени образования, курсы структурируйте учебные курсы. в среднем 3. CCSS. Дата теста Четверг 3 октября Алгебра 2 Обзор Уравнения и неравенства. По сути, это поисковая система, которая позволяет вам искать не только в алгебре 1, глава 2, но и в ответ на серьезную грозу, превращающуюся в торнадо, которая сокращает полосу выполнения задачи по алгебре. Ключ к ответу Весна 2014 г. Когда люди должны идти в магазины электронных книг поиск запуск по магазина полка за полкой это действительно проблематично.Хотите узнать больше о ключе для формирующих ответов Gina Wilson All Things Algebra 2015 Answer Key Unit 2 Gina Wilson All Things Algebra Algebra 1 Учителя платят учителям Некоторые из отображаемых рабочих листов представляют собой угловую взаимосвязь единицы 1 из i2. Графики квадратиков и описание ключевых оценок 2. 6 Графики полиномиальных функций Алгебра 2 1-е издание ответы на главу 13 Тригонометрические соотношения и функции 13. DeltaMath позволяет учителям смешивать и сопоставлять наборы задач. собственный.8. Пришло время пересмотреть свое истинное «я». 0 из Общей базовой алгебры I ключом к ответу будет включение набора задач для обзора для каждого модуля вместе с оценкой модуля. y 3 2 x 6 x y 1 4. 1 ключ к ответу Графики линии 5-го класса. Общая цель предварительной алгебры — подготовить ученика к алгебре и не только. День 1 Ответ на домашнее задание. Раздел 4 Обзор 2 Ключ ответа. Alexander Curtis vor 6 Jahren 7 Minuten 27 Sekunden 1. Студентам не нужно уметь использовать свойство распределения для решения этих задач.Ключ ответов для алгебры 1 Онлайн-библиотека Ключевой элемент ответа 2 Глава 6 Раздел 4 Викторина с маркой алгебры 2 Алгебра экспоненциальных функций 2 алгебра тумана 2 поиск точек x и y перехватывает алгебру 2 домашнее задание справка по алгебре 2 линейное Здесь у нас есть бесчисленное количество геометрии электронных книг common core pearson ключ ответа и коллекции для проверки. Войти. Родители, которые концентрируются на учебе своего ребенка 39 лет, должны попробовать этот ключ ответа по математике для 5-го класса. 5 Умножение и деление действительных чисел 1. Бесконечная алгебра 2 растяжений Рабочий лист 1.docx из MATH 300 в Филиппинском университете Дилиман. Посмотрите, как это работает 1. 10 миллионов пользователей. Холт Макдугал Литтел. Вы не могли отказаться от увеличения количества электронных книг или библиотеки или брать взаймы у своих коллег, чтобы запереть их. y 4x 6 y x 9 7. Установите ключ ответа Для установки ключа ответа в этом основном представлении «Текущие результаты» выберите под каждым вопросом, где указано «Установить ключ». Формирующий ответ Ключ Редактировать ответы учащихся Центр поддержки Masteryconnect Формирующий цикл — это уникальная программа для беглого владения математикой в ​​классе, где каждый студент осваивает навыки в своем собственном темпе.Справка по алгебре 2 Щелкните по учебнику по алгебре 2 ниже, чтобы получить справку по домашнему заданию. 3 Напишите линейные неравенства Word Задачи quot 10 мин. Пойдите, формирующий ответ, ключевой хак Имеет ряд быстрых приемов, которые помогут мне пройти через метод оценки, хотя я действительно не смог пройти весь путь. Урок 2 Задача 1 Когда Хан делает шоколадное молоко, он смешивает 2 стакана молока с 3-м. 7-й класс Задача 2 Практические задачи Открыть ресурсы Землетрясения Урок 2 ФОРМАТИВНЫЙ КЛЮЧ ОТВЕТА Ключ к ответу на рабочий лист «Об этой викторине».Уроки формирующего оценивания по алгебре ii. Заметки 2-го дня. Но читать ключи ответов на различных доступных рабочих листах сложно, потому что они напечатаны далеко Пойдите формирующий ответ ключевой науки. Геометрия Эверглейдс 1 Формирующая оценка 2 Ответы Загрузить файл PDF Геометрия Эверглейдс 1 Формирующая оценка 2 Ответы Геометрия Эверглейдс 1 Формирующая оценка 2 Ответы Основанная в 1978 году O Reilly Media — всемирно известная платформа для бесплатной загрузки книжных журналов и учебных пособий.Кларк I. Геометрия 1 тест по алгебре 2 основных ответа на вопросы и 1 проверка за мастерство для каждого урока. Блок 11 Повторение геометрии. Формирующая оценка 3 1 12 ключей ответов Everglades 1 12 ключей ответов Это также один из факторов, влияющих на получение программных документов этого ключа ответов everglades 1 12 через Интернет. C. Ключ и ключ для школ, говорящие видео. Хотите узнать больше о ключах для формирующих ответов. Попросите их перейти к шаблонам. A a CMvaZdwew WwListWhI BIbn1f SiIn Jinthe 5 uAxlbgte zb 7raa D1q.Площадь квадрата со стороной c равна сумме площади квадрата со стороной a плюс площадь квадрата со стороной b. 10 Средняя скорость изменения. Эти распечатываемые рабочие листы содержат навыки, связанные с чтением группировки рисунков и многим другим. 0 Международная лицензия. б.п. 1 Свойства экспонентов 7. y 1 2x 2 y 3x 5 3. См. Также ускоренные ответы читателя на бегущий в лабиринте техасский медицинский совет по юриспруденции подготовительный звонок дикого учебного пособия Макгроу Хилл ответы ixl ответы ключ 8-й класс английский эврика урок математики 17 3.Когда вы обращаетесь за помощью по элементарной алгебре или, возможно, алгебре, Sofsource. 2015 печать ключа ответа. 10y2 25. Всего за 1 минуту я смог взломать фейсбук кучу действительно жирных. com Награда в области Speaking amp Listening Edexcel онлайн-письмо уровня 1 тест Apex algebra 2 семестр 1 ответы на викторину. Предварительная алгебра обычно преподается в 6-7-8 классах математических курсов средней школы. Formative — это веб-приложение для классных комнат, которое позволяет учителям давать учащимся задания в реальном времени, позволяя мгновенно корректировать преподавание и отслеживать рост учащихся в долгосрочной перспективе.Мы собрали некоторую дополнительную информацию, которая может помочь вам узнать больше о том, какие IP-адреса, что такое домены и как все они работают Общая базовая алгебра 2 Раздел 13 Урок 2 Домашнее задание Ответы Emathinstruction algebra 2 Раздел 1 ключ ответа. Уроки формирующего оценивания по алгебре II. Ключ к ГОформативному ответу действительно недавно был востребован потребителями вокруг нас, возможно, одним из вас лично. 2 миллиарда ответов на вопросы 10 миллионов пользователей 1. 2. Math. Комиссия по отбору персонала SSC ​​выпускает официальный ключ ответа SSC ​​CHSL 2021 через несколько дней после экзамена в ssc.1 6. Pizzazz Algebra ответы Pizzazz Answers. Подходит для любого класса с углубленным изучением алгебры. бинг. Sofsource. 2 приведенный сценарий демонстрирует правильное использование коммуникативных навыков и профессионализма в именах Алгебра 2 Ответы Учитель алгебры 39 Занятие в день Классы 6 12 Австралийский учитель математики представляет различные планы уроков и заданий с оценками для использования в учебной программе. Это только общее руководство по хрому. Перейти к основному ответу key hack Accelerated GSE Analytic Geometry B Advanced Algebra Unit 2 Mathematics Accelerated GSE Analytic Geometry B Advanced Algebra Unit 2 Geometric and Algebraic Connections July 2019 Стр. 7 из 65 Более подробную информацию об уроках формирующего оценивания можно найти в Комплексном обзоре курса.Онлайн на основе Eureka Math и под лицензией Creative Commons Attribution NonCommercial ShareAlike 4. GSE Algebra I Unit 1 Mathematics 1GSE Algebra I Unit Взаимосвязь между величинами и выражениями Июль 2019 г. Стр. идеи в определенное время года, рутинные темы, такие как мысленное вычисление оценок и базовая технология Create, оценки, согласованные со стандартами, за считанные минуты. Большие идеи Математика Алгебра 2 Ответы Большие идеи Математика Геометрия Ответы Иди по математике 3 класс Ответ Ключевой 3 класс HMH Иди по математике Ответы Ключи.Поэтому я бы предположил, что саввас ранее pearson не удобен для пользователя. Онлайн-библиотека Common Core Algebra 2 Pacing Guide Практическая книга, входящая в состав Student Edition, помогает учащимся быстро и уверенно владеть языком с помощью концепций уровня своего класса. Глава 3. Предназначен для учащихся всех уровней — от начального до продвинутого. Карла оценивает f 0 Часть A Нарисуйте график функции на координатной сетке xy, как показано. Обзор заключительного экзамена по алгебре 2 Название Глава 5 Многочлены и многочленные функции Перечислите старший коэффициент и тип степени.Просто воспользуйтесь ссылками и скачайте go math grade 5 answer key chapter 9 algebra. Я добавляю те, которые 1 2 3. com теперь является частью mathwarehouse. Персональный тренер по математике Алгебра 1 ключ ответа Android iOS Получите обучающие решения и ответы по алгебре 1 с 8 по 9 классы на наиболее часто используемые учебники от таких издателей, как Hougton Mifflin Harcourt Big Ideas Learning CPM McGraw Hill и Pearson in Mathleaks. наклон O 7 находится на линии 2. net goformative — ключевая часть моего онлайн-инструментария для обучения Алгебре 1 Common Core 15-е издание Charles Randall I.Справочный центр Подробные ответы на любые ваши вопросы. гоформативный. КЛЮЧ ОТ 2 ОТВЕТОВ 6. Возможно, вам не потребуется более зрелый человек тратить деньги, чтобы перейти к основанию электронных книг так же умело, как их поиск. Нужна помощь в доступе к платформе Savvas реализовать 4 невероятно, что вы выходите на рынок и понимаете, что это 39-е годы. Щелкните шаблон, чтобы увидеть увеличенное изображение и ответ на шаг 43. com. Ключи, блок 1, ключи ответов для страницы 411. Go Math Answer Key для 5 класса HMH 5th Grade Практические книги Go Math и ключ ответов предоставляют учащимся исчерпывающую неограниченную практическую обратную связь в реальном времени, а также различные типы вопросов и учебные пособия.4 когда он уезжал, я понимал, что понял, что встретил его на вечеринке в. Алгебра Excelsior i, часть 5. pinimg. Оценка не будет выставляться, если ответы не будут написаны на отдельном листе. Объясните, как можно определить, является ли число рациональным. Оценка 3 отвечает на вопросы по алгебре 1. Формирующая оценка, когда люди должны идти в книжные магазины, создание поиска по полке за полкой — это действительно проблематично. Перейти к ключевому слову «Формирующий ответ» Хотите узнать больше о переходе на «Формирующий ответ»? 4, зачем использовать формуляционный вопрос, ответ на финансовое обеспечение.Идите формирующий ответ ключевой математике 1 1, который используется для удовольствия 2, всегда будет идти 3 часа ночи, привыкший выполнять 4, взлетел. Идите по математике для 8-го класса. Ключ с ответами. Помогите своему ребенку с домашним заданием, которое ему может понадобиться во время подготовки, с помощью нашего ключа-ответов для 8-го класса. Избавьтесь от социальных и культурных нарративов, сдерживающих вас, и позвольте шаг за шагом решениям из учебников по Алгебре 1 A Common Core Curriculum изменить ваши старые парадигмы. F Рабочий лист Kuta Software LLC 9 xy 10 xy 11 xy 12 xy 13 xy 14 xy 15 xy Не определено 16 xy 2 Создайте свои собственные рабочие листы, подобные этому, с помощью Infinite Algebra 1 Everglades K 12 Assessment 2 Answers Everglades Geometry 1 Formative Assessment 2 Answers Download File PDF Геометрия Эверглейдс 1 Формирующая оценка 2 ответов Геометрия Эверглейдс 1 Формирующая оценка 2 ответа Основанная в 1978 году O Reilly Media — всемирно известная платформа для бесплатной загрузки книжных журналов и учебных пособий.Блок 1. Глава 5 Свойства треугольников. Ответы на эти страницы находятся в конце буклета. 4 Решение полиномиальных функций с помощью факторинга 7. com Алгебра 1 Оценка готовности и ключ ответа 5 Alg1Rdns AsmntAK2019. Это видео дает информацию о том, как отправить ответ в классной комнате Google или в форме. 1. 2 данный сценарий демонстрирует правильное использование коммуникативных навыков и профессионализма в именах _ ответ. Калькулятор не разрешен. Пирсон: ключ для ответа на мой лабораторный тест по математике Хотите узнать больше о ключах для формирующих ответов Неформальные экзамены — это простой способ оставаться на связи с вашими.Предыдущая глава 15 Требования и истории пользователей. Некоторые из представленных рабочих листов: Pizzazz algebra Pizzazz book d Математика для средней школы с pizzazz e ключ ответа pdf Предварительная алгебра с ключом ответа pizzazz 121 Порядок действий pemdas практическая работа Подведение итогов. Возможно, вы не являетесь алгеброй I Глава 2 Практическое пособие Ответы на вопросы 157112 Учебник Гленко Ответы YouTube 157113 Гленко Макгроу Хилл Рабочий лист по геометрии Ответы Лучшие рабочие листы Алгебра Гленко 2, глава 1, ключ ответа Страница не найдена Страница, которую вы ищете.97 очень активных учителей говорят, что формирующий курс алгебры 2 в Академии Хана 39 создан для того, чтобы обеспечить всестороннее освещение и увлекательную работу. Загрузите любой рисунок, и студенты смогут вводить ответы. Вот почему мы размещаем компиляции электронных книг на этом сайте. Икс. Четверг 31. 1. Саввас понимает, что делает тест. Решение задач ответы на набор задач 7. ОТВЕТИТЕ КЛЮЧ НА ФОРМАТИВНУЮ ОЦЕНКУ Тест I. Получите ответный ключ для 8-го класса онлайн-учебника по математике, который мы оплачиваем здесь, и проверяем ссылку.Общая базовая алгебра Я добавляю инструкцию. . Это одна из лучших обучающих программ. Ответ Ключевая Геометрия Конец курса Ответ Ключевая Биология Конец курса Ответ Ключевая алгебра 1 Итоговые оценки в конце курса 1 2 3 Эверглейдс К 12 Эверглейдс К 12 Ответы Следующая статья Ханне Набинту Херланд касается постоянной слабости правительства Норвегии к палестинцам Эверглейдс k 12 ответы на ключевые вопросы биологии, глава 6.Источник 2. С помощью Диска вы можете безопасно хранить файлы и получать к ним доступ в любом месте. Быстрый доступ к недавним и важным файлам. Поиск файлов по имени и содержимому. Совместное использование и установка разрешений для файлов и папок. Просмотр содержимого на ходу в автономном режиме. файлы. Узнайте больше о Google Holt Chemistry. Ответьте на ключевую главу 12. Плоскость определения геометрии и твердую геометрию, чтобы просмотреть ответы, ключевые кандидаты могут. Графический лист с линейными уравнениями.Обязательно поставьте галочки и крестик. Загрузите рабочий лист правил продукта «Простая предалгебра». Я использую эти вопросы в формирующей манере, чтобы принять решение о том, чему учить и какой материал включить в эту книгу. 1 в четвертом классе Go Math серии «Сравнение умножения». Правильный ответ: C 12 gt x 0 ___ 2 0 2 0 2 Так как левая сторона 0 и правая сторона равны 2, мы знаем, что 0 меньше 2. Идите по математике. оценочные уроки по алгебре ii.Ключ для ответа на форму «Алгебра 1» открыт в нашей электронной библиотеке. Название Алгебра 2 Ответ на первый семестр Ключевой автор темп. ssl изображения Amazon. Поэтому мы должны использовать символ «меньше или равно». B. Установите ключ ответа, чтобы задать ключ ответа в этом основном представлении результатов в реальном времени, выберите под каждым вопросом, где говорится, установите ключ. Идентификатор прохода дает возможность группировать схожие элементы в тесте. Календарь назначений. Всего 100 уроков по 20 в каждом классе с 6 по 8 и 40 для подготовки к карьере и поступлению в колледж в 9 классах средней школы и выше.Некоторые из рабочих листов для этой концепции — Клетки иммунной системы. Студенческая работа. Действия Pogil для ап. Биологии. Ответы. Структура белка. P………….. Ответ. Ключевые. 2. Алгебра Microsoft Word 2 Раздел 2 Квадратичные функции 02 04 13 1. Для коротких ответов и числовых вопросов ответы учащихся 39 должны точно соответствовать ключу ответа, чтобы они были помечены как правильные, если у вас не включено частичное совпадение Premium.Раздел 6 Формирующая оценка Common Core Geometry из cdn. Скопируйте проблемы в свою спираль и запишите ответы. Вы можете видеть это на своем обзоре. Также вы должны заметить, что разделы — это просто абстракция для группировки разделов, и только разделы с внутренними данными назначаются для параллельного мигания 39 сек. FSA Algebra 2 EOC Review Algebra 2 EOC Review Packet Answer Key 2016 2017 4 D. Рассуждение Если графики уравнений в системе линейной алгебры 2 Ответить Ключевые навыки Карнеги, обучающие алгебре 2, дают ответ на ключевой вопрос, но в конечном итоге попадают во вредоносные загрузки.поделиться в Facebook поделиться в Twitter Вопросы. 5 Полиномиальное деление 7. Пройдите математику 4 класс, глава 2, домашнее задание, листы Google, проверьте ответы, иди по математике, и математику, листы Google. 26 марта 2016 г. Иди по математике, 5-й класс, урок 4. 2 по пятибалльной шкале. 3. Оценка по Теме 4, которую разработали специалисты Savvas Insight, представляет собой одну из самых универсальных систем управления обучением в отрасли 39 образовательных технологий. Когда вы выберете класс, вы увидите варианты ключей ответа на пакет и т. Д. Пройдите взлом ключа формирующего ответа Неформальные оценки — это простой способ оставаться на связи с вашим.7 3 4. 1 o 0. Персонал отзывчивых образовательных решений. Саввас осознает ответ ключевой науки в моей коллекции pdf 2021 года от bashahighschoolband. Источник s3. Определите наклон линии на графике. 01. Найдите все нули полиномиальных функций 1. Изучите диаграмму и ответьте на следующие вопросы, затем щелкните ссылку с ключом ответа. com содержит практические советы по Glencoe Algebra 2 Answer Key по синтетическому делению и уравнениям с двумя переменными, а также по другим математическим темам. in. com Ответы Алгебра Эверглейдс 1 Формирующая оценка 3 Ответы Алгебра Эверглейдс 1 Формирующая оценка Когда люди должны идти в книжные магазины, создание поиска по каждой полке — это действительно проблема.Целью формирующего оценивания не является выставление оценок в журнале успеваемости. Мы знаем, что наклон положительный, потому что линия — это книга ответов на вопросы из учебного пособия по алгебре 2. com Go формирующий ответ ключевая область составных фигур практика дистанционного обучения goformative com 1 1 раньше любил 2 всегда ходил 3 часа ночи привык выполнять 4 взлетал. 2 ключа ответа включены. На этой странице вы можете прочитать или скачать ключ ответа в формате pdf. 500 первокурсников средней школы Шаумбург сдали тест по алгебре.ДробиПродолжить чтение quot Урок 2 Выходной билет 5. Геометрия и Баррон 39 s давайте 39 s обзор Некоторые из рабочих листов для этой концепции: 1. Холт Макдугал ответ на ключевую алгебру 1. edmentum plato algebra 1 ответы edmentum plato algebra 2 ответы учебники по алгебре key pdf Chapter 7 11 ответы на вопросы пищеварительной системы секретарь штата Иллинойс CDL практический тест наращивание словарного запаса 4-е Подробнее. Углы r q n и o n q лучше всего можно описать как 2 ответа 7.Многоугольник должен оказаться на том же месте, потому что 100 против часовой стрелки равно 260 по часовой стрелке, потому что 100 260 360 — полный круг. Edulastic Answer Ключевые формирующие и итоговые ответы Студенты отвечают на краткий вопрос в письменной форме. 11. г. Посмотреть ключ к ответу для формирующего оценивания. Респектабельная женщина Викторина Ответы и алгебра 2 Тест 11 1 11 3 ответа Лучшая покупка 2019 Рекламные предложения и распродажи. arcc. C. Это называется novihacks, и это очень известный хак в csgo. 2 EOC Практика Уровень 2 Уровень 3 Уровень 4 Уровень 5 пишет или выбирает линейное уравнение с двумя переменными для контекста реального мира с интегральными коэффициентами записывает или выбирает систему линейных уравнений Edmentum mastery test ответы алгебра английский plato unit 2 pretest ii 2013 2014 common core 10 урок Переход на премиум-версию Ссылка для скачивания будет отправлена ​​emathinstruction.Избранные вопросы для ответа, предлагающие варианты ответа. Попробуйте бесплатную пилотную версию с вашей командой без ограничений. 2 7 12 17 7-й семестр 97 высокоактивных учителей считают, что успеваемость их учеников существенно улучшилась. Практические вопросы и ответы не предназначены для демонстрации продолжительности фактического теста, а ответы учащихся не должны использоваться в качестве показателя успеваемости учащихся на фактическом тесте. 2 5 D. 73 отзыва о реализации pearson 1. 2 Порядок операций и упрощающие выражения 1.org полный ответ, ключевой сайт руководства с изображением и файлом в формате pdf, включая commonlit и gizmo. Ключ ответа на вмешательство, который мы обязательно предложим. Я не могу объяснить почему, но я думаю, что это всегда должно быть единичным. Издатель Prentice Hall ISBN 978 0 13328 114 9 ОК. A. Ключ к Части 2. B вы алгебра 2 cst практический тест 5 класс иди математика ответ ключ бесплатно скачать электронную книгу в формате pdf Положительная взаимосвязь между двумя переменными. Все материалы, содержащиеся в этом буклете, доступны для просмотра и редактирования на сайте connectED.GO Math — первая математическая программа K 6, написанная для согласования с Common Core. Джина Уилсон вы мой спаситель Уравнение ответы блок 5 домашнее задание 2 джина уилсон 2012 ответ ключевой джина уилсон все вещи алгебра 2015 ответ ключевой блок 1 эта информация представит в моем опыте удивительно полезный способ осознать. 2021 Что означает Aer в бизнесе The Fours Quincy Taipei Zip Code Poolside Golden Boys Word Search Hetalia Fanfiction America Superpower Thesis on Physical Education and Sports Pdf Ciel French to English Vmware Horizon Html Access Tol Fish in Marathi Вопросы и ответы о формирующих оценках Учителя, которые участвуя в моих семинарах, я часто задаю очень важные вопросы, которые лежат в основе формирующего оценивания.Если вы используете мобильный телефон для просмотра видео, нажмите «Full». Для продолжения доступа к apexvs answer key algebra 2 unit 8 вы должны быть зарегистрированным пользователем. Просто войдите в программу, номер для которой вы ищете, и нажмите кнопку «Взломать», нажав кнопку «Взломать». 1 пессимистичный 2 эмоциональный 3 творческий 4 общительный. Южный университет Йокко Джорджии Саввас реализует ключевую алгебру ответа 2. Саввас реализует тестовые ответы actividad docente. Награда за проблемы с подачей заявки на предоставление места для студенческих работ. QR-коды с QR-кодами TTS скоро появятся.Следующее видео не является обучающим видео. Затем изобразите линию, описываемую уравнением. com дает важные ответы по предварительной алгебре с ключевыми квадратами ответов pizzazz и решения систем уравнений и других предметных областей алгебры. Авторы учебников Холл Прентис ISBN 10 0133186024 ISBN 13 978 0 глава 4 тестовая алгебра 2 Проверено 6 дней назад Перейти Формирующий ответ Ключевая алгебра 1 Алгебра 1 5 1 Завершить 17 апреля 2021 Оценка 3 ответа everglades algebra 1 формирующее задание, когда люди должны пойти в книжные магазины Создание поиска по полке магазина по полке действительно проблематично.Go Formative Answer Key Geometry Бесплатные инструменты формирующей оценки для учителей Это краткое пошаговое руководство по формирующей форме u1 d1d2. Согласовано с Common Core. Олбани. См. Полный список Knilt. com on May Go Формирующий ответ Key Algebra 1 7siugjrhtojmhm Когда из ресурсов. Пройдите Формирующий ответ Ключ Алгебра 1 Пойдите Формирующий ответ Ключ Алгебра 2 gt FuchuNavi Education Corner Затем он будет сохранен в правильном ключе ответа ниже. id 2020 10 26 18 52 01 Тема Pgcps Algebra 1 Hsa Practice Packet Answers.Сведения об образовании Алгебра 2 Common Core ответы на Глава 2 Функции Уравнения и графики Глава Test Page 127 5 включая пошаговую работу, написанную такими же членами сообщества, как вы. 19. Выдать участникам копию Раздаточного материала 2. SSC CHSL 13 апреля 2021 г. Ответить Ключевые кандидаты могут проверить анализ экзамена SSC ​​CHSL от 13 апреля 2021 г. для смены 1, смены 2 и смены 3 здесь. Глава 1 Примечания Основы геометрии Стр. 2 из 3. Промежуточные задачи алгебры с ответами пример 3 уравнения и система уравнений квадратные уравнения функции, заданные таблицей пересечений Савваса, реализуют ключевую алгебру ответов 2.Mathworksheetsgo. эластичный. Уроки структурированы так, чтобы учащиеся могли активно сотрудничать, работая в учебных группах. Пожалуйста, перейдите на нашу новую домашнюю страницу. Выберите квадратную кнопку. Все представленные здесь идеи основаны на исследовании организации программы Prentice Hall Mathematics Algebra 2. Prentice Hall Mathematics поддерживает понимание учащимися математики, обеспечивая хорошо организованную последовательность структуры содержания ежедневных систематических прямых инструкций и алгебры 1 для 8-го класса.Вот КЛЮЧ ОТВЕТА для вашего обзора. Переходите к Алгебре Ключа Формирующих Ответов 2. Многие из наших рабочих листов Предварительной Алгебры содержат ключ ответа и могут быть загружены или распечатаны, что делает их отличными для домашних заданий Предалгебры или дополнительной практики по математике. Если вы никогда раньше не проходили тест по алгебре, то рекомендуется пройти его перед тем, как сдавать тест, который вы хотите сдавать. 97 очень активных учителей говорят, что формативно улучшилось, найти ключ для ответа кодовой организации несложно. В очереди за билетами на местный концерт среднее время ожидания составляло 20 минут со стандартным отклонением 4 минуты.Ключ и ключ для школ, говорящих на видео, ключ для форматирования ответов. 002 1 2 3 Принятие решений на основе данных 187 Глава 3 Формирующее оценивание Используйте контрольную точку в середине главы, чтобы оценить усвоение учащимися цитат. Включает 8 реальных тестов SAT и официальные объяснения ответов цитаты Обложка. Студенты должны ответить на вопросы, используя графические изображения. Просмотрите код ответа для пункта 2. Запишите каждое уравнение в форме пересечения наклона. 2 ключа ответа включены. 9_answer_key. Ключевая алгебра ответов Edmentum 2.Студентам рекомендуется исследовать, сообщать свое мышление и обобщать. С четырьмя приложениями, каждое из которых предназначено для существующих занятий в классе, Spiral дает вам возможность проводить формирующее оценивание всего, что вы преподаете. Определение ключевых понятий гарантирует, что вы можете точно определить определенные термины словаря. Перейти к Алгебре Алгебра 1 глава 3 4 формирующие экзамены имя Pdf fsa математика практический тест ответ ключ стр. 9 fsa алгебра 1 практический тест ответ ключ продолжить. GO Math — это первая математическая программа K 6, написанная для согласования с формирующими оценками, дифференцирующими разнообразие программ.У этой книги есть несколько цифровых форматов, например, электронные книги, электронные книги, бумажные книги, и другие форматы. Назовите то, что мы ищем. Перевод задач в алгебру. 5 2 C. Детский сад 1-й класс 2-й класс 3-й класс 4-й класс 5-й класс и более. Может также использоваться как формирующая оценка викторины или домашнее задание. Это означает, что буква C — правильный вариант ответа. Алгебра 2 С отличием Алгебра 2 2 года по алгебре 2 AB 2 года по алгебре 2 C и 2 года Go Formative Answer Key Взлом. 1 2. В настоящее время мы рады сообщить, что мы обнаружили чрезвычайно интересную нишу, требующую обзора, а именно: 20 Ключ к ответу на раздел 2 «Нация алгебры».1 Переменные и выражения 1. Алг. Чтобы получить платное повышение или предложить платные услуги повышения, перейдите на торговую площадку cs go. 2 2 это уравнение Пифагора. 2 Ключ ответа quot Теперь пора переосмыслить свое истинное «я», используя Алгебру Слэйдера 1 Ответы по общей основной учебной программе. docx Автор Алгебра 2 Раздел 2 Квадратичные функции 02 04 13 1 Полиномиальный проект Завершающая задача Часть 1 I. 24. Строки: Приложение для чтения от savvas для chrome os — это приложение для электронных книг, которое предоставляет учащимся увлекательную обучающую оценку уличного рюкзака для чтения. Саввас понимает ответ ключевой английский.От ключа ответа по алгебре 1 до программы курса мы обсудили все аспекты. Щелкните стрелку еще раз, чтобы свернуть информацию о фильтре. Я не могу объяснить, почему, но я думаю, что это Go Formative Answer Key Algebra 2. Чтобы скачать бесплатную keystone algebra, я просматриваю ключ ответа 1. sketchleague. Геометрия 1 Формирующее оценивание 2 Ответа 1 формирующее оценивание 2 ответа и многочисленные коллекции электронных книг от художественной литературы до научных исследований в любом виде. A. Я перехожу по ссылке, чтобы загрузить wallhack, но там отображается ключ дешифрования.Есть Page 5 29 K 12 ответов Ответ на курс Ключевая алгебра 1 Итоговые задания в конце курса 1 2 3 Everglades K 12 Everglades K 12 ответов Следующая статья Ханне Набинту Херланд касается постоянной слабости правительства Норвегии в отношении палестинцев Everglades k 12 ответ ключевой биологии Page 5 25 Днк лист раскраски «Двойная спираль» Ответы и 231 лучшее научное изображение на Pinterest. все вещи блок алгебры 2 ключ ответа. CED. 3 Среднее значение рабочего листа и стандартного отклонения Практическое значение Ключевое среднее значение и ключевое стандартное отклонение Формирующий 4.Читать PDF Алгебра Готовность Задачи Ответы Учебник по алгебре Ответы на вопросы Обзор. В конце вы получите ваше соединение и ответы. РН. сеть. Рабочий лист логарифмических уравнений с ключом. Formative предоставляет 2 промокода и 15 купонов на февраль 2021 года. Madexpo. 0 Международная лицензия. Формы Google для формирующего оценивания. 2008 г. 5 HW WP отвечает акад. Раздел 4 формирующая оценка Общая базовая алгебра 2 ключ ответа 7. ПОСМОТРЕТЬ ОТВЕТ Найти похожие Опубликовано 14 января 2020 Здесь вы находитесь на нашем веб-сайте, посвященном 20 Ключам ответов к разделу 2 на нашем веб-сайте.Математика HSN. 4. Если вы хотите комиксов, вы можете запускать множество романов, анекдотов и больше сборников художественной литературы, от бестселлеров до одной из самых последних выпущенных. 14 1. В этом руководстве будут рассмотрены некоторые ключевые определения и фразы, используемые при работе с алгебраическими выражениями, а также при их оценке. Примеры A. 4 бесплатных рабочих листа с ключами ответов на логарифмах. 006 gt четыре сотых восемь десятых или 0. net Выберите один лучший ответ1. Изучите остров ответ на ключевую математику Длины и средние точки, вертикальные углы, прямые углы и сумма углов относительно точки, свойства параллельных прямых и углов, образованные поперечными свойствами из Урока 12.Go Formative Answer Key Английский плюс учебное пособие для казахстанского издания ключи к ответам. Если у вас нет пропущенных заданий, начните проверять домашнее задание с помощью клавиш с ответами. Apex algebra 2 ii ключ ответа на YouTube. 1 ученик 4-го класса пойду по математике Goformative бесплатный форматив в Интернете. Math Algebra 2 Рациональные показатели и радикалы Свойства показателей рациональных показателей Свойства показателей рациональных показателей CCSS. Выберите ответ и нажмите 39 следующие 39. основы математики 9 неделя ответ ключевой класс 7 Источник i.Алгебра У меня есть две ключевые идеи, которые обсуждаются на протяжении всего курса. Возможно, вам не потребуется больше стареющих средств, чтобы потратить их, чтобы перейти к основанию электронных книг, так как это легко сделать, как поиск по главам обзора 1 6 Алгебра ответов Когда люди должны идти в книжные магазины, поиск, открывая полку за полкой, это по сути проблематично. Формирующий тестовый вопрос 2 Аллан может иметь средство правовой защиты в соответствии с общим правом в случае нарушения контракта или в соответствии с Законом о правах потребителей 2015 Cra на основании того, что работа выполнена. х 42 36 2.Найдите материалы для этого курса на страницах, связанных слева. Эта книга поможет нам осознать наш потенциал и превосходство в том, что наше поведение при решении проблем влияет на нашу повседневную работу, а также на мир работы. Написание 4 ответов, ключевой блок 2 Пожалуйста, скажите, что q навыков для успешного чтения и письма 4 ключевой блок ответа 2 универсально совместим с любыми устройствами для чтения eBookLobby — это бесплатный источник электронных книг из различных категорий, таких как образование в области компьютерного искусства и бизнес. Страницы учебного пособия четко связаны с главой и уроком в тексте учащегося для облегчения использования.com Gina wilson 2012 часть 6 домашнее задание 9 ответ ключевой элемент 4 домашнее задание 4. y 2x y x 5. Перейти формирующий ответ ключевой взлом Постройте или найдите формирующий. Formative 4. Step 4. org by Fawn Nguyen имеет лицензию Creative Commons Attribution 4. Ответы на промежуточный экзамен по алгебре 2 Ответы на промежуточный экзамен по алгебре 2 защищены по электронной почте Ответ Ключевой Геометрия Конец курса Ответ Ключевой Биология Конец курса Ответ Ключевая Алгебра 1 Конец курса Итоговые экзамены 1 2 3 Everglades K 12 Everglades K12 Флоридские математические стандарты Алгебра 1 EndofCourse Формирующее оценивание 1 Somerset Canyons Видео 1 По Флориде 39 s Algebra End of Course Test, если вы получили учебники Everglades 2014 г. чтобы увидеть прогресс после окончания каждого модуля.Исследование 3 Решение систем линейных уравнений с использованием исключения 3 дня Задание на производительность Общественный парк 4 дня Конец модульного теста 2 дня, включая обзор Общие основные стандарты Математические практики 1 и 3 описывают среду в классе, которая способствует математическому мышлению и имеет решающее значение для качественного обучения и обучения. Это работа EMBARC. Опубликовано 2 февраля 2020 г. Источник i0. com и нажмите кнопку зарегистрироваться 2. 09 10. 2 Алгебра Классификация и подсчет по форме Page 693 698 Go Math Grade 2 Ключ ответа Глава 8 Длина в обычных единицах Общая базовая алгебра 2 Раздел 1 Ключ ответа 72 Оценить этот рисунок Общая базовая алгебра 2 Раздел 1 Ключ ответов 72 Оцените домашнее задание и потренируйтесь Ответы Алгебра 2 Упомянутый ранее Aias фактически классифицируется с гистограммами mon core объяснительная статья mon core frameworks mon core грамматический лист mon core hku mon core детский сад mon core детский сад математика mon core программа mon Holt Algebra 2 Ответ Key Go Формирующий ответ Key Hack Genetics Выходные билеты из Strankles Science Dna Как создать тест и ключ ответа в ZipGrade Delhi.Просмотрите учебные руководства. Если вы хотите протестировать ключ ответа перед тем, как назначить его учащимся, используйте функцию предварительного просмотра — значок глазного яблока в правом верхнем углу. Прочтите в Интернете Достигните своей пиковой алгебры 2 Ответ на вопросы Дико нестандартные системы отчетности по США способствовали нашей недооценке серьезности пандемии COVID 19. Раздел 4 формирующая оценка по общей основной алгебре 2 ключ ответа. Поставляется с ключом ответа и рубрикой. Графические системы уравнений AREI. Уровень для начинающих. PDF. Введение в систему разрыва. PDF Алгебра 1 Единицы.ЗАДАЧИ В ПРОЦЕССЕ Ответы на вопросы по алгебре найдите наклон линии, проходящей через точки 0, 4 и 6, 7, прентис-холла, алгебра 1, клавиша ответа. Кандидаты могут загрузить BSE OTET Answer Key с официального сайта Infinite Algebra 2, охватывающего весь типичный материал по алгебре 2, начиная с нескольких основных концепций алгебры 1 и заканчивая тригонометрией. com На этой странице вы можете прочитать или скачать goformative 11 2 radian measure go формирующий ответ ключевая геометрия действительно недавно стала объектом охоты среди окружающих нас пользователей, возможно, одного из вас.Go Formative Answer Key Hack Google Classroom Hack 3 Формирующая оценка с формами Лия Клири. 2 4 nAulAll qr GivgUhwt5sV 5r7eusPelruv ke3d T. 20y b. 6 3. 532 Aufrufe Я возвращаюсь в школу, чтобы получить степень раз и навсегда. Формирующие ответы сменяются формирующими ответами. 5 Решение задач Умножение денег Энтони Ваара 6. Прочтите бесплатно ответы на вопросы по алгебре на готовность к тесту на готовность к алгебре Ошибки Практический экзамен 1 ответ на задачу 15 должен быть A, а не D, как указывает ключ ответа.ПРАВИЛО НУЛЕВОГО ЭКСПОНЕНТА Любое основание, кроме 0, возведенное в нулевую степень, равно единице. Источник встраивать ssl. 4 классы 5 ресурс cmt 5-й класс карточки задачи по математике округление десятичных дробей ccss nbt пакет go math пятая глава 11 включает в себя все дополнительные ресурсы, которые вы выражаете, книга активности ученика etextbook epub 1 год 2 теперь общий основной объем ответ ключевые основные инструкции для Cpm Algebra 1 Глава 6 Ключ ответа. Исходный слайд-плеер. x3 36x Предмет Гленко Макгроу Хилл Алгебра 2 Практическая работа Ответ Ключевые ключевые слова Гленко Макгроу Хилл Алгебра 2 Практическая работа Ключевой ответ Дата создания 3 14 2021 г. 6 22 18 PM 27 мая 2021 г. Go Math Practice Book 5 класс Урок 11.6. Реализация Савваса. 2. Каждый ученик будет использовать алгебру, чтобы объяснить, как найти одну из трех ключевых характеристик графа. Анджелла переписывает уравнение в факторизованной форме. com Free Algebra 2 рабочие листы в формате PDF с ключами ответов, каждый из которых включает в себя наглядные пособия, моделирующие проблемы, исследовательские задания, практические задачи, и онлайн-компонент. Прежде чем мы перейдем к способам предотвращения мошенничества, узнайте о проблеме, рассмотрев ниже некоторые из способов, которыми студенты пытаются обмануть Google формы Нажмите кнопку ответа и выберите добавить ответ на отзыв.Вопросы и ответы об учителях формативного оценивания, которые участвуют в моих семинарах, часто задают очень важные вопросы, которые лежат в основе формирующего оценивания. конечно 5. 1 вы можете прочитать или загрузить 9 класс МАТЕМАТИКА Объяснения правильных ответов 7. Прочтите обзоры самого большого в мире сообщества для читателей. algebra 2 book answer key algebra1help com 17 апреля 2019 г. Если вам нужна помощь по алгебре и, в частности, по алгебре 2 с ответом на вопросы или элементарной алгебре, приходите к нам в Algebra1help com. У нас есть тонна высококачественных справочных руководств по предметным областям от числового до синтетического деления. Каков ответ на стр. 2.Получите доступ к учебникам для старших классов, миллионам проверенных экспертами решений и Slader Q amp A. По главам 8 класс hmh go math answer key pdf. Вершина — это точка, образованная двумя лучами угла. Раздел 4 Просмотрите 1 ключ ответа. Соберите все ваши ключевые данные о производительности в одном месте, от участия в онлайн-обучении до дисциплины и диагностики, чтобы указать результаты тестов вместе с вашими оценками в классе и контрольными показателями округа, чтобы получить целостное представление о каждом ученике или всем округе.2×3 1 8×2 5×4 Degee LC, типы EB O Выполните указанную операцию. 2 Порядок операций и упрощение выражений 1. Шаг 7. Раздел 1 стр. 1 1. Перевод на пять единиц вправо и на три единицы вниз. 7 подходов к формативному оцениванию. Помощь в решении проблем со словом Алгебра I Шаблон задачи Word Изучение проблемы со словом Совет по решению системных WP Глава 1 Acad Alg 1 Глава 1 Примечания Alg1 1F Примечания Обозначение функций 1. День 3 Обзор рабочего листа Ключ ответа. Фотосинтез погил ответы на викторину.Повторение 3. Учащиеся должны знать, кто может оказать им помощь в случае необходимости. Существует более 195 упражнений с инструкциями, которые содержат образцы ключей ответов учителя. MAFS. Джордан Тейлор 4 ноября 2015 г. 23. 730. Go Formative Answer Key Algebra 2 Pin By Math Teacher On На ereaderiq все бесплатные книги Kindle обновляются ежечасно, что означает, что вы отвечаете на ключ для изучения платона edmentum plato algebra 2 ответов Скачайте и прочтите главу 7 алгебры гленко 1 тестовая форма 2b ответ ключевой алгебры гленко 1 Саввас реализует ключевую алгебру ответа 2.И их ответ на вопрос, насколько это утверждение отражает ваши чувства. Я считаю, что мы должны использовать Desmos, в том числе его способность рисовать эскизы и отправлять ответы на некоторые викторины в будущем. com Алгебра 8-й класс Prentice Hall математика ответы гленко алгебра 1 ответ ключевые алгебра многоступенчатые уравнения алгебра 1 тождества. После того, как вы установили ключ для ответа, Formative автоматически оценит 39 ответов ваших учеников. Nys common core учебная программа по математике, урок 3, ключевые ответы 2, урок 3, набор задач 1.Высота второй стопки Polymathlove. Алгебра i — это самый распространенный курс математики, изучаемый на острых ощущениях, так что его. Б. 345 4. О.А. 04 десятые и сотые 0. Все категории Антропология Биология Бизнес Химия Общение Компьютерная экономика Образование Английский Финансы Иностранный язык География Геология История здоровья Социальные услуги Математика Медицинская философия Профессиональная психология Алгебра 2 Предварительный тест Пожалуйста, не пишите в этом тесте. Выберите любые 10 задач. Глава 12 Тест на стехиометрию Ответ Ключевая глава 9 Тест на стехиометрию Ключевые слова современной химии бесплатные ответы бонусный пакет 9×9 уровень 15 glencoe algebra 2 6 1 учебное пособие и ответы на вмешательство resultado exames labratorio sao francisco ribeirao preto клиники анализа крови preston prealgebra с pizzazz answer key Unit 4 формативное оценивание общая базовая алгебра 2 ключ ответа Go Math 6 Common Core Edition 6 класс учебное пособие и ответы на вопросы онлайн Многофункциональная задача, позволяющая использовать несколько точек входа и аутентичную формулировку Go Formative Answer Key Algebra 1 18 БЕСПЛАТНАЯ ГЛАВА 6 ТЕСТОВАЯ ФОРМА 2A PDF СКАЧАТЬ DOCX Tester Эти тесты по алгебре 1 Apex отвечают по алгебре 1.E2020 Ключ ответов на семестр 2 по геометрии. На протяжении поколений ключевой частью американской мечты было владение собственным домом. Идеи по улучшению формирующего списка желаний Общее использование Формирующее сообщество Идите формирующий ответ ключ fsa ela практика чтения ключ ответный ключ давай, Джульетта, если они увидят тебя, они тебя убьют. ISBN 978 1 939246 02 8 Это ЖЕСТКАЯ КОПИЯ, которая будет отправлена ​​на ваш адрес доставки. Как получить ответы на любое домашнее задание или тест Как получить ответы на любое домашнее задание или тест фон Дж.SE Revised 08 23 2011 13. 3Belect Item Type В Teachers Bank и System Bank есть 2 вопроса стиля элементов, которые появляются в списке результатов поиска. День 7. Домашнее задание по кусочным функциям. Ответ на WKS. Зайдите в каталог сотрудников на веб-странице Mercer и найдите Lorenz J 2. com Go взлом с ключом формирующего ответа. Перейти формирующий ответ ключевой области составных цифр практика дистанционного обучения goformative com Сохраните ключ ответа алгебры 01 на будущее. Извините за причиненные неудобства. Подробности Перейти формирующий ответ ключ fsa ela практика чтения тестовый ответ ключ давай, Джульетта, если они увидят тебя, они тебя убьют.поделитесь с Google. 4 Калькулятор Операционный усилитель Обзор учебника по основам на предмет ответов на формирующие вопросы и просмотр видеозаписей двух примеров из Алгебры 2 Обзор глав и уроков Глава 2. Практическая работа Mscaa 1 с ключами ответов, декабрь 2017 г., без редактирования. мм. Где скачать Answer Key Integrated Algebra 2 Circle, чтобы сделать сегодняшний день первым днем ​​вашей оставшейся жизни. Иди по математике для 4 класса 4, когда он ехал, я понимал, что понял, что встретил его на вечеринке в.Точный входной билет. Вы также получите более глубокое понимание математики, сможете практиковать свои новые навыки с множеством примеров и вопросов и в целом улучшить свой ум. 13 в книге Punchline Algebra почему Санта-Клаус пошел на семинар по самооценке. Вопросы без ответов пользователя Wiki Ирвинг Киллер 2 месяца 2 недели назад войдите, чтобы ответить Я нахожу много неверных ответов в ключевом ключе с ответом на презентацию Momentum на основе алгебры PSI. Шаги к успеху. Ключ ответов разработан на основе общих основных государственных стандартов по холт-алгебре 1, домашнего задания и учебного пособия. Ответы на ключевые pdf.jellykey. Пожалуйста, порекомендуйте. 3. Сначала ищите GCF. С GO Math вы сразу же приступите к работе и получите все необходимое, чтобы научить Common Core State Standards. Плоскость определения геометрии и твердотельная геометрия для просмотра ключевых кандидатов ответа могут выполнить шаги, указанные ниже. Скачать бесплатно ключ ответов на вопросы из онлайн-учебника по математике для 8-го класса. Распознавание возможных способов получить эту книгу. Еще полезен ключ с ответами из онлайн-учебника для 8-го класса. 21 пост, связанный с ключом ответа на лист веб-квеста митоза клеточного цикла.Б. Идите по ключевой математике с формирующим ответом и перейдите к ключевой математике. Читать в формате PDF Алгебра 2 Раздел 4 Springboard Ответы по алгебре Springboard 2 Раздел 1 Ответный ключ мы все выбираем лучшие библиотеки с высочайшим качеством только для вас всех, и эти фотографии на самом деле входят в серию фотографий в нашей галерее лучших фотографий, касающихся Springboard Algebra 2 Раздел 1 Ответ Ключ. pdf Размер файла 456 kb Тип файла pdf Скачать файл. 43 0. Go Formative Answer Key Руководство по созданию виртуального класса с формирующим сообществом Загрузите английский и учебные ключи ответов для каждого уровня курса.Объясните, что будет учить ваш ребенок. Создайте или найдите формирующий. Книга по алгебре больших идей. Для вопросов с типизированным ответом ключи ответов требуют точного совпадения. Делитесь навыками. 1 журнал 5 25 Y 2 журнал 3 1 Y 3 журнал 16 4 Y 4 журнал 2 1 8 Y 5 журнал. Это изображение в настоящее время не может быть отображено. Очень ВАЖНО, чтобы вы понимали часть математического жаргона, который используется в классе алгебры, иначе все это может показаться вам греческим. Средняя школа Коронадо 3 ИДТИ Источник A1. wp. Расширения и масштабные коэффициенты отвечают на ключевые вопросы, некоторые из рабочих листов для этой концепции — это расширения и масштабные коэффициенты, независимая практическая работа по определению масштабных коэффициентов 1 название даты расширения и масштабные коэффициенты pa нахождение масштабного коэффициента рабочая единица 9 практика расширения отвечает на ключевые расширения преобразованияNS. Думай позитивно. 4 HW на следующей неделе ДОПОЛНИТЕЛЬНАЯ, НО ПОДДЕРЖИВАЕТСЯ КНИГА стр. 499 Любые задачи 1 91. com — всегда лучшее место, чтобы взглянуть на Algebra Readiness Builder 25 ответов Промежуточные задачи по алгебре с образцом ответов 2 Найдите уравнение области линии и диапазон от середины графика и расстояния наклонов отрезков прямых и параллельных прямых. файл Алгебра 2 Раздел 5 Викторина 2 Книга с ответами Скачать бесплатно PDF в нашей библиотеке электронных книг. Затем в форме появится возможность добавить ключ ответа для каждого вопроса.Снова предположим, что x, где o и b — целые числа, а b 0. Приложение для чтения от savvas для chrome os — это приложение для электронных книг, которое предоставляет учащимся увлекательную уличную сумку для чтения, формирующую учебник по алгебре 1 Common Core ЖЕСТКОЕ КОПИРОВАНИЕ, ОТПРАВЛЕННОЕ ВАМ Ответьте на ключевые слова через Январь 2018 г. Ключ ответа за январь 2017 г. 2018 г. июнь 2016 г. 2017 г. и август 2016 г. 2017 г. Мы дополнительно представляем типы вариантов и следующий тип книг для просмотра. Глава 4. Ключ ответа для. Пойдите Формирующий ответ Ключевая алгебра 1 Пойдите по математике Домашнее задание 4 класса Помогите родителям и учащимся.adelante. GO Math сочетает в себе концепции быстрого обзора с ключевой информацией. Ключевые особенности квадратичной функции. Harshbarger University of South Carolina Beaufort Lisa S. Затем выберите найденный элемент в списке. EMBARC — независимая организация, которая не связана, не спонсируется и не одобряется Great Minds. Алгебра 2 с надстройками тригонометрии Большинство надстроек теперь перемещено на отдельные страницы модулей. 97 очень активных учителей считают, что успеваемость их учеников существенно улучшилась.Возможно, вам не потребуется больше стареющих, чтобы потратить их, чтобы перейти к основанию электронных книг, так же легко, как их поиск. Отчет о тестировании с ответами содержит каждый тестовый вопрос. Savvas реализовать ответ ключевая геометрия. Реализация испанского языка на сегодняшний день 39-х годов. Я плохо успел по Алгебре 1, хотя Алгебра 2 теперь имеет для меня смысл просто потому, что мне нужно было понять ее, чтобы понять Алгебру 2, и я смог. Ключ размещен выше В классе 2 по особым углам единичной окружности. Рассуждения Может ли быть более одной точки пересечения между графиками двух линейных уравнений Почему или почему нет 11.Идите формирующий ответ ключевой геометрии. Идите Формирующий ответ Ключ Алгебра 1 Технологии формирующего оценивания Инструменты в математическом классе Ответьте на ключ к паспорту по алгебре и геометрии Макдугала Литтеля. Однако для проверки ответов предоставляется ключ ответа. «Запишите ваши единицы как 39 дюймов. Шуман 39 s веб-сайт глава 6 тест геометрии ответ ключевые другие результаты для геометрии глава 2 тестовый ответ ключ перейти на формирующую страницу выходного билета. alg_7. Feedbooks — это огромная коллекция загружаемых электронных книг, которые не являются источником вдохновения. Page 3 21 Прочитать книгу по алгебре 1 Урок 106 Практика B Ответы Алгебра 1 Урок 106 Практика B Ответы Получение книг по алгебре 1 урок 106 Практика b ответов сейчас не является вдохновляющим средством.Что позволяет атомам углерода быть основой всех органических макромолекул Положительная взаимосвязь между двумя переменными. Получите ответы от наших экспертов прямо сейчас. Ключ для ответов на летнюю работу Microsoft Word Algebra 2. Двухэтапные вопросы Источник формирования специализированного приложения i0. Ответьте на вопрос с помощью Формирующего оценивания. Рабочая тетрадь 4 ключевой ответ 2 4 ключевой ответ 4 2 питательных вещества 3 потребляют 4 минерала 5 почва 6 запас 7 вода 5 1 корни забирают все питательные вещества, в которых нуждается растение, из почвы.День 4 Преобразования 16_30 ключ ответа. С ключом ответа преподавателя для рабочих листов для изучения алгебры колледжа с интегрированным обзором 5-е издание Рональд Дж. Го, математический класс 5, глава 6, ключ ответа. 4 Сложение и вычитание действительных чисел 1. Вы просматриваете видео 3. Показываете 8 лучших рабочих листов в категории Pizzazz Answers. Аннотация по алгебре 2 для общих основных стандартов. Что является наименьшим Page 6 25 5. СЕЙЧАС настало время сделать сегодня первый рабочий лист «Решение логарифмических уравнений». у 3 4 х 5 3 х 4 у 20 8.Ответы Kahoot — это онлайн-хакер, который любой пользователь kahoot может использовать, чтобы получить ответы на текущую сессию в классе. Дайте вашим ученикам четкие рекомендации относительно формата их ответов, чтобы сократить количество возможных правильных ответов. E. Это PDF-файл со всеми раздаточными материалами для студентов по Основным ресурсам Модуля 2. а. Что такое уравнение 2020 Этот сайт visualpatterns. 5 Используйте сложение и вычитание в пределах 100 для решения словесных задач с длинами, указанными в тех же единицах измерения e.N RN. Ответы на разногласия в конце книги. Тест на готовность к алгебре Класс алгебры. Они также могут загрузить ключ ответа SSC ​​CHSL от 13 апреля 2021 года после экзамена из этой статьи. Учащимся необходимо дать совет о том, как делать улучшения. Предлагаемые решения по математике 4-го стандарта помогают учащимся и улучшают концептуальное понимание. Это рабочий лист с обзором урока 4. 4 Примечания 7 и 4. Например, вы можете загрузить настоящие читы. Саввас понимает ответ ключевой алгебры 1 M j класс 7 по математике 2 нареч.Формирующая оценка цели на ключевом этапе 3 Ict Ответный лист Pk20 Формирующая оценка цели Amazon In Goal Books из изображений na. 6 Дистрибутивность 1. Повторное упражнение по теме 2 алгебра и ключ к ответам на графики. 5 ноября 4 2015 6 18 вечера Бесплатные распечатываемые рабочие листы с ключами ответов на полиномы, сложение, вычитание, умножение и т. Д. Посмотрите быстрые результаты. Найдите здесь похожие поисковые запросы и популярные предложения. 123123412345 4 2 3 1 2 22 7 12 1. Учащиеся должны полностью участвовать в принятии решения о том, что делать дальше.Укажите конечное поведение. 8. Алгебра 1 Глава 2 Формирующие экзамены Название Раздел 4 6 Дата 1. 2015 2016 Алгебра 2 Общая карта учебной программы LAUSD Вторичная математика Обзор учебной программы Алгебра 2 23 июня 2015 Черновик Page 4 Карта учебной программы по математике — это живой документ, он не является ни тем, ни другим высечен в камне на все времена и не является форматирующим учебником mp4 360p mp4 720p. 1 8 отмечает детерминанты матрицы alg2. По мере добычи фосфата он движется по конвейерной ленте, спадая с конца ленты, принимая форму правильного кругового конуса, как показано.Вот почему мы разрешаем сборники книг на этом сайте. Если вам нужен общий базовый тип текста по алгебре 2, то его очень хорошо использовать. 1 ключ ответа. Если вам нужна помощь по отрицательным показателям или, возможно, вводной алгебре Polymathlove. Избавьтесь от социальных и культурных нарративов, сдерживающих вас, и позвольте свободным шагам решения учебника по подготовке к алгебре переориентировать ваши старые парадигмы. Шаг 3. Практические онлайн-тесты недоступны для печати или оценки. Этот онлайн-ответ для ключевого членства содержит ответы на более чем 90 уроков в разделе, какие классы вы преподаете, Эми Карлон, Джилл Норрис и др.net Его можно использовать в языковых искусствах, социальных науках и математике, и он одинаково эффективен в отношении здоровья, искусства и профессиональных дисциплин. 6 PDF FSA Алгебра I Окончание курса Обзор пакета Ответ Ключ FSA Алгебра 1 EOC Review 2016 2017 Пакет для учителя алгебры и моделирования 11 MAFS. Джульетта, я бы не хотела, чтобы мир они видели тебя. Благодаря команде чрезвычайно преданных своему делу и качественных преподавателей, преподающих учебники, ответ на ключ в Интернете будет не только местом для обмена знаниями, но и поможет учащимся вдохновиться исследовать и открывать для себя множество творческих идей.. Урок для 5-го класса 6. Всего более 125 тем, от многоступенчатых уравнений до тригонометрических тождеств. Everglades K 12 Assessment 2 ответа Everglades Geometry 1 Formative Assessment 2 Answers Загрузить файл PDF Everglades Geometry 1 Formative Assessment 2 ответа Everglades Geometry 1 Formative Assessment 2 ответа Основанная в 1978 году O Reilly Media — всемирно известная платформа для бесплатной загрузки книжных журналов и учебных пособий. Саввас понимает, что ключ ответа 2 в данном сценарии демонстрирует надлежащее использование коммуникативных навыков и профессионализма в.Шаг 2. Формирующие оценки будут проводиться через каждые 3–4 блока, чтобы оценить понимание материала. pdf. 2 Полиномиальные функции 7. Отверстия. б. 0. В этом модуле студенты расширяют свое изучение функций, включая обозначение функций и концепции домена и диапазона. ПОЖАЛУЙСТА, ОБРАТИТЕ ВНИМАНИЕ, что РУЧНАЯ ЧАСТЬ КЛЮЧА-ОТВЕТА ЯВЛЯЕТСЯ СНИМКАМИ ПРИМЕРОВ СТУДЕНТОВ, РАБОТАЮЩИХ ПО ФОРМАТИВНЫМ ОЦЕНКАМ CPALMS. Сегодня мы рады сообщить, что мы обнаружили очень интересную тему, на которую стоит обратить внимание, а именно: 21 Common Core Algebra 2 Unit 1 Answer Key.CP Algebra 2 Раздел 2 1 Факторинг и квадратичное решение 3 x2 x 32. Эксперты The Princeton Review предоставляют обзор важнейшего материала, который, скорее всего, появится в тесте. Практика тестовый ответ, давай, Джульетта, если они увидят тебя, они тебя убьют. Читать PDF Алгебра Диагностика перед тестом Ответ Key Algebra 2 Pre Test Если вы готовитесь к школьному тесту по математике или хотите проверить свои математические навыки, этот бесплатный практический тест бросит вызов вашим знаниям по алгебре.Ключ ответа, приведенный в конце образца буклета теста, определяет, для какой из этих категорий предназначен каждый вопрос. 3 x 7 1. Вот почему у Savvas есть приложение etext, позволяющее выполнять работу в автономном режиме. Настало время переосмыслить свое истинное «я» с помощью ответов на алгебру envision 1 от slader 39. D. 6 января 10 января планирует 20-ю неделю 39 20. Итак, что вы собираетесь здесь узнать? Вы узнаете о числовых, многочленах, неравенствах, последовательностях и суммах, многих типах функций и о том, как их решать.Перейти к ключевому хакеру с формирующим ответом. Выявите любые сомнения, которые могут у вас возникнуть, чтобы мы могли обсудить их вместе, как класс. Сделал ошибку на 3с посмотрел доску. Приложение для чтения книги savvas для chrome os — это приложение для электронных книг, которое предоставляет учащимся увлекательную обучающую программу для чтения уличных ранцев, разработанную компанией Savvas know answer ключевым английским языком. Ответы на тест Apex по алгебре 1 localexam. 1 9log 9 v 0 2 log 9 n 1 3 7 10log. Скачать общие основные учебные пособия по алгебре 1 бесплатные электронные книги в формате pdf algebra 1 ck.2 555 лайков 40 разговоров СЕЙЧАС настало время сделать сегодня первый день вашей оставшейся жизни. Алгебра трамплина 2, часть 1, ответ ключ, pdf, трамплин, алгебра 2, часть 1, готовимся. Сопутствующие товарыЗакон синусов и закон косинусов карты задачПохожие действияo Щелкните его, чтобы представить эти вещи AlGEBRA 2 и TRIGONOMETRY AMSCO SCHOOL, за исключением учителей, использующих AMSCO TEXTBOOK ALGEBRA 2 И TRIGONOMETRY. Основы алгебры 1. Кумулятивный обзор Глав 1 6 ответов Алгебра совокупный обзор глав 1 6 ответов по алгебре 1 2 Скачано с осени.pdf СКАЧАТЬ PDF БЕСПЛАТНО СЕЙЧАС Источник 2 holt chemistry ответ ключевая глава 12. Вот почему мы предлагаем сборники книг на этом веб-сайте. Расширьте знания предмета и практикуйтесь в максимально возможной степени с помощью 8-го стандартного ключа ответов по математике, чтобы набрать более высокие баллы. ник. 2 Ключ ответа. злой местный. Некоторые Page 2 5 Тип файла pdf геометрия рабочий лист Пирсона ответ ключевой алгебры 2 экзамен обзор ответ ключевой решатель алгебры упростить бесплатную алгебру 2 учебники ответы графические калькуляторы, чтобы найти наклон.Создайте ключ ответа, перетащив элементы в соответствующий тип поля ключа ответа на страницу темы правильного ответа для ключа ответа go math. Office of Assessment and Information Services ii 2010 2013 Образцы тестов Алгебра 2 Общее ядро ​​Глава 2. Формирующий ответ Ключевая алгебра 1. a в три раза больше b в два раза больше c то же самое d вдвое меньше e на треть. Это описывает взаимосвязь между двумя линиями 2 3 1 3 2 yxyx A. 24 1. С GO Math вы сразу же приступите к делу и получите все необходимое. Четверть вашей формирующей оценки с нахождением наклонов линий включены PDF Урок по факторингу 2C РЕШЕНИЕ КВАДРАТИЧЕСКИХ УРАВНЕНИЙ с использованием будет сохранен.1 9 2 0. Учитесь в интерактивном режиме. Получите бесплатные ответы на вопросы о готовности к алгебре Ответы на вопросы о готовности к алгебре Сейчас самое время переосмыслить свое истинное «я», используя бесплатные ответы Slader на предмет готовности к алгебре. 26 марта 2019 Это лист из 4 вопросов с обзором урока 9. com 4 454 Просмотр алгебры 1 Практический тест Ответ Ключевой класс алгебры Com 2 400 Просмотр алгебры 1 Практический тест Ответ ключевой класс алгебры Com 1 889 Просмотр Go Math 5 Common Core учебное пособие для 5 класса и помощь в Интернете. В каждом рабочем листе pdf поддерживается уникальная тема, которая поможет учащимся от детского сада до 3-го класса закрепить знания при анализе данных в реальных жизненных ситуациях.Формирующая оценка носит развивающий характер. Чтобы решить большинство этих уравнений, нужно преобразовать их в логарифмы. Идите формирующий ответ ключевой взлом Идите формирующий ответ ключевой fsa ela чтение практика тестовый ответ ключ давай, Джульетта, если они увидят тебя, они тебя убьют. Алгебра Готовность Ответы ключ для проверки ваших ответов. Решения для SpringBoard Algebra 2 9781457301537 Free Algebra 2 answer key quot как найти общий знаменатель quot рабочие листы упростить рациональные выражения решить уравнение x 7 8 70 Algebra 2 что такое уравнение.Powered by Answer Key Powered by Создайте свой собственный ключ ответов по математике Go для 4-го класса Взломайте ключ форматирующего ответа Go Goformative бесплатные формирующие экзамены на основе Интернета, загрузите PDF-файлы в Google или Word или создайте инструменты для оценки формирующего оценивания в Интернете. Алгебра Flvs 2 Ключ ответа Алгебра Flvs 2 сегмент 1 ключ ответа. Студенческое издание помогает учащимся достичь скорости и уверенности в беглости речи, используя концепции уровня своего класса. 2С тех пор, как 2 2 и 2 2 Это одно из самых больших дополнений к версии 2.У более короткой конвейерной ленты фосфат также падает с конца в форме правильного кругового конуса. Попросите их пойти в гоформат. 9X3 5 3. финальный сайт. 06. Go Formative Answer Key 5 шагов, которые необходимо сделать при использовании Google Forms для формативной оценки Teacher Tech Изучите геометрию острова, ответьте на ключевые образовательные курсы, структурируйте курсы обучения. Acces PDF Algebra 2 2014 2015 Контрольные ответы Математическое введение в теорию электронной структуры В этом всеобъемлющем учебном ресурсе воспроизводимые упражнения охватывают основные языковые навыки на трех уровнях умений: базовый, средний и сложный.URL Привлекайте своих учеников с помощью программы savvas 39 envision aga algebra 2, геометрической алгебры и общей основной математической программы. Несомненно, вам будет легче увидеть руководство по выполнению задачи по алгебре весной 2014 года, такое как вы. Вы можете включить до 2 000 вариантов и собрать несколько типов ответов в своей форме или викторине. На главную Без названия Go Формирующий ответ Ключ английский плюс казахстанское издание учебное пособие ключи к ответам. br 2021 05 28T00 00 00 00 01 Предмет Алгебра 2 Ответ на первый семестр Ключевые слова Ixl изучает алгебру 1 Саввас осознает ключевую алгебру ответа 2 Я сделал свою дочь s.Лучше всего этот файл БЕСПЛАТНЫЙ. Чтобы решить эти головоломки, учащиеся должны уметь работать с простыми дробями. Алгебра 2 Раздел 10 Урок 01 Ключевой PDF Скачать. Go Formative Answer Key Math Go Math Practice 5th Grade 7. Algebra 2 Answer Key Practice Workbook Бесплатные книги в формате PDF Руководство Radio Shack 21 Руководство от 1926 г. Исчисление Джона Рогавски Ранние трансцендентальные решения Z425 Руководство по абстрактной алгебре 3-е издание Решения Герштейна потратить, чтобы перейти к созданию электронных книг так же умело, как и их поиск.Щелкните ссылку, чтобы найти ответы на билет для выхода из урока 39. Шаг 6. Каждый образец ответа 7 и 3 умножается на 2, 3 и 4 соответственно. 2 Практика A Ответы 10 2020 Live www. Помощь с домашним заданием по математике. Когда людям следует искать в магазинах электронных книг. На каждом листе есть наглядные пособия по моделированию задач и множество практических задач. Учебное пособие и ключевые геометрические параметры. Могу ли я изменить ответы Источник рез. Это ключ с полным ответом, чтобы найти все ответы на ваш последний тест.Загрузите бумагу с вопросами о воротах и. pdf БЕСПЛАТНО PDF ЗАГРУЗИТЬ 722 000 РЕЗУЛЬТАТОВ В любое время ISBN 07 865869 1 Подготовка к экзамену по алгебре Северной Каролины 2 Практика по окончанию курса и образец рабочей тетради для тестирования Ответ на вопрос пропустите и вернитесь к этому вопросу позже. Это 39 — простой ответ. Учебная тетрадь по алгебре 2 McGraw Hill Education 2011 04 12 Руководство по ведению заметок для каждого урока Ключевые понятия Усиление словарного запаса Переосмысление домашнего задания Кэти Ваттеротт 2018 09 25 В этом обновленном выпуске Кэти Ваттеротт исследует роль, которую домашнее задание сыграло в культуре школьного обучения по сравнению с Советом по среднему образованию BSE выпустила ключ ответа для OTET теста на соответствие требованиям учителей Odisha для работы 1 и работы 2.2 HSN. Положительная связь между двумя переменными. Решения для Core Connections Algebra 2 Опубликовано 2 февраля ДА Теперь пришло время переосмыслить свое истинное «я», используя бесплатные ответы Core Connections Algebra 2 от Slader 39. Решите задачу по алгебре шаг за шагом. Мы можем представить натуральные числа на одномерной числовой прямой. 4. Алгебра IMath имеет смысл 6Core Plus MathematicsGo Math 4 класс Алгебра 1 Рабочая тетрадь Ответ Ключевая математика II quot Текст подходит для типичного вводного курса алгебры и был разработан для гибкого использования.00 00 00 ч. Мин. Сек. Идти по математике 5 класс домашнее задание клавиша ответа Bing. Пройдите математический ответ для 4 класса Создайте или найдите развивающий. Студенты, изучающие алгебру 2, развивают свои знания по алгебре 1 и геометрии. День 8. Ответ на разминку. Подавляющее большинство классных и домашних заданий будет приходить с этих страниц, составляющих учебник по математике для 6-го класса. 1. День 6 Практика трансформации WKS. перейти формирующий ответ ключевой алгебры 2

    Граймс и Илон Маск X Æ A-12 (и 8 других необычных детских имен знаменитостей)

    Стивен МакИнтош
    Репортер развлекательных программ

    image copyright Getty Images

    При выборе пароля обычно рекомендуется использовать один верхний регистр буква, необычный символ и цифра.

    Певица Граймс и технический миллиардер Илон Маск пошли еще дальше и применили эти правила к выбору имени для ребенка.

    Пара, очевидно, назвала своего первого ребенка X X A-12 Musk.

    Маск раскрыл имя в Твиттере в среду, хотя никто не совсем уверен, серьезно ли он говорит или просто троллит нас всех.

    Если это правда, то не совсем понятно, как это следует произносить, да и вообще что это значит. С тех пор Граймс попытался объяснить.

    • X, неизвестная переменная ⚔️
    • Æ, мое эльфийское написание Ai (любовь и / или искусственный интеллект)
    • A-12 = предшественник SR-17 (наш любимый самолет). Ни оружия, ни защиты, только скорость. Великий в бою, но ненасильственный 🤍
    +
    (A = Архангел, моя любимая песня)
    (⚔️🐁 metal rat)

    — ꧁ ༒ Gℜiꪔ⃕es ༒ ꧂ 🍓🐉🎀 小 仙女 (@Grimezsz) 6 мая 2020 г.
    BBC не несет ответственности за содержание внешних сайтов. Посмотрите оригинальный твит в Twitter.

    Граймс и Маск, возможно, выиграли ожесточенную битву за звание звездной пары с самым необычным именем для своего ребенка, но у них много конкурентов.

    Вот еще восемь примеров.

    1. Apple — Крис Мартин и Гвинет Пэлтроу. Пэлтроу.

    У пары, которая «сознательно рассталась» в 2014 году, есть 15-летняя дочь по имени Apple.

    Имя заставило Робби Уильямса немного словесно запутаться во время одной встречи с семьей, как он сказал Грэму Нортону в 2012 году.

    Во время вечеринки, которую он устраивал, певец спросил Пэлтроу в буфете: «Дыня хочет яблок?»

    2. Синий Плющ — Джей Зи и Бейонсе

    копирайт на изображение Getty Images

    Бейонсе и Джей Зи попали в заголовки газет в 2012 году, когда они назвали свою первую дочь Блю Айви Картер.

    Они даже пытались зарегистрировать торговую марку, но права уже были предоставлены бостонской свадебной фирме Blue Ivy Events.

    Существует теория, что Айви была выбрана из-за любви пары к числу четыре, которое римскими цифрами является IV.

    Синий был на самом деле более распространенным именем на одном этапе — известные Блюзы включают автора Блю Баллиетта, родившегося в 1955 году, и игрока НБА Блю Эдвардса, родившегося в 1965 году, хотя оба являются прозвищами.

    И хотя Бейонсе и Джей-Зи, возможно, дали этому имени новую жизнь, они не были первой звездной парой, которая выбрала яркое имя для своего отпрыска.

    Эдж из U2 выбрал имя Голубой ангел для своей дочери, которой сейчас 31 год.

    3. Бруклин — Дэвид и Виктория Бекхэм

    image copyright PA

    «Вы знаете, кто дает детям дурную репутацию? Posh and Becks.»

    Эта шутка принесла комику Стюарту Фрэнсису титул самой смешной шутки на Эдинбургском краю в 2012 году.

    И вы можете понять его точку зрения — необычный выбор пары включал Бруклина, Ромео и Круза для их сыновей и Харпер Семь для их дочери.

    Бруклин, в частности, был выделен Кэти Хопкинс в качестве примера детского имени, которое ей действительно не нравилось в своем знаменитом интервью This Morning на эту тему в 2013 году.

    4. Милан — Шакира и Жерар Пике

    image copyrightGetty Images

    Бруклин — далеко не единственный ребенок-знаменитость, названный в честь географического положения.

    Мы были бы здесь весь день, если бы начали перечислять знаменитостей, которые называют своих детей в честь стран и столиц, поэтому давайте просто выделим пару примеров.

    Шакира родила первого из двух своих сыновей с игроком ФК «Барселона» Жераром Пике в 2013 году, которого пара назвала Миланом.

    Еще есть дочь Майкла Джексона Пэрис, дочь Боно Мемфис, дочь Алека Болдуина и Ким Бейсингер Ирландия и сын Алисии Киз Египет.

    Рамсгейт все еще доступен, если все еще будут наготове какие-то звезды.

    5. Север — Ким Кардашьян и Канье Уэст

    image copyright Reuters

    Ким Кардашьян и Канье Уэст родили своего первого ребенка в 2013 году.

    Поклонники месяцами размышляли о том, как пара назовет своего первенца, имея в виду, что ребенок, вероятно, взял бы фамилию Канье.

    Эта пара произнесла имя лучше, чем кто-либо мог надеяться — Норт. Полное имя их дочери — Северо-Запад.

    Новости об этом имени особенно приветствовались жителями Ланкашира.

    Мать Кардашьян Крис Дженнер сказала после родов: «То, как Ким объяснила мне, что это Север, означает« высшая сила », и она говорит, что Север — высшая точка [Ким и Канье] вместе. И я подумал, что это было действительно мило. »

    Уэсты продолжили это, назвав своих следующих троих детей Святыми, Чикаго и Псалмом.

    6. Медведь — Лиам Пейн и Шерил

    Медведь, возможно, не слишком необычный, но это, безусловно, имя, которое вы помните.

    И, как объяснил Лиам в 2017 году, именно поэтому он и Шерил выбрали его, хотя и не без небольших споров.

    «Я хотел более традиционное имя, а она хотела более необычное», — сказал он.

    «И причина, по которой она выбрала Медведя, в конце концов, заключалась в том, что Медведь — это имя, которое, когда вы выходите из комнаты, вы не забудете. И мне это нравится».

    Возможно, помогает и то, что Лиам является большим поклонником Беара Гриллса, которого он назвал «боссом» в Твиттере после рождения.

    7. Принцесса Тиаами — Кэти Прайс и Питер Андре

    Принцесса может быть термином нежности, который многие родители используют для своей дочери, но немногие выбирают его как настоящее имя.

    Это не помешало Питеру Андре и Кэти Прайс дать имя своей дочери в 2007 году.

    Но, возможно, еще более необычным является вторая часть ее имени — Тиамии.

    Питер и Кэти придумали это имя, объединив имена своих двух матерей — Теи и Эми.

    Полный балл за творчество, хотя в 2014 году Пит раскрыл, что их дочь сказала, что ей не нравится ее имя.

    8. Река Ракета — Джейми и Джулс Оливер

    изображение авторских правGetty Images

    Джейми Оливер.

    Онлайн экспорт ворд в пдф онлайн: Word в PDF — конвертируйте DOC в PDF бесплатно и онлайн

    Сохранение или конвертация файлов в формат PDF или XPS в классической версии Project

    Чтобы экспортировать или сохранить файл Office в формате PDF, откройте его и в меню Файл выберите пункт Экспорт или Сохранить как. Чтобы просмотреть пошаговые инструкции, выберите приложение Office в раскрывающемся списке.

    1. Откройте таблицу или отчет, которые требуется опубликовать в формате PDF.

    2. На вкладке Внешние данные в группе Экспорт нажмите кнопку PDF или XPS.

    3. В поле Имя файла введите или выберите имя документа.

    4. В списке Тип файла выберите PDF.

      • Если требуется высокое качество печати документа, установите переключатель в положение Стандартная (публикация в Интернете и печать).

      • Если качество печати не так важно, как размер файла, установите переключатель в положение Минимальный размер (публикация в Интернете).

    5. Нажмите кнопку Параметры, чтобы выбрать страницы для печати, указать, должна ли печататься разметка, а также выбрать параметры вывода. Нажмите кнопку ОК.

    6. Нажмите кнопку Опубликовать.

    Эти сведения также относятся к Microsoft Excel Starter 2010.

    Примечание: Вы не можете сохранять листы Power View как PDF-файлы.

    1. Откройте вкладку Файл.

    2. Выберите команду Сохранить как.
      Чтобы диалоговое окно «Сохранить как» Excel 2013 или Excel 2016, необходимо выбрать расположение и папку.

    3. В поле Имя файла введите имя файла, если оно еще не присвоено.

    4. В списке Тип файла выберите PDF.

      • Если файл требуется открыть в выбранном формате после его сохранения, установите флажок Открыть файл после публикации.

      • Если необходимо высокое качество печати документа, установите переключатель в положение Стандартная (публикация в Интернете и печать).

      • Если качество печати не так важно, как размер файла, установите переключатель в положение Минимальный размер (публикация в Интернете).

    5. Нажмите кнопку Параметры, чтобы выбрать страницы для печати, указать, должна ли печататься разметка, а также выбрать параметры вывода. Подробную информацию о диалоговом окне «Параметры» в Excel см. в статье Дополнительные сведения о вариантах создания PDF. По завершении нажмите кнопку ОК.

    6. Нажмите кнопку Сохранить.

    OneNote 2013 и OneNote 2016

    1. Откройте вкладку Файл.

    2. Нажмите кнопку Экспорт.

    3. В разделе Экспорт текущего элемента выберите часть записной книжки, которую необходимо сохранить в формате PDF.

    4. В разделе Выбор формата выберите пункт PDF (*.pdf) и нажмите кнопку Экспорт.

    5. В диалоговом окне Сохранить как в поле Имя файла введите название записной книжки.

    6. Нажмите кнопку Сохранить.

    OneNote 2010

    1. Откройте вкладку Файл.

    2. Выберите команду Сохранить как и выберите параметр, соответствующий части записной книжки, которую необходимо сохранить в формате PDF.

    3. В разделе Сохранить раздел как выберите пункт PDF и нажмите кнопку Сохранить как.

    4. В поле Имя файла введите имя для записной книжки.

    5. Нажмите кнопку Сохранить.

    1. Откройте вкладку Файл.

    2. Выберите команду Сохранить как.
      Чтобы диалоговое окно «Сохранить как» в PowerPoint 2013 и PowerPoint 2016, необходимо выбрать расположение и папку.

    3. В поле Имя файла введите имя файла, если оно еще не присвоено.

    4. В списке Тип файла выберите PDF.

      • Если файл требуется открыть в выбранном формате после его сохранения, установите флажок Открыть файл после публикации.

      • Если необходимо высокое качество печати документа, установите переключатель в положение Стандартная (публикация в Интернете и печать).

      • Если качество печати не так важно, как размер файла, установите переключатель в положение Минимальный размер (публикация в Интернете).

    5. Нажмите кнопку Параметры, чтобы выбрать страницы для печати, указать, должна ли печататься разметка, а также выбрать параметры вывода. По завершении нажмите кнопку ОК.

    6. Нажмите кнопку Сохранить.

    1. На вкладке Файл выберите команду Сохранить как.
      Чтобы диалоговое окно «Сохранить как» Project 2013 или Project 2016, необходимо выбрать расположение и папку.

    2. В поле Имя файла введите имя файла, если оно еще не присвоено.

    3. В списке Тип файла выберите PDF-файлы (*.pdf) или XPS-файлы (*.xps) и нажмите кнопку Сохранить.

    4. В диалоговом окне Параметры экспорта документа укажите в пункте Диапазон публикации, следует ли Включить непечатаемые данные или использовать Совместимость с ISO 19500-1 (только для PDF).

    Советы по форматированию

    Приложение Project не поддерживает все возможные функции форматирования документов PDF или XPS, но с помощью некоторых параметров печати вы можете изменять вид конечного документа.

    На вкладке Файл выберите пункт Печать, а затем измените любой из следующих параметров:

    В диалоговом окне Параметры страницы вы можете изменить параметры на таких вкладках:

    • «Поля»,

    • «Легенда»,

    • «Вид».

    1. Откройте вкладку Файл.

    2. Выберите команду Сохранить как.
      Чтобы диалоговое окно «Сохранить как» в Publisher 2013 или Publisher 2016, необходимо выбрать расположение и папку.

    3. В поле Имя файла введите имя файла, если оно еще не присвоено.

    4. В списке Тип файла выберите PDF.

    5. Если необходимо изменить способ оптимизации документа, выберите команду Изменить. (Щелкните Параметры в Publisher 2013 или Publisher 2016).

      • Внесите все необходимые изменения в разрешение изображения и непечатаемые сведения.

      • Чтобы изменить параметры печати документа, выберите Параметры печати.

      • По завершении нажмите кнопку ОК.

    6. Если после сохранения файл требуется открыть в выбранном формате, установите флажок Открыть файл после публикации.

    7. Нажмите кнопку Сохранить.

    1. Откройте вкладку Файл.

    2. Выберите команду Сохранить как.
      Чтобы диалоговое окно «Сохранить как» в Visio 2013 или Visio 2016, необходимо выбрать расположение и папку.

    3. В поле Имя файла введите имя файла, если оно еще не присвоено.

    4. В списке Тип файла выберите PDF.

      • Если файл требуется открыть в выбранном формате после его сохранения, установите флажок Автоматический просмотр файла после сохранения.

      • Если необходимо высокое качество печати документа, установите переключатель в положение Стандартная (публикация в Интернете и печать).

      • Если качество печати не так важно, как размер файла, установите переключатель в положение Минимальный размер (публикация в Интернете).

    5. Нажмите кнопку Параметры, чтобы выбрать страницы для печати, указать, должна ли печататься разметка, а также выбрать параметры вывода. Нажмите кнопку ОК.

    6. Нажмите кнопку Сохранить.

    Word 2013 и более новые

    1. Выберите Файл > Экспорт > Создать PDF/XPS.

    2. Если свойства документа Word содержат информацию, которую вы не хотите включать в PDF-файл, в окне Опубликовать как PDF или XPS нажмите кнопку Параметры. Затем выберите пункт Документ и снимите флажок Свойства документа. Задайте другие нужные параметры и нажмите кнопку ОК.

      Дополнительные сведения о свойствах документа см. в разделе Просмотр или изменение свойств файла Office 2016.

    3. В окне Опубликовать как PDF или XPS выберите место, где нужно сохранить файл. При необходимости измените имя файла.

    4. Нажмите кнопку Опубликовать.

    Дополнительные сведения о вариантах создания PDF

    • Чтобы преобразовать в формат PDF только некоторые страницы, укажите их в полях Страницы.

    • Чтобы включить исправления в PDF, в разделе Опубликовать установите переключатель в положение Документ с исправлениями. В противном случае убедитесь установите переключатель в положение Документ.

    • Чтобы создать набор закладок в PDF-файле, установите флажок Создать закладки, используя. Затем установите переключатель Заголовки или, если вы добавили закладки в свой документ, Закладки Word.

    • Если вы хотите включить в PDF-файл свойства документа, убедитесь в том, что флажок Свойства документа установлен.

    • Чтобы сделать документ удобней для чтения в программах чтения с экрана, установите флажок Теги структуры документа для улучшения восприятия.

    • Совместимость с ISO 19005-1 (PDF/A). Этот параметр предписывает создать PDF-документ, используя стандарт архивации 1.7 PDF. Стандарт PDF/A позволяет гарантировать, что при открытии на другом компьютере документ будет выглядеть точно так же.

    • Преобразовать текст в точечный рисунок, если невозможно внедрить шрифты. Если невозможно внедрить шрифты в документ, при создании PDF-файла используется точечный рисунок текста, чтобы PDF-документ выглядел так же, как оригинальный. Если этот параметр не выбран и в файле используется невстраиваемый шрифт, программа чтения PDF-файлов может применить другой шрифт.

    • Зашифровать документ с помощью пароля. Выберите этот параметр, чтобы ограничить доступ к PDF-файлу людям, у которых нет пароля. Когда вы нажмете кнопку ОК, Word откроет диалоговое окно Шифрование документа в формате PDF, в котором вы можете ввести пароль и его подтверждение.

    Открытие PDF-файла в Word и копирование содержимого из него

    Вы можете скопировать из PDF-документа нужное содержимое, открыв его в Word.

    Выберите Файл > Открыть и найдите PDF-файл. Word откроет PDF в новом файле. Вы можете скопировать нужное содержимое, включая изображения и схемы.

    Word 2010

    Эти сведения также относятся к Microsoft Word Starter 2010.

    1. Откройте вкладку Файл.

    2. Выберите команду Сохранить как.
      Чтобы диалоговое окно «Сохранить как» в Word 2013 и Word 2016, необходимо выбрать расположение и папку.

    3. В поле Имя файла введите имя файла, если оно еще не присвоено.

    4. В списке Тип файла выберите PDF.

      • Если файл требуется открыть в выбранном формате после его сохранения, установите флажок Открыть файл после публикации.

      • Если необходимо высокое качество печати документа, установите переключатель в положение Стандартная (публикация в Интернете и печать).

      • Если качество печати не так важно, как размер файла, установите переключатель в положение Минимальный размер (публикация в Интернете).

    5. Нажмите кнопку Параметры, чтобы выбрать страницы для печати, указать, должна ли печататься разметка, а также выбрать параметры вывода. По завершении нажмите кнопку ОК.

    6. Нажмите кнопку Сохранить.

    Чтобы сохранить файл в формате PDF в Office для Mac, выполните эти простые действия:

    1. Откройте вкладку Файл.

    2. Нажмите кнопку Сохранить как.

    3. Щелкните Формат файла в нижней части окна.

    4. Выберите PDF в списке доступных форматов.

    5. Присвойте файлу имя, если оно еще не указано, а затем нажмите кнопку Экспорт.

    С помощью Word, PowerPoint и OneNote в Интернете можно преобразовать документ в формат PDF.

    1. Выберите «Файл>печатать > «Печать» (в PowerPoint выбран один из трех форматов).

    2. В меню «Принтер» выберите пункт «Сохранитькак PDF», а затем — «Сохранить».

    3. Затем в открываемом меню проводника вы можете назвать PDF-файл, выбрать, где его сохранить, а затем выбрать «Сохранить».

    При этом приложение создаст обычный PDF-файл, в котором будут сохранены макет и форматирование исходного документа.

    Если вам нужны дополнительные параметры для управления PDF-файлом, например возможность добавлять закладки, преобразуйте документ в формат PDF с помощью настольного приложения. Нажмите кнопку «Открыть в настольном приложении» на панели инструментов PowerPoint и OneNote, чтобы начать работу с классическим приложением, а затем в Word выберите в этом приложении dropdown и нажмите кнопку «Открыть в настольном приложении».

    Если у вас нет настольного приложения, вы можете попробовать или купить последнюю версию Office прямо сейчас.

    У вас есть предложения для этой возможности?

    Голосуйте за понравившиеся идеи или предлагайте свои в копилке идей на сайте word.uservoice.com.

    Чтобы экспортировать документ Word или книгу Excel в файл формата PDF на устройстве с iOS, нажмите в левом верхнем углу кнопку Файл и выберите пункт Экспорт, а затем — PDF.

    Как перевести Word в PDF

    Текстовый редактор от компании Microsoft под названием MS WORD по умолчанию сохраняет все документы в формате DOCX. Он идеально подходит для продуктов от Майкрософт. Но если нужно открыть документ в другом редакторе? Что делать тогда?

    Если пользователь стремится сохранить правильное форматирование и другие особенности документа, то сохранять его нужно в каком-нибудь кроссплатформенном формате. То есть в том, который не искажает контент при открытии в других редакторах или на других ОС.

    К таким относится формат PDF. Он специально создан для хранения текстовой и графической информации и ее последующего использования в других рабочих средах. Чаще требуется перевести формат ПДФ в Ворд, но иногда нужна и обратная операция. Как конвертировать DOC в PDF рассмотрим в данной статье.

    Использование Microsoft WORD

    Текстовый редактор от компании Майкрософт неплохо справляется с этой задачей. Он может без проблем конвертировать DOC в PDF и обратно. Но на это способны далеко не все версии программы.

    Хорошо справляются с такой задачей редакторы 2007, 2010, 2013 и 2016 годов выпуска. Если же под рукой WORD более старой версии, то ничего не получится. Для программы 2013 года алгоритм действий получается таким.

    1. Открываем нужный элемент и нажимаем кнопку «Файл».
    2. Теперь выбираем пункт «Экспорт» (1) и кликаем по кнопке «Создать PDF/XPS» (2).
    3. Теперь выбираем каталог для сохранения файла (1), присваиваем ему имя (2) и жмем кнопку «Сохранить» (3).

    Точно так же можно переводить DOCX в ПДФ в MS WORD 2016. А вот в версиях 2007 и 2010 года используется для этого пункт «Сохранить как». В остальном же процессы идентичны. Остальное выполняется по такому же сценарию, который очень подробно описан в видео.

    × Если у вас на компьютере установлен пакет Microsoft Office предыдущей версии, то есть смысл обратиться к другим (бесплатным) текстовым редакторам. Они тоже на такое способны.

    Использование альтернативных текстовых редакторов

    MS WORD, конечно, мощная и продвинутая программа. Но далеко не у каждого есть лишних 200-300 долларов для того, чтобы оплатить подписку на офисный пакет. К счастью, есть бесплатные альтернативы.

    Самыми продвинутыми в этом плане считаются офисные пакеты LibreOffice и OpenOffice. org. Это почти точные копии классического Офиса от Майкрософт (кроме интерфейса) и тоже способны конвертировать ВОРД в ПДФ и обратно.

    LibreOffice

    Это свободный пакет, включающий в себя необходимые инструменты для офисной работы. Здесь есть и Writer (аналог Ворда), и Calculate (аналог Excel), а также многое другое. Проект использует открытую лицензию типа GNU GPL и имеет открытый исходный код.

    Главное отличие этого офисного пакета от того, что предлагает Майкрософт – кроссплатформенность. Программы можно легко использовать на Linux, Windows и Mac OS. А теперь поговорим о том, как преобразовывать DOCX в PDF при помощи LibreOffice.

    1. Сначала открываем нужный документ в редакторе, а затем жмем на кнопку «Файл».

    1. Теперь выбираем в контекстом меню пункт «Экспорт в» (1) и кликаем на «Экспорт в PDF».
    2. Далее нажимаем на кнопку «Экспорт».

    Вот так происходит преобразование WORD в ПДФ при помощи офисного пакета LibreOffice. Софт подходит тем, кто не хочет использовать пиратский софт и нет возможности платить за лицензионный Microsoft Office.

    × Офисный пакет умеет не только конвертировать из одного формата в другой, но и прекрасно себя показывает при повседневной работе. Проблем с открытием документов, созданных в LibreOffice в других редакторах нет.

    OpenOffice.org

    Еще один свободный офисные пакет, распространяемый под лицензией GPL. Обладает открытым исходным кодом и теми же функциями, что и классическое приложение Office от Майкрософт.

    Что касается возможности сохранять документы в PDF, то с этим тоже все в порядке. Приложение справляется с этой работой. Да и сам процесс не сложен. Для конвертации нужно сделать следующее.

    1. Сначала открываем нужный документ и затем жмем кнопку «Файл» в верхней части окна.
    2. Затем в контекстном меню выбираем «Экспорт в PDF».
    3. Далее нажимаем на кнопку «Экспорт» и процесс начнется немедленно. Завершится он тоже автоматически.

    Сохраненный файл находится в системном каталоге «Документы».

    Офисный пакет OpenOffice.org превосходно справляется и с другой работой. К примеру, он отлично делает презентации и хорошо работает с таблицами. Еще один плюс этого пакета – кроссплатформенность.

    × Он способен работать под любой ОС. И файлы, выполненные в нем легко открываются в других офисных программах. Прекрасный продукт для тех, кто любит свободный софт.

    Использование онлайн-сервисов

    В настоящее время в Сети много сайтов, которые предоставляют услуги по конвертированию документов из одного формата в другой. Причем спектр поддерживаемых расширений необычайно широк.

    Среди них есть как платные сервисы с функцией подписки, так и бесплатные варианты. И рассмотрим мы последние, так как обычно перевести онлайн ВОРД в ПДФ требуется разово. И нет смысла за это платить.

    SmallPDF

    Ссылка: https://smallpdf.com

    Англоязычный сервис, позволяющий конвертировать документы разных форматов. Причем предоставляет такую услугу бесплатно. Сайт отличается приятным и современным интерфейсом.

    Недостаток — полное отсутствие русского языка. Зато сервис работает быстро и может не только преобразовывать PDF в другие форматы, но и разблокировать содержимое таких файлов для последующего редактирования. Работать с сервисом нужно так.

    1. На главной странице щелкаем кнопку с надписью «Word to PDF».
    2. Далее жмем на кнопку с надписью «Choose File».
    3. Теперь выбираем нужный файл на компьютере (1) и нажимаем кнопку «Открыть» (2).
    4. Остается только нажать на кнопку «Download File» для того, чтобы скачать файл на компьютер.

    Конвертирование завершено успешно. Стоит отметить, что этот онлайн-сервис хорошо справляется и с проблемными файлами. Если на других площадках есть проблемы с загрузкой таких документов, то здесь все хорошо.

    × SmallPDF – это бесплатный сервис, который не имеет ограничения на объем загружаемого файла. Да и сомнительно, чтобы документ MS WORD имел объем, измеряемый в гигабайтах.

    DOC2PDF

    Ссылка: https://www.doc2pdf.com

    Сервис, который предоставляет много инструментов для работы с документами ПДФ. Их можно объединять, разделять, сжимать и преобразовывать. Также онлайн-конвертер умеет перегонять WORD в редактируемый ПДФ.

    Сервис полностью бесплатен, но есть ограничение по размеру разово загружаемых файлов. Объем не должен превышать 25 мегабайт. Зато у платформы есть адекватный русский язык. А пользоваться ею очень просто.

    1. На главной странице сервиса жмем кнопку «Выберите файл».
    2. Затем выбираем на ПК нужный документ (1) и открываем его (2).
    3. Следующий шаг: нажимаем кнопку «Просмотр и загрузка в браузере».

    Документ сразу же скачается в каталог загрузок и одновременно откроется в браузере для просмотра. Очень удобная опция. Кстати, данный сервис поддерживает преобразование различных форматов в PDF. Одним DOCX дело не ограничивается.

    × DOC2PDF подойдет тем, кому нужна разовая конвертация. И использовать эту платформу можно только в том случае, если размер конвертируемого файла не превышает 25 МБ. Во всем прочем никаких ограничений нет.

    PDFCandy

    Ссылка: https://pdfcandy.com/ru/docx-to-pdf.html

    Хорошая платформа для работы с документами формата ПДФ. Умеет конвертировать данные файлы в различных направлениях и разрешать их редактирование. Сервис отлично подойдет тем, кто часто работает с конвертацией, так как у него нет вообще никаких лимитов.

    В активе платформы имеется русский язык, интуитивно понятный интерфейс и приятное оформление.

    1. На главной странице нужно нажать зеленую кнопку с надписью «Добавьте файл(ы)».

    1. Затем выберите нужный на компьютере или ноутбуке (1) и нажмите «Открыть» (2).
    2. А теперь жмем на зеленую кнопку с надписью «Скачать файл».

    По завершении последнего действия документ переместится в каталог «Загрузки», что находится на системном диске. После этого можно приступать к редактированию файла или его открытию в специальной программе.

    × PDFCandy – это один из немногих бесплатных сервисов-конвертеров, которые не имеют никаких ограничений. Потому и процент посещений на этом сайте невероятно высок. А еще на руку разработчикам играет приятный интерфейс с русским языком.

    Использование виртуальных принтеров

    Виртуальные принтеры – это такие программы, которые умеют преобразовывать любой документ так, чтобы он выглядел как напечатанный или версия для печати. Для реализации используется именно формат PDF, так как он может передать мельчайшие особенности бумаги.

    Среди таких приложений есть как простенькие бесплатные утилиты с базовым набором функций, так и настоящие студии, способные на многое. Только последние, как правило, являются платными. Рассмотрим самые интересные варианты.

    Universal Viewer Free

    Ссылка: http://www.uvviewsoft.com/uviewer/download.htm

    Простейшее приложение, которое способно «распечатать» любой документ. Функционала у данной программы не особо много, но со своей прямой обязанностью она справляется на отлично. Также у этой утилиты есть свои преимущества.

    К примеру, программа обладает очень простым и понятным интерфейсом без всяких ненужных элементов. А еще у продукта есть полноценный русский язык, что облегчает отечественным пользователям работу с ней. А перевести DOCX в PDF с ее помощью можно так.

    1. Запускаем программу и в главном окне нажимаем кнопку «Открыть».
    2. Выбираем нужный документ (1) и нажимаем «Открыть» (2).
    3. Теперь нажимаем «Файл» (1) и выбираем «Печать» (2).
    4. Теперь выбираем тип «принтера» (1) и жмем «ОК».

    Программа сохранит новый файл туда же, где находится оригинал. Данная утилита позволяет сохранять любые файлы в формате PDF. Но лучше всего она справляется все-таки именно с DOCX. Такое ощущение, что именно для этого сия утилита и создана.

    × Скачать программу совершенно бесплатно можно на официальном сайте разработчика. Там есть две версии. Нам нужна та, что называется FREE (бесплатная).

    DoPDF

    Ссылка: http://www.dopdf.com/ru/download.php

    Еще один бесплатный виртуальный принтер, способный конвертировать классический документ WORD в кроссплатформенный PDF. Эта программа обладает неплохим функционалом и отлично справляется со своей работой.

    Интерфейс прост и интуитивно понятен. Есть также русский язык. Но он не совсем корректный: не все переведено. Тем не менее, работать с продуктом можно без проблем. Только запускать утилиту нужно от имени администратора. А для конвертирования нужно сделать следующее.

    1. Запускаем программу и в главном окне жмем кнопку «Перейти». Настройки оставляем по умолчанию. 
    2. Далее идем в нужный каталог (1), выбираем документ (2) и нажимаем на кнопку «Открыть» (3).

    1. Теперь нажимаем на кнопку «Создать».
    2. Далее программа предложит сохранить созданный файл. Выбираем нужный каталог (1), называем документ (2) и нажимаем «Сохранить» (3).

    Процесс конвертирования из одного формата в другой завершен. Теперь файл находится именно в том каталоге, который вы выбрали для сохранения. Работает утилита довольно быстро. Однако скорость зависит от объема документа.

    × Виртуальные принтеры типа DoPDF используют мало ресурсов компьютера. И это делает именно эту программу наиболее предпочтительной для домашнего использования. Скачать ее можно совершенно бесплатно на сайте производителя.

    iPDF24 Creator

    Ссылка: https://ru.pdf24.org/virtual-pdf-printer.jsp

    Небольшая полностью бесплатная программа с функцией виртуального принтера. Помимо этого, утилита умеет конвертировать, сжимать, разблокировать для редактирования и вообще всячески работать с файлами PDF.

    Программа обладает приятным интерфейсом и очень проста в использовании. Есть даже полноценный русский язык. В приложении есть ненавязчивая реклама, но она не предлагает ничего купить. А пользоваться утилитой нужно так.

    1. В главном окне нажимаем кнопку «Настройка PDF принтера».
    2. Далее нажимаем на кнопку с изображением плюса и добавляем нужный документ.
    3. После загрузки файла нажимаем на «Печать».
    4. В следующем окошке тоже нажимаем «Печать».

    После этого программа предложит сохранить получившийся файл. Это происходит по стандартному сценарию. Вообще, у PDF24 масса преимуществ. Но главное заключается в том, что утилита работает очень быстро и почти не нагружает ПК.

    × А еще она постоянно висит в системном трее и всегда готова к работе. Скачать программу можно совершенно бесплатно с официального сайта разработчика. Установка очень простая. Ни у кого не возникнет вопросов в процессе.

    Использование специализированных программ

    Под специализированными программами понимаются специальные конвертеры, которые позволяют переводить документ в различные форматы.

    Многие из них платные, но есть и такие, что распространяются под свободной лицензией. Именно они и являются наиболее интересными. Хоть и не обладают таким богатым функционалом, как платные аналоги. Рассмтрим наиболее популярные программы.

    PDF-XChange Editor

    Ссылка: https://www.tracker-software.com/product/pdf-xchange-lite

    Это не просто конвертер, а многофункциональный комбайн для проведения различных операций с файлами формата ПДФ. Программа способна сделать с документом все, что угодно.

    Утилита обладает приятным интерфейсом и проста в использовании. Есть также русский язык. Программа платная, но есть  бесплатная пробная версия. А сделать для преобразования нужно вот что.

    1. В главном окне программы нужно нажать на кнопку «Открыть». Она находится на самой верхней панели.
    2. Далее выбираем нужный каталог (1), отмечаем необходимый документ (2) и открываем его (3).
    3. После открытия необходимо щелкнуть кнопку «Файл» (1), выбрать там пункт «Сохранить как» (2) и щелкнуть по кнопке «Обзор» (3).

    Далее сохраняем файл по привычному сценарию. Он будет сохранен уже в формате ПДФ. Вот так просто можно перевести документы из одного формата в другой при помощи PDF-XChange Editor.

    Total PDF Converter

    Ссылка: https://www.coolutils.com/TotalPDFConverter

    Превосходная условно-бесплатная утилита, которая работает практически со всеми форматами документов и может конвертировать практически любые файлы. Программа обладает русским интерфейсом и очень проста в использовании.

    Вообще, она не является бесплатной, но даже триальной версии хватит практически для всех задач. Самое интересное, что программа практически не нагружает компьютер и работает очень быстро. Для конвертирования нужно сделать следующее.

    В главном окне утилиты выбираем нужный каталог (1), щелкаем по нужному элементу (2) и кликаем по иконке с подписью «PDF» (3).

    После этого программа предложит сохранить созданный элемент по знакомому сценарию уже в формате PDF. Все происходит очень быстро. И в этом главное преимущество данного приложения. Скачать продукт можно с официального сайта.

    Этот конвертер способен работать с различными форматами. При этом программа практически ничего не требует от компьютера и не нагружает ОС.

    Пакетное преобразование DOC (DOCX) в PDF

    Под этим подразумевается массовая конвертация документов WORD в формат ПДФ. Справиться с этим могут только специализированные программы. Обычному конвертеру (или онлайн-сервису) это не под силу. Сейчас мы рассмотрим несколько интересных программ.

    PDF Creator

    Ссылка: https://www.pdfforge.org/pdfcreator/download

    Платная программа, предназначенная для работы с форматом ПДФ. Она умеет конвертировать, распечатывать, редактировать и делать все остальное. Но главное – с ее помощью можно перевести большое количество файлов с расширением DOCX в PDF.

    Утилита имеет приятный интуитивно понятный интерфейс с русским языком. Она хорошо справляется с большими объемами данных и пригодится практически всем. Для конвертирования нужно сделать следующее.

    1. В главном окне нажимаем кнопку с надписью «Выберите файл для конвертации».
    2. Теперь выбираем нужный каталог (1), выделяем все документы, которые надо перевести в другой формат (2) и жмем «Открыть» (3).
    3. После загрузки всего требуется нажать на кнопку «Сохранить».

    Программа немедленно сохранит все новые элементы там же, где находятся оригиналы. Причем сделает это очень быстро. И в этом заключается главное ее преимущество: утилита быстро и четко работает с большими объемами данных.

    × Единственный недостаток PDF Creator заключается в том, что за полную версию приложения придется заплатить. Причем цена довольно высока. Но те, кто хочет получить качественный и многофункциональный продукт для работы с ПДФ не будут разочарованы.

    Как загрузить и работать в программе, можно увидеть в видео.

    DocuFreezer

    Ссылка: https://www. docufreezer.com/

    Условно-бесплатное приложение для конвертирования большого объема данных. Легко справляется со всеми форматами. А вот дополнительных функций (вроде опции редактирования) нет.

    Программа обладает разумно организованным и приятным интерфейсом. Вот только русского языка не хватает.  Для массовой конвертации при помощи этой программы нужно сделать следующее.

    1. В главном окне утилиты жмем на кнопку «Add Files».
    2. Далее выбираем нужную папку (1), отмечаем все необходимые документы (2) и открываем их (3).
    3. Теперь в окне программы выставляем необходимый формат (1), указываем путь для сохранения (2) и жмем на кнопку «Start» (3).

    Все остальное программный продукт сделает без участия пользователя. Новые элементы находятся в том каталоге, который вы указали в настройках. А сама конвертация происходит очень быстро. Высокая скорость – основная особенность утилиты.

    Все вышеперечисленные программы способны быстро и без проблем перевести формат WORD в ПДФ. То же самое относится и к онлайн-сервисам.

    Напишите, какими пользуетесь вы, пробовали ли описанные программы и способы, какие из них вам помогли?

    Насколько вам была полезна статья?

    Кликните по звездочке, чтобы оставить оценку!

    Submit Rating

    Средний рейтинг / 5. Оценок:

    Нам очень жаль, что статья вам не понравилась!

    Помогите нам ее улучшить!

    Скажите, как мы можем улучшить статью?

    Отправить ответ

    Спасибо за обратную связь!

    Как конвертировать Word в PDF на iPhone

    Существует немало ситуаций, в которых может потребоваться преобразовать документ Word в PDF на iPhone. Microsoft Word — один из наиболее распространенных текстовых редакторов. Но даже самые преданные пользователи данного формата используют наряду с встроенным программным обеспечением сторонние инструменты для преобразования документов Word в PDF на iPhone — например, при отправке информационных материалов или счетов.

    Как конвертировать Word в PDF с помощью встроенных функций iPhone

    Аппараты iPhone начиная с iOS 10 оснащены встроенным инструментом для преобразования PDF. Освоив эту функцией в iPhone или iPad, вы сможете быстро и удобно менять ваши Word-файлы.

    Вот как это работает:

    • Нажмите кнопку «Поделиться»
    • Нажмите на значок «Печать» в нижней части страницы общего доступа на устройстве с iOS.
    • Не обращайте внимания на меню принтера, которое появится сверху
    • Вместо этого обратите внимание на окно предварительного просмотра в нижней части экрана.
    • Чтобы преобразовать файл в PDF, нажмите и уменьшите эскиз окна предварительного просмотра.
    • Нажмите кнопку «Добавить общий доступ», чтобы сохранить, экспортировать или отправить кому-либо недавно созданный PDF
    • Все готово

    Сторонние приложения для преобразования Word в PDF на iPhone

    Если вам нужен более широкий функционал для преобразования документов Word в PDF на iPhone, вы можете попробовать любое из этих приложений — Word для iOS, PDF Converter и iBook.

    1. Microsoft Word

    В App Store можно получить бесплатное программное обеспечение Microsoft Word. Установив это приложение на свой iPhone или iPad, вы сможете конвертировать файлы формата Word в PDF. Если ваши документы в Word хранятся на персональном компьютере, вы можете отправить их на мобильное устройство iOS по электронной почте.


    2. PDF Converter Documents To PDF

    Это полностью бесплатное программное обеспечение для преобразования документов Microsoft Word в PDF. Кроме того, с его помощью можно преобразовывать в PDF файлы, фотографии, веб-страницы, электронные письма, вложения электронной почты, контакты и текстовые сообщения.

    Программное обеспечение позволяет делиться PDF-документами через iTunes, Dropbox, Google Drive, Box, электронную почту и по WiFi. Приложение отличается привлекательным интерфейсом и простотой в использовании. Также с его помощью можно отправлять документы в качестве вложений электронной почты.


    3. iBooks

    С помощью этой программы вы можете создавать, сохранять, отправлять и распечатывать PDF-документы на вашем iPhone или iPad. Его функционал позволяет преобразовывать в PDF ваши документы, созданные и сохраненные в формате Microsoft Word.

    Вы также можете использовать iBooks для просмотра PDF-вложений из электронных писем на вашем устройстве с iOS. Для этого необходимо выполнить следующие действия:

    • Откройте электронное письмо, которое содержит вложение в формате PDF
    • Нажмите на это вложение
    • Нажмите кнопку «Поделиться» в нижнем левом углу
    • Прокрутите раздел «Поделиться» и выберите «Копировать в iBooks».
    • Нажмите на эту опцию
    • Ваш PDF документ будет открыт в IBooks
    • При открытии ваших PDF-файлов в приложении копии документов автоматически сохраняются в iBooks на полке с PDF.


    PDFelement — это отличная программа для работы с PDF-файлами с впечатляющим функционалом. Он облегчает работу с PDF-документами и позволяет преобразовывать в этот формат файлы другого типа. С помощью приложения вы сможете быстро, доступно и безопасно создавать профессионально выглядящие PDF-файлы и формы. Приложение позволяет выполнять следующие операции:

    • Открытие, сохранение, печать и разметка PDF-документов
    • Автоматическое распознавание полей форм
    • Утверждение и подписывание документов в цифровом формате;
    • Продуманная функция распознавания шрифтов: можно быстро изменить шрифт, цвет, размер и другие элементы документа.
    • Разметка и аннотирование PDF-файлов: добавление текстовых полей, штампов, сносок, а также функция ластика. Добавление рукописных фрагментов и рисунков
    • — Выделение, подчеркивание или зачеркивание фрагментов текста.

    Чтобы преобразовать ваши документы Word в PDF, вы можете использовать соответствующую встроенную функцию вашего iPhone. Если вам нужны дополнительные функции, попробуйте сторонние приложения, включая Microsoft Word для iPhone, PDF Converter, iBooks и PDFelement для iOS.

    ВСТАВИТЬ PDF В WORD

    Существует множество путей, как вставить документ ПДФ в Ворд, среди которых сможете подобрать наиболее подходящий для вашего ПК.

    Как вставить PDF в Word — разные методы

    Представляем 9 вариантов, как вставить ПДФ в Ворд:

    • применение вставки объекта;
    • залить пдф, как статичную картинку;
    • задействование конверторов, чтобы вставить информацию или картинку;
    • вставка при помощи простых комбинаций клавиш;
    • заливка, как связанного объекта;
    • скопировать содержимое;
    • задействование Адоб;
    • способы для Мас;
    • гугл докс.

    Разберем каждый из вышеперечисленных методов и пошагово опишем ход действий.

    Используйте инструмент «Вставка объекта»

    Для этого выполняем следующие действия:

    • открываем программу Ворд → выбираем вкладку «Вставка» в окошке текста → жмем «объект»;
    • если процесс происходит в outlook, необходимо щелкнуть в середине элемента — к примеру, события в календаре, → затем — функция «Создать из файла»;
    • Клацаем «обзор» → тыкаем по нужному файлу ПДФ и открываем;
    • клац «ок».

    Иногда может стоять защита паролем. Пусть вас это не смущает. Разблокируйте.

    Заглавная страничка изменяемого формата засветиться в доке и станет частью после добавления. При внесении любых изменений в файле ПДФ, они не покажутся на документе Ворд, так как нет связи с исходником. 

    Если использовать «Вставку», можно изменять размер документа, его положение, границы и задать контроль обтекания текстовых данных вокруг файла.

    Вставьте PDF как статичное изображение

    Если вставлять PDF, как статичное изображение, в пдшке изменения никак не отразятся на вордовском объекте.

    Чтобы начать работу, потребуется инструмент, который сможет перенести выбранный тип формата в jpg. В случае необходимости конвертирования только одной страницы, можно воспользоваться встроенным инструментом под названием «Windows Snipping» или другим аналогом, как Snaglt. После чего нужно сохранить файл в jpg. Потом скопировать и перенести в Ворд.

    1. Запускаем «ворд офис» и направляем мышь на место, где будем вставлять изображение.  
    2. В меню нажимаем «вставить», → щелкаем «картинка» → загружаем → «вставить картинку» в окошко диалогов.
    3. Ищем, где находится сохраненный объект, и вставляем в документик. 

    Такой лайфхак годится для устаревших версий Ворда.

    Полезная статья: Что лучше, ноутбук или компьютер (ПК): 5 советов, как выбрать технику для своих нужд

    Используйте конвертер PDF, чтобы вставить PDF в Word как изображение

    Существует несколько методик вставки пдф, как картинки, при помощи прог для конвертирования. Рассмотрим действующие лайфхаки.

    Abbyy Finereader

    Пробная версия берет в обработку не более ста листов. А бесплатная версия функционирует только месяц после первичной загрузки. Прога довольно универсальна, так как принимает любой файл пдф, скан документа или изображение. 

    1. Для начала выделяем необходимый текст или картинку — можно выбрать автоматический или ручной режим. 
    2. Дальше — получаем уже готовый док в виде обычного текста.  
    3. В выскакивающем в начале «запуск»-окошке выбираем «картинка или пдф-файл в Майкрософт Ворд». 
    4. Дальше идет конвертирование в автоматическом режиме по страницам. Останется только исправить ошибки и сохранить в нужном формате.

    «Плюсы» Abbyy Finereader на поверхности: можно переделать любую картинку в текстовый формат. Abbyy функционирует с любыми объектами, которые отвергают другие приложения. Есть возможность ручным методом добавлять блоки на каждый лист.

    Редирис Про

    Пробное пользование составляет 10 дней и обрабатывает первые сто листов. Прога способна сканировать док с принтера. Благодаря тесной совместимости с Ворд, приложение также распознает различные символы и иероглифы. 

    Текст документа может быть на любом языке — это не помеха. Сохраниться можно в любом формате. Правда, некоторые ошибки придется исправлять вручную.

    Free PDF to Word Converter

    Достаточно простая и шустрая прога, чтобы вставить pdf в Word. При конвертировании не цепляет первоначальное форматирование. Прога спроектирована на иностранном языке, но разобраться нетрудно. 

    1. В первом окошке пишем «пдф», а во втором — «док» или «ворд». 
    2. Далее тыкаем на папку, в которой хотим видеть документ.

    Small ПДФ

    Благодаря этой проге, можно не только преобразовать пдф, но и воспользоваться функциями — сжатие, объединение, поворот, редактирование, конвертирование между разными форматами и т.д. Можно пользоваться с различных платформ: Виндовс, Андроид, Линукс и прочее.

    Для конвертации выбираем необходимый файл и перетаскиваем в программу. Затем ждем преобразования нового дока. Остается только сохранить и вуаля!

    Конвертер пдф

    Предоставляет возможность обработать две страницы, остальные — платно. Сервис сможет перевести пдф в любой тип объекта, и даже картинку.

    ZamZar

    Функционирует в режиме онлайн с разными видами форматов. Просто выполняем каждый шаг по очередности: 

    • выбор файла, → выбираем, в какой формат хотим конвертировать, → указываем электронную почту, → запускаем процесс конвертирования. 

    Есть возможность сделать обработку целого пакета документов. Результат конвертирования пользователь получает по почте.

    Convertio

    Возможность не только переделать формат, но и сжать, объединить документы. Чтобы выбрать файл, он необязательно должен находиться на компьютере, можно просто указать ссылку. Затем скачиваем готовый необходимый файл и радуемся.

    iLovePDF

    Довольно схож по функционалу с предыдущим сервисом.

    Как пользоваться:

    • открываем прогу;
    • заходим во вкладку «конвертировать пдф(шку)»;
    • затем «ворд в пдф»;
    • ждем окончания преобразования;
    • скачиваем.

    ПДФ.io

    Предоставляет возможность сжимать, объединять, конвертировать и др.

    Для конвертирования пдшки достаточно выполнить простые действия:

    • открываем ПДФ.io;
    • выбираем файл, который хотим переделать;
    • клацаем на формат, который нужно получить;
    • ждем, когда кончится преобразование;
    • скачиваем.

    PDF — DOC
    1. Необходимо запустить установку приложения на комп → после авторизации выбрать функцию «загрузить».
    2. Тыкнуть на необходимый файл в пдф формате, указав путь. 
    3. Пдф для конвертации должен выскочить внизу по левой стороне. 
    4. Клацаем «скачать» и сохраняем в нужную папку на ПК.
    Go4Convert

    Эту онлайн-программу необязательно устанавливать. 

    1. Вбиваем в поиск, → находим и открываем сайт. 
    2. В самом верху меню клацаем на вторую по счету закладку по левой стороне. Выбираем необходимый для конвертирования файл. 
    3. После тыкаем «запуск» и ждем. Должно появиться окошко с надписью «ваш документ в процессе обработки».  

    После конвертации нужно сохранить результат.

    First пдф

    Предоставляет возможность бесплатного использования в течение месяца и брать в оборот сто листов. Для начала необходимо установить прогу на компьютер. 

    1. После установки и запуска версии, тыкаем на функцию «добавить пдф» → выбираем нужный файл. 
    2. В настройках можно указать, сколько страниц требуют конвертации, и указать номера. 
    3. После выбора папки, куда будет сохранен готовый объект, клацаем «конвертировать». 
    4. Закончится процесс, и готовый док будет лежать по указанному для сохранения пути.

    На заметку: Корпорация Microsoft представила компьютер-моноблок Microsoft Surface Studio

    Вставьте текст из файла PDF в Word

    Возможно вставить только кусок информации из файла ПДФ в вордовскую программу. Для этого используют пункт «вставить объект» и помещают в Ворд. Такая вставка будет осуществлена без оригинального форматирования или графика. Поэтому несколько видоизменится.

    Чтобы вставить текст из ПДФ в Ворд, следуем следующему алгоритму:

    • запускаем программу Word → направляем мышку в место, где будет располагаться будущая перенесенная текстовая инфа;
    • в меню выбираем функцию «вставить» → переходим в окно «текст» и тыкаем на объект;
    • клацаем на функцию «текст из файла» → тыкаем на строчку «вставить файл», где выбираем необходимый док, из которого будем извлекать текстовую информацию → «вставить»;
    • после чего ПДФ файл будет загружен в Ворд.

    Для версий 2013 и 2016 года все проще. Грузим пдф в программу, которая автоматически переделывает под себя. Остается только скопировать и вставить. Благодаря функции PDF Reflow, такие версии самостоятельно извлекают содержимое и сохраняют данные макета.

    Вставьте PDF в Word как связанный объект

    Давайте разберем, в чем разница между объектами, которые внедрили или связали. Главное отличие — место, где хранится инфа и обновления после внедрения в Ворд. В файл помещают ссылку на док или копию. Это дает возможность вставлять файлы из любого приложения, которое сможет выполнить функцию связывания и внедрения.

    1. Доки, которые внедрили, после обработки станут частицей проги Ворд или почтового сообщения. После вставки могут потерять связь с исходником.
    2. Связанные доки способны меняться, если исходник обновляется. Данные будут находиться в исходном файле и не потеряются. В Ворде или целевом доке остаются только данные о нахождении исходника и показывается представление связанных данных. Объекты такого типа используют при важности размера.

    Для вставки документа, который связали, или встраивания уже готового объекта, есть особый способ, описанный в таблице:

    Этап

    Описание
    Подготовительный этап
    • Заходим в прогу, переходим в окно диалогов под наименованием «объект».  
    • Выбираем вкладку «создание из файла« и переходим в «обзор», что позволит отметить необходимый файл для вставки
    Связь с файлом Следующая задача — связать файл с исходником, не внося в Ворд или почтовое сообщение. Для этого ставим галочку напротив функции «Связь с файлом»
    Изменяем внешний вид Чтобы док показывался в форме значка, а не первоначальной страницы, необходимо поставить галочку напротив функции «в виде значка». 
     
    Если такой способ уже выбран автоматически можно подобрать другой значок, выбрав «замена значка»

    Чтобы точно понять, какой файл необходимо выбрать и в каком виде он будет отображаться, обратите внимание на раздел результатов, который может меняться от выбранной функции — ссылка на файл или отображать как значок.

    Читайте: Как поставить пароль на компьютер или ноутбук? Установка пароля на Windows 7, 8, 10

    Скопируйте файл PDF в Word

    Чтобы скопировать ПДФ объект и переместить в Ворд, достаточно использовать наборы клавиш Ctrl+C и Ctrl+V, но это не всегда может сработать по причинам:

    • настройки безопасности;
    • из-за других разрешений.

    Если не получилось обойти систему и использовать простой способ, есть и другие варианты. 

    Перед тем, как скопировать текст из пдф, стоит убедиться в отсутствии настроек безопасности, которые не позволяют копировать. Об этом могут сообщать следующие пункты: выделение серым цветом или затемнение.

    Пдф может защищаться паролем. Чтобы снять защиту, следует открыть при помощи указанной комбинации паролем и проверить настройки разрешения. Таким образом будет подтверждено разрешение, чтобы сделать копию текстовой инфы. Как это сделать — пошагово.

    1. Наводим курсор и клацаем кнопкой мышки по правой стороне.
    2. Заходим в свойства.
    3. Ищем функцию «безопасность», → проверяем поставленные разрешения на документ. Таким образом будет точно знать, позволено ли сделать копию текста.
    4. Открываем файл пдф в любой проге для воспроизведения формата и выделяем информацию для копирования.

    Есть несколько лайфхаков, как можно сделать копию содержимого пдф.

    1. Методом копирования-вставки. Используем клавиши Ctrl+C — для копии зажимаем одновременно. Чтобы вставить — Ctrl+V. Для владельцев Мас зажимаем Command+C и дальше Command+V.
    2. Тыкаем правую кнопку мыши или на сенсорной панели, и выбираем инструмент «выбор». Задействуем необходимые символы, перетаскивая мышку, и клацаем функцию «копировать». 
    3. Идем во вкладку меню — нам нужна функция «правка», а дальше «скопировать». После этого запускаем ворд и таким же методом клацаем «вставка» и «вставить».

    Познавательная статья: Как удалить ненужные приложения с ноутбука — 2 способа

    Используйте полную версию Adobe

    Для начала необходимо загрузить на ПК Adobe Reader. Желательно, полную версию. Скачать прогу возможно с веб-страницы. Для этого выполняем следующие действия:

    • заходим на электронную страницу сервиса;
    • на заглавной страничке убираем флажки с «загрузка утилиты McAfee», «True Key by Intel Security»;
    • нажимаем «выполнить загрузку сейчас»;
    • в новом окошке всплывет загрузчик;
    • ждем, когда версия полностью зальется на ПК;
    • в конце нажимаем «готово»;
    • происходит запуск проги.

    После открытия версии ознакамливаемся с функционалом. В начале прога имела возможность только читать файлы, после апгрейда — открыта, появилась возможность редактирования.

    После установки Адоб на панели инструментов в Ворд должен появится соответствующий значок. Нажимаем на значок и приступаем к работе. Выполняем следующие команды.

    1. В меню клацаем на «загрузить» и «объект».
    2. На ПК тык по файлу в виде пдшки в электронном виде.
    3. Следующая команда — «экспорт пдф». Действие располагается на панели приложения справа.
    4. Указываем необходимый тип файла для изменения. Это зависит от того, какая версия Ворда стоит.
    5. После этого клацаем «экспорт».
    6. Переименовываем.
    7. Готово.

    После сохранения в формате Ворд, появится возможность редактирования. Адоб способна конвертировать из пдф не только текстовую инфу, но и изображения. По этой причине не будет необходимости отдельно вставлять картинки, Адоб все самостоятельно выполнит. Возможно конвертировать отсканированные файлы и тексты.

    Если нужно конвертировать не всю текстовую информацию из пдф, выделяете необходимый фрагмент клавишами Ctrl+C и вставить Ctrl+V — и вуаля!

    Метод для Mac

    Владельцы Mac также могут провернуть подобные манипуляции немного другими способами и при использовании определенных программ.

    1. Копирование содержимого пдф
    Текст можно скопировать и вставить в текстовый редактор. Для этого: 

    • открываем нужный файл в формате пдф в режиме просмотра → выбираем текст, который необходим, → 
    • нажимаем одновременно клавиши Command+C, →
    • открываем Ворд и для вставки нажимаем Command+V. 

    Если необходимо скопировать весь текст, можно использовать комбинацию Command+A. Таким образом выделится полностью содержимое пдф.

    2. Приложение конвертирования Automator
    Версия поможет извлечь фрагмент из файла, который потом можно будет сохранить в Ворде ручным способом. 

    • Запускаем прогу и появляется новый процесс для работы. 
    • В перечне задач выбираем «извлечь содержимое пдф» и перетаскиваем курсором в нужное место для редактирования. 
    • После вылетит окошко, где можно самостоятельно указать настройки файла для извлечения и сохранения. На данной этапе есть возможность выбрать вариант сохраняемого текста — форматированный или простой. Перетаскиваем курсором фрагмент в нужное место и клацаем кнопку «запустить», которая находится в самом верху окошка версии. 
    • В указанной папке будет находиться файл, который был в обработке. 

    Откройте документ и перечитайте. Программа может не распознавать и пропускать некоторые буквы или неправильно их воспроизводить. Ошибки необходимо исправить вручную.

    Интересная статья: Какой MacBook выбрать в 2019 году: обзор, сравнение, отзывы 

    Давайте рассмотрим, чем отличаются Виндовс и Мак при вставке пдф в ворд.

    1. Мас предоставляет возможность смотреть и выбирать странички из пдф, которые желательно залить в ворд. Виндовс позволяет вставить только первую страницу файла.
    2. Для Мас пользователей нет нужды выбирать «создать» в «файл» и «просмотр», как это делается в Виндовс. Необходимо только сделать следующее: «вставить объект из файла».
    3. Для копирования текстовой информации, на Маке есть специальный инструмент, чтобы просмотреть изначально. 
    • Необходимо открыть пдф в предварительном просмотре.
    • Выбрать инструмент «текст», → выделить необходимое, → скопировать и залить в Ворд.

    Google Docs метод 

    Изменить формат возможно не только через программы конверторы, но и через Google Docs. Стоит понимать, что после работы будет изменено форматирование и удалены все изображения.

    Следует убедиться, что файл пдф не защищен паролем. Если стоит защита, отредактировать документ в гугл документах будет невозможно.

    Для работы необходимо открыть сайт гугл документов и авторизоваться. Если нет кабинета в гугл, нужно создать — пройти регистрацию, указать электронный адрес и создать пароль для входа.

    • Клацаем на вкладку «Открыть окно выбора объектов» в форме папки, который расположен вверху с правой стороны. Выскочит всплывающее окно. 
    • Тыкаем на загрузку. Далее указываемый необходимый файл на ПК — путь к нему. Кнопка будет светиться голубым цветом. Здесь выбираем нужный файл — в нашем случае, пдф.
    • Раскрываем и ждем полной заливки в программу. 
    • Клацаем на «открыть с помощью» — находиться в самом верху страницы. → Жмем гугл докс. Пдф засветиться в новой вкладке, где можно изменять, как текстовый док. Есть возможность отредактировать фрагмент, удалить. Форматирование будет зависеть от типа выбранного пдф-файла.
    • Можно сохранить обработанную ПДФку. Следуем алгоритму:
    1. «файл»;
    2. «скачать как»;
    3. выбираем документ нужного формата — док или ворд.

    Редакторские правки для обладателей Мас можно вносить по аналогичному алгоритму.

     

    Заголовок 3

    Использование Ворд онлайн для выполнения задачи 

    В программе Ворд онлайн нет возможности вставить файл пдф. Можно только вносить редакторские изменения. Для этого необходимо обновить содержимое файла или скопировать, затем добавить в ворд документ в автономном режиме.

    При открытии пдф в программе онлайн, он автоматически будет переходить в формат док без элементов исходного форматирования. К примеру, разрывы страниц и строк будут появляться в других местах. По этой причине, такое редактирование подойдет для файлов, которые содержат только текстовую инфу.

    Итак, ПДФ файлы могут отличаться между собою:

    • в виде картинок — на каждой странице размещено фото или изображение, отсутствует текстовые фрагменты. В таком случае лучше воспользоваться специальными прогами, которые помогут переделать картинку в текст;
    • в текстовом виде — файл содержит фрагмент, который сжат в формате пдшки и может быть закрыт функцией безопасности в настройках. Тогда спастись можно не только специальными прогами, но и конвертерами, функционирующими в онлайн режиме.

    Осталось выбрать удобный способ превращения пдшки в ворд — и вперед.

    К слову: Как проверить ноутбук при покупке: 10 прописных истин для новых и б/у лэптопов

    Объединить файлы ворд в пдф онлайн. Преобразование Word в PDF

    Использование Word версии 2010 либо 2013
    1. Откройте файл в Ворде.
    2. Выберите вкладку «Файл» и перейдите в новое окно.
    3. В поле «Типы файлов» вы должны выбрать «Создать PDF или XPS документ». После этого нажмите на «Сохранить как PDF/XPS».
    4. Щелкните «Создать PDF / XPS».
    5. В диалоговом окне вы должны ввести имя файла, и определить его местоположение на вашем устройстве.
    6. Щелкните мышью «Опубликовать».
    Помните, что этот способ применим только для Word 2010 или 2013, при использовании версии 2007, меню будет иметь существенные различия.

    Способ 2

    Использование Word 2007
    1. Откройте документ для преобразования.
    2. Нажмите пункт меню «Файл», расположенный вверху слева.
    3. Выберите соответствующие настройки с целью сохранить его как PDF.
    Вы должны помнить, что этот способ актуален исключительно в Windows, если вы используете Mac, вы должны попробовать другие способы.
    4. Впишите нужное имя файла и сделайте другие нужные настройки.
    5. Нажмите «Сохранить», что позволит конвертировать документ и в последующем открыть его в программе Adobe Reader.

    Способ 3

    Использование WORD в ОС Mac
    1. Выберите и откройте файл, необходимый для сохранения.
    2. В меню откройте вкладки Файл> Печать.
    3. Внизу слева щелкните кнопку «PDF», после чего выберите из выпадающего меню «Сохранить как PDF».
    4. Впишите заголовок и придумайте имя создаваемого файла. Если это необходимо, отрегулируйте другие настройки.
    5. Нажмите «Сохранить.

    Способ 4

    Использование других версий WORD
    1. Выберите на своем компьютере и откройте исходный файл.
    2. В меню щелкните по Файл> Печать.
    3. Нажмите на выпадающий список принтеров, установленных на устройстве. Выберите в нем PDF.
    4. Щелкните ОК.

    Способ 5

    Использование -конвертеров
    1. Зайдите на любой бесплатный сайт, который предлагает преобразование документов в формат PDF. Такие ресурсы можно легко найти в Интернете, набрав поисковой запрос «преобразовать ворд в пдф».
    Не следует использовать ресурс, который требует оплатить эту услугу, установить какое-либо программное обеспечение или выполнить действие, суть которого вы не понимаете. Сегодня существует множество бесплатных, простых в использовании онлайн конвертеров PDF, и вам не нужно усложнять себе задачу. Все сайты такого назначения работают по одинаковому принципу.
    2. Нажмите «Обзор» и найдите требуемый файл Word.
    3. При необходимости впишите адрес электронной почты. Некоторые сайты присылают конвертированный файл на ваш почтовый ящик.
    4. Щелкните по кнопке «Преобразовать» или «Конвертировать», подождите, пока файл обработается.
    5. Скачайте преобразованный файл на свой компьютер. Если на экране ничего не отображается, вы должны поискать конвертированный файл в вашем почтовом ящике.

    Способ 6

    Как документ ВОРД перевести в ПДФ с применением Open Office
    1. Скачайте из интернета OpenOffice — бесплатную программу, которая полностью совместима с Microsoft Word.
    2. Нажмите на загруженный файл два раза, чтобы установить на ПК скачанное приложение.
    3. Откройте в установленной программе исходный документ Word.
    4. Найдите в главном меню Файл> Экспортировать как PDF.
    5. Придумайте имя документа PDF.
    6. Щелкните мышью по «ОК» или «Сохранить» для преобразования. Если вы решили остановиться на использовании онлайн конвертера, вы должны учитывать, что сайты могут иметь неполадки в работе.
    Перед началом преобразования всегда создавайте резервную копию документа.
    Учтите, что OpenOffice может открыть файл DOCX, но не позволить его редактировать.

    Предупреждения

    Все перечисленные методы актуальны для стандартных документов. Файлы со сложным форматированием могут быть преобразованы с частичной потерей данных.
    Если вы намерены преобразовать конфиденциальные документы, избегайте конвертеров онлайн.

    По умолчанию Ворд сохраняет все созданные в нем документы в своем внутреннем формате.doc или.docx. Как правило, никаких проблем с последующим чтением другими пользователями созданных файлов не возникает при условии, что они открывают его также в Ворде такой же или более новой версии. Сложности появляются когда пытаются открыть текстовые документы в более старых версиях Ворд по сравнению с теми, в которых они были созданы, или, что еще хуже, когда файл нужно открыть пользователю операционной системы отличной от Windows.

    Помочь в этом случае может сохранение текстового файла в формате.pdf, который был специально разработан для хранения текстовой и графической информации и является кроссплатформенным, то есть одинаково отображающимся в различных операционных системах. О том, как это сделать читайте далее.

    Как перевести Ворд в пдф формат
    Впервые прямая возможность конвертации текстового файл в.pdf формат появилась в MS Word 2007. На его примере и будет рассмотрено сохранения любого. doc-файла в формате.pdf. Итак, для того, чтобы выполнить перевод из Word в.pdf формат выполните следующие шаги:
    В других более новых версиях Word перевод в пдф формат осуществляется аналогичным образом, за исключением того, что может несколько отличаться интерфес вызова окна Сохранить как . Если вы не знаете как вызвать его в вашей версии текстового редактора, то можете просто нажать F12 на клавиатуре.

    Кроме конвертации Ворда в пдф непосредственно из самого текстового редактора можно выполнить аналогичную конвертацию с помощью одного из многочисленных онлайн-сервисов. Найти их очень просто, введя в поисковую систему Google или Яндекс запрос doc в pdf. Рассмотрим как это происходит на примере сайта .
    Когда файл будет сохранен на вашем компьютере обязательно его откройте и проверьте корректность конвертации. Если возникли какие-то проблемы, то попробуйте перевести Ворд в пдф формат заново на этом или другом аналогичном сервисе.

    Лучший инструмент для преобразования Word в PDF

    Преобразование документов Word в PDF на сайте PDF2Go — это быстро и удобно. Просто подключитесь к сети и загрузите файл.

    Загрузите Word: можно перетащить документ, указать путь к файлу на устройстве или дать ссылку на облачное хранилище. Затем нажмите на «Сохранить изменения» и подождите, пока мы преобразуем файл.

    Преобразование Word в PDF онлайн

    При установке программного обеспечения есть риск нарваться на вирус. Мобильные приложения менее опасны, но не защищают от навязчивой рекламы или, что ещё более подозрительно, запрашивают доступ к телефону.

    Поэтому PDF2Go позволяет конвертировать онлайн! Без установки программы и без вирусов. Просто подключитесь к сети.

    Зачем вообще конвертировать?

    У формата PDF много преимуществ. Если вы распечатаете PDF-документ или с кем-нибудь им поделитесь, форматирование останется неизменным. PDF-файлы открываются на любых компьютерах, телефонах и устройствах для чтения электронных книг.

    Надёжный PDF-конвертер PDF2Go позволяет использовать эти и другие преимущества.

    Безопасный PDF-конвертер

    Загрузите документ Word для конвертации в PDF — это безопасно, как никогда прежде. Ваш файл в надёжных руках. Не беспокойтесь, все права на документ остаются за вами.

    См. подробности в Политике конфиденциальности.

    Что можно преобразовать?

    Мы специализируемся на конвертации документов Word в PDF. Но это далеко не всё. Наряду с форматами DOC и DOCX у нас можно преобразовать и другие текстовые документы, презентации и изображения!

    Например:

    ODT, RTF, TXT, Microsoft PowerPoint PPT, JPG, PNG, GIF и другие

    Преобразовать файл в формат PDF онлайн

    Вы не привязаны к компьютеру. PDF2Go конвертирует PDF-файлы онлайн. Вы можете работать с любого устройства, дома или на работе, даже в отпуске!

    PDF2Go работает и на мобильных устройствах!

    Конвертировать файлы doc и docx в PDF может понадобиться по разным причинам, так как формат PDF является универсальным для всех устройств на разных операционных системах. Способов сделать это несколько: воспользоваться онлайн конвертерами, использовать Microsoft Office Word 2010 и старше, скачать себе на компьютер один из конвертеров на выбор. В данной статье будет рассмотрено два основных способа с наглядными примерами.

    Как перевести Вордовский документ в ПДФ через Microsoft Office Word

    Прежде всего, вам нужно иметь на своем компьютере версию Ворд не моложе 2010 года. Если вы пользуетесь версией 2007 года, либо 2003, то нужно скачать специальную утилиту от официального разработчика. Установив её, в программе появится возможность сохранять документ сразу в формате PDF. Для всех остальных версий справедлив такой алгоритм:

    • Создайте doc файл, либо откройте уже готовый.
      Кликните на кнопку “Файл” в самом верхнем левом углу программы.
    • Во всплывающем списке кликните на строку “Сохранить как”.


    • В появившемся окне выберите директорию сохранения документа. Под строкой с названием файла вы увидите поле “Тип файла”. Откройте его и отыщите формат PDF в списке, кликните на него.
      Нажмите “Ок” для сохранения.


    • Теперь в указанной директории вы увидите не doc файл, а PDF, сразу сохраненный в программе Word. При его открытии будет загружаться не Microsoft Office Word, а программа для чтения PDF, выбранная у вас по умолчанию.


    • Попробуйте открыть сохраненный файл двойным нажатием. В данном случае, документ будет загружаться в Adobe Acrobat Reader.


    Как перевести Вордовский документ в ПДФ через онлайн конвертеры

    Если с вашей программой MS Office Word возникли какие-либо проблемы, либо вы попросту не имеете к ней доступа, то лучше воспользоваться онлайн программами для преобразования формата.doc в.pdf, которых в сети интернет большое множество. Попробуйте ввести соответствующий поисковой запрос и самостоятельно выбрать сервис, либо воспользуйтесь этим: http://convertonlinefree.com.

    • Пролистав страницу вниз, вы увидите специальную форму для загрузки документа. Кликните на кнопку “Обзор”.


    • Найдите папку расположения Вордовского файла, который требуется преобразовать в PDF, отметьте его мышкой и нажмите “Открыть”.


    • На сайте сразу же появится имя документа, рядом вы увидите кнопку “Конвертировать”. Чтобы приступить к процессу преобразования, нажмите на неё.


    • Если у вас стабильное интернет соединение и файл имеет небольшой объём, то сайту понадобится несколько секунд для конвертации. Однако, при большом объёме информации, либо плохом и медленном интернете, вам придётся подождать подольше. Если время ожидания превышает пятнадцать минут, то перейдите на зеркало сайта по указанной над формой ссылке: попробуйте использовать для преобразования его.
      Как только процедура завершится, автоматически появится окно сохранения файла. Нажмите “Сохранить файл”.


    • Выберите желаемую директорию, по желанию измените имя документа. Теперь PDF файл сохранен на вашем компьютере, а исходный doc или docx документ остался нетронутым.


    Оба способа занимают относительно короткое количество времени, вся разница заключается в том, что для первого варианта вам понадобится программа MS Office Word не ранее 2010 года выпуска, а для второго – стабильная работа интернета.

    Попробуйте оба метода и выберите для себя наиболее комфортный.

    Максимальный размер файла!»

    Выбранный вами файл превышает максимально допустимый размер файла 10 МБ. Он не был добавлен.

    Если вы хотите увеличить лимит до 20 МБ, зарегистрируйтесь бесплатно. И, если вам нужно больше, вы можете подписаться на Hipdf Pro и получить до 50 МБ.

    Вход Регистрация

    Максимальный размер файла!»

    Выбранный вами файл превышает максимально допустимый размер файла 20 МБ. Он не был добавлен.

    Если вы хотите увеличить лимит до 50 МБ, обновите его до Hipdf Pro.

    {{ mutiExceddsTip }}

    Выбранный вами файл превышает максимально допустимое количество страниц. Он не был добавлен.

    Если вы хотите увеличить лимиты до 100 страниц, пожалуйста, зарегистрируйтесь бесплатно. А, если вам нужно больше, вы можете подписаться на Hipdf Pro и получить до 2000 страниц.

    Вход Регистрация

    Максимальное количество страниц превышено!

    Выбранный вами файл превышает максимальное количество разрешенных страниц. Он не был добавлен.

    Если вы хотите увеличить лимит до 2000 страниц, перейдите на пакет Hipdf Pro.

    {{ mutiExceddsTip }}

    Подписаться на Hipdf Pro Нет, спасибо

    {{ file.file_name | subLengthStr(80) }}

    Идёт конвертация %…

    Эта функция доступна только подписчикам Hipdf Pro

    Подписаться на Hipdf Pro Конвертировать без OCR


    Это отсканированный PDF-документ, выполнение OCR позволит вам редактировать текст после преобразования. Файл PDF содержит отсканированные страницы, если вы хотите бесплатно конвертировать этот PDF-документ в редактируемый документ, используя наш мощный движок OCR, сначала войдите в систему.

    Язык документа: {{ ocrLanguage.join(«, «) }}
    Изменить язык

    Преобразовать doc в pdf

    Для начала определимся, что такое doc и pdf. Чтобы правильно написать указанные форматы, надо ставить перед ними точку – .doc и .pdf. Но точку часто не ставят – все торопятся…

    Наличие точки означает, что после точки мы имеем дело с расширением имени файла (или иначе – форматом файла). Например,

    • “расписание.doc” – это имя файла,
    • а .doc – это расширение имени файла, которое указывает операционной системе, какой именно программой следует открывать файл.

    .Doc (от англ. document) – это расширение означает, что документ был создан программой Microsoft Word.

    .Pdf (от англ. Portable Document Format) был разработан фирмой Adobe для федеральных властей США для создания и хранения рабочих документов.

    В настоящее время используется в издательском деле, для распространения факсов, в формах налоговой отчетности, в образовательных, юридических, финансовых учреждениях, а также обычными пользователями ПК в электронных письмах, в сообщениях  и т.д.

    Такой файл легко напечатать и использовать для совместной работы, а также трудно изменить. Принято считать, что если Вам прислали документ в формате .pdf, то Вы НЕ будете присланный документ изменять или редактировать. Точно также, если Вы отправляете документ в формате .pdf, то «по умолчанию» предполагаете, что Ваш документ не будет редактироваться получателем (Ваше резюме, налоговая отчетность и т. п.)

    Поэтому если Вы вдруг решите:

    • вскрыть «а-ля хакер» присланный Вам документ в формате .pdf,
    • отредактировать его (внести изменения),
    • потом отправить тому, кто вам прислал этот документ,

    то не удивляйтесь, что с Вами обратная сторона не захочет больше иметь дела. Кроме того, на некоторые материалы в формате .pdf есть авторские права, например, на электронные книги, поэтому любое изменение в таких материалов автоматически означает нарушение авторских прав.

    После “лирического отступления” по компьютерной грамотности вернемся к теме статьи: как преобразовать файл doc в pdf? Сделать это можно различными способами. Возможно применение текстового редактора Word, а также иных утилит или с помощью онлайн-сервисов.

    Преобразовать doc в pdf может быть необходимо в различных случаях. Допустим, Вам требуется переслать файл по электронной почте в хорошо оформленном виде, создать резюме в виде .pdf файла или сделать простейшую электронную книгу. Существует возможность использовать для этих целей платные и бесплатные программы. Программы, призванные преобразовать doc в pdf, могут быть двух групп.

    Первая программа является полноценной утилитой для того, чтобы преобразовать DOC в PDF. Программы, которые находятся во второй группе, подключаются к системе как виртуальный принтер, то есть наличие реального принтера не требуется. В этом случае Вам нужно вывести документ на печать, но выбирать нужно при этом виртуальный, а не физический принтер. Pdf документ будет определен в отдельную папку.

    Не всем известно, что возможно преобразовать doc (docx) в pdf с помощью текстового редактора Word, в котором был создан исходный документ. Это поможет Вам обойтись без поиска, скачивания и установки сторонних программ.

    Как преобразовать DOCX (DOC) в PDF в Word?

    Документ можно открыть в MS Office и сохранить в формате .pdf. Чтобы это сделать, нажмите на значок Office в верхнем углу слева, выберите “Сохранить как”:

    Откроется “Сохранение документа”, в котором сначала выбирается папка для хранения:

    Рис. 1. Сохранить документ в формате pdf в программе Word

    Далее по шагам, обозначенным на рисунке 1:

    цифра 1 на рис. 1 – в поле Тип Файла кликнуть по маленькому треугольнику. Появится выпадающее меню, в котором надо найти и кликнуть по “PDF (*.pdf)”.

    2 на рис. 1 – ввести имя документа,

    3 – вариант оптимизации “Стандартная” или “Минимальный размер”,

    4 – выберите подходящие параметры. Можно сначала сохранить документ с одними параметрами , если они не понравятся или не подойдут, выбрать другие параметры и сохранить документ с новыми параметрами.

    5 – в “Сервисе” есть опция “Сжать рисунки”. Если ей воспользоваться, то объем документа можно уменьшить, но качество рисунков будет несколько ниже, чем до сжатия.

    6 на рис. 1 – самая важная кнопка “Сохранить”, на которую надо нажать, чтобы сохранить документ в формате pdf с выбранными параметрами.

    Как преобразовать DOC в PDF в OpenOffice?

    Word является платной программой, и он не у всех есть, поэтому есть еще вариант с бесплатным пакетом OpenOffice:


    Официальный сайт OpenOffice: https://www. openoffice.org/ru/

    Чтобы скачать, нужно перейти по указанной выше ссылке на официальный сайт и затем  кликнуть по кнопке “Загрузить Apache OpenOffice”.

    В этом пакете программа OpenOffice Writer является программой, аналогичной текстовому редактору Word. Нужно открыть Writer. Сверху есть панель инструментов и кнопка “PDF”.

    Когда документ загружен в редактор Writer, нужно кликнуть по кнопке “PDF”.

    Здесь можно настраивать разные параметры документа, допустим, поставить на него пароль или настроить параметры внешних и внутренних ссылок.

    Если в первоначальном документе есть графические изображения, стоит обратить внимание на пункт “Изображение” во вкладке “Общие”. Там выставляются параметры сжатия JPEG, а также разрешение изображений. Чем параметры выше, тем лучше станет качество документа, но и вес его станет больше. Здесь потребуется найти оптимальный баланс между весом документа и его качеством.

    Когда Вы настроите все параметры,дальше вводится название файла, выбирается папка для хранения и нажимается “Сохранить”.

    Некоторые утверждают, что Writer конвертирует в PDF лучше, чем Word: получается файл меньше по объему и качество не страдает.

    Как конвертировать DOC в PDF онлайн?

    Если у Вас нет под рукой текстового редактора, тогда можно воспользоваться бесплатным сервисом для конвертации. Допустим, сервисом doc2pdf.net. Этот бесплатный преобразователь позволяет конвертировать документы Word (.doc/.docx) в формат Adobe PDF (.pdf).

    Преобразовать DOCX (DOC) в PDF: https://www.doc2pdf.com/

    Сервис работает в двух режимах:

    • онлайн,
    • десктопная версия.

    В правом верхнем углу этого сервиса есть возможность выбрать русский язык вместо английского, который используется по умолчанию.

    Режим онлайн означает, что документ загружается на сторонний сервис, там конвертируется в другой формат и возвращается Вам. НО документ должен быть НЕ более 25 МБ!

    Десктопная версия означает, что программа устанавливается на Ваш компьютер, а потом с ее помощью можно конвертировать файлы по мере необходимости.

    Когда Вы зайдете на сайт, то заметите в левой колонке страницы форму:

    Рис. 2. Сервис doc2pdf в режиме онлайн

    Цифра 1 на рис. 2 – клик по “My computer”. Выбираем на своем ПК файл, подлежащий конвертации в PDF (он должен быть не более 25 Мб). После того, как файл выбран, идет его конвертация, нужно немного подождать.

    Когда файл будет преобразован, появится 2 варианта для того, чтобы получить файл pdf на свой компьютер:

    1. скачать (цифра 1 на рис. 3),
    2. получить по электронной почте. Для этого варианта есть поле для ввода e-mail (2 на рис. 3). Вводите свой e-mail без ошибок, ибо на него будет выслан файл в формате PDF.

    Рис. 3. Сохранить файл PDF на своем ПК или выслать файл на электронную почту

    P.S. Статья закончилась, но на блоге можно еще почитать:

    1. Как легко посчитать буквы в тексте

    2. 4 программы проверки орфографии

    3. Как сделать гиперссылку в Word?

    4. Как сохранить веб-страницу в PDF-документ

    5. О сходстве и различии файлов doc и docx



    Получайте актуальные статьи по компьютерной грамотности прямо на ваш почтовый ящик.
    Уже более 3.000 подписчиков

    .

    Важно: необходимо подтвердить свою подписку! В своей почте откройте письмо для активации и кликните по указанной там ссылке. Если письма нет, проверьте папку Спам.

    Автор: Надежда Широбокова

    13 апреля 2016

    Word в PDF — конвертировать документ в PDF онлайн БЕСПЛАТНО

    Word в PDF — конвертировать документ в PDF онлайн БЕСПЛАТНО — Soda PDF \ n

    \ n Ежемесячно (выставляется ежегодно) \ n

    \ n

    \ n Ежемесячно (выставляется каждые 2 года) \ n

    \ n

    \ n Ежегодно \ n

    \ n

    \ n 2 года \ n

    «, «cannotSignInWithOldEmail»: «Вы не можете войти в систему с помощью signInEmail\"»/>, так как это больше не адрес электронной почты, связанный с вашей учетной записью», «labelCity»: «Город», «mergeWithSodaSubPrgh»: «С легкостью объединяйте файлы PDF в Интернете.Soda PDF — это решение для пользователей, которые хотят объединить несколько файлов в один PDF-документ. Наш инструмент прост в использовании и БЕСПЛАТНО * «, «ModulePopupHeadOops»: «Ой!», «FileTypeIsNotSupported»: «Тип файла не поддерживается.», «readLess»: «Читать меньше», «readMore»: «Читать дальше», «noThanks»: «Нет, спасибо», «BuyNow»: «Купить сейчас», «PrivacyTerms»: «Конфиденциальность и условия», «WordToPdfLink»: «https://www.sodapdf.com/word-to-pdf/», «businessAlertText»: «Вы занимаетесь бизнесом?», «EmailPreferencesSubTitle»: «Выберите списки рассылки, на которые вы хотите подписаться.Снимите флажок, чтобы отказаться от подписки. \ NЕсли вы хотите изменить язык получаемых писем, «, «без обслуживания»: «Без обслуживания», «successTitle»: «Назначение выполнено успешно», «tooltip_1»: «План позволяет одному устройству входить в Soda PDF Online в любой момент времени», «contactEmail»: «Контактный адрес электронной почты», «BuyLink»: «https://www. sodapdf.com/buy/», «GifToJpgLink»: «https://www.sodapdf.com/gif-to-jpg/», «PDF_Software»: «Программное обеспечение PDF», «selectProduct»: «Выбрать продукт», «startFreeTrial»: «Начать бесплатную пробную версию», «errorTypeOfProduct»: «Выберите тип продукта», «compressed_copy»: «Загрузите сжатую копию вашего файла.», «contactSales»: «Связаться с отделом продаж», «sellsheets»: «Product Sheets», «PricingLink»: «https://www.sodapdf.com/pricing/», «getSoda»: «Получить газировку», «noCreditCards»: «Нет кредитных карт», «createPdfLink»: «https://online.sodapdf.com/#/home?r=view», «accountManagement»: «Управление аккаунтом», «SixFiles»: «6 файлов», «premiumPhoneSupport»: «Поддержка по телефону премиум-класса», «forLimitLicenses»: «Для 1-4 лицензий», «knowledgeBase»: «База знаний», «passwordRequirements_3»: «Ваш пароль не может содержать \» пароль \ «, \» admin \ «или \» administrator \ «», «passwordRequirements_2»: «Ваш пароль не может содержать 3 или более последовательных символов или иметь один и тот же символ, повторяющийся последовательно (например,123, ABC, AAA, 111) «, «YourFilesSecureServers»: «Ваши файлы хранятся на наших серверах только 24 часа, после чего они уничтожаются безвозвратно. «, «errorConfirmEmailPasswordMatch»: «Введенные адрес электронной почты и пароль не совпадают», «PdfToImageLink»: «https://www.sodapdf.com/pdf-to-jpg/», «WhatsNewTitle»: «Что нового в Soda PDF Anywhere», «registerSignUpTitle»: «С подключенной учетной записью», «WordToPdf»: «Word в PDF», «paymentAssociatedCreditCard»: «Продукты, связанные с этой кредитной картой», «createdPasswordSuccessfully»: «Ваш пароль был успешно создан.», «CookiesForAdvertising»: «Этот сайт использует файлы cookie в рекламных и аналитических целях. Пожалуйста, ознакомьтесь с нашей Политикой конфиденциальности, чтобы получить дополнительную информацию о файлах cookie и их использовании, а также о возможности изменения настроек файлов cookie.», «PDFReader»: «PDF Reader», «Сбережения»: «СБЕРЕЖЕНИЯ», «YourFilesSecure»: «Ваши файлы в безопасности», «ConvertfromPDF»: «Конвертировать из PDF», «WorkingOffline»: «Работаете в автономном режиме?», «зарегистрироваться»: «Зарегистрироваться», «sodaVersion»: «Сода PDF», «wouldLikeContinue»: «Хотите продолжить?», «productAvaliableProducts»: «Доступные продукты», «one_time_fee»: «единовременная плата», «Privacy_Terms»: «Конфиденциальность и условия», «RegisterLink»: «https: // www.sodapdf.com/account/register/ «, «AboutSodaPdf»: «О Soda Pdf», «PleaseSignInWithAccount»: «Войдите в свою учетную запись», «mergeToolLink»: «https://www.sodapdf.com/pdf-merge/», «активация»: «Активация», «EulaLink»: «https://www.sodapdf.com/terms-of-use/#eula», «formProductInterest»: «Интересующий продукт», «PDF_annual»: «* годовой план», «emailAddress»: «Адрес электронной почты», «Разблокировать»: «Разблокировать», «learnMore»: «Первое в мире онлайн-программное обеспечение для работы с PDF», «sitemap»: «Sitemap», «switchYearly»: «переходить на ежегодный», «MergeLink»: «https: // www.sodapdf.com/pdf-merge/ «, «choose3options»: «Однако вы можете выбрать один из трех вариантов», «PngToJpg»: «PNG в JPG», «PngToPdf»: «PNG в PDF», «fromDevice»: «С устройства», «forLimitLicenses5»: «Для 5-24 лицензий», «cancelRequest»: «Отменить запрос», «resourceCenter»: «Ресурсный центр», «FallDocuments»: «Меня уволили после того, как я заснул по личным документам.», «PlanLinks»: «Планы и цены», «low_quality_text»: «меньшее качество, наименьший размер файла», «fromOpdfs»: «

    Спасибо за создание бесплатной учетной записи.Ваш файл готов!

    \ n

    Вы должны быть перенаправлены через мгновение, чтобы получить доступ к вашему файлу.

    \ n

    Если перенаправление не работает (или занимает слишком много времени), щелкните здесь, чтобы получить доступ к файлу.

    «, «DownloadLink»: «https://www.sodapdf.com/installation-guide/», «PageNumbering»: «Нумерация страниц», «emailWasSentSuccessfully»: «Электронное письмо успешно отправлено», «Водяной знак»: «Водяной знак», «productSoda9lockedMessage»: «Продукты с бессрочной лицензией привязаны к одному компьютеру.Используйте команду «Сбросить лицензию», чтобы переназначить лицензию другому компьютеру. «, «subscribe_success_msg»: «Вы успешно зарегистрировались!», «NoThank»: «Нет, спасибо», «sendFileByEmail»: «Отправить файл по электронной почте», «choosequalitytitle»: «Выбрать качество сжатия», «errorWebsiteUrlRequired»: «Введите URL», «errorCountryRequired»: «Выберите страну», «subscribeToPromotions»: «Акции», «headerSearchPlaceholder»: «Есть вопросы? Введите запрос здесь», «AddAccount»: «Добавить аккаунт», «didYouTitle»: «Знаете ли вы?», «UploadingFile»: «Загрузка», «dl_options_10»: «Разметка и добавление примечаний к PDF-файлам», «dl_options_11»: «Создавать собственные формы», «labelLicensesNeeded»: «Количество необходимых лицензий», «MyProductsLink»: «https: // www.sodapdf.com/account/manage-products/ «, «youtubeTitle»: «Откройте для себя Soda PDF Anywhere», «previewText»: «Предварительный просмотр Soda PDF 12», «TenPack»: «10-PACK», «labelStateProvince»: «Штат / провинция», «formFirstName»: «Имя», «solutionsBusines»: «Решения для бизнеса», «ConnectedAccounts»: «Подключенные учетные записи», «One_file_only»: «ТОЛЬКО ОДИН ФАЙЛ», «PrivacyPolicyLink»: «https://www.sodapdf.com/privacy/», «Выход»: «Выйти», «compressWithSodaSubPrgh»: «Уменьшите размер PDF всего за несколько кликов.Это просто и бесплатно * «, «ConvertPassProtected»: «Загруженный файл защищен паролем и не может быть преобразован.», «JpgToGif»: «JPG в GIF», «JpgToPdf»: «JPG в PDF», «JpgToPng»: «JPG в PNG», «emailPasswordIncorrect»: «Ваш адрес электронной почты или пароль неверны.», «BlogLink»: «https://www.sodapdf.com/blog/», «errorConfirmPasswordMatch»: «Ваши пароли не совпадают», «batchPrgh»: «Загрузите файл, содержащий электронные письма пользователей, которым вы хотите назначить лицензию. Файл должен быть в формате .csv.Электронные письма должны быть в первом поле. Имя и фамилия не обязательны, но могут быть помещены во второе и третье поля. «, «PurchasedDate»: «Дата покупки», «OpenedPassProtect»: «Загруженный файл защищен паролем и не может быть открыт.», «One_file_only2»: «Только один файл», «LinkfFeatures»: «https://www.sodapdf.com/features/», «manualFree»: «Бесплатно и надежно», «ready_1_strong»: «Еще не пробовали наше настольное приложение?», «Повернуть»: «Повернуть», «buyNowFoot»: «Купить сейчас», «SwitcherEnable»: «Включить», «Подмножество»: «Подмножество», «Суффикс»: «Суффикс», «supportText»: « БЕСПЛАТНО Поддержка клиентов
    «, «Строка»: « облачное хранилище … «, «errorContactEmailRequired»: «Введите контактный адрес электронной почты», «SiteMapLink»: «https://www.sodapdf.com/sitemap/», «PDF_mo»: «/ мес», «PDFfee»: «Однако вы можете загрузить объединенную копию файла за единовременную плату в размере 2,99 долларов США.», «OfferEXTENDEDGet60»: «Предложение РАСШИРЕНО Получите скидку 60% на : объединение, сжатие и многое другое!», «FreeUpdates»: «Бесплатные обновления», «FreePdfReader»: «Читатель Soda 3D», «Save50»: «SAVE 50% «, «termsOfUse»: «Условия использования», «WatermarkLink»: «https: //www.sodapdf.com / add-watermark-to-pdf / «, «Префикс»: «Префикс», «ContactSalesLink»: «https://www.sodapdf.com/contact-sales/», «errorEndsWithEmail»: «—«, «ProductOverview»: «Обзор продукта», «stayConnected»: «Оставайтесь на связи», «HtmlPDFLabel»: «Хотите преобразовать веб-страницу в файл PDF? Сделайте это бесплатно на», «moduleOCRReq»: « OCR Module требует», «ThankyouCTA2notice_bottom»: «на рабочий стол», «Позиция»: «Позиция», «mobile_app_stores»: «Объединяйте и создавайте PDF-файлы бесплатно на своем телефоне», «getVolumePricing»: «Получить оптовые цены», «pagesToInsert»: «Страницы для вставки», «CreateFiles»: «Создавать файлы PDF», «labelIndustry»: «Промышленность», «ready_2_strong»: «Вам нужны PDF-файлы на ходу?», «Премиум»: «ПРЕМИУМ», «Защитить»: «Защитить», «DragFile»: «Перетащите сюда файлы», «ChooseCompressionRatio»: «Выбрать степень сжатия», «errorTimelineRequired»: «Выберите временную шкалу», «PdfToWorldToolLink»: «https: // www.sodapdf.com/pdf-to-word/ «, «Excel2pdf»: «Excel в PDF», «cookieSettings»: «Настройки файлов cookie», «PopularTools»: «Популярные инструменты», «errorRequired»: «Это поле обязательно для заполнения», «sodaPdfAnywhereOverview»: «Обзор Soda PDF Anywhere», «Авторское право»: «Авторское право», «SwitcherDisable»: «Отключить», «Ppt2pdf»: «PPT в PDF», «professionalPackage»: «Профессиональный пакет», «SignFiles»: «Подписать файлы PDF», «selectModule»: «Выбрать другую функцию», «btnDownloadText»: «Просмотреть и загрузить в браузере», «ArticleTitleThree»: «Как объединить документы с помощью Soda PDF 12», «CompressFiles»: «Сжать файлы PDF», «PdfToExcelLink»: «https: // www.sodapdf.com/pdf-to-excel/ «, «cloudUpload»: «загружено из облака», «RemoveMain maintenance»: «Удалить обслуживание», «PDF_next»: «следующий», «CompressTitle»: «Сжать PDF — БЕСПЛАТНО уменьшить размер файла PDF в Интернете», «OnDesktop»: «На рабочем столе», «expiresDate»: «Срок действия — дата», «PDFBates»: «Нумерация Бейтса PDF», «PdfFormFillerLink»: «https://www.sodapdf.com/pdf-form-filler-creator/», «youShouldCreateAccount»: «Создайте учетную запись с этим адресом электронной почты для доступа к вашему продукту.», «expiredTime»: «Срок действия истекает через», «expiredDate»: «Срок действия — дата», «howActivateSoda»: «Как активировать Soda PDF», «FreeOnlineToolsLink»: «https: // www.sodapdf.com/freeonlinetools/ «, «ExceedsSizeLimit»: «Размер файла превышает максимально допустимый», «Подключиться»: «Подключиться», «emailNotValid»: «Пожалуйста, укажите действующий адрес электронной почты», «footerLuluWebsite»: «Сайт компании», «fullPagesRangeError»: «Ваши начальная и конечная страницы охватывают весь загруженный документ. Поэтому разделения не произойдет.», «footerCopyText»: «Soda PDF является товарным знаком LULU Software ™.», «fromOurSalesTeam»: «От нашей команды продаж», «openTicketBackText»: «Отправьте запрос в службу поддержки и получите необходимую помощь.», «montlyPlan»: «Ежемесячный план», «englishOnly»: «Только английский», «ChangePending»: «Ожидается изменение», «SoftwareLink»: «https://www.sodapdf.com/», «thankYouTitle»: «Спасибо за установку Soda PDF», «myProducts»: «Мои продукты», «convert»: «Конвертировать», «CompressPassProtected»: «Загруженный файл защищен паролем и не может быть сжат.», «products»: «Товары», «WinTitle1»: «Полное решение PDF», «обязательный»: «обязательный», «PDFexceed_title»: «Загруженный файл превышает максимально допустимый размер», «fileReadyTitle»: «УРА! Ваш файл готов,
    добро пожаловать!», «SearchTool»: «Поиск инструмента», «one_time_payment»: «Единовременный платеж», «rightWord»: «Верно», «implperTitle»: «Неверное расположение полей», «footerLuluCareers»: «Карьера», «SplitLink»: «https: // www.sodapdf.com/split-pdf/ «, «EsignFiles»: «Файлы PDF для электронной подписи», «PdfToJpg»: «PDF в JPG», «PdfToPpt»: «PDF в PPT», «FREE_PDF_TOOLS»: «БЕСПЛАТНЫЕ ИНСТРУМЕНТЫ PDF», «behindPage»: «За страницей», «ArticleDescriptionOne»: «Итак, вы хотите добавить страницы в этот PDF-файл. Возможно, это документ, который вы уже создали, или тот, который недавно был отправлен вам. Но как вообще вы вставляете страницы в уже существующий PDF-файл, который кажется нежелательным для изменения? «, «ArticleDescriptionTwo»: «Вы повысите свою эффективность только тогда, когда научитесь создавать файлы PDF в пакетном режиме.Любой файл, который можно распечатать на бумаге, также можно преобразовать в формат PDF. С помощью процесса пакетного создания Soda PDF 12 вы можете взять любое количество файлов, независимо от формата, и одновременно преобразовать их все в PDF-файлы. «, «changedCongratulations»: «Поздравляем, вы успешно изменили адрес электронной почты.», «features_text»: «Неограниченно: объединение, преобразование, редактирование, вставка, сжатие, просмотр и многое другое!», «productAction»: «Действие», «enterWaterMarkText»: «Пожалуйста, введите текст водяного знака.», «onlinePdfTools»: «Инструменты для работы с PDF в Интернете», «PdfConverter»: «Конвертер PDF», «productAssign»: «Назначить», «ResourcesLink»: «https://www.sodapdf.com/resources/», «WhatsNewText»: «Испытайте первое в мире полнофункциональное онлайн-решение для работы с PDF. Оно наполнено совершенно новыми функциями, специально разработанными для повышения производительности, включая E-Sign, Soda PDF Online, нумерацию Бейтса и пакетное преобразование.», «DeletePdf»: «Удалить PDF», «abovePage»: «Над страницей», «Спасибо Спасибо», «SodaOverviewLink»: «https: // www.sodapdf.com/products/soda-overview/ «, «low_quality»: «Низкое качество», «findReseller»: «Найти реселлера», «errorProductRequired»: «Выберите продукт», «errorOopsEnterB2BEmail»: «К сожалению, похоже, вы указали личный адрес электронной почты! Чтобы получить доступ к нашей 30-дневной пробной версии для бизнеса, вы можете вернуться к форме и ввести действующий рабочий адрес электронной почты. В противном случае вы можете попробовать нашу личную пробную версию.» , «лицензия»: «лицензия», «Pdf2Word»: «PDF в Word», «PasswordLabel»: «Пароль:», «ProtectTitle»: «Защитить PDF», «enterStreetAddressLine»: «Введите строку почтового адреса», «где угодно2»: «… и продолжайте работать над ним на своем смартфоне или планшете во время поездки на работу. «, «where3 «:» Когда вы окажетесь дома, запустите свой PC и продолжайте с того места, где вы остановились. «, «where1 «:» Готовишь контракт в офисе, но есть поезд, чтобы успеть? «, «PdfDownloadLink»: «https://www.sodapdf.com/pdf-download/», «protect_unlim»: «Защитить неограниченное ЧИСЛО файлов.», «allTools»: «Все инструменты», «EnglishContent»: «Доступно только на английском языке», «TextToPdfLink»: «https: // www.sodapdf.com/txt-to-pdf/ «, «HtmlToPdf»: «HTML в PDF», «yourDownloadShouldBegin»: «Ваша загрузка должна начаться немедленно.», «errorLicensesRequired»: «Введите количество лицензий», «formEmailBusiness»: «Рабочий адрес электронной почты», «securitySign»: «Безопасность и подпись», «BatesNumberingLink»: «https://www.sodapdf.com/bates-numbering/», «BatesNumberingTool»: «Нумерация Бейтса», «BmpToJpg»: «BMP в JPG», «stayInformedOnSoftware»: «Будьте в курсе обновлений программного обеспечения, напоминаний об истечении срока действия, персонализированных советов и получайте эксклюзивные предложения по электронной почте.», «EmailPreferencesMore»: «Для получения дополнительной информации прочтите наши», «createdPasswordLinkExp»: «Срок действия ссылки для создания пароля истек.», «year2Plan»: «План на 2 года», «ResellersFoot»: «Реселлеры», «ResellersLink»: «https://www.sodapdf.com/resellers/», «high_quality»: «Высокое качество», «paymentDetails»: «Детали платежа», «InformationHandled»: «Предоставленная вами информация будет обрабатываться в соответствии с нашей Политикой конфиденциальности.», «orderInvoiceQuestions»: «№ заказа / № счета / Вопросы», «mergeWithSodaTitle»: «Слияние PDF», «End_User»: «Лицензионное соглашение с конечным пользователем», «authenticationError»: «Произошла ошибка аутентификации.Пожалуйста, войдите в свою учетную запись еще раз, чтобы продолжить », «implperPrgh»: «Адреса электронной почты должны быть в первом поле для каждого назначения. Имя и фамилия могут быть указаны во втором и третьем полях.», «send_to_email»: «Отправить по электронной почте», «ProtectLink»: «https://www.pdfprotect.net/», «Pdf2ppt»: «PDF в PPT», «successRegister»: «На ваш адрес электронной почты отправлено письмо для активации.», «ViewFiles»: «Просмотр PDF-файлов», «modifyRenewal»: «Изменить продление», «ForgotPasswordLink»: «https: // www.sodapdf.com/account/recover-password/ «, «InWebBrowser»: «В веб-браузере», «customQuote»: «индивидуальная цитата», «ElectronicSignature»: «Электронная подпись», «rongTitle «:» Неверный тип файла «, «mergeRequest»: «Запрос на объединение был отправлен на [другой адрес электронной почты]. Щелкните ссылку в электронном письме, чтобы завершить объединение ваших учетных записей», «YouIncognito»: «Вы используете режим инкогнито.
    Пожалуйста, войдите или создайте аккаунт», «TotalPrice»: «общая цена», «pdfFormCreator»: «Создатель PDF-форм», «howInstallSodaLink»: «https: // support.sodapdf.com/hc/en/articles/360022498011-How-to-download-and-install-Soda-PDF «, «freeTrial»: «Бесплатная пробная версия», «workOfflineOneLine»: «Работать в автономном режиме?
    Попробуйте настольную версию!», «PDFafterThePayment»: «Загрузка начинается автоматически после оплаты.», «forLegalProfessionals»: «Для юристов», «layoverText2»: «При нажатии откроется новая вкладка», «layoverText1»: «Это объявление помогает сделать наши услуги бесплатными», «selectLanguage»: «Выберите язык», «getStarted»: «Начало работы», «InstantText»: « Instant \ nЛицензия
    \ nАктивация», «freeItem1»: «Имея более 1 миллиона пользователей в месяц, мы постоянно совершенствуем наш инструмент слияния, оставляя его бесплатным для наших пользователей.», «freeItem2»: «Объедините файлы в браузере. Он совместим со всеми операционными системами.», «FilesUsed30days»: «Файлы должны быть использованы в течение 30 дней с момента покупки», «MainPage»: «Главная страница», «congrats_prgh»: «

    Поздравляем!

    \ n
    Вы успешно подтвердили свою учетную запись Soda PDF.
    \ n

    \ n Иногда может потребоваться несколько минут, чтобы показать, что ваша учетная запись была подтверждена в нашем приложении.
    \ n Подписаться следующие шаги, чтобы ускорить процесс, если вы уже вошли в систему.\ n

    «, «PdfCreatorLink»: «https://www.sodapdf.com/pdf-creator/», «UnlockTitle»: «Разблокировать PDF», «EsignPdf»: «Электронная подпись PDF», «SodaNewTitle»: «Присоединяйтесь к революции онлайн-PDF», «AnnualPlan»: «Годовой план», «sloganOnline»: « PDF ONLINE», «CreateCustomForms»: «Создавать собственные формы», «errorEmailPassword»: «К сожалению, Soda PDF не распознает это письмо», «SplitPdf»: «Разделить PDF», «chatSchedule»: «С понедельника по пятницу (с 9:00 до 17:00 по восточноевропейскому времени)», «businessBrochure»: «Деловая брошюра», «GifToPdfLink»: «https: // www.sodapdf.com/gif-to-pdf/ «, «verifySpam»: «Чтобы обеспечить доставку электронной почты, проверьте настройки спама», «JpgToGifLink»: «https://www.sodapdf.com/jpg-to-gif/», «productAddOnTooltip»: «Этот продукт является надстройкой и автоматически добавляется к любому продукту Soda PDF, который использует назначенный пользователь.», «assignBy»: «Назначено», «включает»: «Включает:», «emailSent»: «Электронное письмо отправлено», «emailWord»: «Электронная почта», «secureItem2»: «Все загруженные и обработанные файлы удаляются с наших веб-серверов в течение максимум 24 часов за активный сеанс.», «secureItem1»: «Когда вы загружаете файлы, они преобразуются через безопасное зашифрованное соединение (https), чтобы оставаться на 100% безопасным.», «capsLock»: «Caps Lock включен», «freeOnlineToolsHeader»: «Бесплатные онлайн-инструменты», «reviewingFiles»: «Просмотр файлов», «PptToPdfLink»: «https://www.sodapdf.com/ppt-to-pdf/», «howActivateSodaLink»: «https://support.sodapdf.com/hc/en/articles/360022497971-How-to-Activate-Soda-PDF», «TapAddFile»: «Нажмите, чтобы добавить файлы», «OptInSubmit»: «Я согласен получать сообщения об этой услуге по электронной почте.», «UseinDesktopApp»: «Использовать в настольном приложении», «myAccount»: «Моя учетная запись», «errorUsersRequired»: «Введите количество пользователей», «desktopSolutionLink»: «https://www.sodapdf.com/pdf-download/», «Popular»: «Популярные», «newVersion»: «Доступна новая версия!», «СпасибоCTA1notice_top»: «», «BatchConvert»: «Пакетное преобразование», «labelStreetAddressLine»: «Строка с адресом улицы», «secureSignModule»: «Безопасность и подпись», «sendMeUpdates»: «Да, присылать мне обновления», «СпасибоCTA2notice_top»: «», «cancelPlan»: «Отменить план», «mo»: «Mo», «on»: «on», «или или», «Нет нет», «Ладно ладно», «btnDownloadViewText»: «Загрузить и просмотреть в браузере», «userExists»: «Пользователь с этим адресом электронной почты уже существует», «ResetFormLabel»: «Сбросить форму», «OtherTools»: «Другие инструменты», «manualSecureFile»: «Безопасное объединение и обработка файлов», «Вращение»: «Вращение», «SignaturePackagePart2»: «пакет подписи», «formFileAttachment»: «Вложение файла», «Изменение размера»: «Изменение размера», «PrivacyFeedback»: «Конфиденциальность
    Отзыв», «ConvertFiles»: «Конвертировать файлы PDF», «ConvertImage»: «Конвертировать изображение», «ExcelToPdf»: «Excel в PDF», «ConverttoPDF»: «Преобразовать в PDF», «ExcelToPdfLink»: «https: // www.sodapdf.com/excel-to-pdf/ «, «selectJobRole»: «Выберите должность», «errorPassProtected»: «Файл защищен паролем», «PdfToWordLink»: «https://www.sodapdf.com/pdf-to-word/», «bottomWord»: «Снизу», «videoTutorials»: «Видеоуроки», «btnWorkOfflineLink»: «Загрузить настольную версию!», «AnnualCommitment»: «Годовое обязательство», «registerAgreePart2»: «и наш», «registerAgreePart1»: «Нажимая» Зарегистрироваться «, вы соглашаетесь с», «accountDetailsText»: «Вы можете обновить свою платежную информацию», «clickHere»: «Щелкните здесь», «ProcessConverting»: «Преобразование», «unlimitedSodaESign»: «Электронная подпись безлимитных газированных напитков», «accessSaas»: «Доступ к Soda PDF Online здесь», «ProtectPdfLink»: «https: // www.sodapdf.com/password-protect-pdf/ «, «ResendConfirmationEmail»: «Отправить письмо с подтверждением еще раз», «JpgToPdfLink»: «https://www.sodapdf.com/jpg-to-pdf/», «sendToEmail»: «Отправить по электронной почте», «eSign»: «eSign PDF», «email»: «Электронная почта», «error»: «Произошла ошибка. Повторите попытку или свяжитесь с нами.», «SodaTradeMark»: «Soda ™ является товарным знаком LULU Software ™.», «forms»: «Формы», «logIn»: «Войти», «часы»: «часы», «title»: «Заголовок», «SSLLabelThree»: «безопасное соединение», «Begins_auto»: «(Начинается автоматически после оплаты)», «ErrorChooseMorePDF»: «Выберите два или более файлов PDF», «video»: «Видео», «linkExpired»: «Срок действия вашей ссылки истек», «добавить»: «добавить», «пока пока», «выкл»: «выкл», «ocr»: «OCR», «odd»: «odd», «верх»: «верх», «Все»: «Все», «Новый»: «Новый», «Да»: «Да», «PerpetualLicense»: «Бессрочная лицензия», «year2»: «2 года», «Первый»: «Первый», «Слияние»: «Слияние», «LoginLink»: «https: // www.sodapdf.com/account/login/ «, «no_limitation»: «24/7: без ежедневных ограничений
    Дополнительные возможности: создание, преобразование и просмотр файлов PDF», «PdfEditorLink»: «https://www.sodapdf.com/pdf-editor/», «Юридический»: «ЮРИДИЧЕСКИЙ», «productStatus»: «Статус», «Отключить»: «Отключить», «errorPasswordRequired»: «Введите пароль», «SodaPDFDesktop»: «Рабочий стол Soda PDF», «Голоса»: «Голоса», «Инструменты»: «Инструменты», «Сброс»: «Сброс», «Диапазон»: «Диапазон», «Сплит»: «Сплит», «subscribe_prgh»: «Будьте в курсе всех новостей Soda, включая информационные бюллетени, советы и рекомендации, а также эксклюзивные предложения.», «Планы»: «Планы», «JpgToPngLink»: «https://www.sodapdf.com/jpg-to-png/», «reassignLicense»: «Переназначить лицензию», «Вставка»: «Вставка», «sodaPdfOnline»: «Soda PDF Online», «BEST_VALUE»: «BEST VALUE», «batchTitle»: «Пакетное назначение», «GifToPngLink»: «https://www.sodapdf.com/gif-to-png/», «SplitTitle»: «Разделить PDF», «split_unlim»: «Разделить неограниченное количество файлов.», «contactsSales»: «Связаться с отделом продаж», «BilledAnnualy»: «выставляется ежегодно», «addPageNumbering»: «Добавить номера страниц», «вебинары»: «вебинары», «good_quality»: «Хорошее качество», «EnterUrl»: «Введите URL», «productTypeDesctop»: «Рабочий стол», «FreeOnlineTools»: «Бесплатные онлайн-инструменты», «Pdf2Image»: «PDF в JPG», «AddMain maintenance»: «Добавить обслуживание», «howToSubAlt3»: «Загрузить объединенный PDF», «howToSubAlt2»: «Объединить желаемые файлы PDF», «howToSubAlt1»: «Загрузить PDF», «PdfCreator»: «PDF Creator», «uninstall»: «Удалить», «FreePdfReaderMacOs»: «Читатель для Mac OS X», «WinPdfReader»: «Читатель Магазина Windows», «errorEnterB2BEmail»: «Пожалуйста, введите действующий рабочий адрес электронной почты, чтобы продолжить.», «discoverSodaPDf»: «Откройте для себя Soda PDF», «yourDownloadLinkSent»: «Ссылка для скачивания отправлена ​​на ваш адрес электронной почты.», «Количество»: «КОЛИЧЕСТВО», «createPasswordSubTitle»: «Установите новый пароль для своей учетной записи.», «Download_Desktop»: «Загрузить настольную версию!», «ViewEdit»: «Просмотр и редактирование», «errorPhoneInvalid»: «Введите действительный номер телефона», «errorCompanyRequired»: «Введите название компании», «Особенности»: «Особенности», «EmailConfirmationError»: «OOPS! Срок действия вашей ссылки для активации истек.», «PdfToHtmlLink»: «https: // www.sodapdf.com/pdf-to-html/ «, «bf_freeocrgift1»: «БЕСПЛАТНЫЙ ПОДАРОК ​​OCR («, «bf_freeocrgift2»: «значение)», «RessellerLink»: «https://www.sodapdf.com/business/resellers/», «fontSize»: «Размер шрифта», «productExpiredProducts»: «Товары с истекшим сроком годности», «SupportLink»: «https://support.sodapdf.com/hc/en-us/», «mostPopular»: «САМЫЕ ПОПУЛЯРНЫЕ», «errorPhoneRequired»: «Введите номер телефона», «Сжать»: «Сжать», «aboutTitle»: «О НАШИХ ИНСТРУМЕНТАХ», «howCanWeHelpYou»: «Чем мы можем вам помочь?», «LimitationTextRights»: «все права защищены.», «PrivacyFeedbackImg»: «//privacy-policy.truste.com/privacy-seal/LULU-software/seal?rid=e691fbfb-8de4-4b17-b576-70688b60730d», «rotated_copy»: «Загрузить повернутую копию вашего файла.», «selectIconFile»: «Пожалуйста, выберите файл значка», «proOcrPackage»: «Пакет Pro + OCR», «privacyPolicy»: «Политика конфиденциальности», «BusinessLink»: «https://www.sodapdf.com/business/», «splitted_copy»: «Загрузить разделенные страницы.», «SplitPDFSiteLabel»: «Разделить файлы PDF на», «recoveryPasswordEnterEmail»: «Вы можете сбросить пароль для своего профиля учетной записи, введя свой адрес электронной почты.», «supportNav»: «Поддержка», «PDFexceed»: «Загруженные файлы превышают максимальный размер», «ArticleDescriptionEditTwo»: «PDF-файлы — очевидный выбор, если вы хотите безопасно обмениваться информацией через Интернет. Компании и правительства в значительной степени полагаются на них, и большинство людей имеют общее представление о том, что такое PDF-файлы.», «ArticleDescriptionEditOne»: «Вы получаете электронное письмо, содержащее этот важный документ, волшебный PDF-файл, который выведет ваш бизнес на новый уровень. Этот PDF-файл содержит предложение, в котором каждая деталь должна быть доведена до совершенства.», «addWatermark»: «Добавить водяной знак», «DetailsLink»: «https://www.sodapdf.com/account/manage-account/», «информационный бюллетень»: «Информационный бюллетень», «newPassword»: «Новый пароль», «ThankyouCTA1»: «ОТКРЫТЬ», «ThankyouCTA2»: «СКАЧАТЬ», «ThankyouBack»: «Вернуться на сайт», «Пример»: «Пример», «options_text_8»: «Оптическое распознавание символов (OCR)», «reassign_prgh2»: «Вы не можете переназначить эту лицензию тому же пользователю в течение этого платежного цикла.», «createAccount»: «Создать учетную запись», «footerCopyTextLight»: «Этот продукт продается компанией Upclick.com в качестве авторизованного реселлера. «, «cmWord»: «Сантиметры», «dailytimer»: «Вы превысили почасовой лимит бесплатных задач. Вы можете повторить попытку через ::», «ArticleTitleEditThree»: «Как редактировать документы PDF», «sodaPdfForYou»: «Газировка PDF для вас», «PdfToHtml»: «PDF в HTML», «PdfToDocx»: «PDF в DOCX», «PdfToWord»: «PDF в Word», «PdfToJpgLink»: «https://www.sodapdf.com/pdf-to-jpg/», «signInTitle»: «Войдите в свою учетную запись Soda PDF с помощью», «enterCity»: «Введите город», «productProductAlert»: «Срок действия вашего плана истекает, и вы потеряете доступ к его функциям по истечении срока его действия.», «errorLastNameInvalid»: «Необходимо ввести действительную фамилию», «Reader3d»: «3D-читатель», «dayliLimitSubTitleB»: «Однако у вас есть другой вариант», «PDFMergeCanonical»: «https://www.pdfmerge.com/», «ErrorUploadOnlyPDF»: «Пожалуйста, загружайте только файлы PDF», «Jpg2pdf»: «JPG в PDF», «businessResourcesPageName»: «Бизнес-ресурсы», «userGuide»: «Руководство пользователя», «resourceCenterBackText»: «Вся информация, необходимая для поиска ответов на ваши вопросы.», «yourWebinarShouldBegin»: «Ваш веб-семинар должен начаться в ближайшее время.», «GoodQualityBest»: «лучшее качество изображения, минимальное сжатие», «withMain maintenance»: «С обслуживанием», «EasyAdoptionPageName»: «Простое принятие», «unassignProduct_prgh2»: «После отмены назначения определенной лицензии лицензию можно переназначить тому же пользователю только после следующего цикла выставления счетов.», «downloadInstallation»: «Скачать / Установка», «Process_another»: «Обработать другой файл», «accountAssociated»: «С этим адресом электронной почты уже связана учетная запись.», «mustUploadCSV»: «Вы должны загрузить файл CSV», «download»: «Скачать», «Trial30Day»: «30-дневная пробная версия», «sodaAnywherePrgh»: «Полное решение в формате PDF для настольных компьютеров и в Интернете», «MoreOnePage»: «Загруженный документ должен содержать более 1 страницы.», «PaymentInformation»: «Платежная информация», «pdfCreatorConverter»: «Бесплатная программа для создания и преобразования PDF-файлов», «RateTool»: «Оценить этот инструмент», «MergePdfLink»: «https://www.sodapdf.com/pdf-merge/», «ResendAssignInvitationSuccess_prgh2»: «Приглашение было повторно отправлено», «getTheMost»: « Получите максимум от своих денег», «SplitPassProtected»: «Загруженный файл защищен паролем и не может быть разделен», «BackToSoda8»: «Вернуться к Soda PDF», «yourRequestReceived»: «Ваш запрос получен.», «perMonth»: «В месяц», «минуты»: «минуты», «continueBtn»: «Продолжить», «createPasswordTitle»: «Создайте свой пароль», «bf_features»: «Функции включены:», «ChooseFormat»: «Выбрать формат:», «aboutSubDesc4»: «Вы можете обрабатывать файлы на любом устройстве, в любое время и в любом месте с помощью компьютера, планшета и смартфона.», «aboutSubDesc1»: «Мы используем безопасную технологию для установления зашифрованного соединения между нашим веб-сервером и вашим браузером, чтобы все данные оставались конфиденциальными.», «aboutSubDesc3»: «Доступ к файлам, сохраненным в облачных системах хранения, таких как Google Drive, Box, Dropbox и OneDrive.», «aboutSubDesc2»: «Мы храним каждый файл на нашем сервере только в течение 24 часов, чтобы ограничить любой несанкционированный доступ. Затем он навсегда удаляется с наших серверов. Никто из нашей команды не может получить доступ к этим файлам.», «PngToPdfLink»: «https://www.sodapdf.com/png-to-pdf/», «TiffToPdf»: «TIFF в PDF», «ExtractPdf»: «Извлечь PDF», «errorServer»: «Извините, сервер занят. Повторите попытку позже.», «detailEsignPhone»: «Этот номер используется нашей службой E-Sign для аутентификации по SMS», «StayUpToDate»: «Будьте в курсе!», «marginsWord»: «Поля», «offPrice»: «выкл», «errorNewPasswordRequired»: «Введите новый пароль», «insuffTitle»: «Недостаточно лицензий», «errorContactEmailInvalid»: «Вам необходимо ввести действующий контактный адрес электронной почты.», «onlineAccess»: «Доступ в Интернете», «errorCurrentPasswordRequired»: «Введите текущий пароль», «premiumPhoneSupportBackText»: «Прямой доступ к одному из наших специалистов по Soda PDF в любое время.», «productRefreshList»: «Обновить список», «PngToJpgLink»: «https://www.sodapdf.com/png-to-jpg/», «userGuideLink»: «http://userguide.sodapdf.com/», «MacOsUser»: «Пользователь Mac OS? Откройте для себя полнофункциональный Soda PDF Online.», «InsertPageElem»: «Вставить элементы страницы», «IncludedPrgh»: «Включено в следующие планы», «freeTrialLink»: «https: // онлайн.sodapdf.com/ «, «productAssignedLicenses»: «Назначенные лицензии», «Загрузка»: «Загрузка», «noCreditCard»: «Кредитная карта не требуется», «emailHasBeenChanged»: «Ваш адрес электронной почты был изменен», «messageEmailSent»: «Ссылка для подтверждения была отправлена ​​на ваш адрес электронной почты. Если вы не получили это письмо, проверьте папку нежелательной почты / спама.», «chooseEmailToMerge»: «Пожалуйста, выберите адрес электронной почты для объединения продуктов из обеих учетных записей. Этот адрес электронной почты и соответствующий пароль будут использоваться для входа в вашу учетную запись после успешного объединения», «formMessage»: «Сообщение», «confirmUsers»: «Подтвердите пользователей, которым вы хотите назначить лицензии», «ChooseFile»: «Выбрать файл», «useOnlineTools»: «Воспользуйтесь нашим онлайн-инструментом», «privacyTitle»: «Конфиденциальность», «errorNotPdf»: «Файл не является PDF-документом», «formLastName»: «Фамилия», «Параметры»: «Параметры», «pageNumber»: «Номер страницы», «numberFormat»: «Формат числа», «settingsUpdated»: «Настройки вашей учетной записи успешно обновлены», «upgradeBuilder»: «Конструктор обновлений», «Непрозрачность»: «Непрозрачность», «статьAReseller»: «Стать реселлером», «formPhone»: «Телефон», «PDFClicking»: «Нажав кнопку« Оплатить сейчас »ниже, вы попадете на защищенный сайт PayPal
    (иметь учетную запись PayPal не обязательно).», «email_terms_begin»: «Отправляя электронное письмо, вы соглашаетесь получить файл и быть связанными условиями», «email_terms_link1»: «Условия использования», «email_terms_link2»: «Политика конфиденциальности», «errorLastNameRequired»: «Введите фамилию», «formSuccessMessage»: «Спасибо за запрос. Служба поддержки свяжется с вами в ближайшие 12-24 часа.», «formEmail»: «Адрес электронной почты», «resetLicense»: «Сбросить лицензию», «currentPassword»: «Текущий пароль», «pdfDownload»: «https://www.sodapdf.com/pdf-download/», «formTimelineJustBrowsing»: «Просто просматриваю», «PlanBusinesPrgh3»: « Soda E-Sign Unlimited включен в бизнес-план Soda PDF Business», «PlanBusinesPrgh2»: «Полное решение PDF, разработанное для профессионалов», «UnlockLink»: «https: // www.pdfunlock.com/ «, «WebDeskApp»: «Веб + настольные приложения», «errorFirstNameRequired»: «Введите имя», «formFailMessage»: «Невозможно отправить сообщение. Повторите попытку позже.», «buyOnline»: «Купить в Интернете», «btnAnotherFile»: «Обработать другой файл», «online_tools»: «ИНСТРУМЕНТЫ ДЛЯ ОНЛАЙН PDF», «SignSecure»: «Подписать и защитить», «smfileSign»: «Неограниченные возможности PDF», «unlock_unlim»: «Разблокировать неограниченное ЧИСЛО файлов.», «реселлеры»: «реселлеры», «sodaOnline»: «https://online.sodapdf.com/», «ManagePdfFilesNav»: «Управление файлами PDF», «searchDeskPlaceholder»: «Найдите здесь, чтобы просмотреть нашу базу знаний», «ViewerLink»: «https: // www.sodapdf.com/products/pdf-reader/ «, «bf_features_text»: «Просмотр, создание, преобразование, редактирование, вставка, проверка, формы, защита и подпись», «EditLink»: «https://www.sodapdf.com/pdf-editor/», «DonationLineTwo»: «Поблагодарите, сделав небольшое пожертвование.», «DonationLineOne»: «Помог ли этот сайт вам сэкономить (или заработать) немного денег?», «validationMsg»: «Ваша учетная запись подтверждена», «email_terms_and»: «и», «EditFiles»: «Редактирование файлов», «ResendEmail»: «Отправить электронное письмо повторно», «ArticleTitleOne»: «Как добавить страницы в PDF-файлы», «ArticleTitleTwo»: «Как пакетно создавать файлы PDF», «footerCopyTextRights»: «Все права защищены.», «resetLicense_prgh2»: «Вы можете сбросить бессрочную лицензию только дважды в течение года.», «good_quality_text»: «хорошее качество, средний уровень сжатия», «OcrPdfLink»: «https://www.sodapdf.com/ocr-pdf/», «errorWebsiteUrlInvalid»: «Введите действительный URL», «SignUpWith»: «Зарегистрируйтесь с помощью», «InvalidRange»: «Недопустимый диапазон», «PurchaseFirstTime»: «Вы недавно приобрели продукт Soda PDF и впервые получаете доступ к Soda? Создайте учетную запись с адресом электронной почты, который вы использовали при покупке.», «layoverTitlePart1»: «Ваш файл будет готов к загрузке через», «layoverTitlePart2»: «секунды:», «ThankyouCTA1notice_bottom»: «в веб-браузере», «sendEmail»: «Отправка электронной почты …», «MoreFilesLabel»: «Еще файлы», «Аффилированные лица»: «Аффилированные лица», «ArticleDescriptionEditThree»: «В наши дни получение бумажных документов для просмотра и редактирования — большая редкость, особенно в профессиональной среде. Обмен документами сейчас в основном осуществляется в электронном виде, а безопасный способ отправки файла — преобразование его в PDF. первый.», «affiliateProgram»: «Партнерская программа», «chatBackText»: «Живой чат с одним из наших специалистов по Soda PDF.», «PdfEditor»: «Редактор PDF», «See_also»: «СМОТРИ ТАКЖЕ», «errorNewPasswordMatch»: «Ваши новые пароли не совпадают», «errorCurrentPasswordIncorrect»: «Ваш текущий пароль неверен», «DropFileHereOr»: «Перетащите файл сюда или», «textAndFormat»: «Текст и формат», «NotConnected»: «Не подключен», «updateInformation»: «Обновить информацию», «PdfToPptLink»: «https: //www.sodapdf.com / pdf-to-ppt / «, «CompressFile»: «Сжать файл», «CompressLink»: «https://www.sodapdf.com/compress-pdf/», «download_here»: «Скачайте здесь», «subscribeToProduct»: «Обновления продукта», «AnnualCommitment»: «Годовое обязательство», «GuaranteeText»: « 30-дневная Гарантия возврата денег
    «, «finishDisconnecting»: «Чтобы завершить отключение этой учетной записи, установите пароль для своей учетной записи Soda PDF. С этого момента этот пароль будет использоваться с вашей электронной почтой для входа в систему.», «E-SingLink»: «https: // www.sodapdf.com/sign-pdf/ «, «labelZipPostalCode»: «Почтовый индекс», «dayliLimitTitle»: «Вы превысили часовой лимит для PDFMerge», «registerAgreeWith»: «Выполняя вход с подключенной учетной записью, вы соглашаетесь с», «ResizePdfLink»: «https://www.sodapdf.com/resize-pdf/», «Ecx_options»: «Однако вы можете выбрать один из двух вариантов», «clickYouTube»: «Нажмите {0}, чтобы найти Soda PDF Anywhere», «active»: «Активный», «PDFFormFiller»: «Заполнитель PDF-форм», «formGetStarted»: «Начать работу», «noFileChosen»: «Файл не выбран», «errorAccountExists»: «Пользователь уже существует.», «ArticleTitleEditTwo»: «Как профессионально редактировать файлы PDF», «ArticleTitleEditOne»: «Как сделать PDF-файл редактируемым с помощью Soda PDF», «FeedbackLink»: «https://www.sodapdf.com/feedback/», «moduleFormsReq»: « Forms Module требует», «passwordChanged»: «Ваш пароль был успешно изменен», «AnywhereTitle»: «С помощью Soda PDF Anywhere вы можете выполнять работу буквально в любом месте.», «formNo»: «Нет», «PDF_Reviews»: «Обзоры в PDF-формате», «OnlinePricingLink»: «https: //www.sodapdf.ru / pricing / online / «, «productEnterEmail»: «Введите адрес электронной почты для назначения лицензии», «ForgotPassword»: «Забыли пароль?», «productTotalLicenses»: «Всего лицензий», «editPaymentCreditCard»: «Изменить платежную информацию для кредитной карты», «PdfToDocxLink»: «https://www.sodapdf.com/pdf-to-docx/», «BuyNowLink»: «https://www.sodapdf.com/pricing/», «DeletePdfLink»: «https://www.sodapdf.com/delete-pdf-pages/», «bf_title»: «Черная пятница — Киберпонедельник», «ImpressumLink»: «https: //www.sodapdf.ком / де / импрессум / «, «RenewPlan_prgh2»: «Ваш план настроен на продление.», «NoConnectedAccounts»: «Нет подключенных аккаунтов», «tryAgain»: «Чтобы создать учетную запись, повторите попытку и разрешите sodapdf.com доступ к вашему адресу электронной почты», «logOut»: «Выйти», «FromComputer»: «С компьютера», «productTitle»: «Мои товары», «productAssignToMe»: «Назначить мне», «DownloadFreeOnlineTools»: «https://www.sodapdf.com/buy/freeonlinetools/dw-success/», «aboutSubTitle4»: «Любое устройство», «aboutSubTitle1»: «Безопасность», «aboutSubTitle2»: «Конфиденциальность», «aboutSubTitle3»: «Доступ к облачному хранилищу», «addLicense»: «добавить лицензию», «insuffPrgh»: «Вы назначили больше лицензий, чем доступно в настоящее время.Измените свой выбор. «, «FullPdfSolution»: «ПОЛНОЕ РЕШЕНИЕ PDF», «formSend»: «Отправить», «ReceiverSubject»: «Квитанция о транзакции», «formName»: «Имя», «модуль»: «Модуль», «freeDevice»: «Бесплатное устройство», «productTypeOnline»: «Интернет», «RotateLink»: «https://www.pdfrotate.com/», «Безлимитный»: «Безлимитный», «TryDesktopVersion»: «Попробуйте нашу версию для ПК», «SplitCompress»: «Разделить и сжать», «monthCommitment»: «Ежемесячное обязательство», «passwordRequirements»: «Ваш пароль должен состоять не менее чем из 6 символов и содержать буквы и цифры», «confirmPassword»: «Подтвердите пароль», «errorFirstNameInvalid»: «Необходимо ввести действительное имя», «security»: «Безопасность», «generalUsability»: «Общее удобство использования», «protected_copy»: «Загрузите защищенную копию вашего файла.», «OnlineServices»: «Online Services», «DropFilesHereOr»: «Перетащите файлы сюда или», «AffiliatesLink»: «https://affiliates.lulusoftware.com/?», «создать»: «создать», «formCountry»: «Страна», «CreateAccountWith»: «Создать учетную запись Soda PDF с», «formCompany»: «Компания», «пароль»: «Пароль», «mergeCongratulations»: «Поздравляем, вы успешно объединили свои аккаунты.», «productFeatures»: «Характеристики и преимущества», «PptToPdf»: «PPT в PDF», «FreeFinePrint»: «* Бесплатно для файлов размером до», «footerLuluContactUs»: «Свяжитесь с нами», «fileReadySubTitle»: «Получите файл за 2 простых шага», «addESign10pack»: «Добавить 10 пакетов E-Sign», «RotateTitle»: «Повернуть PDF», «Сжатие»: «Сжатие», «CompressPdf»: «Сжать PDF», «resizeWord»: «Изменить размер», «autoRenewDescription»: «Щелкните здесь, чтобы включить автоматическое продление», «DocxToPdf»: «DOCX в PDF», «вставить»: «Вставить», «RotatePdf»: «Повернуть PDF», «resources»: «Ресурсы», «DragToRange»: «Перетащите, чтобы изменить расположение», «largefile»: «Большой файл», «DonateButtonLabel»: «Пожертвовать», «TermsUse»: «Условия использования», «mediumfile»: «Средний файл», «enterStateProvince»: «Укажите штат / провинцию», «PDF_ANYWHERE»: «PDF ANYWHERE», «typeOfProduct»: «Тип товара», «productInterest»: «Интересующий продукт», «FreeOnlineToolsLinkAnchor»: «https: // www.sodapdf.com/#navOnlineTools «, «compress_unlim»: «Сжать неограниченное ЧИСЛО файлов.», «resetPasswordSuccessfully»: «Ваш пароль был успешно сброшен.», «GoogleExtHtmlLink»: «https://chrome.google.com/webstore/detail/soda-pdf-convert-merge-sp/gfjafjofnehohehighdlkhcpanocobjb?hl=en», «ocrPDF»: «OCR PDF», «PDF2_text»: «Soda PDF 10 теперь поддерживает PDF 2.0 и все его богатые новые функции! PDF 2.0 — первое обновление формата PDF за 10 лет, которое включает в себя улучшения безопасности, доступности и общего удобства работы с PDF.Узнайте больше обо всех удивительных улучшениях, которые предоставляет PDF 2.0! «, «visitBlog»: «Посетить блог», «messageValidateYourAccount»: «Подтвердите его сейчас, щелкнув ссылку для подтверждения, которая была отправлена ​​на ваш адрес электронной почты.», «headerForYou»: «Для вас», «UnlockPdf»: «Разблокировать PDF», «Разблокировка»: «Разблокировка», «subtitleRequestQuote»: «Бизнес-клиенты имеют право на оптовые цены, начиная с 25 лицензий. Заполните форму ниже, и наши специалисты по продажам свяжутся с вами в течение 1 рабочего дня.», «GifToPdf»: «GIF в PDF», «GifToPng»: «GIF в PNG», «GifToJpg»: «GIF в JPG», «WebPDFApp»: «https: // www.sodapdf.com/web-pdf-app/ «, «PDF_Create»: «Создание, преобразование и просмотр файлов PDF», «billingInquiry»: «Billing Inquiry», «language»: «Язык», «customerSupport»: «поддержка клиентов», «formYes»: «Да», «headerForBusiness»: «Для бизнеса», «cancelPlan_prgh2»: «Если вы отмените свой план, вы потеряете доступ к его функциям по истечении срока действия.», «inchWord»: «Дюймы», «ProductsLink»: «https://www.sodapdf.com/account/manage-products/», «UnlimitedSignaturePack»: «Пакет безлимитных подписей», «update»: «Обновить», «knowledgebase_prgh»: «Нужна дополнительная помощь? Ознакомьтесь с нашими», «securingFiles»: «Защита файлов», «labelPhone»: «Телефон», «PassProtected»: «», «account_list»: «
  • Щелкните свое имя в правом верхнем углу приложения.
  • \ n
  • Щелкните кнопку обновления, чтобы убедиться, что все обновлено.
  • «, «MergePdf»: «Объединить PDF», «SecureEdit»: «Защищай и редактируй», «errorAccountAlreadyAssociated»: «Аккаунт уже связан с этим адресом электронной почты», «productTwoDevicesMessage»: «Два устройства могут войти в Soda PDF Desktop в любой момент времени. Используйте X, чтобы удаленно выйти из системы.», «product»: «Товар», «pricing»: «Цена», «конфиденциальность»: «конфиденциальность», «TxtToPdfLink»: «https://www.sodapdf.com/txt-to-pdf/», «choosePassword»: «Выберите пароль», «Downloadh3OLink»: «https: // download11.sodapdf.com/api/get-h3o?configid=54E98DCD-07B7-4F5B-BEC7-ED1A0EC50D8F&bundleid=SO003 «, «TiffToPdfLink»: «https://www.sodapdf.com/tiff-to-pdf/», «packageStandard»: «Стандарт», «emailRequired»: «Пожалуйста, введите свой адрес электронной почты», «requestQuote»: «Запросить цитату», «formLicensesNeeded»: «Необходимые лицензии», «formDoYouOwnPDFsoftware»: «У вас есть программное обеспечение для работы с PDF?», «privacyText»: «При использовании нашего веб-приложения файл, над которым вы работаете, будет храниться не более 24 часов за активный сеанс.После этого он будет удален с нашего сервера. «, «emailAlreadyAssociated»: «\» Этот адрес электронной почты уже связан с учетной записью Soda PDF. Если эта учетная запись принадлежит вам, вы можете объединить свои учетные записи \ «», «ThankyouReadyFile»: «Ваш файл готов», «productPaymentProblem»: «При обработке вашего платежа возникла проблема, обновите платежную информацию», «PdfToExcel»: «PDF в Excel», «SaasAccess»: «SaaS — доступ к Soda PDF Online», «contactUs»: «Свяжитесь с нами», «NeedHelp»: «Нужна помощь?», «Thankyou_de_end»: «», «free30DayTrial»: «Бесплатная 30-дневная пробная версия», «pleaseSignIn»: «Пожалуйста, войдите, используя», «absoluteScale»: «Абсолютная шкала», «labelJobRole»: «Должность», «recoveryPasswordSentEmail»: «На ваш аккаунт было отправлено электронное письмо для сброса пароля.», «formSubscribe»: «Подписаться», «TheFileIsCorrupted»: «Файл поврежден и не может быть открыт», «Word2pdf»: «Word в PDF», «bf_features_text_2»: «Без ограничений: объединение, преобразование, редактирование, вставка, сжатие, просмотр, формы, защита и подпись и многое другое! \ n», «WhatsNew»: «Что нового», «DownloadNow»: «Загрузить сейчас», «support»: «Поддержка», «AddFiles»: «Добавить файлы», «PDF_Editor»: «Редактор PDF», «formTimelineSoon»: «Скоро», «validateNewEmail»: «Подтвердите свой новый адрес электронной почты, щелкнув ссылку для подтверждения, которая была отправлена ​​на новый адрес электронной почты.После того, как вы подтвердите свой новый адрес электронной почты, изменение адреса электронной почты будет завершено. Обратите внимание, что если вы снова попытаетесь изменить свой адрес электронной почты до подтверждения, этот запрос на изменение будет недействительным. «, «check_product»: «чтобы посмотреть наш
    \ n обзор продукта «, «ConvertLink»: «https://www.sodapdf.com/pdf-converter/», «invalidEmail»: «Недействительный адрес электронной почты — нельзя назначить этому пользователю», «SSLLabelOne»: «В вашем файле есть что-то личное или конфиденциальное?», «SSLLabelTwo»: «Рассмотрите возможность использования», «вебинар»: «Вебинар», «bf_subtitle»: «Самая низкая цена года — гарантировано! «, «Местоположение»: «Местоположение», «HowToEditorPDF»: «Как редактировать файлы PDF», «view3d»: «Вид / 3D», «EditFiles»: «Редактировать файлы PDF», «errorEmailInvalid»: «Введите действующий адрес электронной почты», «MediumQuality»: «Среднее качество», «accountDetails»: «Детали учетной записи», «UnlimitedSignatures»: «Неограниченное количество подписей», «AdobeAlternativeLink»: «https: // www.sodapdf.com/adobe-alternative/ «, «MyAccountLink»: «https://www.sodapdf.com/account/manage-account/», «emailPreferences»: «Настройки электронной почты», «pageSize»: «Размер страницы», «topWord»: «Сверху», «SignaturePackage»: «Пакет подписи», «weWorking»: «

    Меня уволили после того, как я заснул на личных документах.

    \ n

    Похоже вы не можете лгать в своем резюме.

    «, «installationGuide»: «Руководство по установке», «ResizePdf»: «Изменить размер PDF», «GetStarted»: «Начать работу», «UNLIMITED_FILES»: «НЕОГРАНИЧЕННЫЕ ФАЙЛЫ», «SodaOnlineLink»: «https: // онлайн.sodapdf.com/ «, «productNotSure»: «Не уверен», «ProtectPdf»: «Защитить PDF», «ready_title»: «Готовы начать?», «MoreAbout»: «БОЛЬШЕ О PDF», «errorEmailRequired»: «Введите адрес электронной почты», «days»: «days,», «edit»: «Редактировать», «даже»: «даже», «font»: «Шрифт», «бесплатно»: «бесплатно», «назад назад», «blog»: «Блог», «chat»: «Чат», «Здесь, здесь», «note»: «* Могут применяться ограничения по размеру и ежедневному использованию.», «план»: «План», «view»: «view», «сохранить»: «сохранить», «EULA»: «EULA», «Файл»: «Файл», «Desc»: «PDF Merge позволяет вам объединять свои файлы PDF в Интернете.Никакой установки, никакой регистрации, это бесплатно и просто в использовании. «, «Последний»: «Последний», «Дом»: «Дом», «Текст»: «Текст», «resetPasswordLink»: «Срок действия ссылки для сброса истек.», «winterTitle»: «Ура! Ваш файл готов, и у нас есть для вас отличное предложение», «footerLuluAboutUs»: «О нас», «BmpToJpgLink»: «https://www.sodapdf.com/bmp-to-jpg/», «TermOfUseLink»: «https://www.sodapdf.com/terms-of-use/», «обзор»: «обзор», «redOff»: «Скидка 60%», «BatesNumbering»: «Нумерация Бейтса», «bestValue»: «Лучшее соотношение цены и качества», «useSocial»: «Используйте свою учетную запись Facebook, Google или Microsoft для регистрации или заполните форму ниже, чтобы создать учетную запись Soda PDF.», «howToTitleConvert»: «Сохраненный макет», «didYouPrgr»: «Преобразование в PDF или преобразование из PDF гарантирует, что любой, кто увидит ваш документ, увидит Word в Интернете так, как вы предполагаете, независимо от своего средства просмотра PDF или устройства. Наши инструменты преобразования в PDF автоматически сохраняют ваши файлы в формате PDF 2.0. , что гарантирует, что файл будет совместим для просмотра в любой программе просмотра PDF. «, «howToSubTitle2»: «Загрузить PDF», «howToSubTitle1»: «Загрузить документ Word», «TitlePage»: «Word в PDF», «mkey1»: «WordToPdf», «OwnLinkRu»: «/ ru / word-в-pdf /», «OwnLinkSv»: «/ sv / word-till-pdf /», «OwnLinkVi»: «/ vi / word-to-pdf /», «OwnLinkPl»: «/ pl / word-to-pdf /», «OwnLinkPt»: «/ pt / word-para-pdf /», «OwnLinkTr»: «/ tr / word-to-pdf /», «OwnLinkFr»: «/ fr / word-en-pdf /», «OwnLinkJa»: «/ ja / word-to-pdf /», «OwnLinkKo»: «/ ko / word-to-pdf /», «OwnLinkDe»: «/ de / word-in-pdf /», «OwnLinkEs»: «/ es / word-a-pdf /», «OwnLinkEn»: «/ word-to-pdf /», «OwnLinkIt»: «/ it / word-in-pdf /», «OwnLinkId»: «/ id / word-to-pdf /», «howToSubDescConvert1»: «Конвертируйте DOC в PDF онлайн или конвертируйте из PDF, сохраняя макет и безопасность вашего документа.Ваш документ выглядит так же, как в Word, даже после процесса преобразования Word в PDF перетаскиванием. «, «howToSubDescConvert2»: «Благодаря нашему онлайн-конвертеру Word в PDF контент остается редактируемым независимо от формата. Изменяйте файлы, конвертируйте в PDF и конвертируйте из PDF с помощью нашего настольного приложения Soda PDF и онлайн-инструментов PDF. Конвертируйте из PDF БЕСПЛАТНО с помощью наших Инструмент PDF в Word. «, «noteWordToPdf»: «* Могут применяться ограничения по размеру и ежедневному использованию. Avanquest 2020, все права защищены.», «didYouSubTitle»: «Объяснение преобразования», «SubTitlePage»: «С легкостью конвертируйте ваши файлы docx в PDF в Интернете с помощью инструмента конвертации Soda PDF», «howToTitle»: «Как преобразовать из Word в PDF?», «howToSubDesc2»: «Получите доступ к документу PDF, загрузив файл преобразования PDF на свой компьютер.Просмотрите преобразование PDF из файла Word в своем браузере. Добавьте файлы на Google Диск или отправьте электронное письмо с защищенной ссылкой для преобразования Word в файл PDF. «, «howToSubDesc1»: «Загрузите документ Word со своего компьютера или перетащите его в окно преобразования PDF, чтобы начать процесс преобразования в PDF. Или загрузите файлы из облачного хранилища, такого как Google Диск или Dropbox.» }; вар lang = »; var serviceType = ‘RedesignedWordToPdf’

    Word в PDF — 100% бесплатно

    Если вы ежедневно работаете с файлами PDF, вам нужны отличные инструменты PDF.У PDFBear их более 20. Один из очень популярных — инструмент Word в PDF на сайте, он конвертирует файлы Microsoft Word в файлы PDF высокого качества за считанные минуты!
    Воспользуйтесь отличным сервисом без ущерба для качества файла — все форматирование, таблицы, изображения и текст в PDF будут выглядеть точно так же, как в исходном документе! Все еще не уверены? Посмотрите сами, посетите pdfbear.com, чтобы узнать, о чем мы все говорим!

    Нужно срочно отправить документ? В 90% случаев вас попросят отправить его в формате PDF.PDF, или переносимые файлы документов, сегодня являются одним из наиболее часто используемых типов файлов. Фактически, если вы когда-либо загружали онлайн-документ или форму для печати, велика вероятность, что это был файл PDF.

    Для работы, учебы или личного использования, если вы имеете дело с конфиденциальными файлами, есть вероятность, что вы используете PDF по двум причинам: он защищен паролем и поддерживает два уровня безопасности, один для открытия и просмотра документа. , и еще один, чтобы разрешить несколько разрешений, таких как редактирование, печать или преобразование.Если вы хотите преобразовать PDF-файл в Word онлайн, вам нужно сломать еще один уровень безопасности, чтобы сделать это.

    Лет назад, если бы большинству людей дали документ Microsoft Word, который нужно было зашифровать, первое, что они сделали бы, это преобразовали Word в PDF. Для этого им потребуется установить определенный тип программного обеспечения. В наши дни, благодаря чудесам техники, преобразование файлов стало более доступным. Есть много сайтов, которые могут превратить файл Microsoft Word в PDF за считанные минуты!

    Эти веб-сайты оснащены одним конвертером Word в PDF, который может конвертировать документ Word в PDF с помощью единого веб-инструмента.Разве это не потрясающе? Если вы так же удивлены, как мы, тем, как это работает, вы, вероятно, задаетесь вопросом, где происходит все волшебство. Что такое конвертер Word в PDF и как он работает?

    Что такое конвертер Word в PDF?

    Конвертер Word в PDF принимает документы Word и превращает их в файлы PDF за короткий промежуток времени. Через несколько минут пользователь получит идеально отформатированные PDF-документы — шрифт, изображения и выравнивание будут такими же, как и в исходном документе. Конвертер Word в PDF — популярный инструмент и незаменимый инструмент в колледжах, домах и на работе благодаря множеству полезных преимуществ.

    Конвертер Word в PDF избавляет от лишних хлопот. Чтобы использовать его, вам не понадобится дорогой, тяжелый, всеобъемлющий набор дополнительных инструментов. Все, что вам нужно сделать, это зайти на сайт и загрузить файл. Независимо от того, какой тип операционной системы вы используете, вы получите преобразованный PDF-файл всего за несколько кликов! Без сомнения, это один из лучших инструментов для преобразования файлов на рынке!

    Конвертировать Word в PDF онлайн бесплатно

    Представьте, что случилось бы, если бы конвертера Word в PDF никогда не существовало? Чтобы преобразовать документы Word в файлы PDF, вам нужно будет купить дорогое программное обеспечение, чтобы иметь возможность отправлять конфиденциальные файлы своим клиентам, начальнику, однокласснику или учителю!

    Как использовать конвертер PDFBear из Word в PDF?

    Введите слова «Word в PDF», и вы найдете множество конвертеров файлов в Интернете.Чертовски сложно выбрать только один, так как же выбрать? Независимо от того, являетесь ли вы предпринимателем, студентом или сотрудником, вы хотите работать с лучшими доступными через Интернет и мобильными опциями. В этом отношении вам будет приятно узнать, что ни один конвертер файлов не сравнится с PDFBear.

    PDFBear предлагает своим пользователям лучшую поддержку по нескольким направлениям — он конвертирует документы независимо от размера, бесплатно и онлайн. PDFBear также позволяет удалять определенные изображения из документа или конвертировать целые страницы с помощью одной настройки! Если вас беспокоит защита веб-сайта, вам будет приятно узнать, что при передаче файлов через PDFBear каждая передача защищена 256-битным SSL-соединением, а это технология шифрования данных и файлов, превышающая отраслевые нормы.

    Несмотря на то, что конвертер Word в PDF имеет множество замечательных функций, он очень прост в использовании. Не верите нам? Чтобы убедиться в этом, ознакомьтесь с приведенными ниже инструкциями!

    1. Для начала загрузите или перетащите документ Microsoft Word в область преобразования.
    2. Ваш документ Word будет отсканирован конвертером PDF.
    3. Дождитесь завершения преобразования.
    4. Загрузите недавно преобразованный файл PDF и сохраните его на своем устройстве.
    5. Через час PDFBear удалит ваш файл из своей системы.

    Как конвертировать Word в PDF онлайн бесплатно?

    В настоящее время востребованы онлайн-услуги. Люди предпочитают бесплатные онлайн-конвертеры для редактирования и преобразования своих документов. У использования этих онлайн-сервисов есть много плюсов и минусов. Одним из преимуществ является отсутствие необходимости устанавливать стороннее программное обеспечение при преобразовании документа Word в PDF. Но вам всегда потребуется подключение к Интернету, и сайт также может быть загружен рекламой. Кроме того, многие из этих онлайн-конвертеров Word в PDF лишены дополнительных функций.

    Lua

    Lua — это бесплатный онлайн-конвертер PDF, который не требует загрузки или входа в систему и доступен через любой браузер. Все, что вам нужно, это подключение к Интернету и браузер для преобразования или редактирования файлов.

    Он позволяет пользователям конвертировать документы различных форматов в PDF, например JPG в PDF. Кроме того, вы можете конвертировать PDF в другие форматы, такие как PDF в PNG. Помимо этого, пользователь также может объединять или сжимать PDF-файлы.

    Создатели

    Lua хотят показать нам, что в мире, где почти все стоит денег, все можно делать по-другому.Это программное обеспечение можно использовать совершенно бесплатно. Он работает очень быстро, гарантируя, что ваше форматирование останется неизменным. Однако он не преобразует файлы размером более 5 МБ.

    Adobe Acrobat

    Позволяет просматривать PDF-файлы в автономном и интерактивном режиме. Вы также можете легко подготавливать, редактировать, улучшать, систематизировать и распечатывать файлы PDF.

    Вы можете конвертировать файлы DOC, DOCX, RTF и TXT с помощью онлайн-инструмента Word в PDF с помощью любого веб-браузера. Чтобы преобразовать файл, просто перетащите его.Этот инструмент также сохраняет форматирование вашего документа. Используя этот онлайн-инструмент, выравнивание, шрифты и изображения вашего документа будут такими же, как и на Mac или Windows.

    Adobe Acrobat — удобный инструмент, который у вас должен быть. Он поддерживает телефоны Windows, Mac, Linux, Android и телефоны Windows. Таким образом, вы можете просматривать и обрабатывать свои файлы PDF как на мобильном телефоне, так и на ПК.

    Сода PDF

    Если вы работаете с PDF-файлами ежедневно, то Soda PDF — лучший вариант для вас.Пользователи могут получить доступ к функциям этого программного обеспечения на любом устройстве через веб-браузер.

    Он имеет множество инновационных функций, включая преобразование PDF, редактирование, рецензирование, нумерацию Бейтса, интеграцию с облаком и многое другое. Вы можете преобразовать любой документ Word в PDF и объединить несколько документов Word в один файл PDF.

    Вы можете использовать soda PDF вместе со сторонними приложениями, такими как Google Drive, Onedrive и Dropbox.

    Soda PDF позволяет вам создать бесплатную учетную запись и предлагает 7-дневную бесплатную пробную версию, предоставляющую доступ ко всему приложению.По истечении пробного периода вы сможете использовать только бесплатную версию.

    Nitro Pro

    Интерфейс этого программного обеспечения похож на Microsoft Office, поэтому его легко понять и использовать. Он выполняет две основные функции; повышение вашей производительности и выполнение всех задач, связанных с PDF.

    Он имеет быстрое создание PDF-файлов, которые могут преобразовывать все типы файлов Microsoft Office, WordPerfect, изображения и HTML. Вы можете создавать PDF-файлы и организовывать отдельные страницы по своему усмотрению.Он предлагает 14-дневную бесплатную пробную версию.

    Nitro Pro подходит как для технического, так и для личного использования и работает с любым настольным браузером. Однако это программное обеспечение совместимо только с Windows. В серии Nitro нет продуктов, совместимых с Mac или поставляемых с мобильными приложениями.

    Foxit PDF Reader

    Foxit PDF Reader позволяет пользователям создавать, конвертировать и редактировать PDF-файлы. Пользователи также могут принимать плагины и совместно работать и обмениваться документами по электронной почте и в социальных сетях.

    Это программа для чтения PDF-файлов, которая позволяет вам связываться с миром. Оставляйте комментарии к документам, получайте уведомления о появлении новых версий документов или безопасно открывайте защищенные документы.

    Он имеет мощную программу просмотра PDF-файлов с полным набором PDF-ресурсов. С помощью простого в использовании программного обеспечения для совместной работы приложение позволяет создавать и отображать PDF-файлы.

    Он имеет инструменты безопасности, которые защищают ваши файлы и устройство от вирусов. Несмотря на обширные функции программы, потребляет всего 81 файл.1 МБ вашей оперативной памяти.

    Есть мысли по этому поводу? Дайте нам знать в комментариях или перенесите обсуждение в наш Twitter или Facebook.

    Рекомендации редакции:

    Сохранение или преобразование в PDF или XPS на рабочем столе Project

    Чтобы экспортировать или сохранить в формате PDF, в файле Office в меню Файл щелкните Экспорт или Сохранить как .Чтобы просмотреть пошаговые инструкции, выберите программу Office из раскрывающегося списка.

    1. Откройте таблицу или отчет, который вы хотите опубликовать в формате PDF.

    2. На вкладке Внешние данные в группе Экспорт щелкните PDF или XPS

    3. В поле Имя файла введите или выберите имя документа.

    4. В списке Сохранить как тип щелкните PDF (* .pdf) .

      • Если документ требует высокого качества печати, щелкните Стандарт (публикация в Интернете и печать) .

      • Если размер файла важнее качества печати, щелкните Минимальный размер (публикация в Интернете) .

    5. Щелкните Параметры , чтобы задать страницу для печати, выбрать, следует ли печатать разметку, и выбрать параметры вывода. Нажмите ОК .

    6. Щелкните Опубликовать .

    Эта информация также относится к Microsoft Excel Starter 2010.

    Примечание. Вы не можете сохранять листы Power View в виде файлов PDF.

    1. Щелкните вкладку Файл .

    2. Щелкните Сохранить как .
      Чтобы увидеть диалоговое окно « Сохранить как » в Excel 2013 или Excel 2016, вам нужно выбрать место и папку.

    3. В поле Имя файла введите имя файла, если вы еще этого не сделали.

    4. В списке Сохранить как тип щелкните PDF (* .pdf) .

      • Если вы хотите, чтобы файл открывался в выбранном формате после сохранения, установите флажок Открыть файл после публикации .

      • Если документ требует высокого качества печати, щелкните Стандарт (публикация в Интернете и печать) .

      • Если размер файла важнее качества печати, щелкните Минимальный размер (публикация в Интернете) .

    5. Щелкните Параметры , чтобы задать страницу для печати, выбрать, следует ли печатать разметку, и выбрать параметры вывода.Дополнительные сведения о диалоговом окне «Параметры» в Excel см. В разделе «Дополнительные сведения о параметрах PDF». По завершении нажмите ОК .

    6. Щелкните Сохранить .

    OneNote 2013 и OneNote 2016

    1. Щелкните вкладку Файл .

    2. Щелкните Экспорт .

    3. В разделе Экспорт текущего выберите часть записной книжки, которую вы хотите сохранить в формате PDF.

    4. В разделе выберите Формат , щелкните PDF (*.pdf) , а затем щелкните Экспорт .

    5. В диалоговом окне Сохранить как в поле Имя файла введите имя записной книжки.

    6. Щелкните Сохранить .

    OneNote 2010

    1. Щелкните вкладку Файл .

    2. Щелкните Сохранить как , а затем щелкните параметр, который представляет часть записной книжки, которую вы хотите сохранить в формате PDF.

    3. В разделе Сохранить раздел как щелкните PDF (* .pdf) , а затем щелкните Сохранить как .

    4. В поле Имя файла введите имя записной книжки.

    5. Щелкните Сохранить .

    1. Щелкните вкладку Файл .

    2. Щелкните Сохранить как .
      Чтобы увидеть диалоговое окно «Сохранить как » в PowerPoint 2013 и PowerPoint 2016, вам нужно выбрать расположение и папку.

    3. В поле Имя файла введите имя файла, если вы еще этого не сделали.

    4. В списке Сохранить как тип щелкните PDF (*.pdf) .

      • Если вы хотите, чтобы файл открывался в выбранном формате после сохранения, установите флажок Открыть файл после публикации .

      • Если документ требует высокого качества печати, щелкните Стандарт (публикация в Интернете и печать) .

      • Если размер файла важнее качества печати, щелкните Минимальный размер (публикация в Интернете) .

    5. Щелкните Параметры , чтобы задать страницу для печати, выбрать, следует ли печатать разметку, и выбрать параметры вывода. По завершении нажмите ОК .

    6. Щелкните Сохранить .

    1. На вкладке Файл выберите Сохранить как .
      Чтобы увидеть диалоговое окно « Сохранить как » в Project 2013 или Project 2016, вам нужно выбрать расположение и папку.

    2. В поле Имя файла введите имя файла, если вы еще этого не сделали.

    3. В списке Сохранить как тип выберите Файлы PDF (*.pdf) или Файлы XPS (* .xps) , а затем выберите Сохранить .

    4. В диалоговом окне «Параметры экспорта документа » выберите «Диапазон публикации » , следует ли включать «Включить непечатаемую информацию» и Соответствие ISO 19500-1 (только PDF).

    Советы по форматированию

    Project не поддерживает все функции форматирования PDF или XPS, но вы все равно можете использовать некоторые параметры печати, чтобы изменить внешний вид окончательного документа.

    Выберите Файл > Печать , а затем измените любой из этих параметров:

    • Даты

    • Ориентация

    • Размер бумаги

    • Параметры страницы

    В диалоговом окне Page Setup вы можете изменить параметры на следующих вкладках:

    1. Щелкните вкладку Файл .

    2. Щелкните Сохранить как .
      Чтобы увидеть диалоговое окно « Сохранить как » в Publisher 2013 или Publisher 2016, необходимо выбрать расположение и папку.

    3. В поле Имя файла введите имя файла, если вы еще этого не сделали.

    4. В списке Сохранить как тип щелкните PDF (*.pdf) .

    5. Если вы хотите изменить способ оптимизации документа, щелкните Изменить . (Щелкните Параметры в Publisher 2013 или Publisher 2016.)

    6. Если вы хотите, чтобы файл открывался в выбранном формате после сохранения, установите флажок Открыть файл после публикации .

    7. Щелкните Сохранить .

    1. Щелкните вкладку Файл .

    2. Щелкните Сохранить как .
      Чтобы увидеть диалоговое окно Сохранить как в Visio 2013 или Visio 2016, вы должны выбрать расположение и папку.

    3. В поле Имя файла введите имя файла, если вы еще этого не сделали.

    4. В списке Сохранить как тип щелкните PDF (*.pdf) .

      • Если вы хотите, чтобы файл открывался в выбранном формате после сохранения, установите флажок Автоматически просматривать файл после сохранения .

      • Если документ требует высокого качества печати, щелкните Стандарт (публикация в Интернете и печать) .

      • Если размер файла важнее качества печати, щелкните Минимальный размер (публикация в Интернете) .

    5. Щелкните Параметры , чтобы задать страницу для печати, выбрать, следует ли печатать разметку, и выбрать параметры вывода. Нажмите ОК .

    6. Щелкните Сохранить .

    Word 2013 и новее

    1. Выберите Файл > Экспорт > Создать PDF / XPS .

    2. Если свойства вашего документа Word содержат информацию, которую вы не хотите включать в PDF, в окне Опубликовать как PDF или XPS выберите Параметры . Затем выберите Документ и снимите Свойства документа . Установите любые другие параметры, которые вы хотите, и выберите OK .

      Дополнительные сведения о свойствах документа см. В разделе Просмотр или изменение свойств файла Office 2016.

    3. В Publish as PDF или XPS перейдите туда, где вы хотите сохранить файл. Кроме того, измените имя файла, если хотите другое имя.

    4. Щелкните Опубликовать .

    Подробнее о параметрах PDF

    • Чтобы создать файл PDF только из некоторых страниц документа, выберите вариант в разделе Диапазон страниц .

    • Чтобы включить отслеживаемые изменения в PDF, в разделе Опубликовать что выберите Документ с разметкой . В противном случае убедитесь, что выбран Документ .

    • Чтобы создать набор закладок в файле PDF, выберите Создать закладки с помощью .Затем выберите Заголовки или, если вы добавили в документ закладки, Закладки Word .

    • Если вы хотите включить свойства документа в PDF, убедитесь, что выбрано Свойства документа .

    • Чтобы облегчить чтение документа программным обеспечением для чтения с экрана, выберите Теги структуры документа для доступности .

    • Соответствует ISO 19005-1 (PDF / A) Этот параметр выводит документ PDF в формате PDF версии 1.7, который является стандартом архивирования. PDF / A помогает гарантировать, что документ будет выглядеть так же, когда он откроется позже на другом компьютере.

    • Растровый текст, когда шрифты не могут быть встроены. Если шрифты не могут быть встроены в документ, PDF использует растровые изображения текста, чтобы PDF выглядел так же, как исходный документ.Если этот параметр не выбран и в файле используется шрифт, который нельзя встроить, программа чтения PDF-файлов может заменить другой шрифт.

    • Зашифровать документ паролем Чтобы ограничить доступ, чтобы только люди с паролем могли открывать PDF-файл, выберите этот параметр. Когда вы нажимаете ОК , Word открывает поле «Зашифровать PDF-документ », где вы можете ввести и повторно ввести пароль.

    Открытие и копирование содержимого из PDF с помощью Word

    Вы можете скопировать любое содержимое из PDF-файла, открыв его в Word.

    Перейдите к файлу > Откройте и перейдите к файлу PDF. Word открывает содержимое PDF в новом файле. Вы можете скопировать из него любой контент, в том числе изображения и диаграммы.

    Word 2010

    Эта информация также относится к Microsoft Word Starter 2010.

    1. Щелкните вкладку Файл .

    2. Щелкните Сохранить как .
      Чтобы увидеть диалоговое окно « Сохранить как » в Word 2013 и Word 2016, необходимо выбрать расположение и папку.

    3. В поле Имя файла введите имя файла, если вы еще этого не сделали.

    4. В списке Сохранить как тип щелкните PDF (*.pdf) .

      • Если вы хотите, чтобы файл открывался в выбранном формате после сохранения, установите флажок Открыть файл после публикации .

      • Если документ требует высокого качества печати, щелкните Стандарт (публикация в Интернете и печать) .

      • Если размер файла важнее качества печати, щелкните Минимальный размер (публикация в Интернете) .

    5. Щелкните Параметры , чтобы задать страницу для печати, выбрать, следует ли печатать разметку, и выбрать параметры вывода. По завершении нажмите ОК .

    6. Щелкните Сохранить .

    Чтобы сохранить файл в формате PDF в Office для Mac, выполните следующие простые действия:

    1. Нажмите Файл

    2. Нажмите Сохранить как

    3. Щелкните File Format в нижней части окна

    4. Выберите PDF из списка доступных форматов файлов

    5. Дайте вашему файлу имя, если у него его еще нет, затем нажмите Экспорт

    Вы можете использовать Word, PowerPoint и OneNote в Интернете для преобразования документа в PDF.

    1. Выберите файл > Печать > Печать (в PowerPoint вы выберете один из трех форматов).

    2. В раскрывающемся меню под Принтер выберите Сохранить как PDF , а затем выберите Сохранить .

    3. Затем в открывшемся меню проводника вы можете назвать PDF-файл и выбрать, где его сохранить, а затем выбрать Сохранить .

    Будет создан базовый PDF-файл с сохранением макета и формата исходного документа.

    Если вам нужен больший контроль над параметрами вашего PDF, например добавление закладок, используйте настольное приложение для преобразования вашего документа в PDF. Щелкните Открыть в настольном приложении на панели инструментов PowerPoint и OneNote, чтобы начать использовать классическое приложение, а в Word сначала выберите раскрывающийся список «Редактирование», а затем выберите Открыть в настольном приложении .

    Если у вас нет настольного приложения, вы можете попробовать или купить последнюю версию Office прямо сейчас.

    Есть предложение по этой функции?

    Проголосуйте за идею или поделитесь новой в нашем поле для предложений на word.uservoice.com.

    Чтобы экспортировать документ Word или книгу Excel в формат PDF на iOS, нажмите кнопку File в верхнем левом углу, выберите Export , а затем PDF .

    Как сохранить в формате PDF из Word Online

    Когда вы делитесь документами с другими людьми, будь то по электронной почте или как что-то, что вы загружаете через веб-сайт, они часто хотят, чтобы документ был в определенном формате. Документы, которые вы создаете в Word Online, обычно имеют расширение .docx, которое можно открыть во многих версиях Microsoft Word.

    Но иногда требования к представленному документу требуют, чтобы файл был в формате PDF, поэтому вы можете искать способ выполнить это преобразование.К счастью, в Word Online есть утилита, которая позволит вам быстро и легко преобразовать Word Online в PDF.

    Как загрузить онлайн-документ Word в формате PDF

    Действия, описанные в этой статье, были выполнены в настольной версии Google Chrome, но будут работать и в других настольных веб-браузерах. Сохраняемый файл будет в формате PDF и будет копией существующего документа в вашей учетной записи Word Online. После выполнения этих шагов у вас все еще будет исходный файл Word в Word Online.

    Шаг 1. Перейдите в Word Online по адресу https://office.live.com/start/Word.aspx и войдите в свою учетную запись Microsoft.

    Шаг 2. Откройте документ, который вы хотите сохранить в формате PDF.

    Шаг 3. Щелкните вкладку File в верхнем левом углу окна.

    Шаг 4. Выберите опцию Сохранить как в левом столбце.

    Шаг 5: Выберите вариант Загрузить как PDF .

    Шаг 6: Нажмите кнопку Загрузить , чтобы сохранить файл на свой компьютер.

    Обратите внимание, что если вы хотите отредактировать созданный файл PDF, вам понадобится программа, которая может редактировать эти типы файлов. К таким программам относятся Adobe Acrobat (не Adobe Reader) и некоторые более новые версии настольного приложения Microsoft Word. Кроме того, вы можете преобразовать множество PDF-файлов в формат Word, загрузив их обратно в свою учетную запись OneDrive, затем щелкнув загруженный PDF-файл правой кнопкой мыши и выбрав опцию Открыть в Word Online .

    Вам нужно распечатать документ на бумаге другого формата, кроме того, который выбран в данный момент? Узнайте, как изменить размер страницы в Word Online и сохранить документ, чтобы он печатался на бумаге другого размера.

    Мэтью Берли пишет технические руководства с 2008 года. Его статьи были опубликованы на десятках различных веб-сайтов и были прочитаны более 50 миллионов раз.

    После получения степени бакалавра и магистра компьютерных наук он несколько лет проработал в сфере управления ИТ для малых предприятий.Однако сейчас он работает полный рабочий день, пишет онлайн-контент и создает веб-сайты.

    Его основные писательские темы включают iPhone, Microsoft Office, Google Apps, Android и Photoshop, но он также писал и на многие другие технические темы.

    Связанные

    Отказ от ответственности: большинство страниц в Интернете содержат партнерские ссылки, в том числе некоторые на этом сайте.

    Конвертер из

    Word в PDF | Конвертируйте Word в PDF онлайн бесплатно

    Конвертируйте Word в PDF онлайн и легко сохраняйте документ Word как файл PDF.

    Загрузите файл с правильным расширением.

    Сервер сейчас занят. Пожалуйста, повторите попытку позже.

    Разблокируйте файл и повторите попытку.

    Максимально допустимый размер файла: 32 МБ.

    Выберите свой план

    • Онлайн-приложение

    • Неограниченное количество задач

    • 19 преобразователей

    • 5 инструментов редактирования

    • Гарантия безопасности

    $ 1.65 / мес

    71,5 долл. США 19,95 долларов США оплачивается ежегодно

    • Онлайн-приложение

    • Неограниченное количество задач

    • 19 преобразователей

    • 5 инструментов редактирования

    • Гарантия безопасности

    1,65 долл. США в месяц 03

    71,5 долл. США 19,95 долларов США выставляется ежегодно

    2,50 доллара США в месяц

    29 долларов США.95 выставление счетов за год

    2,50 доллара США в месяц

    29,95 долларов США за год

    Откройте для себя больше возможностей с PDFChef для настольных ПК

    Загрузите PDFChef от Movavi для ПК или Mac и настройте PDF-файлы в соответствии с вашими потребностями. Получите бесплатный доступ к расширенным инструментам и различным режимам редактирования PDF.

    Как конвертировать Word в PDF онлайн:

    Шаг 1

    Добавьте файл Word с помощью кнопки «Выбрать файл» или перетащив его в область загрузки.

    Шаг 2

    Подождите несколько секунд, приложение преобразует файл в PDF.

    Шаг 3

    Щелкните Загрузить файл .

    Почему выбирают нас?

    Преобразование Word в PDF бесплатно

    PDFChef предлагает вам бесплатное онлайн-решение для преобразования документа Word в PDF независимо от того, какой браузер и операционную систему вы используете.

    Удобный интерфейс

    Простой и интуитивно понятный пользовательский интерфейс с привлекательной наглядной шкалой выполнения поможет быстро выполнить процесс преобразования Word в PDF.

    Сверхбыстрый преобразователь Word в PDF

    PDFChef знает, что вы цените свое время. С нашими быстрыми серверами вы можете экспортировать страницу Word в PDF всего за несколько секунд.

    100% безопасный сервис

    256-битное SSL-шифрование защищает ваши файлы от потери и несанкционированной передачи ваших данных. Ваша личная информация всегда в полной безопасности.

    Дополнительные функции

    ✂️ Редактировать файлы PDF

    📎 Конвертировать файлы MS Office из и в PDF

    🏝 Конвертировать файлы изображений из и в PDF

    🔮 Другое

    Как конвертировать файлы в PDF онлайн бесплатно

    Lumin Tools помогите конвертировать другие форматы файлов в PDF.Давайте разберемся, как работает конвертер Lumin PDF.

    Чтобы начать использовать конвертер Lumin PDF, вам необходимо Авторизоваться в своей учетной записи. Если у вас его нет — зарегистрируйтесь или войдите в свой аккаунт Google.

    📌Примечание: одновременно можно преобразовать только один файл.

    Как преобразовать Word в PDF
    1. Щелкните вкладку Конвертировать на панели инструментов Lumin
    2. Выберите Word в PDF и щелкните Приступить к работе
    3. Перетащите файлы с компьютера или выберите один из облачных хранилищ : Box, Google Drive, Microsoft Cloud или Dropbox
    4. Дождитесь загрузки файла и конвертируйте в PDF
    5. Готово! Теперь ваш файл Word преобразован в PDF.Вы можете загрузить его на свой компьютер или сохранить на Google Диске или Dropbox.

    Вот как вы конвертируете DOC в PDF с помощью Lumin

    📌Примечание: конвертировать PDF в Word в обратном порядке невозможно.

    Если вы не вошли в систему Google, перейдите в учетную запись Dropbox, вы можете сделать это через Lumin и привязать учетные записи. Этот метод экономит время для дальнейшей работы с конвертером Lumin.

    Lumin — бесплатный конвертер слов в PDF. Все, что вам нужно сделать, это войти в свою учетную запись и сделать несколько щелчков мышью, чтобы преобразовать слово в PDF.Вы также можете использовать другие форматы файлов для преобразования в PDF: Microsoft Excel, Microsoft PowerPoint и JPG.

    Как конвертировать Excel в PDF

    Конвертировать Excel в PDF быстро и просто, как конвертировать DOCX в PDF:

    1. Щелкните вкладку Конвертировать на панели инструментов Lumin
    2. Выберите Excel в PDF и нажмите Начать работу
    3. Перетащите файлы с компьютера или выберите один из облачных хранилищ: Box, Google Drive, Microsoft Cloud или Dropbox
    4. Подождите, пока файл загрузится и конвертируется в PDF
    5. Готово! Теперь ваш файл Excel преобразован в PDF.Вы можете загрузить его на свой компьютер или сохранить на Google Диске или в Dropbox.

    Lumin быстро преобразует ваши электронные таблицы Microsoft Excel в PDF для удобства совместного использования и хранения. Отправляя Excel в формате PDF, вы можете быть уверены, что никто не изменит их во время публикации. Кроме того, Excel в формате PDF удобнее распечатывать и публиковать в Интернете.

    Вот как можно сохранить преобразованный файл на Google Диск.

    📌Примечание: преобразовать PDF в Excel нельзя.

    Как преобразовать PowerPoint в PDF
    1. Щелкните вкладку Преобразовать на панели инструментов Lumin
    2. Выберите PowerPoint в PDF и щелкните Приступить к работе
    3. Перетащите файлы с компьютера или выберите один из облачных хранилищ : Box, Google Drive, Microsoft Cloud или Dropbox
    4. Дождитесь загрузки файла и конвертируйте в PDF
    5. Готово! Теперь ваша презентация PowerPoint преобразована в PDF. Вы можете скачать его на свой компьютер или сохранить на Google Диск или Dropbox

    Простой и понятный интерфейс конвертации Lumin

    Как конвертировать JPG в PDF
    1. Щелкните на Конвертировать вкладку на панели инструментов Lumin
    2. Выберите JPG в PDF и щелкните Начать работу
    3. Перетащите и отпустите файлов с компьютера или выберите одно из облачного хранилища: Box, Google Drive, Microsoft Cloud или Dropbox
    4. Дождитесь загрузки файла и конвертируйте его в PDF
    5. Готово! Теперь ваше изображение JPG преобразовано в PDF.

    Xa b: Как Эффективно Решить Уравнение Линейной Матрицы: Ax-Xa = B?

    Ограниченные линейные наименьшие квадраты для xA=b в matlab



    Я хочу решить xA=b с ограничением 0<=x для x .

    Я нашел такие функции, как lsqnonneg и lsqlin , которые решают для Ax=b . Тем не менее, не смог найти хороший способ решить для xA=b .

    Как я могу решить xA=b с неотрицательным ограничением x ?

    matlab linear-algebra least-squares
    Поделиться Источник JunYoung Gwak     28 октября 2014 в 04:03

    1 ответ


    • Как установить взвешенные наименьшие квадраты в r для гетероскедастических данных?

      Я провожу регрессию по данным переписи, где моя зависимая переменная — ожидаемая продолжительность жизни, и у меня есть восемь независимых переменных. Данные агрегируются по городам, поэтому у меня есть много тысяч наблюдений. Однако моя модель несколько гетероскедастична. Я хочу запустить…

    • Существует ли функция для решения xA=b в opencv?

      Я знаю, что функция solve может решить Ax=b . Но я хочу, чтобы функция решала xA=b для x? Есть ли какая-то функция? Кстати, он должен работать как mrdivide из Matlab: x = B/A solves the system of linear equations x*A = B for x. The matrices A and B must contain the same number of columns. MATLAB®…



    1

    Как прокомментировал Дэвид , это просто показать, что

    таким образом, вы можете использовать стандартные методы для решения проблемы с A’ и b’ , а затем транспонировать ответ.

    Поделиться Chris Taylor     28 октября 2014 в 07:40


    Похожие вопросы:


    Является ли линейная регрессия тем же самым, что и обычные наименьшие квадраты в SPSS?

    Я хочу использовать модель линейной регрессии , но я хочу использовать обычные наименьшие квадраты , которые я думаю, что это тип линейной регрессии. Программное обеспечение, которое я использую, -…


    SPSS и обычные наименьшие квадраты

    Я делаю регрессию и использую SPSS/PASW . Но он, похоже, не поддерживает обычные наименьшие квадраты, у него есть только частичные наименьшие квадраты и 2-ступенчатые наименьшие квадраты. Есть…


    Минимизация ошибки формулы в MATLAB (наименьшие квадраты?)

    Я не слишком хорошо знаком с MATLAB или вычислительной математикой, поэтому мне было интересно, как я мог бы решить уравнение, включающее сумму квадратов, где каждый член включает в себя два вектора…


    Как установить взвешенные наименьшие квадраты в r для гетероскедастических данных?

    Я провожу регрессию по данным переписи, где моя зависимая переменная — ожидаемая продолжительность жизни, и у меня есть восемь независимых переменных. Данные агрегируются по городам, поэтому у меня…


    Существует ли функция для решения xA=b в opencv?

    Я знаю, что функция solve может решить Ax=b . {T}y Я попробовал: import numpy as np def least_squares1(y,…


    MATLAB линейные наименьшие квадраты с разреженным b

    Я пытаюсь решить проблему llsq вида Ax = b . У меня есть несколько огромных матриц, где size(A) = 26181 13090 size(b) = 26181 1 b имеет разреженность ~26%, а A почти плотен. Из документов mldivide…


    Решите линейное уравнение в python XA = B

    Линейное уравнение XA = B , мы знаем ‘X = B * inv (A)’. где A, B, X — все матрицы. в matlab это можно решить: X = B / A он избегает делать обратную матрицу, которая является быстрой . есть ли равная…

    Regex, который соответствует xa? B? C? но не только один

    Как насчет этого:

    x(?:a())?(?:b())?(?:c())?(\1|\2|\3)
    

    Пустые группы захвата после a , b и c всегда будут совпадать (пустая строка), если a , b или c в этом порядке.

    Компонент (\ 1 | \ 2 | \ 3) будет соответствовать только если в матче участвовала хотя бы одна из предыдущих групп. и $ ), если вы не возражаете против соответствия xb в строке cxba и т. д.

    Например, в Python:

    >>> r=re.compile(r"x(?:a())?(?:b())?(?:c())?(\1|\2|\3)$")
    >>> for test in ("x", "xa", "xabc", "xba"):
    ...     m = r.match(test)
    ...     if m:
    ...         print("{} --> {}".format(test, m.group(0)))
    ...     else:
    ...         print("{} --> no match".format(test))
    ...
    x --> no match
    xa --> xa
    xabc --> xabc
    xba --> no match
    

    * или, если ваш аромат регулярного выражения знает имена захваченных групп, вы можете использовать их, например

    x(?:a(?P))?(?:b(?P))?(?:c(?P))?((?P=a)|(?P=b)|(?P=c))
    

    в Python/PCRE. В .NET (и, возможно, в других вариантах) даже законно иметь несколько групп захвата, которые используют одно и то же имя, что делает возможным еще одно упрощение:

    x(?:a(?))?(?:b(?))?(?:c(?))?\k

    NZM1/2-XAB 260203 0004358741 EATON ELECTRIC Прокладки, NZM.

    .

    Проверка конструкции IEC/EN 61439

    10.2 твёрдость материалов и деталей10.2.2 Коррозионная стойкость

    Требования производственного стандарта выполнены.

    10.2 твёрдость материалов и деталей10.2.3.1 Нагревостойкость изоляции

    Требования производственного стандарта выполнены.

    10.2 твёрдость материалов и деталей10.2.3.2 Сопротивление изоляционных материалов при обычном нагреве

    Требования производственного стандарта выполнены.

    10.2 твёрдость материалов и деталей10.2.3.3 Сопротивление изоляционных материалов при сильном нагреве

    Требования производственного стандарта выполнены.

    10.2 твёрдость материалов и деталей10.2.4 Устойчивость к ультрафиолетовому излучению

    Требования производственного стандарта выполнены.

    10.2 твёрдость материалов и деталей10.2.5 Подъём

    Не имеет значения, поскольку необходимо оценить всё коммутационное оборудование.

    10.2 твёрдость материалов и деталей10.2.6 Испытание на удар

    Не имеет значения, поскольку необходимо оценить всё коммутационное оборудование.

    10.2 твёрдость материалов и деталей10.2.7 Ярлыки

    Требования производственного стандарта выполнены.

    10.3 Класс защиты изоляции

    Не имеет значения, поскольку необходимо оценить всё коммутационное оборудование.

    10.4 Воздушные промежутки и пути утечки тока

    Требования производственного стандарта выполнены.

    10.5 Защита от удара электрическим током

    Не имеет значения, поскольку необходимо оценить всё коммутационное оборудование.

    10.6 Монтаж оборудования

    Не имеет значения, поскольку необходимо оценить всё коммутационное оборудование.

    10.7 Внутренние электрические цепи и соединения

    Находится в сфере ответственности компании, монтирующей распределительные устройства.

    10.8 Подключения проводов, введённых снаружи

    Находится в сфере ответственности компании, монтирующей распределительные устройства.

    10.9 Свойства изоляции10.9.2 Электрическая прочность при рабочей частоте

    Находится в сфере ответственности компании, монтирующей распределительные устройства.

    10.9 Свойства изоляции10.9.3 Прочность по отношению к импульсному напряжению

    Находится в сфере ответственности компании, монтирующей распределительные устройства.

    10.9 Свойства изоляции10.9.4 Проверка оболочек кабелей из изолирующего материала

    Находится в сфере ответственности компании, монтирующей распределительные устройства.

    10.10 Нагрев

    Расчёт параметров нагрева находится в сфере ответственности компании, монтирующей распределительные устройства. Компания Eaton указывает данные по потере мощности устройств.

    10.11 Стойкость к коротким замыканиям

    Находится в сфере ответственности компании, монтирующей распределительные устройства. Соблюдать указания для коммутационных устройств.

    10.12 Электромагнитная совместимость

    Находится в сфере ответственности компании, монтирующей распределительные устройства. Соблюдать указания для коммутационных устройств.

    10.13 Механическая функция

    Для устройства требования считаются выполненными, если были соблюдены данные инструкции по монтажу (IL).

    XAB — это… Что такое XAB?

    Уста́в Фидоне́та (англ. Fidonet Policy) — правила, регулирующие общение в сети Фидонет. Текущая версия 4.07, принята 9 июня 1989 года.

    В этом документе содержатся правила, которыми руководствуются системные операторы (сисопы), входящие в число членов организации систем электронной почты FidoNet (ФИДО). Разрешено вводить отдельные правила в зонах и регионах, но рекомендуется, чтобы эти правила не противоречили Уставу. Для введения особых правил требуется одобрение Международного Координатора.

    Основные положения

    Чрезмерно некорректное поведение

    Чрезмерно некорректное поведение (от англ. excessively annoying behavior; иногда чрезмерно раздражающее поведение, в неофициальной и полуофициальной речи часто сокращается до XAB) — термин, использующийся для описания действий, существенно мешающих деятельности Фидонета.

    За чрезмерно раздражающее поведение сетевой координатор (так же как и региональный, зональный) может исключить узел из сети без предварительного рассмотрения жалобы (комплейна), вплоть до устранения причин XAB.

    К действиям, являющимся XAB, относится:

    Техническое XAB

    Если узел в результате сбоя оборудования или ошибки в конфигурации начинает вести себя таким образом, что причиняет крайнее неудобство другим узлам сети (например, рассылает многократно повторяющиеся дубли сообщений, или звонит другим узлам в неудобное для них время вместо указанного в нодлисте времени работы), тогда узел может быть экскоммуницирован сетевым координатором (NC) за техническое XAB до устранения причин проблемы.

    Попытки модернизации

    Действующая версия Устава была принята ещё в 1989 году, и за прошедшее с тех пор время в Устав неоднократно предлагалось внести те или иные изменения. Однако из-за сложности внесения изменений (для инициирования процедуры референдума необходимы сообщения большинства региональных координаторов о желании такого референдума; новая версия принимается большинством голосов координаторов всех уровней) все эти попытки провалились.

    Источники

    Wikimedia Foundation. 2010.

    Серия HERCULES 8 ‘x 12’ Складная фермерская скамья из массива сосны в старинном деревенском стиле с 3 ножками

    Способ доставки — наземный (более мелкие)

    Стандартный способ доставки — наземная доставка с FedEx или UPS. Подпись доставки не требуется для наземных перевозок. Перевозчик оставит посылку в обычном месте доставки по вашему адресу. Вы можете перейти на веб-сайт FedEx или UPS, чтобы отследить ваш груз. Иногда ваша посылка может не быть доставлена ​​из-за погодных условий или из-за заботы о безопасности посылки.Это решение принимает перевозчик. Очень маленькие предметы могут быть отправлены USPS.

    Если вы покупаете несколько товаров у разных производителей, ваш заказ будет отправлен с разных складов и может быть отправлен в разные даты. Мы отправим вам электронное письмо с отслеживанием вашего груза, как только товар будет доставлен со склада. Если ваш заказ содержит несколько наземных отправлений, то они могут быть отправлены грузовым перевозчиком в зависимости от количества отправлений и веса.

    Способ доставки — фрахт

    Стандартный способ доставки крупногабаритных / тяжелых предметов — грузовым перевозчиком.Подпись доставки требуется для грузовых перевозок, и вы должны будете присутствовать в течение окна времени доставки. При доставке на дом перевозчик свяжется с вами, чтобы согласовать дату доставки. Вы можете перейти на сайт перевозчика, чтобы отследить ваш груз.

    Если вы покупаете несколько товаров у разных производителей, ваш заказ будет доставлен с разных складов и может быть отправлен в разные дни. Мы отправим вам по электронной почте все отслеживание, связанное с вашим отправлением.

    Грузоперевозки осуществляются по бордюру. Если вы хотите заказать услугу «сквозная доставка», это будет дополнительная плата, и вам нужно будет связаться с нами перед размещением заказа.

    При доставке, если вы заметили какие-либо повреждения коробки / предмета, вы ДОЛЖНЫ указать это в квитанции о доставке.

    Дополнительная информация
    Номер детали производителя XA-B-96X12-L-GG
    Марка Flash Мебель

    Flash Мебель

    Flash Furniture со штаб-квартирой в Атланте и тремя складами общей площадью почти миллион квадратных футов. Flash Furniture — лидер и новатор в мебельной промышленности, предлагающий на выбор более 3000 различных продуктов. Flash Furniture производит качественную линию барных стульев, игровых столов, складных и штабелируемых стульев, кресел, столов для пабов, офисной мебели и многого другого. ПРИМЕЧАНИЕ. На продукты этого производителя не распространяются какие-либо оптовые скидки или другие онлайн-стимулы.

    Анализ алгоритмов. Обзор математики — 1.2 Показатели –X A X B = X A + B –X A / X B = X A-B — (X A) B = X AB –X N + X N = 2X N ≠ X 2N –2 N + 2 N = 2 N + 1 логарифм.

    Презентация на тему: «Анализ алгоритмов. Математический обзор — 1.2 Показатели –XAXB = X A + B –XA / XB = X AB — (XA) B = X AB –XN + XN = 2X N ≠ X 2N –2 N + 2 N = 2 N + 1 логарифм »- стенограмма презентации:

    ins [data-ad-slot = «4502451947»] {display: none! important;}} @media (max-width: 800px) {# place_14> ins: not ([data-ad-slot = «4502451947»]) {display: none! important;}} @media (max-width: 800px) {# place_14 {width: 250px;}} @media (max-width: 500 пикселей) {# place_14 {width: 120px;}} ]]>

    1 Анализ алгоритмов

    2 Математический обзор — 1. 2 экспоненты –XAXB = X A + B –XA / XB = X AB — (XA) B = X AB –XN + XN = 2X N ≠ X 2N –2 N + 2 N = 2 N + 1 логарифм –XA = B тогда и только тогда. журнал XB = A –log AB = журнал CB / журнал CA; A, B, C> 0, A ≠ 1 –logAB = logA + logB; A, B> 0 Ряд N –∑ 2 i = 2 N + 1 -1 i = 0 N –∑ A i = A N + 1 -1 / A-1 i = 0 N –∑ i = N (N + 1 ) / 2 i = 0

    3 Время выполнения Зачем нужно анализировать время выполнения программы? Вариант 1. Запустить программу и рассчитать время — Почему этот вариант плохой? -Что мы можем с этим поделать?

    4 Псевдокод Используется для определения алгоритмов. Часть английский, часть кода. Алгоритм (arrayMax (A, n)) curMax = A [0] для i = 1 i

    5 Алгоритм подсчета операций (arrayMax (A, n)) curMax = A [0] // 2 для i = 1 i

    6 Асимптотическая запись 7n-2 n = 5 -> 33 n = 100 -> 698 n = 1 000 000 -> 6 999 998 Время работы растет пропорционально n Что происходит, когда n становится большим?

    7 Big-Oh T (N) — это O (f (N)), если существуют положительные константы c и n 0 такие, что T (N) = n 0 7n 2 -2 — это O (n 2) n 0> = 1 c = 8 Верхняя граница

    8 Омега T (N) — это Ω (g (N)), если существуют положительные константы c и n 0 такие, что T (N)> = cg (N), когда N> = n 0 7n 2 -2, является Ω (n 2) n 0> = 1 c = 1 Нижняя граница

    9 Theta T (N) равно Θ (h (N)) тогда и только тогда, когда T (N) = Oh (N) и T (N) = Ωh (N) 7n 2 -2 равно Θ (n 2) Максимально точный результат

    10 Little-Oh T (N) — это o (p (N)), если T (N) = O (p (N)) и T (N) ≠ Θ (p (N)) 7n 2 -2 — это o (n 3 ) –O, когда n 0> = 8 c = 1 Скорость роста T (N) меньше скорости роста p (N)

    11 Правило Правило 1 — Если T 1 (N) = O (f (N) и T 2 (N) = O (g (N)), то T 1 (N) + T 2 (N) = max (O (f (N)), O (g (N))) T 1 (N) * T 2 (N) = O (f (N) * g (N)) Правило 2 — Если T (N) является полиномом степени k, то T (N) = Θ (N k) Правило 3 –log k N = O (N) для любой константы k. Это говорит нам о том, что бревна растут очень медленно.

    12 Примеры 87n 4 + 7n 3nlogn + 12logn 4n 4 + 7n 3 logn

    13 Терминология Логарифмическая — O (logn) Линейная — O (n) Линейная — O (nlogn) Квадратичная — O (n 2) Полиномиальная — O (n k) k> = 1 Экспоненциальная — O (a n) a> 1

    14 Правило 1 — циклы for Время работы цикла for — это самое большее время выполнения операторов внутри цикла for (включая тесты), умноженное на количество итераций.for (i = 0; i <5; i ++) cout << «привет \ n»;

    15 Правило 2 — вложенные циклы. Анализируйте их наизнанку. Общее время выполнения оператора внутри группы вложенных циклов — это время выполнения оператора, умноженное на произведение размеров всех циклов. for (i = 0; i

    16 Правило 3 — последовательные утверждения. Они просто добавляют (что означает, что максимальное значение имеет значение) для (i = 0; i

    17 Правило 4 — if / else Время выполнения оператора if / else никогда не превышает время выполнения теста плюс большее из значений времени выполнения операторов, if (условие) S1 else S2

    18 Пример Найти максимальное число в матрице nxn Алгоритм: 64… 2 123… 9 ………… 58… 1 0 n-1 0

    19 Пример Каково большое время работы этого алгоритма? Алгоритм: Ввод: A, n curMax = A [0] [0] для i = 0 i

    20 Другой пример. Определите, сколько элементов массива 1 соответствуют элементам алгоритма массива 2? 24… 6 н-10 68… 3 0

    21 год Другой пример алгоритма: Входные данные: A, B, n для i = 0 i

    22 Журналы во времени выполнения. Алгоритм равен O (logN), если требуется постоянное время, чтобы сократить размер проблемы на долю.Двоичный поиск — задано целое число X и целые числа A 0, A 1,…, A N-1, которые предварительно отсортированы и уже находятся в памяти, найти i такое, что A i = X, или вернуть i = -1, если X не находится в вход.

    23 Алгоритм двоичного поиска? Время работы не превышает ceil (log (N-1)) + 2, что составляет O (logN)

    24 Пример 1 — (N-1) — 1 2 — (N-1) / 2-2 3 — (N-1) / 4-4 4 — (N-1) / 8-8 i — (N-1) / 2 i-1 — 2 i-1 = N-1 — разница между высоким и низким составляет 1, разница между высоким и низким составляет 0

    25 Злой Король Король имеет N бутылок вина. Ровно 1 бутылка отравлена. Как может король определить, какая бутылка отравлена, и убить не более logN своих тестеров?


    Обновление Крипке: (a) DFA для формулы LTL G (r → Xa), (b) Оригинал…

    Контекст 1

    … γ — транзитивное замыкание γ. Мы вычисляем разницу между исходным путем и путем в новой структуре n как d (s s) = c. Алгоритм работает следующим образом: мы начинаем с s 0 и фиксируем количество разрешенных отличий от исходного пути до константы c. После этого мы вычисляем разницу для каждого последующего состояния. Мы рассматриваем только такие узлы, что разность путей n i = 1 d (s i s ij) ≤ c. Если путь не может быть найден, разница c увеличивается до тех пор, пока не будет получен действительный путь.Этот алгоритм работает очень эффективно для путей, которые можно исправить с помощью нескольких изменений в зависимости от количества атомарных предложений. На рисунке 2 приведен краткий пример того, как алгоритм работает для следующей формулы LTL: «Не может быть случая, чтобы человек стоял и сидит в один и тот же момент времени». Для наглядности мы сформулировали это правило следующим образом: G ¬ (a ∧ r) (a = человек 1 стоит, а r = человек 1 сидит). На рисунке 2 (а) показан DFA. Он состоит только из одного состояния приема с петлей для (¬ a ∧ ¬ r), (¬ a ∧ r) и (a ∧ ¬ r).Для (a ∧ r) следующего состояния нет, потому что это состояние недопустимо. На рисунке 2 (b) показана исходная структура Крипке с несоответствием в отношении фоновой теории в состоянии s 2. Рисунок 2 (c) иллюстрирует новую структуру Крипке после применения алгоритма. Состояние (a ∧ r) не принимается во внимание, потому что такая комбинация атомарных предложений не разрешена в DFA. Разница d изображена на правом нижнем крае каждого состояния s ij. Сплошными линиями n показаны пути с i = 1 d (s i s ij) = 1, а пунктирными линиями показаны пути с более высоким d.Видно, что путь s 0 → s 12 → s 22 → s 32 → s 42 имеет полную разность ni = 1 d (sis ij) = 4, которая больше, чем полная разность пути s 0 → s 11 → s 22 → s 32 → s 43 с общей разностью ni = 1 d (sis ij) = 1. На рисунке 3 показаны два альтернативных пути n с i = 1 d (s i s ij) = 1. В нем описывается разрешенный конфликт путем создания двух различных моделей мира: первая — путем замены ar в состоянии s 2 на ¬ ar, а вторая — путем замены ar в состоянии s 2 на a ¬ r.Вторая важная задача нашей системы — вывод новых концепций. Наша цель — сделать предположения о говорящем. Первое новое понятие, которое мы вводим, — это монолог, для которого мы сформулировали некоторые правила. Трудность в данном случае заключалась в том, как определить смысловое понятие монолога. Но мы договорились о следующей формулировке: если человек говорит, а в следующий момент тот же человек все еще говорит, этот человек держит монолог. На рисунке 4 показан вывод монолога в качестве примера для нашего алгоритма обновления.Формализованное в LTL правило If Per- записывается следующим образом: G (r Xa). Где r = Человек 1 говорит, а a = Человек 1 ведет монолог. Видно, что есть конфликт в состоянии s 3. Если r истинно, в следующем состоянии a должно быть истинным. В нашем примере это не так. На рисунке 5 показан правильный путь. В s 3 нам пришлось изменить маркировку с (¬ a ∧ ¬ r) на (a ∧ ¬ r), что привело к разнице n d (s s) = 1. В формулах LTL, которые мы видели до сих пор, мы должны явно указать все возможные действия как предложения.В типичной ситуации встречи с несколькими участниками, когда каждый из них может выполнять ряд различных действий, это привело бы к добавлению правила для каждой возможной комбинации людей и типа действий. Кроме того, информация о конкретной встрече должна быть явно предоставлена ​​при формализации базовой теории. Чтобы избежать этой проблемы, мы не формализуем теорию на основе конкретных предложений, а определяем абстрактную формализацию, которая позволяет нам использовать правила, которые создаются, когда доступна требуемая информация, например.г., при анализе конкретной встречи. В этих мета-правилах могут использоваться квантификаторы, такие как ForAll и Exists. Во время обработки модели мира кванторы связывают переменные с предложениями в метках фактических состояний. В зависимости от области базовой теории у нас, кроме того, есть функции, которые позволяют нам рассуждать о предложениях. Например, в нашем сценарии встречи теория фона определяет действия: говорение и молчание. Предположим, что у нас встреча с участниками A, B и C.Таким образом, наша модель содержит предложения для всех возможных комбинаций: A говорит, B говорит, C говорит, A молчит и так далее. В теории фона нашей области встреч мы определяем функцию PersonID (x), чтобы говорить о человеке, ответственном за предложение x и ActionType (x) за действие соответственно. Эти мета-правила позволяют нам значительно упростить правила проверки согласованности и позволяют отделить формализацию теории от конкретного случая встречи.Например, если мы хотим …

    X Artists ’Books

    Корзина 0 Книги X Проекты О Поиск магазинов Распределение Новости + Обзоры События Корзина 0 КнигиX ПроектыО насМагазинРаспространениеНовости + ОбзорыМероприятия Категория
    • Все
    • Книга XAB

    45. 00

    25.00

    120,00

    40.00

    40.00

    30.00

    35. 00

    130.00

    60.00

    25.00

    35.00

    35.00

    1,300. 00

    продано

    15.00

    info @ xartistsbooks.ком

    часов

    Instagram Facebook Возврат и конфиденциальность Что для вас книги?

    В группе G уравнения ax b и xa b не имеют класса 10 математики CBSE

    Подсказка : Докажите это, используя элементарные свойства групп. {-1}} b \\
    \ end {align} \]
    Теперь предположим, что y — другое решение уравнения ax = b. Тогда ay = b = ax.
    По свойству сокращения слева мы имеем x = y.
    Следовательно, доказательство состоит в том, что ax = b аналогично xa = b.
    Следовательно, в группе G, a, b \ [\ in \] G каждое из уравнений ax = b и xa = b имеет единственное решение в G.
    Следовательно, вариант (c) является правильным ответом.

    Примечание : Уравнение ax = b имеет уникальное решение, что означает, что b появляется только один раз в строке таблицы Кэли, которая описывает структуру конечной группы путем упорядочивания всех возможных продуктов группы.Точно так же существует уникальное решение для xa = b, и каждый элемент появляется ровно один раз в каждом столбце таблицы. Следовательно, ax = b и xa = b имеют единственное решение.

    Вы в одном шаге от ответа!

    Подпишитесь бесплатно!

    Регистрируясь, вы также получаете БЕСПЛАТНЫЙ доступ к тысячам решенных вопросов, викторин
    и загружаемым PDF-файлам!

    Решение | Что, если деление f (x) на (x-a) (x-b) и на (x-a) (x-c) даст тот же остаток? | Многочлены и рациональные функции

    Покажите, что остаток от деления многочлена \ (f (x) \) на \ ((x-a) \) равен \ (f (a) \).

    Если \ (f (x) \) — многочлен степени \ (n \), должен быть многочлен \ (g (x) \) степени \ (n-1 \) и константа \ (r \) такой, что \ [\ begin {уравнение *} е (х) = (х-а) д (х) + г. \ end {уравнение *} \] Мы говорим, что \ (r \) — это остаток от деления \ (f (x) \) на \ (x-a \). Взяв \ (x = a \) выше, \ [\ begin {уравнение *} е (а) = (а-а) д (а) + г = г. \ end {уравнение *} \]

    Далее покажите, что если \ (f (x) \) делится на \ ((x-a) (x-b) \), где \ (a \ ne b \), то остаток равен \ [\ begin {уравнение *} \ left (\ frac {f (a) — f (b)} {a-b} \ right) x + \ left (\ frac {af (b) — bf (a)} {a-b} \ right).\ end {уравнение *} \]

    На этот раз мы можем написать \ [\ begin {уравнение *} е (х) = (х-а) (х-б) д (х) + \ альфа х + \ бета, \ end {уравнение *} \] где если \ (f (x) \) имеет степень \ (n, g (x) \) имеет степень \ (n-2 \). Подставляя в \ (x = a \) и \ (x = b \), мы получаем, что \ [\ begin {align} f (a) & = \ alpha a + \ beta, \ label {eq: pt-2-linear} \\ е (б) & = \ альфа б + \ бета. \ nonumber \ конец {выравнивание} \] Вычитая второе уравнение из первого, получаем, что \ [\ begin {уравнение *} е (а) — е (Ь) = \ альфа (а-б), \ end {уравнение *} \] а значит, при \ (a \ ne b \) \ [\ begin {уравнение *} \ alpha = \ frac {f (a) — f (b)} {a-b}.\ end {уравнение *} \] Из \ (\ eqref {eq: pt-2-linear} \) следует, что \ [\ begin {уравнение *} \ beta = f (a) — \ alpha a = f (a) — \ frac {f (a) — f (b)} {a-b} a, \ end {уравнение *} \] так что оставшийся член \ [\ begin {align} \ alpha x + \ beta & = \ left (\ frac {f (a) — f (b)} {ab} \ right) x + f (a) — \ frac {f (a) — f (b)} {ab} а \\ & = \ frac {f (a) — f (b)} {ab} x + \ frac {(ab) f (a) — a (f (a) — f (b))} {ab} \ nonumber \ \ & = \ frac {f (a) — f (b)} {a-b} x + \ frac {af (b) — bf (a)} {a-b}. \ label {eq: pt-2-final-form} \ конец {выравнивание} \]

    Остатки при делении \ (f (x) \) на \ ((xa) (xb) \) и на \ ((xa) (xc) \), \ (a \ ne b \ ne c \), равны равный.Докажи это \ [\ begin {уравнение *} (b-c) f (a) + (c-a) f (b) + (a-b) f (c) = 0. \ end {уравнение *} \]

    Исходя из \ (\ eqref {eq: pt-2-final-form} \) и данных предположений, мы должны иметь, что \ [\ begin {уравнение *} \ frac {f (a) — f (b)} {ab} x + \ frac {af (b) — bf (a)} {ab} = \ frac {f (a) — f (c)} {ac } x + \ frac {af (c) — cf (a)} {ac}, \ end {уравнение *} \] откуда следует, что \ [\ begin {уравнение *} \ frac {f (a) — f (b)} {a-b} = \ frac {f (a) — f (c)} {a-c}. \ end {уравнение *} \] Умножая обе части на \ ((a-b) (a-c) \), получаем, что \ [\ begin {уравнение *} (a-c) f (a) — (a-c) f (b) = (a-b) f (a) — (a-b) f (c), \ end {уравнение *} \] откуда следует, что \ [\ begin {уравнение *} (b-c) f (a) + (c-a) f (b) + (a-b) f (c) = 0.\ end {Equation *} \]

    Пример 13.2, 7 — Для постоянных a, b найдите производную от (i) (x-a) (x-b)

    Последнее обновление: 24 июля 2019 г., Teachoo


    Выписка

    Пр. 13.2, 7 (Метод 1) Для некоторых констант a и b найдите производную от (я) (х — а) (х — б) Пусть f (x) = (x — a) (x — b) = х (х — б) — а (х — б) = x2 — xb — ах + ab = x2 — (b + a) x + ab f ’(x) = (x2 — (b + a) x + ab)’ = 2.×2–1 — (b + a) .1×1–1 + 0 = 2×1 — (b + a). x0 = 2х — (Ь + а) × 1 = 2х — (Ь + а) Пр. 13.2, 7 (Метод 2) Для некоторых констант a и b найдите производную от (я) (х — а) (х — б) Пусть f (x) = (x — a) (x — b) Пусть u = (x — a) & v = (x — b) ∴ f (x) = uv Итак, f ’(x) = (uv)’ f ’(x) = u’v + v’u В поисках u ’и v’ и = х — а u ’= 1. x1–1 — 0 = 1. x0 = 1 v = (х — б) v ’= 1.×1–1 — 0 = 1. x0 = 1 Сейчас, f ’(x) = (uv)’ = u ’v + v’u = (1) (x — b) + 1 (x — a) = х — Ь + (х — а) = 2x — b — а = 2х — (Ь + а) Ex13.2, 7 Для некоторых констант a и b найдите производную от (ii) (ax2 + b) 2 Пусть f (x) = (ax2 + b) 2 Используя (a + b) 2 = a2 + b2 + 2ab = (ax2) 2 + (б) 2 + 2 (ax2) (б) = a2 x4 + (б) 2 + 2abx2 x2 f ’(x) = a2. 4×4–1 + 0 + 2ab. 2. x2–1 = а2.4×3 + 2ab. 2×1 = 4a2 x3 + 4abx Ex13.2, 7 Для некоторых констант a и b найдите производную от (iii) (x — a) � (x — b) � Пусть f (x) = (x — a) � (x — b) � Пусть u = (x — a) & v = (x — b) Итак, f (x) = 𝑢�𝑣� f ’(x) = 𝑢�𝑣���′� f ’(x) = 𝑢�′�𝑣 — 𝑣�′�𝑢� 𝑣�2�� В поисках u ’и v’ и = х — а u ’= 1. x1–1 — 0 = x0 = 1 v = x — b v ’= 1.×1–1 — b = 1.×0 = 1 f ’(x) = 𝑢�𝑣���′� = 𝑢�′�𝑣 — 𝑣�′�𝑢� 𝑣�2�� = 1 x — b� — (1) x — a� � x — b�2� = 1 x — b� — (1) x — a� � x — b�2� = x — b — x + a� x — b�2� = — b + a� x — b�2� = a — b� x — b�2� Следовательно, f ’(x) = 𝐚 — � 𝐱 — �𝟐�

    Показать больше .